Sei sulla pagina 1di 221

GROSS INCOME

G.R. No. L-12954 February 28, 1961


COLLECTOR OF INTERNAL REVENUE, petitioner,
vs.
ARTHUR HENDERSON, respondent.
x---------------------------------------------------------x
G.R. No. L-13049 February 28, 1961
ARTHUR HENDERSON, petitioner,
vs.
COLLECTOR OF INTERNAL REVENUE, respondent.
Office of the Solicitor General for petitioner.
Formilleza & Latorre for respondent.
PADILLA, J.:
These are petitioner filed by the Collector of Internal Revenue (G.R. No. L-
12954) and by Arthur Henderson (G.R. No. L-13049) under the provisions of
section 18, Republic Act No. 1125, for review of a judgment dated 26 June
1957 and a resolution dated 28 September 1957 rendered and adopted by
the Court of Tax Appeals in Case No. 237.
The spouses Artuhur Henderson and Marie B. Henderson (later referred to as
the taxpayers) filed with the Bureau of Internal Revenue returns of annual net
income for the years 1948 to 1952, inclusive, where the following net
incomes, personal exemptions and amounts subject to tax appear:
1948:

Net Income .......................................................


P29,573.79

Less:Personal Exemption ..............................


2,500.00

Amount subject to tax .......................................


P27,073.79
1949:

Net Income .......................................................


P31,817.66

Less:Personal Exemption ..............................


2,500.00

Amount subject to tax .......................................


P29,317.66
1950:

Net Income .......................................................


P34,815.74

Less:Personal Exemption ..............................


3,000.00

Amount subject to tax .......................................


P31,815.74
1951:

Net Income ........................................................


P32,605.83

Less:Personal Exemption ..............................


3,000.00

Amount subject to tax .......................................


P29,605.83
1952:

Net Income .......................................................


P36,780.11

Less:Personal Exemption ..............................


3,000.00

Amount subject to tax .......................................


P33,780.11
(Exhibits 1, 3, 5, 7, 9, A, F, J, N, R). In due time the taxpayers received from
the Bureau of Internal Revenue assessment notices Nos. 15804-48, 25450-
49, 15255-50, 25705-51 and 22527-52 and paid the amounts assessed as
follows:
1948:

14 May 1949, O.R. No. 52991, Exhibit B ......


P2,068.12

12 September 1950, O.R. No. 160473, Exhibit B-1 .


2,068.11

Total Paid .........................................................


P4,136.23
1949:

13 May 1950, O.R. No. 232366, Exhibit G ...........


P2,314.95

15 September 1950, O.R. No. 247918, Exhibit G-1 .


2,314.94

Total Paid .........................................................


P4,629.89
1950:

27 April 1951, O.R. No. 323173, Exhibit K ....


P7,273.00
1951:

Amount withheld from salary and paid by employer .


P5,780.40

15 May 1952, O.R. No. 33250, Exhibit O .................


360.50

15 August 1952, O.R. No. 383318, Exhibit O-1 ....


361.20

Total Paid .........................................................


P6,502.10
1952:

Amount withheld from salary and paid by employer .


P5,660.40

18 May 1953, O.R. No. 438026, Exhibit T ..


1,160.30

13 August 1953, O.R. No. 443483, Exhibit T-1 .....


1,160.30

Total Paid .........................................................


P7,981.00
On 28 November 1953, after investigation and verification, the Bureay of
Internal Revenue reassessed the taxpayers'income for the years 1948 to
1952, inclusive, as follows:
1948:

Net income per return ..................................


P29,573.79
Add:

Rent expense ...........................................................


7,200.00

Additional bonus for 1947 received May 13, 1948 .


6,500.00

Other income:

Manager's residential expense (2/29/48 a/c/#4.51)


1,400.00
Manager's residential expense (refer to 1948 P & L) ..
1,849.32

Entrance fee Marikina Gun & Country Club ....


200.00
Net income per investigation ............................................
P46,723.11
Less: Personal exemption ................................................
2,500.00
Net taxable income ..........................................................
P44,223.11
Tax due thereon ...............................................................
P8,562.47

Less: Amount of tax already paid per OR #52991 & 160473 ..

4,136.23
Deficiency tax still due & assessable ............................
P4,426.24

1949:

Net income per return ..................................


P31,817.66
Add: disallowances

Capital loss (no capital gain) ...................


P3,248.84

Undeclared bonus .....................


3,857.75

Rental allowance from A.I.U. ...................


1,800.00

Subsistence allowance from A.I.U. ...


6,051.30
14,958.09
Net income per investigation ............................................
P46,775.75
Less: Personal exemption .................................................
2,500.00
Amount of income subject to tax ...................................
43,275.75
Tax due thereon ................................................................
P8,292.21
Less: tax already assessed & paid per OR Nos. 232366 & 247918
4,629.89
Deficiency tax due .............................................................
P3,662.23
(Should be) ......................................................................
3,662.32

1950:

Net income per return ..................................


P34,815.74
Add:

Rent, electricity, water allowances .........................


8,373.73
Net income per investigation ............................................
P43,189.47
Less: Personal exemption .................................................
3,000.00
Net taxable income ............................................................
P40,189.47
Tax due thereon ................................................................
P10,296.00
Less: tax already paid per OR No. #323173
7,273.00
Deficiency tax due & assessable ...................
P3,023.00

1951:
Net income per return ..................................
P32,605.83
Add: house rental allowance from AIU
5,782.91
Net income per investigation ............................................
P83,388.74
Less: Personal exemption .................................................
3,000.00
Amount of income subject to tax ....................................
P35,388.74
Tax due thereon ................................................................
P 8,560.00
Less: tax already assessed and paid per O.R. Nos. A33250
& 383318 .......................
6,502.00
Deficiency tax due ..................
P2,058.00

1952:

Net income per return ..................................


P36,780.11
Add:

Withholding tax paid by company .....................................


600.00

Travelling allowances .......................................................


3,247.40

Allowances for rent, telephone, water, electricity, etc. .....


7,044.67
Net income per investigation ............................................
P47,672.18
Less: Personal exemption .................................................
3,000.00
Net taxable income ..................................
P44,672.18
Tax due thereon ................................................................
P12,089.00
Less: Tax already withheld
P5,660.40

Tax already paid per O.R. Nos. #438026, 443484


2,320.60
7,981.00
Deficiency tax still due & collectible ...............................
P4,108.00
(Exhibits 2, 4, 6, 8, 10) and demanded payment of thedeficiency taxes on or
before 28 February 1954 with respectto those due for the years 1948, 1949,
1950 and 1952and on or before 15 February 1954 with respect to thatdue for
the year 1951 (Exhibits B-2, H, L, P, S).
In the foregoing assessments, the Bureau of InternalRevenue considered as
part of their taxable income thetaxpayer-husband's allowances for rental,
residential expenses,subsistence, water, electricity and telephone; bonuspaid
to him; withholding tax and entrance fee to the Marikinagun and Country Bluc
paid by his employer for hisaccount; and travelling allowance of his wife. On
26 and27 January 1954 the taxpayers asked for reconsiderationof the
foregoing assessment (pp. 29, 31, BIR rec.) andon 11 Februayr 1954 and 28
February 1955 stated thegrounds and reasons in support of their request for
reconsideration (pp. 36-38, 62-66, BIR rec.). The claimthat as regards the
husband-taxpayer's allowances forrental and utilities such as water, electricity
and telephone,he did not receive the money for said allowances, but thatthey
lieved in the apartment furnished and paid for byhis employer for its
convenience; that they had no choicebut live in the said apartment furnished
by his employer,otherwise they would have lived in a less expensive one;that
as regards his allowances for rental of P7,200 andresidential expenses of
P1,400 and P1,849.32 in 1948, rentalof P1,800 and subsistence of P6,051.50
(the latter merelyconsisting of allowances for rent and utilities such as
light,water, telephone, etc.) in 1949 rental, electricity and waterof P8,373.73 in
1950, rental of P5,782.91 in 1951 and rental,telephone, water, electricity, etc.
of P7,044.67 in 1952, onlythe amount of P3,900 for each year, which is the
amountthey would have spent for rental of an apartment includingutilities,
should be taxed; that as regards the amount ofP200 representing entrance
fee to the Marikina Gun andCountry Club paid for him by his employer in
1948, thesame should not be considered as part of their income forit was an
expense of his employer and his membershiptherein was merely incidental to
his duties of increasingand sustaining the business of his employer; and that
asregards the wife-taxpayer's travelling allowance of P3,247.40 in 1952, it
should not be considered as part of theirincome because she merely
accompanied him in his businesstrip to New York as his secretary and, at the
behestof her husband's employer, to study and look into the detailsof the
plans and decorations of the building intendedto be constructed byn his
employer in its property at DeweyBoulevard. On 15 and 27 February 1954,
the taxpayerspaid the deficiency taxes assessed under Official ReceiptsNos.
451841, 451842, 451843, 451748 and 451844 (ExhibitsC, I, M, Q, and Y).
After hearing conducted by theConference Staff of the Bureau of Internal
Revenue on5 October 1954 (pp. 74-85, BIR rec.), on 27 May 1955the Staff
recommended to the Collector of Internal Revenuethat the assessments
made on 28 November 1953 (Exhibits2, 4, 6, 8, 10) be sustained except that
the amountof P200 as entrance fee to the Marikina Gun and CountryClub
paid for the husband-taxpayer's account by his employerin 1948 should not
be considered as part of thetaxpayers' taxable income for that year (pp. 95-
107, BIRrec.). On 14 July 1955, in line with the recommendationof the
Conference Staff, the Collector of Internal Revenuedenied the taxpayers'
request for reconsideration, exceptas regards the assessment of their income
tax due for theyear 1948, which was modified as follows:
Net income per return
P29,573.79
Add: Rent expense
7,200.00

Additional bonus for 1947 received on May 13, 1948


6,500.00

Manager's residential expense (2/29/48 a/c #4.41)


1,400.00

Manager's residential expense (1948 profit and loss)


1,849.32
Net income per investigation
P46,523.11
Less: Personal exemption
2,500.00
Net taxable income
P44,023.11
Tax due thereon
P 8,506.47
Less; Amount already paid
4,136.23
Deficiency tax still due
P 4,370.24
and demanded payment of the deficiency taxes of P4,370.24for 1948,
P3,662.23 for 1949, P3,023 for 1950, P2,058 for1951 and P4,108 for 1952,
5% surcharge and 1% monthlyinterest thereon from 1 March 1954 to the date
of paymentand P80 as administrative penalty for late payment,to the City
Treasurer of Manila not later than 31 July1955 (Exhibit 14). On 30 January
1956 the taxpayersagain sought a reconsideration of the denial of their
requestfor reconsideration and offered to settle the case ona more equitable
basis by increasing the amount of thetaxable portion of the husband-
taxpayer's allowances forrental, etc. from P3,000 yearly to P4,800 yearly,
which "isthe value to the employee of the benefits he derived
therefrommeasured by what he had saved on account thereof'in the ordinary
course of his life ... for which hewould have spent in any case'". The
taxpayers also reiteratedtheir previous stand regarding the
transportationallowance of the wife-taxpayer of P3,247.40 in 1952
andrequested the refund of the amounts of P3,477.18, P569.33,P1,294, P354
and P2,164, or a total of P7,858.51, (Exhibit Z). On 10 February 1956 the
taxpayers again requestedthe Collector of Internal Revenue to refund to them
theamounts allegedly paid in excess as income taxes for theyears 1948 to
1952, inclusive (Exhibit Z-1). The Collectorof Internal Revenue did not take
any action on the taxpayers'request for refund.
On 15 February 1956 the taxpayers filed in the Courtof Tax Appeals a petition
to review the decision of theCollector of Internal Revenue (C.T.A. Case No.
237). Afterhearing, on 26 June 1957 the Court rendered judgmentholding
"that the inherent nature of petitioner's(the husband-taxpayer) employment as
president of theAmerican International Underwriters as president of
theAmerican International Underwriters of the Philippines,Inc. does not
require him to occupy the apartments suppliedby his employer-corporation;"
that, however, onlythe amount of P4,800 annually, the ratable value to him
ofthe quarters furnished constitutes a part of taxable income;that since the
taxpayers did not receive any benefitout of the P3,247.40 traveling expense
allowance grantedin 1952 to the wife-taxpayer and that she merely
undertookthe trip abroad at the behest of her husband's employer,the same
could not be considered as income; andthat even if it were considered as
such, still it could not besubject to tax because it was deductible as travel
expense;and ordering the Collector of Internal Revenue to refundto the
taxpayers the amount of P5,109.33 with interestfrom 27 February 1954,
without pronouncement as tocosts. The taxpayers filed a motion for
reconsiderationclaiming that the amount of P5,986.61 is the amount
refundableto them because the amounts of P1,400 and P1,849.32 as
manager's residential expenses in 1948 shouldnot be included in their taxable
net income for the reasonthat they are of the same nature as the rentals for
theapartment, they being mainly expenses for utilities aslight, water and
telephone in the apartment furnished bythe husbant-taxpayer's employer. The
Collector of InternalRevenue filed an opposition to their motion for
reconsideration.He also filed a separate motion for reconsiderationof the
decision claiming that his assessmentunder review was correct and should
have been affirmed.The taxpayers filed an opposition to this motion for
reconsiderationof the Collector of Internal Revenue; thelatter, a reply thereto.
On 28 September 1957 the Courtdenied both motions for reconsideration. On
7 October1957 the Collector of Internal Revenue filed a notice ofappeal in the
Court of Tax Appeals and on 21 October1957, within the extension of time
previously granted bythis Court, a petition for review (G.R. No. L-12954).
On29 October 1957 the taxpayers filed a notice of appealin the Court of Tax
Appeals and a petition for review inthis Court (G.R. No. L-13049).
The Collector of Internal Revenue had assigned the followingerrors allegedly
committed by the Court of TaxAppeals:
I. The Court of Tax Appeals erred in finding that theherein respondent did not
have any choice in the selection ofthe living quarters occupied by him.
II. The Court of Tax Appeals erred in not consideringthe fact that respondent
is not a minor company official butthe President of his employer-corporation,
in the appreciationof respondent's alleged lack of choice in the matter of the
selectionof the quarters occupied by him.
III. The Court of Tax Appeals erred in giving full weightand credence to
respondent's allegation, a self-serving and unsupported declaration that the
ratable value to him of the living quarters and subsistence allowance was only
P400.00 a month.
IV. The Court of Tax Appeals erred in holding that only the ratable value of
P4,800.00 per annum, or P400.00 a month constitutes income to respondent.
V. The Court of Tax Appeals erred in arbitrarily fixing the amount of P4,800.00
per annum, or P400.00 a month as the only amount taxable aganst
respondent during the five tax years in question.
VI. The Court of Tax Appeals erred in not finding that travelling allowance in
the amount of P3,247.40 constituted income to respondent and, therefore,
subject to the income tax.
VII. The Court of Tax Appeals erred in ordering the refund of the sum of
P5,109.33 with interest from February 17, 1954. (G.R. No. L-12954.)
The taxpayers have assigned the following errors allegedly committed by the
Court of Tax Appeals:
I. The Court of Tax Appeals erred in its computation of the 1948 income tax
and consequently in the amount that should be refunded for that year.
II. The Court of Tax Appeals erred in denying our motion for reconsideration
as contained in its resolution dated September 28, 1957. (G.R. No. L-13049.)
The Government's appeal:
The Collector of Internal Revenue raises questions of fact. He claims that the
evidence is not sufficient to support the findings and conclusion of the Court
of Tax Appeals that the quarters occupied by the taxpayers were not of their
choice but that of the husband-taxpayer's employer; that it did not take into
consideration the fact that the husband-taxpayer is not a mere minor
company official, but the highest executive of his employer-corporation; and
that the wife-taxpayer's trip abroad in 1952 was not, as found by the Court, a
business but a vacation trip. In Collector of Internal Revenue vs. Aznar, 56,
Off. Gaz. 2386, this Court held that in petitions for review under section 18,
Republic Act No. 1125, it may review the findings of fact of the Court of Tax
Appeals.
The determination of the main issue in the case requires a review of the
evidence. Are the allowances for rental of the apartment furnished by the
husband-taxpayer's employer-corporation, including utilities such as light,
water, telephone, etc. and the allowance for travel expenses given by his
employer-corporation to his wife in 1952 part of taxable income? Section 29,
Commonwealth Act No. 466, National Internal Revenue Code, provides:
"Gross income" includes gains, profits, and income derived from salaries,
wages, or compensation for personal service of whatever kind and in
whatever form paid, or from professions, vocations, trades, businesses,
commerce, sales, or dealings in property, whether real or personal, growing
out of the ownership or use of or interest in such property; also from interest,
rents dividend, securities, or the transaction of any business carried on for
gain or profit, or gains, profits, and income derived from any source whatever.
(Emphasis ours.)
The Court of Tax Appeals found that the husband-taxpayer "is the president of
the American International Underwriters for the Philippines, Inc., a domestic
corporation engaged in insurance business;" that the taxpayers "entertained
officials, guests and customers of his employer-corporation, in apartments
furnished by the latter and successively occupied by him as president thereof;
that "In 1952, petitioner's wife, Mrs. Marie Henderson, upon request o Mr. C.
V. Starr, chairman of the parent corporation of the American International
Underwriters for the Philippines, Inc., undertook a trip to New York in
connection with the purchase of a lot in Dewey Boulevardby petitioner's
employer-corporatio, the construction of a building thereon, the drawing of
prospectus and plans for said building, and other related matters."
Arthur H. Henderson testified that he is the President of American
International Underwriters for the Philippines, Inc., which representa a group
of American insurance companies engagad in the business of general
insurance except life insurance; that he receives a basic annual salary of
P30,000 and allowance for house rental and utilities like light, water,
telephone, etc.; that he and his wife are childless and are the only two in the
family; that during the years 1948 to 1952, they lived in apartments chosen by
his employer; that from 1948 to the early part of 1950, they lived at the
Embassy Apartments on Dakota Street, Manila, where they had a large sala,
three bedrooms, dining room, two bathrooms, kitchen and a large porch, and
from the early part of 1950 to 1952, they lived at the Rosaria Apartments on
the same street where they had a kitchen, sala, dining room two bedrooms
and bathroom; that despite the fact that they were the only two in the family,
they had to live in apartments of the size beyond their personal needs
because as president of the corporation, he and his wife had to entertain and
put up houseguests; that during all those years of 1948 to 1952, inclusive,
they entertained and put up houseguests of his company's officials, guests
and customers such as the president of C, V. Starr & Company, Inc., who
spent four weeks in his apartment, Thomas Cocklin, a lawyer from
Washington, D.C., and Manuel Elizalde, a stockholder of AIUPI; that were he
not required by his employer to live in those apartments furnished to him, he
and his wife would have chosen an apartment only large enough for them and
spend from P300 to P400 monthly for rental; that of the allowances granted to
him, only the amount of P4,800 annually, the maximum they would have
spent for rental, should be considered as taxable income and the excess
treated as expense of the company; and that the trip to New York undertaken
by his wife in 1952, for which she was granted by his employer-corporation
travelling expense allowance of P3,247.40, was made at the behest of his
employer to assist its architect in the preparation of the plans for a proposed
building in Manila and procurement of supplies and materials for its use,
hence the said amount should not be considered as part of taxable income. In
support of his claim, letters written by his wife while in New York concerning
the proposed building, inquiring about the progress made in the acquisition of
the lot, and informing him of the wishes of Mr. C. V. Starr, chairman of the
board of directors of the parent-corporation (Exhibits U-1, U-1-A, V, V-1 and
W) and a letter written by the witness to Mr. C. V. Starr concerning the
proposed building (Exhibits X, X-1) were presented in evidence.
Mrs. Marie Henderson testified that for almost three years, she and her
husband gave parties every Friday night at their apartment for about 18 to 20
people; that their guests were officials of her husband's employer-corporation
and other corporations; that during those parties movies for the entertainment
of the guests were shown after dinner; that they also entertained during
luncheons and breakfasts; that these involved and necessitated the services
of additional servants; and that in 1952 she was asked by Mr. C. V. Starr to
come to New York to take up problems concerning the proposed building and
entertainment because her husband could not make the trip himself, and
because "the woman of the family is closer to those problems."
The evidence presented at the hearing of the case substantially supports the
findings of the Court of Tax Appeals. The taxpayers are childless and are the
only two in the family. The quarters, therefore, that they occupied at the
Embassy Apartments consisting of a large sala, three bedrooms, dining room,
two bathrooms, kitchen and a large porch, and at the Rosaria Apartments
consisting of a kitchen, sala dining room, two bedrooms and a bathroom,
exceeded their personal needs. But the exigencies of the husband-taxpayer's
high executive position, not to mention social standing, demanded and
compelled them to live in amore spacious and pretentious quarters like the
ones they had occupied. Although entertaining and putting up houseguests
and guests of the husbnad-taxpayer's employer-corporation were not his
predominand occupation as president, yet he and his wife had to entertain
and put up houseguests in their apartments. That is why his employer-
corporation had to grant him allowances for rental and utilities in addition to
his annual basic salary to take care of those extra expenses for rental and
utilities in excess of their personal needs. Hence, the fact that the taxpayers
had to live or did not have to live in the apartments chosen by the husband-
taxpayer's employer-corporation is of no moment, for no part of the
allowances in question redounded to their personal benefit or was retained by
them. Their bills for rental and utilities were paid directly by the employer-
corporation to the creditors (Exhibit AA to DDD, inclusive; pp. 104, 170-193,
t.s.n.). Neverthelss, as correctly held by the Court of Tax Appeals, the
taxpayers are entitled only to a ratable value of the allowances in question,
and only the amount of P4,800 annually, the reasonable amount they would
have spent for house rental and utilities such as light, water, telephone, etc.,
should be the amount subject to tax, and the excess considered as expenses
of the corporation.
Likewise, the findings of the Court of Tax Appeals that the wife-taxpayer had
to make the trip to New York at the behest of her husband's employer-
corporation to help in drawing up the plans and specificatins of a proposed
building, is also supported by the evidence. The parts of the letters written by
the wife-taxpayer to her husband while in New York and the letter written by
the husband-taxpayer to Mr. C. V. Starr support the said findings (Exhibits U-
2, V-1, W-1, X). No part of the allowance for travellking expenses redounded
to the benefit of the taxpayers. Neither was a part thereof retained by them.
The fact that she had herself operated on for tumors while in New York wsa
but incidental to her stay there and she must have merely taken advantage of
her presence in that city to undergo the operation.
The taxpayers' appeal:
The taxpayers claim that the Court of Tax Appeals erred in considering the
amounts of P1,400 and P1,849.32, or a total of P3,249.32, for "manager's
residential expense" in 1948 as taxable income despite the fact "that they
were of the same nature as the rentals for the apartment, they being
expenses for utilities, such as light, water and telephone necessarily
incidental to the apartment furnished to him by his employer."
Mrs. Crescencia Perez Ramos, an examiner of the Bureau of Internal
Revenue who examined the books of accound of the American International
Underwriters for the Philippines, Inc., testified that he total amount of
P3,249.32 was reflected in its books as "living expenses of Mr. and Mrs.
Arthur Henderson in the quarters they occupied in 1948;" and that "the
amount of P1,400 was included as manager's residential expense while the
amount of P1,849.32 was entered as profit and loss account."
Buenaventura Loberiza, acting head of the accouting department of the
American International Underwriters for the Philippines, Inc., testified that
rentals, utilities, water, telephone and electric bills of executives of the
corporation were entered in the books of account as "subsistence allowances
and expenses;" that there was a separate account for salaries and wages of
employees and officers; and that expenses for rentals and other utilities were
not charged to salary accounts.
The taxpayers' claim is supported by the evidence. The total amount of
P3,249.32 "for manager's residential expense" in 1948 should be treated as
rentals for apartments and utilities and should not form part of the ratable
value subject to tax.
The computation made by the taxpayers is correct. Adding to the amount of
P29,573.79, their net income per return, the amount of P6,500, the bonus
received in 1948, and P4,800, the taxable ratable value of the allowances,
brings up their gross income to P40,873.79. Deducting therefrom the amount
of P2,500 for personal exemption, the amount of P38,373.79 is the amount
subject to income tax. The income tax due on this amount is P6,957.19 only.
Deducting the amount of income tax due, P6,957.19, from the amount
already paid, P8,562.47 (Exhibits B, B-1, C), the amount of P1,605.28 is the
amount refundable to the taxpayers. Add this amount to P563.33, P1,294.00,
P354.00 and P2,154.00, refundable to the taxpayers for 1949, 1950, 1951
and 1952 and the total is P5,986.61.
The judgment under review is modified as above indicated. The Collector of
Internal Revenue is ordered to refund to the taxpayers the sum of P5,986.61,
without pronouncement as to costs.

G.R. No. 96016 October 17, 1991


COMMISSIONER OF INTERNAL REVENUE, petitioner,
vs.
THE COURT OF APPEALS and EFREN P. CASTANEDA, respondents.
Leovigildo Monasterial for private respondent.
RESOLUTION

PADILLA, J.:p
The issue to be resolved in this petition for review on certiorari is whether or
not terminal leave pay received by a government official or employee on the
occasion of his compulsory retirement from the government service is subject
to withholding (income) tax.
We resolve the issue in the negative.
Private respondent Efren P. Castaneda retired from the government service
as Revenue Attache in the Philippine Embassy in London, England, on 10
December 1982 under the provisions of Section 12 (c) of Commonwealth Act
186, as amended. Upon retirement, he received, among other benefits,
terminal leave pay from which petitioner Commissioner of Internal Revenue
withheld P12,557.13 allegedly representing income tax thereon.
Castaneda filed a formal written claim with petitioner for a refund of the
P12,557.13, contending that the cash equivalent of his terminal leave is
exempt from income tax. To comply with the two-year prescriptive period
within which claims for refund may be filed, Castaneda filed on 16 July 1984
with the Court of Tax Appeals a Petition for Review, seeking the refund of
income tax withheld from his terminal leave pay.
The Court of Tax Appeals found for private respondent Castaneda and
ordered the Commissioner of Internal Revenue to refund Castaneda the sum
of P12,557.13 withheld as income tax. (,Annex "C", petition).
Petitioner appealed the above-mentioned Court of Tax Appeals decision to
this Court, which was docketed as G.R. No. 80320. In turn, we referred the
case to the Court of Appeals for resolution. The case was docketed in the
Court of Appeals as CA-G.R. SP No. 20482.
On 26 September 1990, the Court of Appeals dismissed the petition for
review and affirmed the decision of the Court of Tax Appeals. Hence, the
present recourse by the Commissioner of Internal Revenue.
The Solicitor General, acting on behalf of the Commissioner of Internal
Revenue, contends that the terminal leave pay is income derived from
employer-employee relationship, citing in support of his stand Section 28 of
the National Internal Revenue Code; that as part of the compensation for
services rendered, terminal leave pay is actually part of gross income of the
recipient. Thus
. . . It (terminal leave pay) cannot be viewed as salary for purposes which
would reduce it. . . . there can thus be no "commutation of salary" when a
government retiree applies for terminal leave because he is not receiving it as
salary. What he applies for is a "commutation of leave credits." It is an
accumulation of credits intended for old age or separation from service. . . .
The Court has already ruled that the terminal leave pay received by a
government official or employee is not subject to withholding (income) tax. In
the recent case of Jesus N. Borromeo vs. The Hon. Civil Service
Commission, et al., G.R. No. 96032, 31 July 1991, the Court explained the
rationale behind the employee's entitlement to an exemption from withholding
(income) tax on his terminal leave pay as follows:
. . . commutation of leave credits, more commonly known as terminal leave,
is applied for by an officer or employee who retires, resigns or is separated
from the service through no fault of his own. (Manual on Leave Administration
Course for Effectiveness published by the Civil Service Commission, pages
16-17). In the exercise of sound personnel policy, the Government
encourages unused leaves to be accumulated. The Government recognizes
that for most public servants, retirement pay is always less than generous if
not meager and scrimpy. A modest nest egg which the senior citizen may
look forward to is thus avoided. Terminal leave payments are given not only
at the same time but also for the same policy considerations governing
retirement benefits.
In fine, not being part of the gross salary or income of a government official or
employee but a retirement benefit, terminal leave pay is not subject to income
tax.
ACCORDINGLY, the petition for review is hereby DENIED.
SO ORDERED.

[G.R. No. 108576. January 20, 1999]


COMMISSIONER OF INTERNAL REVENUE, petitioner,
vs. THE COURT OF APPEALS, COURT OF TAX
APPEALS and A. SORIANO CORP., respondents.
DECISION
MARTINEZ, J.:
Petitioner Commissioner of Internal Revenue (CIR) seeks the reversal
of the decision of the Court of Appeals (CA)[if !supportFootnotes][1][endif] which
affirmed the ruling of the Court of Tax Appeals (CTA) [if !supportFootnotes][2][endif] that
private respondent A. Soriano Corporations (hereinafter ANSCOR)
redemption and exchange of the stocks of its foreign stockholders cannot
be considered as essentially equivalent to a distribution of taxable
dividends under Section 83(b) of the 1939 Internal Revenue Act [if !supportFootnotes]
[3][endif]

The undisputed facts are as follows:


Sometime in the 1930s, Don Andres Soriano, a citizen and resident of
the United States, formed the corporation A. Soriano Y Cia, predecessor of
ANSCOR, with a P1,000,000.00 capitalization divided into 10,000
common shares at a par value of P100/share. ANSCOR is wholly owned
and controlled by the family of Don Andres, who are all non-resident
aliens.[if !supportFootnotes][4][endif] In 1937, Don Andres subscribed to 4,963 shares of
the 5,000 shares originally issued.[if !supportFootnotes][5][endif]
On September 12, 1945, ANSCORs authorized capital stock was
increased to P2,500,000.00 divided into 25,000 common shares with the
same par value. Of the additional 15,000 shares, only 10,000 was issued
which were all subscribed by Don Andres, after the other stockholders
waived in favor of the former their pre-emptive rights to subscribe to the
new issues.[if !supportFootnotes][6][endif] This increased his subscription to 14,963
common shares.[if !supportFootnotes][7][endif] A month later,[if !supportFootnotes][8][endif] Don Andres
transferred 1,250 shares each to his two sons, Jose and Andres, Jr., as their
initial investments in ANSCOR.[if !supportFootnotes][9][endif] Both sons are foreigners.[if !
supportFootnotes][10][endif]

By 1947, ANSCOR declared stock dividends. Other stock dividend


declarations were made between 1949 and December 20, 1963. [if !supportFootnotes]
[11][endif]
On December 30, 1964 Don Andres died. As of that date, the records
revealed that he has a total shareholdings of 185,154 shares [if !supportFootnotes][12]
[endif]
- 50,495 of which are original issues and the balance of 134,659 shares
as stock dividend declarations.[if !supportFootnotes][13][endif] Correspondingly, one-half
of that shareholdings or 92,577[if !supportFootnotes][14][endif] shares were transferred to
his wife, Doa Carmen Soriano, as her conjugal share. The other half
formed part of his estate.[if !supportFootnotes][15][endif]
A day after Don Andres died, ANSCOR increased its capital stock to
P20M[if !supportFootnotes][16][endif] and in 1966 further increased it to P30M.[if !supportFootnotes]
[17][endif]
In the same year (December 1966), stock dividends worth 46,290
and 46,287 shares were respectively received by the Don Andres estate [if !
supportFootnotes][18][endif]
and Doa Carmen from ANSCOR. Hence, increasing their
accumulated shareholdings to 138,867 and 138,864 [if !supportFootnotes][19][endif]
common shares each.[if !supportFootnotes][20][endif]
On December 28, 1967, Doa Carmen requested a ruling from the
United States Internal Revenue Service (IRS), inquiring if an exchange of
common with preferred shares may be considered as a tax avoidance
scheme[if !supportFootnotes][21][endif] under Section 367 of the 1954 U.S. Revenue Act.
[if !supportFootnotes][22][endif]
By January 2, 1968, ANSCOR reclassified its existing
300,000 common shares into 150,000 common and 150,000 preferred
shares.[if !supportFootnotes][23][endif]
In a letter-reply dated February 1968, the IRS opined that the
exchange is only a recapitalization scheme and not tax avoidance. [if !
supportFootnotes][24][endif]
Consequently,[if !supportFootnotes][25][endif] on March 31, 1968 Doa
Carmen exchanged her whole 138,864 common shares for 138,860 of the
newly reclassified preferred shares. The estate of Don Andres in turn,
exchanged 11,140 of its common shares for the remaining 11,140 preferred
shares, thus reducing its (the estate) common shares to 127,727.[if !supportFootnotes]
[26][endif]

On June 30, 1968, pursuant to a Board Resolution, ANSCOR


redeemed 28,000 common shares from the Don Andres estate. By
November 1968, the Board further increased ANSCORs capital stock to
P75M divided into 150,000 preferred shares and 600,000 common shares.[if
!supportFootnotes][27][endif]
About a year later, ANSCOR again redeemed 80,000
common shares from the Don Andres estate, [if !supportFootnotes][28][endif] further
reducing the latters common shareholdings to 19,727. As stated in the
board Resolutions, ANSCORs business purpose for both redemptions of
stocks is to partially retire said stocks as treasury shares in order to reduce
the companys foreign exchange remittances in case cash dividends are
declared.[if !supportFootnotes][29][endif]
In 1973, after examining ANSCORs books of account and records,
Revenue examiners issued a report proposing that ANSCOR be assessed
for deficiency withholding tax-at-source, pursuant to Sections 53 and 54 of
the 1939 Revenue Code,[if !supportFootnotes][30][endif] for the year 1968 and the second
quarter of 1969 based on the transactions of exchange and redemption of
stocks.[if !supportFootnotes][31][endif] The Bureau of Internal Revenue (BIR) made the
corresponding assessments despite the claim of ANSCOR that it availed of
the tax amnesty under Presidential Decree (P.D.) 23[if !supportFootnotes][32][endif] which
were amended by P.D.s 67 and 157.[if !supportFootnotes][33][endif] However, petitioner
ruled that the invoked decrees do not cover Sections 53 and 54 in relation
to Article 83(b) of the 1939 Revenue Act under which ANSCOR was
assessed.[if !supportFootnotes][34][endif] ANSCORs subsequent protest on the
assessments was denied in 1983 by petitioner.[if !supportFootnotes][35][endif]
Subsequently, ANSCOR filed a petition for review with the CTA
assailing the tax assessments on the redemptions and exchange of stocks.
In its decision, the Tax Court reversed petitioners ruling, after finding
sufficient evidence to overcome the prima facie correctness of the
questioned assessments.[if !supportFootnotes][36][endif] In a petition for review, the CA,
as mentioned, affirmed the ruling of the CTA.[if !supportFootnotes][37][endif] Hence, this
petition.
The bone of contention is the interpretation and application of Section
83(b) of the 1939 Revenue Act[if !supportFootnotes][38][endif] which provides:
Sec.83.Distributionofdividendsorassetsbycorporations.
(b)StockdividendsAstockdividendrepresentingthetransferofsurplusto
capitalaccountshallnotbesubjecttotax.However,ifacorporation
cancelsorredeemsstockissuedasadividendatsuchtimeandinsuch
mannerastomakethedistributionandcancellationorredemption,in
wholeorinpart,essentiallyequivalenttothedistributionofataxable
dividend,theamountsodistributedinredemptionorcancellationofthe
stockshallbeconsideredastaxableincometotheextentitrepresentsa
distributionofearningsorprofitsaccumulatedafterMarchfirst,nineteen
hundredandthirteen.(Italicssupplied).
Specifically, the issue is whether ANSCORs redemption of stocks from its
stockholder as well as the exchange of common with preferred shares can
be considered as essentially equivalent to the distribution of taxable
dividend, making the proceeds thereof taxable under the provisions of the
above-quoted law.
Petitioner contends that the exchange transaction is tantamount to
cancellation under Section 83(b) making the proceeds thereof taxable. It
also argues that the said Section applies to stock dividends which is the
bulk of stocks that ANSCOR redeemed. Further, petitioner claims that
under the net effect test, the estate of Don Andres gained from the
redemption. Accordingly, it was the duty of ANSCOR to withhold the tax-
at-source arising from the two transactions, pursuant to Section 53 and 54
of the 1939 Revenue Act.[if !supportFootnotes][39][endif]
ANSCOR, however, avers that it has no duty to withhold any tax
either from the Don Andres estate or from Doa Carmen based on the two
transactions, because the same were done for legitimate business purposes
which are (a) to reduce its foreign exchange remittances in the event the
company would declare cash dividends,[if !supportFootnotes][40][endif] and to (b)
subsequently filipinized ownership of ANSCOR, as allegedly envisioned
by Don Andres.[if !supportFootnotes][41][endif] It likewise invoked the amnesty
provisions of P.D. 67.
We must emphasize that the application of Sec. 83(b) depends on the
special factual circumstances of each case. [if !supportFootnotes][42][endif] The findings of
facts of a special court (CTA) exercising particular expertise on the subject
of tax, generally binds this Court,[if !supportFootnotes][43][endif] considering that it is
substantially similar to the findings of the CA which is the final arbiter of
questions of facts.[if !supportFootnotes][44][endif] The issue in this case does not only deal
with facts but whether the law applies to a particular set of facts.
Moreover, this Court is not necessarily bound by the lower courts
conclusions of law drawn from such facts.[if !supportFootnotes][45][endif]
AMNESTY:
We will deal first with the issue of tax amnesty. Section 1 of P.D. 67 [if !
supportFootnotes][46][endif]
provides:
I.Inallcasesofvoluntarydisclosuresofpreviouslyuntaxedincome
and/orwealthsuchasearnings,receipts,gifts,bequestsoranyother
acquisitionsfromanysourcewhatsoeverwhicharetaxableunderthe
NationalInternalRevenueCode,asamended,realizedhereorabroadby
anytaxpayer,naturalorjuridical;thecollectionofallinternalrevenue
taxesincludingtheincrementsorpenaltiesoraccountofnonpaymentas
wellasallcivil,criminaloradministrativeliabilitiesarisingfromor
incidenttosuchdisclosuresundertheNationalInternalRevenueCode,the
RevisedPenalCode,theAntiGraftandCorruptPracticesAct,theRevised
AdministrativeCode,theCivilServicelawsandregulations,lawsand
regulationsonImmigrationandDeportation,oranyotherapplicablelaw
orproclamation,areherebycondonedand,inlieuthereof,ataxoften
(10%)percentumonsuchpreviouslyuntaxedincomeorwealthishereby
imposed,subjecttothefollowingconditions:(conditionsomitted)
[Emphasissupplied].
The decree condones the collection of all internal revenue taxes including
the increments or penalties or account of non-payment as well as all civil,
criminal or administrative liabilities arising from or incident to (voluntary)
disclosures under the NIRC of previously untaxed income and/or wealth
realized here or abroad by any taxpayer, natural or juridical.
May the withholding agent, in such capacity, be deemed a taxpayer
for it to avail of the amnesty? An income taxpayer covers all persons who
derive taxable income.[if !supportFootnotes][47][endif] ANSCOR was assessed by
petitioner for deficiency withholding tax under Section 53 and 54 of the
1939 Code. As such, it is being held liable in its capacity as a withholding
agent and not in its personality as a taxpayer.
In the operation of the withholding tax system, the withholding agent
is the payor, a separate entity acting no more than an agent of the
government for the collection of the tax[if !supportFootnotes][48][endif] in order to ensure
its payments;[if !supportFootnotes][49][endif] the payer is the taxpayer he is the person
subject to tax impose by law;[if !supportFootnotes][50][endif] and the payee is the taxing
authority.[if !supportFootnotes][51][endif] In other words, the withholding agent is merely
a tax collector, not a taxpayer. Under the withholding system, however, the
agent-payor becomes a payee by fiction of law. His (agent) liability is
direct and independent from the taxpayer,[if !supportFootnotes][52][endif] because the
income tax is still impose on and due from the latter. The agent is not
liable for the tax as no wealth flowed into him he earned no income. The
Tax Code only makes the agent personally liable for the tax [if !supportFootnotes][53]
[endif]
(c) 1939 Tax Code, as amended by R.A. No. 2343 which provides in
part that xxx Every such person is made personally liable for such tax
xxx.53 arising from the breach of its legal duty to withhold as distinguish
from its duty to pay tax since:
thegovernmentscauseofactionagainstthewithholdingagentisnotfor
thecollectionofincometax,butfortheenforcementofthewithholding
provisionofSection53oftheTaxCode,compliancewithwhichis
imposedonthewithholdingagentandnotuponthetaxpayer.[if!supportFootnotes][54]
[endif]

Not being a taxpayer, a withholding agent, like ANSCOR in this


transaction, is not protected by the amnesty under the decree.
Codalprovisionsonwithholdingtaxaremandatoryandmustbecomplied
withbythewithholdingagent.[if!supportFootnotes][55][endif]Thetaxpayershouldnot
answerforthenonperformancebythewithholdingagentofitslegalduty
towithholdunlessthereiscollusionorbadfaith.Theformercouldnotbe
deemedtohaveevadedthetaxhadthewithholdingagentperformedits
duty.Thiscouldbethesituationforwhichtheamnestydecreewas
intended.Thus,tocurtailtaxevasionandgivetaxevadersachanceto
reform,[if!supportFootnotes][56][endif]itwasdeemedadministrativelyfeasibletogrant
taxamnestyincertaininstances.Inaddition,ataxamnesty,muchlikea
taxexemption,isneverfavorednorpresumedinlawandifgrantedbya
statute,thetermsoftheamnestylikethatofataxexemptionmustbe
construedstrictlyagainstthetaxpayerandliberallyinfavorofthetaxing
authority.[if!supportFootnotes][57][endif]Theruleonstrictissimijurisequallyapplies.[if!
supportFootnotes][58][endif]
Sothat,anydoubtintheapplicationofanamnesty
law/decreeshouldberesolvedinfavorofthetaxingauthority.
Furthermore, ANSCORs claim of amnesty cannot prosper. The
implementing rules of P.D. 370 which expanded amnesty on previously
untaxed income under P.D. 23 is very explicit, to wit:
Section4.Casesnotcoveredbyamnesty.Thefollowingcasesarenot
coveredbytheamnestysubjectoftheseregulations:
xxx xxx xxx
(2)Taxliabilitieswithorwithoutassessments,onwithholdingtaxat
sourceprovidedunderSections53and54oftheNationalInternal
RevenueCode,asamended;[if!supportFootnotes][59][endif]
ANSCOR was assessed under Sections 53 and 54 of the 1939 Tax Code.
Thus, by specific provision of law, it is not covered by the amnesty.
TAX ON STOCK DIVIDENDS
General Rule
Section 83(b) of the 1939 NIRC was taken from Section 115(g)(1) of
the U.S. Revenue Code of 1928.[if !supportFootnotes][60][endif] It laid down the general
rule known as the proportionate test[if !supportFootnotes][61][endif] wherein stock
dividends once issued form part of the capital and, thus, subject to income
tax.[if !supportFootnotes][62][endif] Specifically, the general rule states that:
Astockdividendrepresentingthetransferofsurplustocapitalaccount
shallnotbesubjecttotax.
Having been derived from a foreign law, resort to the jurisprudence of
its origin may shed light. Under the US Revenue Code, this provision
originally referred to stock dividends only, without any exception. Stock
dividends, strictly speaking, represent capital and do not constitute income
to its recipient.[if !supportFootnotes][63][endif] So that the mere issuance thereof is not yet
subject to income tax[if !supportFootnotes][64][endif] as they are nothing but an
enrichment through increase in value of capital investment. [if !supportFootnotes][65]
[endif]
As capital, the stock dividends postpone the realization of profits
because the fund represented by the new stock has been transferred from
surplus to capital and no longer available for actual distribution. [if !supportFootnotes]
[66][endif]
Income in tax law is an amount of money coming to a person within
a specified time, whether as payment for services, interest, or profit from
investment.[if !supportFootnotes][67][endif] It means cash or its equivalent. [if !supportFootnotes][68]
[endif]
It is gain derived and severed from capital, [if !supportFootnotes][69][endif] from labor
or from both combined[if !supportFootnotes][70][endif] - so that to tax a stock dividend
would be to tax a capital increase rather than the income. [if !supportFootnotes][71][endif]
In a loose sense, stock dividends issued by the corporation, are considered
unrealized gain, and cannot be subjected to income tax until that gain has
been realized. Before the realization, stock dividends are nothing but a
representation of an interest in the corporate properties. [if !supportFootnotes][72][endif] As
capital, it is not yet subject to income tax. It should be noted that capital
and income are different. Capital is wealth or fund; whereas income is
profit or gain or the flow of wealth.[if !supportFootnotes][73][endif] The determining factor
for the imposition of income tax is whether any gain or profit was derived
from a transaction.[if !supportFootnotes][74][endif]
The Exception

However,ifacorporationcancelsorredeemsstockissuedasadividendat
suchtimeandinsuchmannerastomakethedistributionandcancellation
orredemption,inwholeorinpart,essentiallyequivalenttothe
distributionofataxabledividend,theamountsodistributedinredemption
orcancellationofthestockshallbeconsideredastaxableincometothe
extentitrepresentsadistributionofearningsorprofitsaccumulatedafter
Marchfirst,nineteenhundredandthirteen.(Emphasissupplied).
InaresponsetotherulingoftheAmericanSupremeCourtinthecaseof
Eisnerv.Macomber[if!supportFootnotes][75][endif](thatproratastockdividendsarenot
taxableincome),theexemptingclauseabovequotedwasaddedbecause
corporationsfoundaloopholeintheoriginalprovision.Theyresortedto
deviousmeanstocircumventthelawandevadethetax.Corporate
earningswouldbedistributedundertheguiseofitsinitialcapitalizationby
declaringthestockdividendspreviouslyissuedandlaterredeemsaid
dividendsbypayingcashtothestockholder.Thisprocessofissuance
redemptionamountstoadistributionoftaxablecashdividendswhichwas
justdelayedsoastoescapethetax.Itbecomesaconvenienttechnical
strategytoavoidtheeffectsoftaxation.
Thus,toplugtheloopholetheexemptingclausewasadded.Itprovides
thattheredemptionorcancellationofstockdividends,dependingonthe
timeandmanneritwasmadeisessentiallyequivalenttoadistributionof
taxabledividends,makingtheproceedsthereoftaxableincometothe
extentitrepresentsprofits.Theexceptionwasdesignedtopreventthe
issuanceandcancellationorredemptionofstockdividends,whichis
fundamentallynottaxable,frombeingmadeuseofasadeviceforthe
actualdistributionofcashdividends,whichistaxable.[if!supportFootnotes][76][endif]
Thus,
theprovisionhadtheobviouspurposeofpreventingacorporationfrom
avoidingdividendtaxtreatmentbydistributingearningstoitsshareholders
intwotransactionsaproratastockdividendfollowedbyaprorata
redemptionthatwouldhavethesameeconomicconsequencesasasimple
dividend.[if!supportFootnotes][77][endif]
Although redemption and cancellation are generally considered capital
transactions, as such, they are not subject to tax. However, it does not
necessarily mean that a shareholder may not realize a taxable gain from
such transactions.[if !supportFootnotes][78][endif] Simply put, depending on the
circumstances, the proceeds of redemption of stock dividends are
essentially distribution of cash dividends, which when paid becomes the
absolute property of the stockholder. Thereafter, the latter becomes the
exclusive owner thereof and can exercise the freedom of choice [if !supportFootnotes]
[79][endif]
Having realized gain from that redemption, the income earner cannot
escape income tax.[if !supportFootnotes][80][endif]
As qualified by the phrase such time and in such manner, the
exception was not intended to characterize as taxable dividend every
distribution of earnings arising from the redemption of stock dividends. [if !
supportFootnotes][81][endif]
So that, whether the amount distributed in the redemption
should be treated as the equivalent of a taxable dividend is a question of
fact,[if !supportFootnotes][82][endif] which is determinable on the basis of the particular
facts of the transaction in question.[if !supportFootnotes][83][endif] No decisive test can be
used to determine the application of the exemption under Section 83(b)
The use of the words such manner and essentially equivalent negative any
idea that a weighted formula can resolve a crucial issue Should the
distribution be treated as taxable dividend. [if !supportFootnotes][84][endif] On this aspect,
American courts developed certain recognized criteria, which includes the
following:[if !supportFootnotes][85][endif]
1) the presence or absence of real business purpose,
2) the amount of earnings and profits available for the declaration of a regular
dividend and the corporations past record with respect to the declaration of
dividends,
3) the effect of the distribution as compared with the declaration of regular
dividend,
4) the lapse of time between issuance and redemption,[if !supportFootnotes][86][endif]
5) the presence of a substantial surplus[if !supportFootnotes][87][endif] and a generous supply of
cash which invites suspicion as does a meager policy in relation both to current
earnings and accumulated surplus.[if !supportFootnotes][88][endif]
REDEMPTION AND CANCELLATION

For the exempting clause of Section 83(b) to apply, it is indispensable


that: (a) there is redemption or cancellation; (b) the transaction involves
stock dividends and (c) the time and manner of the transaction makes it
essentially equivalent to a distribution of taxable dividends. Of these, the
most important is the third.
Redemption is repurchase, a reacquisition of stock by a corporation
which issued the stock[if !supportFootnotes][89][endif] in exchange for property, whether
or not the acquired stock is cancelled, retired or held in the treasury.[if !
supportFootnotes][90][endif]
Essentially, the corporation gets back some of its stock,
distributes cash or property to the shareholder in payment for the stock,
and continues in business as before. The redemption of stock dividends
previously issued is used as a veil for the constructive distribution of cash
dividends. In the instant case, there is no dispute that ANSCOR redeemed
shares of stocks from a stockholder (Don Andres) twice (28,000 and
80,000 common shares). But where did the shares redeemed come from? If
its source is the original capital subscriptions upon establishment of the
corporation or from initial capital investment in an existing enterprise, its
redemption to the concurrent value of acquisition may not invite the
application of Sec. 83(b) under the 1939 Tax Code, as it is not income but
a mere return of capital. On the contrary, if the redeemed shares are from
stock dividend declarations other than as initial capital investment, the
proceeds of the redemption is additional wealth, for it is not merely a
return of capital but a gain thereon.
It is not the stock dividends but the proceeds of its redemption that
may be deemed as taxable dividends. Here, it is undisputed that at the time
of the last redemption, the original common shares owned by the estate
were only 25,247.5.[if !supportFootnotes][91][endif] This means that from the total of
108,000 shares redeemed from the estate, the balance of 82,752.5 (108,000
less 25,247.5) must have come from stock dividends. Besides, in the
absence of evidence to the contrary, the Tax Code presumes that every
distribution of corporate property, in whole or in part, is made out of
corporate profits,[if !supportFootnotes][92][endif] such as stock dividends. The capital
cannot be distributed in the form of redemption of stock dividends without
violating the trust fund doctrine wherein the capital stock, property and
other assets of the corporation are regarded as equity in trust for the
payment of the corporate creditors.[if !supportFootnotes][93][endif] Once capital, it is
always capital.[if !supportFootnotes][94][endif] That doctrine was intended for the
protection of corporate creditors.[if !supportFootnotes][95][endif]
With respect to the third requisite, ANSCOR redeemed stock
dividends issued just 2 to 3 years earlier. The time alone that lapsed from
the issuance to the redemption is not a sufficient indicator to determine
taxability. It is a must to consider the factual circumstances as to the
manner of both the issuance and the redemption. The time element is a
factor to show a device to evade tax and the scheme of cancelling or
redeeming the same shares is a method usually adopted to accomplish the
end sought.[if !supportFootnotes][96][endif] Was this transaction used as a continuing plan,
device or artifice to evade payment of tax? It is necessary to determine the
net effect of the transaction between the shareholder-income taxpayer and
the acquiring (redeeming) corporation.[if !supportFootnotes][97][endif] The net effect test
is not evidence or testimony to be considered; it is rather an inference to be
drawn or a conclusion to be reached.[if !supportFootnotes][98][endif] It is also important to
know whether the issuance of stock dividends was dictated by legitimate
business reasons, the presence of which might negate a tax evasion plan. [if !
supportFootnotes][99][endif]

The issuance of stock dividends and its subsequent redemption must


be separate, distinct, and not related, for the redemption to be considered a
legitimate tax scheme.[if !supportFootnotes][100][endif] Redemption cannot be used as a
cloak to distribute corporate earnings.[if !supportFootnotes][101][endif] Otherwise, the
apparent intention to avoid tax becomes doubtful as the intention to evade
becomes manifest. It has been ruled that:
[A]noperationwithnobusinessorcorporatepurposeisameredevise
whichputontheformofacorporatereorganizationasadisguisefor
concealingitsrealcharacter,andthesoleobjectandaccomplishmentof
whichwastheconsummationofapreconceivedplan,nottoreorganizea
businessoranypartofabusiness,buttotransferaparcelofcorporate
sharestoastockholder.[if!supportFootnotes][102][endif]
Dependingoneachcase,theexemptingprovisionofSec.83(b)ofthe
1939Codemaynotbeapplicableiftheredeemedshareswereissuedwith
bonafidebusinesspurpose,[if!supportFootnotes][103][endif]whichisjudgedaftereach
andeverystepofthetransactionhavebeenconsideredandthewhole
transactiondoesnotamounttoataxevasionscheme.
ANSCOR invoked two reasons to justify the redemptions (1) the
alleged filipinization program and (2) the reduction of foreign exchange
remittances in case cash dividends are declared. The Court is not
concerned with the wisdom of these purposes but on their relevance to the
whole transaction which can be inferred from the outcome thereof. Again,
it is the net effect rather than the motives and plans of the taxpayer or his
corporation[if !supportFootnotes][104][endif] that is the fundamental guide in administering
Sec. 83(b). This tax provision is aimed at the result. [if !supportFootnotes][105][endif] It also
applies even if at the time of the issuance of the stock dividend, there was
no intention to redeem it as a means of distributing profit or avoiding tax
on dividends.[if !supportFootnotes][106][endif] The existence of legitimate business
purposes in support of the redemption of stock dividends is immaterial in
income taxation. It has no relevance in determining dividend equivalence. [if
!supportFootnotes][107][endif]
Such purposes may be material only upon the issuance of
the stock dividends. The test of taxability under the exempting clause,
when it provides such time and manner as would make the redemption
essentially equivalent to the distribution of a taxable dividend, is whether
the redemption resulted into a flow of wealth. If no wealth is realized from
the redemption, there may not be a dividend equivalence treatment. In the
metaphor of Eisner v. Macomber, income is not deemed realize until the
fruit has fallen or been plucked from the tree.
The three elements in the imposition of income tax are: (1) there must
be gain or profit, (2) that the gain or profit is realized or received, actually
or constructively,[if !supportFootnotes][108][endif] and (3) it is not exempted by law or
treaty from income tax. Any business purpose as to why or how the
income was earned by the taxpayer is not a requirement. Income tax is
assessed on income received from any property, activity or service that
produces the income because the Tax Code stands as an indifferent neutral
party on the matter of where income comes from.[if !supportFootnotes][109][endif]
As stated above, the test of taxability under the exempting clause of
Section 83(b) is, whether income was realized through the redemption of
stock dividends. The redemption converts into money the stock dividends
which become a realized profit or gain and consequently, the stockholders
separate property.[if !supportFootnotes][110][endif] Profits derived from the capital invested
cannot escape income tax. As realized income, the proceeds of the
redeemed stock dividends can be reached by income taxation regardless of
the existence of any business purpose for the redemption. Otherwise, to
rule that the said proceeds are exempt from income tax when the
redemption is supported by legitimate business reasons would defeat the
very purpose of imposing tax on income. Such argument would open the
door for income earners not to pay tax so long as the person from whom
the income was derived has legitimate business reasons. In other words,
the payment of tax under the exempting clause of Section 83(b) would be
made to depend not on the income of the taxpayer but on the business
purposes of a third party (the corporation herein) from whom the income
was earned. This is absurd, illogical and impractical considering that the
Bureau of Internal Revenue (BIR) would be pestered with instances in
determining the legitimacy of business reasons that every income earner
may interposed. It is not administratively feasible and cannot therefore be
allowed.
The ruling in the American cases cited and relied upon by ANSCOR
that the redeemed shares are the equivalent of dividend only if the shares
were not issued for genuine business purposes[if !supportFootnotes][111][endif] or the
redeemed shares have been issued by a corporation bona fide[if !supportFootnotes][112]
[endif]
bears no relevance in determining the non-taxability of the proceeds of
redemption. ANSCOR, relying heavily and applying said cases, argued
that so long as the redemption is supported by valid corporate purposes the
proceeds are not subject to tax. [if !supportFootnotes][113][endif] The adoption by the courts
below [if !supportFootnotes][114][endif] of such argument is misleading if not misplaced. A
review of the cited American cases shows that the presence or absence of
genuine business purposes may be material with respect to the issuance or
declaration of stock dividends but not on its subsequent redemption. The
issuance and the redemption of stocks are two different transactions.
Although the existence of legitimate corporate purposes may justify a
corporations acquisition of its own shares under Section 41 of the
Corporation Code,[if !supportFootnotes][115][endif] such purposes cannot excuse the
stockholder from the effects of taxation arising from the redemption. If the
issuance of stock dividends is part of a tax evasion plan and thus, without
legitimate business reasons the redemption becomes suspicious which may
call for the application of the exempting clause. The substance of the
whole transaction, not its form, usually controls the tax consequences. [if !
supportFootnotes][116][endif]

The two purposes invoked by ANSCOR under the facts of this case
are no excuse for its tax liability. First, the alleged filipinization plan
cannot be considered legitimate as it was not implemented until the BIR
started making assessments on the proceeds of the redemption. Such
corporate plan was not stated in nor supported by any Board Resolution
but a mere afterthought interposed by the counsel of ANSCOR. Being a
separate entity, the corporation can act only through its Board of Directors.
[if !supportFootnotes][117][endif]
The Board Resolutions authorizing the redemptions state
only one purpose reduction of foreign exchange remittances in case cash
dividends are declared. Not even this purpose can be given credence.
Records show that despite the existence of enormous corporate profits no
cash dividend was ever declared by ANSCOR from 1945 until the BIR
started making assessments in the early 1970s. Although a corporation
under certain exceptions, has the prerogative when to issue dividends, yet
when no cash dividends was issued for about three decades, this
circumstance negates the legitimacy of ANSCORs alleged purposes.
Moreover, to issue stock dividends is to increase the shareholdings of
ANSCORs foreign stockholders contrary to its filipinization plan. This
would also increase rather than reduce their need for foreign exchange
remittances in case of cash dividend declaration, considering that
ANSCOR is a family corporation where the majority shares at the time of
redemptions were held by Don Andres foreign heirs.
Secondly, assuming arguendo, that those business purposes are
legitimate, the same cannot be a valid excuse for the imposition of tax.
Otherwise, the taxpayers liability to pay income tax would be made to
depend upon a third person who did not earn the income being taxed.
Furthermore, even if the said purposes support the redemption and justify
the issuance of stock dividends, the same has no bearing whatsoever on the
imposition of the tax herein assessed because the proceeds of the
redemption are deemed taxable dividends since it was shown that income
was generated therefrom.
Thirdly, ANSCOR argued that to treat as taxable dividend the
proceeds of the redeemed stock dividends would be to impose on such
stock an undisclosed lien and would be extremely unfair to intervening
purchasers, i.e. those who buys the stock dividends after their issuance. [if !
supportFootnotes][118][endif]
Such argument, however, bears no relevance in this case as
no intervening buyer is involved. And even if there is an intervening buyer,
it is necessary to look into the factual milieu of the case if income was
realized from the transaction. Again, we reiterate that the dividend
equivalence test depends on such time and manner of the transaction and
its net effect. The undisclosed lien[if !supportFootnotes][119][endif] may be unfair to a
subsequent stock buyer who has no capital interest in the company. But the
unfairness may not be true to an original subscriber like Don Andres, who
holds stock dividends as gains from his investments. The subsequent buyer
who buys stock dividends is investing capital. It just so happen that what
he bought is stock dividends. The effect of its (stock dividends)
redemption from that subsequent buyer is merely to return his capital
subscription, which is income if redeemed from the original subscriber.
After considering the manner and the circumstances by which the
issuance and redemption of stock dividends were made, there is no other
conclusion but that the proceeds thereof are essentially considered
equivalent to a distribution of taxable dividends. As taxable dividend under
Section 83(b), it is part of the entire income subject to tax under Section 22
in relation to Section 21[if !supportFootnotes][120][endif] of the 1939 Code. Moreover,
under Section 29(a) of said Code, dividends are included in gross income.
As income, it is subject to income tax which is required to be withheld at
source. The 1997 Tax Code may have altered the situation but it does not
change this disposition.
EXCHANGE OF COMMON WITH PREFERRED SHARES[if !
supportFootnotes][121][endif]
Exchange is an act of taking or giving one thing for another [if !supportFootnotes]
[122][endif]
involving reciprocal transfer[if !supportFootnotes][123][endif] and is generally
considered as a taxable transaction. The exchange of common stocks with
preferred stocks, or preferred for common or a combination of either for
both, may not produce a recognized gain or loss, so long as the provisions
of Section 83(b) is not applicable. This is true in a trade between two (2)
persons as well as a trade between a stockholder and a corporation. In
general, this trade must be parts of merger, transfer to controlled
corporation, corporate acquisitions or corporate reorganizations. No
taxable gain or loss may be recognized on exchange of property, stock or
securities related to reorganizations.[if !supportFootnotes][124][endif]
Both the Tax Court and the Court of Appeals found that ANSCOR
reclassified its shares into common and preferred, and that parts of the
common shares of the Don Andres estate and all of Doa Carmens shares
were exchanged for the whole 150, 000 preferred shares. Thereafter, both
the Don Andres estate and Doa Carmen remained as corporate subscribers
except that their subscriptions now include preferred shares. There was no
change in their proportional interest after the exchange. There was no cash
flow. Both stocks had the same par value. Under the facts herein, any
difference in their market value would be immaterial at the time of
exchange because no income is yet realized it was a mere corporate paper
transaction. It would have been different, if the exchange transaction
resulted into a flow of wealth, in which case income tax may be imposed.
[if !supportFootnotes][125][endif]

Reclassification of shares does not always bring any substantial


alteration in the subscribers proportional interest. But the exchange is
different there would be a shifting of the balance of stock features, like
priority in dividend declarations or absence of voting rights. Yet neither the
reclassification nor exchange per se, yields realize income for tax
purposes. A common stock represents the residual ownership interest in the
corporation. It is a basic class of stock ordinarily and usually issued
without extraordinary rights or privileges and entitles the shareholder to a
pro rata division of profits.[if !supportFootnotes][126][endif] Preferred stocks are those
which entitle the shareholder to some priority on dividends and asset
distribution.[if !supportFootnotes][127][endif]
Both shares are part of the corporations capital stock. Both
stockholders are no different from ordinary investors who take on the same
investment risks. Preferred and common shareholders participate in the
same venture, willing to share in the profits and losses of the enterprise. [if !
supportFootnotes][128][endif]
Moreover, under the doctrine of equality of shares all
stocks issued by the corporation are presumed equal with the same
privileges and liabilities, provided that the Articles of Incorporation is
silent on such differences.[if !supportFootnotes][129][endif] In this case, the exchange of
shares, without more, produces no realized income to the subscriber. There
is only a modification of the subscribers rights and privileges - which is
not a flow of wealth for tax purposes. The issue of taxable dividend may
arise only once a subscriber disposes of his entire interest and not when
there is still maintenance of proprietary interest.[if !supportFootnotes][130][endif]
WHEREFORE, premises considered, the decision of the Court of
Appeals is MODIFIED in that ANSCORs redemption of 82,752.5 stock
dividends is herein considered as essentially equivalent to a distribution of
taxable dividends for which it is LIABLE for the withholding tax-at-
source. The decision is AFFIRMED in all other respects.
SO ORDERED.

DEDUCTIONS

G.R. Nos. L-28508-9 July 7, 1989


ESSO STANDARD EASTERN, INC., (formerly, Standard-Vacuum Oil
Company), petitioner,
vs.
THE COMMISSIONER OF INTERNAL REVENUE, respondent.
Padilla Law Office for petitioner.

CRUZ, J.:
On appeal before us is the decision of the Court of Tax Appeals 1 denying
petitioner's claims for refund of overpaid income taxes of P102,246.00 for 1959 and
P434,234.93 for 1960 in CTA Cases No. 1251 and 1558 respectively.
I
In CTA Case No. 1251, petitioner ESSO deducted from its gross income for
1959, as part of its ordinary and necessary business expenses, the amount it
had spent for drilling and exploration of its petroleum concessions. This claim
was disallowed by the respondent Commissioner of Internal Revenue on the
ground that the expenses should be capitalized and might be written off as a
loss only when a "dry hole" should result. ESSO then filed an amended return
where it asked for the refund of P323,279.00 by reason of its abandonment
as dry holes of several of its oil wells. Also claimed as ordinary and necessary
expenses in the same return was the amount of P340,822.04, representing
margin fees it had paid to the Central Bank on its profit remittances to its New
York head office.
On August 5, 1964, the CIR granted a tax credit of P221,033.00 only,
disallowing the claimed deduction for the margin fees paid.
In CTA Case No. 1558, the CR assessed ESSO a deficiency income tax for
the year 1960, in the amount of P367,994.00, plus 18% interest thereon of
P66,238.92 for the period from April 18,1961 to April 18, 1964, for a total of
P434,232.92. The deficiency arose from the disallowance of the margin fees
of Pl,226,647.72 paid by ESSO to the Central Bank on its profit remittances to
its New York head office.
ESSO settled this deficiency assessment on August 10, 1964, by applying the
tax credit of P221,033.00 representing its overpayment on its income tax for
1959 and paying under protest the additional amount of P213,201.92. On
August 13, 1964, it claimed the refund of P39,787.94 as overpayment on the
interest on its deficiency income tax. It argued that the 18% interest should
have been imposed not on the total deficiency of P367,944.00 but only on the
amount of P146,961.00, the difference between the total deficiency and its
tax credit of P221,033.00.
This claim was denied by the CIR, who insisted on charging the 18% interest
on the entire amount of the deficiency tax. On May 4,1965, the CIR also
denied the claims of ESSO for refund of the overpayment of its 1959 and
1960 income taxes, holding that the margin fees paid to the Central Bank
could not be considered taxes or allowed as deductible business expenses.
ESSO appealed to the CTA and sought the refund of P102,246.00 for 1959,
contending that the margin fees were deductible from gross income either as
a tax or as an ordinary and necessary business expense. It also claimed an
overpayment of its tax by P434,232.92 in 1960, for the same reason.
Additionally, ESSO argued that even if the amount paid as margin fees were
not legally deductible, there was still an overpayment by P39,787.94 for 1960,
representing excess interest.
After trial, the CTA denied petitioner's claim for refund of P102,246.00 for
1959 and P434,234.92 for 1960 but sustained its claim for P39,787.94 as
excess interest. This portion of the decision was appealed by the CIR but was
affirmed by this Court in Commissioner of Internal Revenue v. ESSO, G.R.
No. L-28502- 03, promulgated on April 18, 1989. ESSO for its part appealed
the CTA decision denying its claims for the refund of the margin fees
P102,246.00 for 1959 and P434,234.92 for 1960. That is the issue now
before us.
II
The first question we must settle is whether R.A. 2009, entitled An Act to
Authorize the Central Bank of the Philippines to Establish a Margin Over
Banks' Selling Rates of Foreign Exchange, is a police measure or a revenue
measure. If it is a revenue measure, the margin fees paid by the petitioner to
the Central Bank on its profit remittances to its New York head office should
be deductible from ESSO's gross income under Sec. 30(c) of the National
Internal Revenue Code. This provides that all taxes paid or accrued during or
within the taxable year and which are related to the taxpayer's trade,
business or profession are deductible from gross income.
The petitioner maintains that margin fees are taxes and cites the background
and legislative history of the Margin Fee Law showing that R.A. 2609 was
nothing less than a revival of the 17% excise tax on foreign exchange
imposed by R.A. 601. This was a revenue measure formally proposed by
President Carlos P. Garcia to Congress as part of, and in order to balance,
the budget for 1959-1960. It was enacted by Congress as such and,
significantly, properly originated in the House of Representatives. During its
two and a half years of existence, the measure was one of the major sources
of revenue used to finance the ordinary operating expenditures of the
government. It was, moreover, payable out of the General Fund.
On the claimed legislative intent, the Court of Tax Appeals, quoting
established principles, pointed out that
We are not unmindful of the rule that opinions expressed in debates, actual
proceedings of the legislature, steps taken in the enactment of a law, or the
history of the passage of the law through the legislature, may be resorted to
as an aid in the interpretation of a statute which is ambiguous or of doubtful
meaning. The courts may take into consideration the facts leading up to,
coincident with, and in any way connected with, the passage of the act, in
order that they may properly interpret the legislative intent. But it is also well-
settled jurisprudence that only in extremely doubtful matters of interpretation
does the legislative history of an act of Congress become important. As a
matter of fact, there may be no resort to the legislative history of the
enactment of a statute, the language of which is plain and unambiguous,
since such legislative history may only be resorted to for the purpose of
solving doubt, not for the purpose of creating it. [50 Am. Jur. 328.]
Apart from the above consideration, there are at least two cases where we
have held that a margin fee is not a tax but an exaction designed to curb the
excessive demands upon our international reserve.
In Caltex (Phil.) Inc. v. Acting Commissioner of Customs, 2 the Court stated
through Justice Jose P. Bengzon:
A margin levy on foreign exchange is a form of exchange control or restriction
designed to discourage imports and encourage exports, and ultimately,
'curtail any excessive demand upon the international reserve' in order to
stabilize the currency. Originally adopted to cope with balance of payment
pressures, exchange restrictions have come to serve various purposes, such
as limiting non-essential imports, protecting domestic industry and when
combined with the use of multiple currency rates providing a source of
revenue to the government, and are in many developing countries regarded
as a more or less inevitable concomitant of their economic development
programs. The different measures of exchange control or restriction cover
different phases of foreign exchange transactions, i.e., in quantitative
restriction, the control is on the amount of foreign exchange allowable. In the
case of the margin levy, the immediate impact is on the rate of foreign
exchange; in fact, its main function is to control the exchange rate without
changing the par value of the peso as fixed in the Bretton Woods Agreement
Act. For a member nation is not supposed to alter its exchange rate (at par
value) to correct a merely temporary disequilibrium in its balance of
payments. By its nature, the margin levy is part of the rate of exchange as
fixed by the government.
As to the contention that the margin levy is a tax on the purchase of foreign
exchange and hence should not form part of the exchange rate, suffice it to
state that We have already held the contrary for the reason that a tax is levied
to provide revenue for government operations, while the proceeds of the
margin fee are applied to strengthen our country's international reserves.
Earlier, in Chamber of Agriculture and Natural Resources of the Philippines v.
Central Bank, 3 the same idea was expressed, though in connection with a
different levy, through Justice J.B.L. Reyes:
Neither do we find merit in the argument that the 20% retention of exporter's
foreign exchange constitutes an export tax. A tax is a levy for the purpose of
providing revenue for government operations, while the proceeds of the 20%
retention, as we have seen, are applied to strengthen the Central Bank's
international reserve.
We conclude then that the margin fee was imposed by the State in the
exercise of its police power and not the power of taxation.
Alternatively, ESSO prays that if margin fees are not taxes, they should
nevertheless be considered necessary and ordinary business expenses and
therefore still deductible from its gross income. The fees were paid for the
remittance by ESSO as part of the profits to the head office in the Unites
States. Such remittance was an expenditure necessary and proper for the
conduct of its corporate affairs.
The applicable provision is Section 30(a) of the National Internal Revenue
Code reading as follows:
SEC. 30. Deductions from gross income in computing net income there shall
be allowed as deductions
(a) Expenses:
(1) In general. All the ordinary and necessary expenses paid or incurred
during the taxable year in carrying on any trade or business, including a
reasonable allowance for salaries or other compensation for personal
services actually rendered; traveling expenses while away from home in the
pursuit of a trade or business; and rentals or other payments required to be
made as a condition to the continued use or possession, for the purpose of
the trade or business, of property to which the taxpayer has not taken or is
not taking title or in which he has no equity.
(2) Expenses allowable to non-resident alien individuals and foreign
corporations. In the case of a non-resident alien individual or a foreign
corporation, the expenses deductible are the necessary expenses paid or
incurred in carrying on any business or trade conducted within the Philippines
exclusively.
In the case of Atlas Consolidated Mining and Development Corporation v.
Commissioner of Internal Revenue, 4 the Court laid down the rules on the
deductibility of business expenses, thus:
The principle is recognized that when a taxpayer claims a deduction, he must
point to some specific provision of the statute in which that deduction is
authorized and must be able to prove that he is entitled to the deduction
which the law allows. As previously adverted to, the law allowing expenses as
deduction from gross income for purposes of the income tax is Section 30(a)
(1) of the National Internal Revenue which allows a deduction of 'all the
ordinary and necessary expenses paid or incurred during the taxable year in
carrying on any trade or business.' An item of expenditure, in order to be
deductible under this section of the statute, must fall squarely within its
language.
We come, then, to the statutory test of deductibility where it is axiomatic that
to be deductible as a business expense, three conditions are imposed,
namely: (1) the expense must be ordinary and necessary, (2) it must be paid
or incurred within the taxable year, and (3) it must be paid or incurred in
carrying on a trade or business. In addition, not only must the taxpayer meet
the business test, he must substantially prove by evidence or records the
deductions claimed under the law, otherwise, the same will be disallowed.
The mere allegation of the taxpayer that an item of expense is ordinary and
necessary does not justify its deduction.
While it is true that there is a number of decisions in the United States delving
on the interpretation of the terms 'ordinary and necessary' as used in the
federal tax laws, no adequate or satisfactory definition of those terms is
possible. Similarly, this Court has never attempted to define with precision the
terms 'ordinary and necessary.' There are however, certain guiding principles
worthy of serious consideration in the proper adjudication of conflicting
claims. Ordinarily, an expense will be considered 'necessary' where the
expenditure is appropriate and helpful in the development of the taxpayer's
business. It is 'ordinary' when it connotes a payment which is normal in
relation to the business of the taxpayer and the surrounding circumstances.
The term 'ordinary' does not require that the payments be habitual or normal
in the sense that the same taxpayer will have to make them often; the
payment may be unique or non-recurring to the particular taxpayer affected.
There is thus no hard and fast rule on the matter. The right to a deduction
depends in each case on the particular facts and the relation of the payment
to the type of business in which the taxpayer is engaged. The intention of the
taxpayer often may be the controlling fact in making the determination.
Assuming that the expenditure is ordinary and necessary in the operation of
the taxpayer's business, the answer to the question as to whether the
expenditure is an allowable deduction as a business expense must be
determined from the nature of the expenditure itself, which in turn depends on
the extent and permanency of the work accomplished by the expenditure.
In the light of the above explanation, we hold that the Court of Tax Appeals
did not err when it held on this issue as follows:
Considering the foregoing test of what constitutes an ordinary and necessary
deductible expense, it may be asked: Were the margin fees paid by petitioner
on its profit remittance to its Head Office in New York appropriate and helpful
in the taxpayer's business in the Philippines? Were the margin fees incurred
for purposes proper to the conduct of the affairs of petitioner's branch in the
Philippines? Or were the margin fees incurred for the purpose of realizing a
profit or of minimizing a loss in the Philippines? Obviously not. As stated in
the Lopez case, the margin fees are not expenses in connection with the
production or earning of petitioner's incomes in the Philippines. They were
expenses incurred in the disposition of said incomes; expenses for the
remittance of funds after they have already been earned by petitioner's
branch in the Philippines for the disposal of its Head Office in New York which
is already another distinct and separate income taxpayer.
xxx
Since the margin fees in question were incurred for the remittance of funds to
petitioner's Head Office in New York, which is a separate and distinct income
taxpayer from the branch in the Philippines, for its disposal abroad, it can
never be said therefore that the margin fees were appropriate and helpful in
the development of petitioner's business in the Philippines exclusively or were
incurred for purposes proper to the conduct of the affairs of petitioner's
branch in the Philippines exclusively or for the purpose of realizing a profit or
of minimizing a loss in the Philippines exclusively. If at all, the margin fees
were incurred for purposes proper to the conduct of the corporate affairs of
Standard Vacuum Oil Company in New York, but certainly not in the
Philippines.
ESSO has not shown that the remittance to the head office of part of its
profits was made in furtherance of its own trade or business. The petitioner
merely presumed that all corporate expenses are necessary and appropriate
in the absence of a showing that they are illegal or ultra vires. This is error.
The public respondent is correct when it asserts that "the paramount rule is
that claims for deductions are a matter of legislative grace and do not turn on
mere equitable considerations ... . The taxpayer in every instance has the
burden of justifying the allowance of any deduction claimed." 5
It is clear that ESSO, having assumed an expense properly attributable to its
head office, cannot now claim this as an ordinary and necessary expense
paid or incurred in carrying on its own trade or business.
WHEREFORE, the decision of the Court of Tax Appeals denying the
petitioner's claims for refund of P102,246.00 for 1959 and P434,234.92 for
1960, is AFFIRMED, with costs against the petitioner.

G.R. No. L-15290 May 31, 1963


MARIANO ZAMORA, petitioner,
vs.
COLLECTOR OF INTERNAL REVENUE and COURT OF TAX APPEALS,
respondents.
-----------------------------
G.R. No. L-15280 May 31, 1963
COLLECTOR OF INTERNAL REVENUE, petitioner,
vs.
MARIANO ZAMORA, respondent.
-----------------------------
G.R. No. L-15289 May 31, 1963
ESPERANZA A. ZAMORA, as Special Administratrix of Estate of
FELICIDAD ZAMORA, petitioner,
vs.
COLLECTOR OF INTERNAL REVENUE and COURT OF TAX APPEALS,
respondents.
-----------------------------
G.R. No. L-15281 May 31, 1963
COLLECTOR OF INTERNAL REVENUE, petitioner,
vs.
ESPERANZA A. ZAMORA, as Special Administratrix, etc. respondent.
Office of the Solicitor General for petitioner.
Rodegelio M. Jalandoni for respondents.
PAREDES, J.:
In the above-entitled cases, a joint decision was rendered by the lower court
because they involved practically the same issues. We do so, likewise, for the
same reason.
Cases Nos. L-15290 and L-15280
Mariano Zamora, owner of the Bay View Hotel and Farmacia Zamora, Manila,
filed his income tax returns the years 1951 and 1952. The Collector of
Internal Revenue found that he failed to file his return of the capital gains
derived from the sale of certain real properties and claimed deductions which
were not allowable. The collector required him to pay the sums of P43,758.50
and P7,625.00, as deficiency income tax for the years 1951 and 1952,
respectively (C.T.A. Case No. 234, now L-15290). On appeal by Zamora, the
Court of Tax Appeals on December 29, 1958, modified the decision appealed
from and ordered him to pay the reduced total sum of P30,258.00
(P22,980.00 and P7,278.00, as deficiency income tax for the years 1951 and
1952, respectively), within thirty (30) days from the date the decision
becomes final, plus the corresponding surcharges and interest in case of
delinquency, pursuant to section 51(e), Int. Revenue Code. With costs
against petitioner.
Having failed to obtain a reconsideration of the decision, Mariano Zamora
appealed (L-15290), alleging that the Court of Tax Appeals erred
(1) In dissallowing P10,478.50, as promotion expenses incurred by his wife
for the promotion of the Bay View Hotel and Farmacia Zamora (which is of
P20,957.00, supposed business expenses):
(2) In disallowing 3-% per annum as the rate of depreciation of the Bay
View Hotel Building;
(3) In disregarding the price stated in the deed of sale, as the costs of a
Manila property, for the purpose of determining alleged capital gains; and
(4) In applying the Ballantyne scale of values in determining the cost of said
property.
The Collector of Internal Revenue (L-15280) also appealed, claiming that the
Court of Tax Appeals erred
(1) In giving credence to the uncorroborated testimony of Mariano Zamora
that he bought the said real property in question during the Japanese
occupation, partly in Philippine currency and partly in Japanese war notes,
and
(2) In not holding that Mariano Zamora is liable for the payment of the sums
of P43,758.00 and P7,625.00 as deficiency income taxes, for the years 1951
and 1952, plus the 5% surcharge and 1% monthly interest, from the date said
amounts became due to the date of actual payment.
Wherefore, the parties respectfully pray that the foregoing stipulation of facts
be admitted and approved by this Honorable Court, without prejudice to the
parties adducing other evidence to prove their case not covered by this
stipulation of facts.
1wph1.t

Cases Nos. L-15289 and L-15281


Mariano Zamora and his deceased sister Felicidad Zamora, bought a piece of
land located in Manila on May 16, 1944, for P132,000.00 and sold it for
P75,000.00 on March 5, 1951. They also purchased a lot located in Quezon
City for P68,959.00 on January 19, 1944, which they sold for P94,000 on
February 9, 1951. The CTA ordered the estate of the late Felicidad Zamora
(represented by Esperanza A. Zamora, as special administratrix of her
estate), to pay the sum of P235.50, representing alleged deficiency income
tax and surcharge due from said estate. Esperanza A. Zamora appealed and
alleged that the CTA erred:
The Commissioner of Internal Revenue likewise appealed from the decision,
claiming that the lower court erred:
(1) In giving credence to the uncorroborated testimony of Mariano Zamora
that he bought the real property involved during the Japanese occupation,
partly in genuine Philippine currency and partly in Japanese war notes; and
(2) In not holding that Esperanza A. Zamora, as administratrix, is liable for the
payment of the sum of P613.00 as deficiency income tax and 50% surcharge
for 1951, plus 50% surcharge and 1% monthly interest from the date said
amount became due, to the date of actual payment.
It is alleged by Mariano Zamora that the CTA erred in disallowing P10,478.50
as promotion expenses incurred by his wife for the promotion of the Bay View
Hotel and Farmacia Zamora. He contends that the whole amount of
P20,957.00 as promotion expenses in his 1951 income tax returns, should be
allowed and not merely one-half of it or P10,478.50, on the ground that, while
not all the itemized expenses are supported by receipts, the absence of some
supporting receipts has been sufficiently and satisfactorily established. For,
as alleged, the said amount of P20,957.00 was spent by Mrs. Esperanza A.
Zamora (wife of Mariano), during her travel to Japan and the United States to
purchase machinery for a new Tiki-Tiki plant, and to observe hotel
management in modern hotels. The CTA, however, found that for said trip
Mrs. Zamora obtained only the sum of P5,000.00 from the Central Bank and
that in her application for dollar allocation, she stated that she was going
abroad on a combined medical and business trip, which facts were not denied
by Mariano Zamora. No evidence had been submitted as to where Mariano
had obtained the amount in excess of P5,000.00 given to his wife which she
spent abroad. No explanation had been made either that the statement
contained in Mrs. Zamora's application for dollar allocation that she was going
abroad on a combined medical and business trip, was not correct. The
alleged expenses were not supported by receipts. Mrs. Zamora could not
even remember how much money she had when she left abroad in 1951, and
how the alleged amount of P20,957.00 was spent.
Section 30, of the Tax Code, provides that in computing net income, there
shall be allowed as deductions all the ordinary and necessary expenses paid
or incurred during the taxable year, in carrying on any trade or business (Vol.
4, Mertens, Law of Federal Income Taxation, sec. 25.03, p. 307). Since
promotion expenses constitute one of the deductions in conducting a
business, same must testify these requirements. Claim for the deduction of
promotion expenses or entertainment expenses must also be substantiated
or supported by record showing in detail the amount and nature of the
expenses incurred (N.H. Van Socklan, Jr. v. Comm. of Int. Rev.; 33 BTA 544).
Considering, as heretofore stated, that the application of Mrs. Zamora for
dollar allocation shows that she went abroad on a combined medical and
business trip, not all of her expenses came under the category of ordinary
and necessary expenses; part thereof constituted her personal expenses.
There having been no means by which to ascertain which expense was
incurred by her in connection with the business of Mariano Zamora and which
was incurred for her personal benefit, the Collector and the CTA in their
decisions, considered 50% of the said amount of P20,957.00 as business
expenses and the other 50%, as her personal expenses. We hold that said
allocation is very fair to Mariano Zamora, there having been no receipt
whatsoever, submitted to explain the alleged business expenses, or proof of
the connection which said expenses had to the business or the
reasonableness of the said amount of P20,957.00. While in situations like the
present, absolute certainty is usually no possible, the CTA should make as
close an approximation as it can, bearing heavily, if it chooses, upon the
taxpayer whose inexactness is of his own making.
In the case of Visayan Cebu Terminal Co., Inc. v. Collector of Int. Rev., G.R.
No. L-12798, May 30, 1960, it was declared that representation expenses fall
under the category of business expenses which are allowable deductions
from gross income, if they meet the conditions prescribed by law, particularly
section 30 (a) [1], of the Tax Code; that to be deductible, said business
expenses must be ordinary and necessary expenses paid or incurred in
carrying on any trade or business; that those expenses must also meet the
further test of reasonableness in amount; that when some of the
representation expenses claimed by the taxpayer were evidenced by
vouchers or chits, but others were without vouchers or chits, documents or
supporting papers; that there is no more than oral proof to the effect that
payments have been made for representation expenses allegedly made by
the taxpayer and about the general nature of such alleged expenses; that
accordingly, it is not possible to determine the actual amount covered by
supporting papers and the amount without supporting papers, the court
should determine from all available data, the amount properly deductible as
representation expenses.
In view hereof, We are of the opinion that the CTA, did not commit error in
allowing as promotion expenses of Mrs. Zamora claimed in Mariano Zamora's
1951 income tax returns, merely one-half or P10,478.50.
Petitioner Mariano Zamora alleges that the CTA erred in disallowing 3-%
per annum as the rate of depreciation of the Bay View Hotel Building but only
2-%. In justifying depreciation deduction of 3-%, Mariano Zamora
contends that (1) the Ermita District, where the Bay View Hotel is located, is
now becoming a commercial district; (2) the hotel has no room for
improvement; and (3) the changing modes in architecture, styles of furniture
and decorative designs, "must meet the taste of a fickle public". It is a fact,
however, that the CTA, in estimating the reasonable rate of depreciation
allowance for hotels made of concrete and steel at 2-%, the three factors
just mentioned had been taken into account already. Said the CTA
Normally, an average hotel building is estimated to have a useful life of 50
years, but inasmuch as the useful life of the building for business purposes
depends to a large extent on the suitability of the structure to its use and
location, its architectural quality, the rate of change in population, the shifting
of land values, as well as the extent and maintenance and rehabilitation. It is
allowed a depreciation rate of 2-% corresponding to a normal useful life of
only 40 years (1955 PH Federal Taxes, Par 14 160-K). Consequently, the
stand of the petitioners can not be sustained.
As the lower court based its findings on Bulletin F, petitioner Zamora, argues
that the same should have been first proved as a law, to be subject to judicial
notice. Bulletin F, is a publication of the US Federal Internal Revenue Service,
which was made after a study of the lives of the properties. In the words of
the lower court: "It contains the list of depreciable assets, the estimated
average useful lives thereof and the rates of depreciation allowable for each
kind of property. (See 1955 PH Federal Taxes, Par. 14, 160 to Par. 14, 163-0).
It is true that Bulletin F has no binding force, but it has a strong persuasive
effect considering that the same has been the result of scientific studies and
observation for a long period in the United States after whose Income Tax
Law ours is patterned." Verily, courts are permitted to look into and investigate
the antecedents or the legislative history of the statutes involved (Director of
Lands v. Abaya, et al., 63 Phil. 559). Zamora also contends that his basis for
applying the 3-% rate is the testimony of its witness Mariano Katipunan,
who cited a book entitled "Hotel Management Principles and Practice" by
Lucius Boomer, President, Hotel Waldorf Astoria Corporation. As well
commented by the Solicitor General, "while the petitioner would deny us the
right to use Bulletin F, he would insist on using as authority, a book in Hotel
management written by a man who knew more about hotels than about
taxation. All that the witness did (Katipunan) . . . is to read excerpts from the
said book (t.s.n. pp. 99-101), which admittedly were based on the decision of
the U.S. Tax Courts, made in 1928 (t.s.n. p. 106)". In view hereof, We hold
that the 2-% rate of depreciation of the Bay View Hotel building, is
approximately correct.
The next items in dispute are the undeclared capital gains derived from the
sales in 1951 of certain real properties in Malate, Manila and in Quezon City,
acquired during the Japanese occupation.
The Manila property (Esperanza Zamora v. Coll. of Int. Rev., Case No. L-
15289). The CTA held in this case, that the cost basis of property acquired in
Japanese war notes is the equivalent of the war notes in genuine Philippine
currency in accordance with the Ballantyne Scale of values, and that the
determination of the gain derived or loss sustained in the sale of such
property is not affected by the decline at the time of sale, in the purchasing
power of the Philippine currency. It was found by the CTA that the purchase
price of P132,000.00 was not entirely paid in Japanese War notes but
thereof or P66,000.00 was in Philippine currency, and that during certain
periods of the enemy occupation, the value of the Japanese war notes was
very much less than the value of the genuine Philippine currency. On this
point, the CTA declared
Finally, it is alleged that the purchase price of P132,000.00 was not entirely
paid in Japanese war notes, Mariano Zamora, co-owner of the property in
question, testified that P66,000.00 was paid in Philippine currency and the
other P66,000.00 was paid in Japanese war notes. No evidence was
presented by respondent to rebut the testimony of Mariano Zamora; it is
assailed merely as being improbable. We have examined this question
thoroughly and we are inclined to give credence to the allegation that a
portion of the purchase price of the property was paid in Philippine money. In
the first place, it appears that the Zamoras owned the Farmacia Zamora
which continued to engage in business during the war years and that a
considerable portion of its sales was paid for in genuine Philippine currency.
This circumstance enabled the Zamoras to accumulate Philippine money
which they used in acquiring the property in question and another property in
Quezon City. In the second place, P132,000.00 in Japanese war notes in
May, 1944 is equivalent to only P11,000.00. The property in question had at
the time an assessed value of P27,031.00 (in Philippine currency).
Considering the well known fact that the assessed value of real property is
very much below the fair market value, it is incredible that said property
should have been sold by the owner thereof for less than one-half of its
assessed value. These facts have convinced us of the veracity of the
allegation that of the purchase price of P132,000.00 the sum of P66,000.00
was paid in Philippine currency, so that only the sum of P66,000.00 was paid
in Japanese War notes.
This being the case, the Ballantyne Scale of values, which was the result of
an impartial scientific study, adopted and given judicial recognition, should be
applied. As the value of the Japanese war notes in May, 1944 when the
Manila property was bought, was 1 of the genuine Philippine Peso
(Ballantyne Scale), and since the gain derived or loss sustained in the
disposition of this property is to reckoned in terms of Philippine Peso, the
value of the Japanese war notes used in the purchase of the property, must
be reduced in terms of the genuine Philippine Peso to determine the cost of
acquisition. It, therefore, results that since the sum of P66,000.00 in
Japanese war notes in May, 1944 is equivalent to P5,500.00 in Philippine
currency (P66,000.00 divided by 12), the acquisition cost of the property in
question is P66,000.00 plus P5,500.00 or P71,500.00 and that as the
property was sold for P75,000.00 in 1951, the owners thereof Mariano and
Felicidad Zamora derived a capital gain of P3,500.00 or P1,750.00 each.
The Quezon City Property (Mariano Zamora v. Coll. of Customs, Case No.
15290). The Zamoras alleged that the entire purchase price of P68,959.00
was paid in Philippine currency. The collector, on the other hand, contends
that the purchase price of P68,959.00 was paid in Japanese war notes. The
CTA, however, giving credence to Zamora's version, said
. . . If , as contended by respondent, the purchase price of P68,959.00 was
paid in Japanese war notes, the purchase price in Philippine currency would
be only P17,239.75 (P68,959.00 divided by 4, 34.00 in war notes being
equivalent to P1.00 in Philippine currency). The assessed value of said
property in Philippine currency at the time of acquisition was P46,910.00. It is
quite incredible that real property with an assessed value of P46,910.00
should have been sold by the owner thereof in Japanese war notes with an
equivalent value in Philippine currency of only P17,239.75. We are more
inclined to believe the allegation that it was purchased for P68,959.00 in
genuine Philippine currency. Since the property was sold for P94,000.00 on
February 9, 1951, the gain derived from the sale is P15,361.75, after
deducting from the selling price the cost of acquisition in the sum of
P68,959.00 and the expense of sale in the sum of P9,679.25.
The above appraisal is correct, and We have no plausible reason to disturb
the same.
Consequently, the total undeclared income of petitioners derived from the
sales of the Manila and Quezon City properties in 1951 is P17,111.75
(P1,750.00 plus P15,361.75), 50% of which in the sum of P8,555.88 is
taxable, the said properties being capital assets held for more than one year.
IN VIEW HEREOF, the petition in each of the above-entitled cases is
dismissed, and the decision appealed from is affirmed, without special
pronouncement as to costs.

G.R. No. L-24059 November 28, 1969


C. M. HOSKINS & CO., INC., petitioner,
vs.
COMMISSIONER OF INTERNAL REVENUE, respondent.
Ross, Salcedo, Del Rosario, Bito and Misa for petitioner.
Office of the Solicitor General Arturo A. Alafriz, Assistant Solicitor General
Felicisimo R. Rosete and Special Attorney Michaelina R. Balasbas for
respondent.
TEEHANKEE, J.:
We uphold in this taxpayer's appeal the Tax Court's ruling that payment by the
taxpayer to its controlling stockholder of 50% of its supervision fees or the
amount of P99,977.91 is not a deductible ordinary and necessary expense
and should be treated as a distribution of earnings and profits of the taxpayer.
Petitioner, a domestic corporation engaged in the real estate business as
brokers, managing agents and administrators, filed its income tax return for
its fiscal year ending September 30, 1957 showing a net income of
P92,540.25 and a tax liability due thereon of P18,508.00, which it paid in due
course. Upon verification of its return, respondent Commissioner of Internal
Revenue, disallowed four items of deduction in petitioner's tax returns and
assessed against it an income tax deficiency in the amount of P28,054.00
plus interests. The Court of Tax Appeals upon reviewing the assessment at
the taxpayer's petition, upheld respondent's disallowance of the principal item
of petitioner's having paid to Mr. C. M. Hoskins, its founder and controlling
stockholder the amount of P99,977.91 representing 50% of supervision fees
earned by it and set aside respondent's disallowance of three other minor
items. The Tax Court therefore determined petitioner's tax deficiency to be in
the amount of P27,145.00 and on November 8, 1964 rendered judgment
against it, as follows:
WHEREFORE, premises considered, the decision of the respondent is
hereby modified. Petitioner is ordered to pay to the latter or his representative
the sum of P27,145.00, representing deficiency income tax for the year 1957,
plus interest at 1/2% per month from June 20, 1959 to be computed in
accordance with the provisions of Section 51(d) of the National Internal
Revenue Code. If the deficiency tax is not paid within thirty (30) days from the
date this decision becomes final, petitioner is also ordered to pay surcharge
and interest as provided for in Section 51 (e) of the Tax Code, without costs.
Petitioner questions in this appeal the Tax Court's findings that the disallowed
payment to Hoskins was an inordinately large one, which bore a close
relationship to the recipient's dominant stockholdings and therefore amounted
in law to a distribution of its earnings and profits.
We find no merit in petitioner's appeal.
As found by the Tax Court, "petitioner was founded by Mr. C. M. Hoskins in
1937, with a capital stock of 1,000 shares at a par value of P1.00 each share;
that of these 1,000 shares, Mr. C. M. Hoskins owns 996 shares (the other 4
shares being held by the other four officers of the corporation), which
constitute exactly 99.6% of the total authorized capital stock (p. 92, t.s.n.);
that during the first four years of its existence, Mr. C. M. Hoskins was the
President, but during the taxable period in question, that is, from October 1,
1956 to September 30, 1957, he was the chairman of the Board of Directors
and salesman-broker for the company (p. 93, t.s.n.); that as chairman of the
Board of Directors, he received a salary of P3,750.00 a month, plus a salary
bonus of about P40,000.00 a year (p. 94, t.s.n.); that he was also a
stockholder and officer of the Paradise Farms, Inc. and Realty Investments,
Inc., from which petitioner derived a large portion of its income in the form of
supervision fees and commissions earned on sales of lots (pp. 97-99, t.s.n.;
Financial Statements, attached to Exhibit '1', p. 11, BIR rec.); that as
chairman of the Board of Directors of petitioner, his duties were: "To act as a
salesman; as a director, preside over meetings and to get all of the real estate
business I could for the company by negotiating sales, purchases, making
appraisals, raising funds to finance real estate operations where that was
necessary' (p. 96, t.s.n.); that he was familiar with the contract entered into by
the petitioner with the Paradise Farms, Inc. and the Realty Investments, Inc.
by the terms of which petitioner was 'to program the development, arrange
financing, plan the proposed subdivision as outlined in the prospectus of
Paradise Farms, Inc., arrange contract for road constructions, with the
provision of water supply to all of the lots and in general to serve as
managing agents for the Paradise Farms, Inc. and subsequently for the
Realty Investment, Inc." (pp. 96-97. t.s.n.)
Considering that in addition to being Chairman of the board of directors of
petitioner corporation, which bears his name, Hoskins, who owned 99.6% of
its total authorized capital stock while the four other officers-stockholders of
the firm owned a total of four-tenths of 1%, or one-tenth of 1% each, with their
respective nominal shareholdings of one share each was also salesman-
broker for his company, receiving a 50% share of the sales commissions
earned by petitioner, besides his monthly salary of P3,750.00 amounting to
an annual compensation of P45,000.00 and an annual salary bonus of
P40,000.00, plus free use of the company car and receipt of other similar
allowances and benefits, the Tax Court correctly ruled that the payment by
petitioner to Hoskins of the additional sum of P99,977.91 as his equal or 50%
share of the 8% supervision fees received by petitioner as managing agents
of the real estate, subdivision projects of Paradise Farms, Inc. and Realty
Investments, Inc. was inordinately large and could not be accorded the
treatment of ordinary and necessary expenses allowed as deductible items
within the purview of Section 30 (a) (i) of the Tax Code.
If such payment of P99,977.91 were to be allowed as a deductible item, then
Hoskins would receive on these three items alone (salary, bonus and
supervision fee) a total of P184,977.91, which would be double the
petitioner's reported net income for the year of P92,540.25. As correctly
observed by respondent. If independently, a one-time P100,000.00-fee to
plan and lay down the rules for supervision of a subdivision project were to be
paid to an experienced realtor such as Hoskins, its fairness and deductibility
by the taxpayer could be conceded; but here 50% of the supervision fee of
petitioner was being paid by it to Hoskins every year since 1955 up to 1963
and for as long as its contract with the subdivision owner subsisted,
regardless of whether services were actually rendered by Hoskins, since his
services to petitioner included such planning and supervision and were
already handsomely paid for by petitioner.
The fact that such payment was authorized by a standing resolution of
petitioner's board of directors, since "Hoskins had personally conceived and
planned the project" cannot change the picture. There could be no question
that as Chairman of the board and practically an absolutely controlling
stockholder of petitioner, holding 99.6% of its stock, Hoskins wielded
tremendous power and influence in the formulation and making of the
company's policies and decisions. Even just as board chairman, going by
petitioner's own enumeration of the powers of the office, Hoskins, could
exercise great power and influence within the corporation, such as directing
the policy of the corporation, delegating powers to the president and advising
the corporation in determining executive salaries, bonus plans and pensions,
dividend policies, etc.1
Petitioner's invoking of its policy since its incorporation of sharing equally
sales commissions with its salesmen, in accordance with its board resolution
of June 18, 1946, is equally untenable. Petitioner's Sales Regulations
provide:
Compensation of Salesmen
8. Schedule I In the case of sales to prospects discovered and worked by
a salesman, even though the closing is done by or with the help of the Sales
Manager or other members of the staff, the salesmen get one-half (1/2) of the
total commission received by the Company, but not exceeding five percent
(5%). In the case of subdivisions, when the office commission covers general
supervision, the 1/2-rule does not apply, the salesman's share being
stipulated in the case of each subdivision. In most cases the salesman's
share is 4%. (Exh. "N-1").2
It will be readily seen therefrom that when the petitioner's commission covers
general supervision, it is provided that the 1/2 rule of equal sharing of the
sales commissions does not apply and that the salesman's share is stipulated
in the case of each subdivision. Furthermore, what is involved here is not
Hoskins' salesman's share in the petitioner's 12% sales commission, which
he presumably collected also from petitioner without respondent's questioning
it, but a 50% share besides in petitioner's planning and supervision fee of 8%
of the gross sales, as mentioned above. This is evident from petitioner's
board's resolution of July 14, 1953 (Exhibit 7), wherein it is recited that in
addition to petitioner's sales commission of 12% of gross sales, the
subdivision owners were paying to petitioner 8% of gross sales as
supervision fee, and a collection fee of 5% of gross collections, or total fees of
25% of gross sales.
The case before us is similar to previous cases of disallowances as
deductible items of officers' extra fees, bonuses and commissions, upheld by
this Court as not being within the purview of ordinary and necessary
expenses and not passing the test of reasonable compensation. 3 In Kuenzle
& Streiff, Inc. vs. Commissioner of Internal Revenue decided by this Court on
May 29, 1969,4 we reaffirmed the test of reasonableness, enunciated in the
earlier 1967 case involving the same parties, that: "It is a general rule that
'Bonuses to employees made in good faith and as additional compensation
for the services actually rendered by the employees are deductible, provided
such payments, when added to the stipulated salaries, do not exceed a
reasonable compensation for the services rendered' (4 Mertens Law of
Federal Income Taxation, Sec. 25.50, p. 410). The conditions precedent to
the deduction of bonuses to employees are: (1) the payment of the bonuses
is in fact compensation; (2) it must be for personal services actually rendered;
and (3) the bonuses, when added to the salaries, are 'reasonable . . . when
measured by the amount and quality of the services performed with relation
to the business of the particular taxpayer' (Idem., Sec. 25, 44, p. 395).
"There is no fixed test for determining the reasonableness of a given bonus
as compensation. This depends upon many factors, one of them being 'the
amount and quality of the services performed with relation to the business.'
Other tests suggested are: payment must be 'made in good faith'; 'the
character of the taxpayer's business, the volume and amount of its net
earnings, its locality, the type and extent of the services rendered, the salary
policy of the corporation'; 'the size of the particular business'; 'the employees'
qualifications and contributions to the business venture'; and 'general
economic conditions' (4 Mertens, Law of Federal Income Taxation, Secs.
25.44, 25.49, 25.50, 25.51, pp. 407-412). However, 'in determining whether
the particular salary or compensation payment is reasonable, the situation
must be considered as whole. Ordinarily, no single factor is decisive. . . . it is
important to keep in mind that it seldom happens that the application of one
test can give satisfactory answer, and that ordinarily it is the interplay of
several factors, properly weighted for the particular case, which must furnish
the final answer."
Petitioner's case fails to pass the test. On the right of the employer as against
respondent Commissioner to fix the compensation of its officers and
employees, we there held further that while the employer's right may be
conceded, the question of the allowance or disallowance thereof as
deductible expenses for income tax purposes is subject to determination by
respondent Commissioner of Internal Revenue. Thus: "As far as petitioner's
contention that as employer it has the right to fix the compensation of its
officers and employees and that it was in the exercise of such right that it
deemed proper to pay the bonuses in question, all that We need say is this:
that right may be conceded, but for income tax purposes the employer cannot
legally claim such bonuses as deductible expenses unless they are shown to
be reasonable. To hold otherwise would open the gate of rampant tax
evasion.
"Lastly, We must not lose sight of the fact that the question of allowing or
disallowing as deductible expenses the amounts paid to corporate officers by
way of bonus is determined by respondent exclusively for income tax
purposes. Concededly, he has no authority to fix the amounts to be paid to
corporate officers by way of basic salary, bonus or additional remuneration
a matter that lies more or less exclusively within the sound discretion of the
corporation itself. But this right of the corporation is, of course, not absolute. It
cannot exercise it for the purpose of evading payment of taxes legitimately
due to the State."
Finally, it should be noted that we have here a case practically of a sole
proprietorship of C. M. Hoskins, who however chose to incorporate his
business with himself holding virtually absolute control thereof with 99.6% of
its stock with four other nominal shareholders holding one share each. Having
chosen to use the corporate form with its legal advantages of a separate
corporate personality as distinguished from his individual personality, the
corporation so created, i.e., petitioner, is bound to comport itself in
accordance with corporate norms and comply with its corporate obligations.
Specifically, it is bound to pay the income tax imposed by law on corporations
and may not legally be permitted, by way of corporate resolutions authorizing
payment of inordinately large commissions and fees to its controlling
stockholder, to dilute and diminish its corresponding corporate tax liability.
ACCORDINGLY, the decision appealed from is hereby affirmed, with costs in
both instances against petitioner.

G.R. No. L-54108 January 17, 1984


COMMISSIONER OF INTERNAL REVENUE, petitioner,
vs.
COURT OF TAX APPEALS and SMITH KLINE & FRENCH OVERSEAS
CO. (PHILIPPINE BRANCH), respondents.
The Solicitor General for petitioner.
Siguion Reyna, Montecillo & Ongsiako and J.C. Castaeda, Jr. and E.C.
Alcantara for respondents.

AQUINO, J.:
This case is about the refund of a 1971 income tax amounting to P324,255.
Smith Kline and French Overseas Company, a multinational firm domiciled in
Philadelphia, Pennsylvania, is licensed to do business in the Philippines. It is
engaged in the importation, manufacture and sale of pharmaceuticals drugs
and chemicals.
In its 1971 original income tax return, Smith Kline declared a net taxable
income of P1,489,277 (Exh. A) and paid P511,247 as tax due. Among the
deductions claimed from gross income was P501,040 ($77,060) as its share
of the head office overhead expenses. However, in its amended return filed
on March 1, 1973, there was an overpayment of P324,255 "arising from
underdeduction of home office overhead" (Exh. E). It made a formal claim for
the refund of the alleged overpayment.
It appears that sometime in October, 1972, Smith Kline received from its
international independent auditors, Peat, Marwick, Mitchell and Company, an
authenticated certification to the effect that the Philippine share in the
unallocated overhead expenses of the main office for the year ended
December 31, 1971 was actually $219,547 (P1,427,484). It further stated in
the certification that the allocation was made on the basis of the percentage
of gross income in the Philippines to gross income of the corporation as a
whole. By reason of the new adjustment, Smith Kline's tax liability was greatly
reduced from P511,247 to P186,992 resulting in an overpayment of
P324,255.
On April 2, 1974, without awaiting the action of the Commissioner of Internal
Revenue on its claim Smith Kline filed a petition for review with the Court of
Tax Appeals.
In its decision of March 21, 1980, the Tax Court ordered the Commissioner to
refund the overpayment or grant a tax credit to Smith Kline. The
Commissioner appealed to this Court.
The governing law is found in section 37 of the old National Internal Revenue
Code, Commonwealth Act No. 466, which is reproduced in Presidential
Decree No. 1158, the National Internal Revenue Code of 1977 and which
reads:
SEC. 37. Income form sources within the Philippines.
xxx xxx xxx
(b) Net income from sources in the Philippines. From the items of gross
income specified in subsection (a) of this section there shall be deducted the
expenses, losses, and other deductions properly apportioned or allocated
thereto and a ratable part of any expenses, losses, or other deductions which
cannot definitely be allocated to some item or class of gross income. The
remainder, if any, shall be included in full as net income from sources within
the Philippines.
xxx xxx xxx
Revenue Regulations No. 2 of the Department of Finance contains the
following provisions on the deductions to be made to determine the net
income from Philippine sources:
SEC. 160. Apportionment of deductions. From the items specified in
section 37(a), as being derived specifically from sources within the
Philippines there shall be deducted the expenses, losses, and other
deductions properly apportioned or allocated thereto and a ratable part of any
other expenses, losses or deductions which can not definitely be allocated to
some item or class of gross income. The remainder shall be included in full as
net income from sources within the Philippines. The ratable part is based
upon the ratio of gross income from sources within the Philippines to the total
gross income.
Example: A non-resident alien individual whose taxable year is the calendar
year, derived gross income from all sources for 1939 of P180,000, including
therein:
Interest on bonds of a domestic corporation P9,000
Dividends on stock of a domestic corporation 4,000
Royalty for the use of patents within the Philippines 12,000
Gain from sale of real property located within the Philippines 11,000
Total P36,000
that is, one-fifth of the total gross income was from sources within the
Philippines. The remainder of the gross income was from sources without the
Philippines, determined under section 37(c).
The expenses of the taxpayer for the year amounted to P78,000. Of these
expenses the amount of P8,000 is properly allocated to income from sources
within the Philippines and the amount of P40,000 is properly allocated to
income from sources without the Philippines.
The remainder of the expense, P30,000, cannot be definitely allocated to any
class of income. A ratable part thereof, based upon the relation of gross
income from sources within the Philippines to the total gross income, shall be
deducted in computing net income from sources within the Philippines. Thus,
these are deducted from the P36,000 of gross income from sources within the
Philippines expenses amounting to P14,000 [representing P8,000 properly
apportioned to the income from sources within the Philippines and P6,000, a
ratable part (one-fifth) of the expenses which could not be allocated to any
item or class of gross income.] The remainder, P22,000, is the net income
from sources within the Philippines.
From the foregoing provisions, it is manifest that where an expense is clearly
related to the production of Philippine-derived income or to Philippine
operations (e.g. salaries of Philippine personnel, rental of office building in the
Philippines), that expense can be deducted from the gross income acquired
in the Philippines without resorting to apportionment.
The overhead expenses incurred by the parent company in connection with
finance, administration, and research and development, all of which direct
benefit its branches all over the world, including the Philippines, fall under a
different category however. These are items which cannot be definitely
allocated or Identified with the operations of the Philippine branch. For 1971,
the parent company of Smith Kline spent $1,077,739. Under section 37(b) of
the Revenue Code and section 160 of the regulations, Smith Kline can claim
as its deductible share a ratable part of such expenses based upon the ratio
of the local branch's gross income to the total gross income, worldwide, of the
multinational corporation.
In his petition for review, the Commissioner does not dispute the right of
Smith Kline to avail itself of section 37(b) of the Tax Code and section 160 of
the regulations. But the Commissioner maintains that such right is not
absolute and that as there exists a contract (in this case a service agreement)
which Smith Kline has entered into with its home office, prescribing the
amount that a branch can deduct as its share of the main office's overhead
expenses, that contract is binding.
The Commissioner contends that since the share of the Philippine branch has
been fixed at $77,060, Smith Kline itself cannot claim more than the said
amount. To allow Smith Kline to deduct more than what was expressly
provided in the agreement would be to ignore its existence. It is a cardinal
rule that a contract is the law between the contracting parties and the
stipulations therein must be respected unless these are proved to be contrary
to law, morals, good customs and public policy. There being allegedly no
showing to the contrary, the provisions thereof must be followed.
The Commissioner also argues that the Tax Court erred in relying on the
certification of Peat, Marwick, Mitchell and Company that Smith Kline is
entitled to deduct P1,427,484 ($219,547) as its allotted share and that Smith
Kline has not presented any evidence to show that the home office expenses
chargeable to Philippine operations exceeded $77,060.
On the other hand, Smith Kline submits that the contract between itself and
its home office cannot amend tax laws and regulations. The matter of
allocated expenses which are deductible under the law cannot be the subject
of an agreement between private parties nor can the Commissioner
acquiesce in such an agreement.
Smith Kline had to amend its return because it is of common knowledge that
audited financial statements are generally completed three or four months
after the close of the accounting period. There being no financial statements
yet when the certification of January 11, 1972 was made the treasurer could
not have correctly computed Smith Kline's share in the home office overhead
expenses in accordance with the gross income formula prescribed in section
160 of the Revenue Regulations. What the treasurer certified was a mere
estimate.
Smith Kline likewise submits that it has presented ample evidence to support
its claim for refund. To this end, it has presented before the Tax Court the
authenticated statement of Peat, Marwick, Mitchell and Company to show
that since the gross income of the Philippine branch was P7,143,155
($1,098,617) for 1971 as per audit report prepared by Sycip, Gorres, Velayo
and Company, and the gross income of the corporation as a whole was
$6,891,052, Smith Kline's share at 15.94% of the home office overhead
expenses was P1,427,484 ($219,547) (Exh. G to G-2, BIR Records, 4-5).
Clearly, the weight of evidence bolsters its position that the amount of
P1,427,484 represents the correct ratable share, the same having been
computed pursuant to section 37(b) and section 160.
In a manifestation dated July 19, 1983, Smith Kline declared that with respect
to its share of the head office overhead expenses in its income tax returns for
the years 1973 to 1981, it deducted its ratable share of the total overhead
expenses of its head office for those years as computed by the independent
auditors hired by the parent company in Philadelphia, Pennsylvania U.S.A.,
as soon as said computations were made available to it.
We hold that Smith Kline's amended 1971 return is in conformity with the law
and regulations. The Tax Court correctly held that the refund or credit of the
resulting overpayment is in order.
WHEREFORE, the decision of the Tax Court is hereby affirmed. No costs.

G.R. No. L-13325 April 20, 1961


SANTIAGO GANCAYCO, petitioner,
vs.
THE COLLECTOR OF INTERNAL REVENUE, respondent.
Benjamin J. Molina for petitioner.
Office of the Solicitor General and Special Attorney Antonio A. Garces for
respondent.
CONCEPCION, J.:
Petitioner Santiago Gancayco seeks the review of a decision of the Court of
Tax Appeals, requiring him to pay P16,860.31, plus surcharge and interest, by
way of deficiency income tax for the year 1949.
On May 10, 1950, Gancayco filed his income tax return for the year 1949.
Two (2) days later, respondent Collector of Internal Revenue issued the
corresponding notice advising him that his income tax liability for that year
amounted P9,793.62, which he paid on May 15, 1950. A year later, on May
14, 1951, respondent wrote the communication Exhibit C, notifying
Gancayco, inter alia, that, upon investigation, there was still due from him, a
efficiency income tax for the year 1949, the sum of P29,554.05. Gancayco
sought a reconsideration, which was part granted by respondent, who in a
letter dated April 8, 1953 (Exhibit D), informed petitioner that his income tax
defendant efficiency for 1949 amounted to P16,860.31. Gancayco urged
another reconsideration (Exhibit O), but no action taken on this request,
although he had sent several communications calling respondent's attention
thereto.
On April 15, 1956, respondent issued a warrant of distraint and levy against
the properties of Gancayco for the satisfaction of his deficiency income tax
liability, and accordingly, the municipal treasurer of Catanauan, Quezon
issued on May 29, 1956, a notice of sale of said property at public auction on
June 19, 1956. Upon petition of Gancayco filed on June 16, 1956, the Court
of Tax Appeal issued a resolution ordering the cancellation of the sale and
directing that the same be readvertised at a future date, in accordance with
the procedure established by the National Internal Revenue Code.
Subsequently, or on June 22, 1956, Gancayco filed an amended petition
praying that said Court:
(a) Issue a writ of preliminary injunction, enjoining the respondents from
enforcing the collection of the alleged tax liability due from the petitioner
through summary proceeding pending determination of the present case;
(b) After a review of the present case adjudge that the right of the government
to enforce collection of any liability due on this account had already
prescribed;
(c) That even assuming that prescription had not set in the objections of
petitioner to the disallowance of the entertainment, representation and
farming expenses be allowed;
xxx xxx xxx
In his answer respondent admitted some allegations the amended petition,
denied other allegations thereof an set up some special defenses. Thereafter
Gancayco received from the municipal treasurer of Catanauan, Quezon,
another notice of auction sale of his properties, to take place on August 29,
1956. On motion of Gancayco, the Court of Tax Appeals, by resolution dated
August 27, 1956, "cancelled" the aforementioned sale and enjoined
respondent and the municipal treasurer of Catanauan, Quezon, from
proceeding with the same. After appropriate proceedings, the Court of Tax
Appeals rendered, on November 14, 1957, the decision adverted to above.
Gancayco maintains that the right to collect the deficiency income tax in
question is barred by the statute of limitations. In this connection, it should be
noted, however, that there are two (2) civil remedies for the collection of
internal revenue taxes, namely: (a) by distraint of personal property and levy
upon real property; and (b) by "judicial action" (Commonwealth Act 456,
section 316). The first may not be availed of except within three (3) years
after the "return is due or has been made ..." (Tax Code, section 51 [d] ). After
the expiration of said Period, income taxes may not be legally and validly
collected by distraint and/or levy (Collector of Internal Revenue v. Avelino, L-
9202, November 19, 1956; Collector of Internal Revenue v. Reyes, L-8685,
January 31, 1957; Collector of Internal Revenue v. Zulueta, L-8840, February
8, 1957; Sambrano v. Court of Tax Appeals, L-8652, March 30, 1957).
Gancayco's income tax return for 1949 was filed on May 10, 1950; so that the
warrant of distraint and levy issued on May 15, 1956, long after the expiration
of said three-year period, was illegal and void, and so was the attempt to sell
his properties in pursuance of said warrant.
The "judicial action" mentioned in the Tax Code may be resorted to within five
(5) years from the date the return has been filed, if there has been no
assessment, or within five (5) years from the date of the assessment made
within the statutory period, or within the period agreed upon, in writing, by the
Collector of Internal Revenue and the taxpayer. before the expiration of said
five-year period, or within such extension of said stipulated period as may
have been agreed upon, in writing, made before the expiration of the period
previously situated, except that in the case of a false or fraudulent return with
intent to evade tax or of a failure to file a return, the judicial action may be
begun at any time within ten (10) years after the discovery of the falsity, fraud
or omission (Sections 331 and 332 of the Tax Code). In the case at bar,
respondent made three (3) assessments: (a) the original assessment of
P9,793.62, made on May 12, 1950; (b) the first deficiency income tax
assessment of May 14, 1951, for P29,554.05; and (c) the amended deficiency
income tax assessment of April 8, 1953, for P16,860.31.
Gancayco argues that the five-year period for the judicial action should be
counted from May 12, 1950, the date of the original assessment, because the
income tax for 1949, he says, could have been collected from him since then.
Said assessment was, however, not for the deficiency income tax involved in
this proceedings, but for P9,793.62, which he paid forthwith. Hence, there
never had been any cause for a judicial action against him, and, per force, no
statute of limitations to speak of, in connection with said sum of P9,793.62.
Neither could said statute have begun to run from May 14, 1951, the date of
the first deficiency income tax assessment or P29,554.05, because the same
was, upon Gancayco's request, reconsidered or modified by the assessment
made on April 8, 1953, for P16,860.31. Indeed, this last assessment is what
Gancayco contested in the amended petition filed by him with the Court of
Tax Appeals. The amount involved in such assessment which Gancayco
refused to pay and respondent tried to collect by warrant of distraint and/or
levy, is the one in issue between the parties. Hence, the five-year period
aforementioned should be counted from April 8, 1953, so that the statute of
limitations does not bar the present proceedings, instituted on April 12, 1956,
if the same is a judicial action, as contemplated in section 316 of the Tax
Code, which petitioner denies, upon the ground that
a. "The Court of Tax Appeals does not have original jurisdiction to entertain an
action for the collection of the tax due;
b. "The proper party to commence the judicial action to collect the tax due is
the government, and
c. "The remedies provided by law for the collection of the tax are exclusive."
Said Section 316 provides:
The civil remedies for the collection of internal revenue taxes, fees, or
charges, and any increment thereto resulting from delinquency shall be (a) by
distraint of goods, chattels, or effects, and other personal property of
whatever character, including stocks and other securities, debts, credits, bank
accounts, and interest in and rights to personal property, and by levy upon
real property; and (b) by judicial action. Either of these remedies or both
simultaneously may be pursued in the discretion of the authorities charged
with the collection of such taxes.
No exemption shall be allowed against the internal revenue taxes in any
case.
Petitioner contends that the judicial action referred to in this provision is
commenced by filing, with a court of first instance, of a complaint for the
collection of taxes. This was true at the time of the approval of
Commonwealth Act No. 456, on June 15, 1939. However, Republic Act No.
1125 has vested the Court of Tax Appeals, not only with exclusive appellate
jurisdiction to review decisions of the Collector (now Commissioner) of
Internal Revenue in cases involving disputed assessments, like the one at
bar, but, also, with authority to decide "all cases involving disputed
assessments of Internal Revenue taxes or customs duties pending
determination before the court of first instance" at the time of the approval of
said Act, on June 16, 1954 (Section 22, Republic Act No. 1125). Moreover,
this jurisdiction to decide all cases involving disputed assessments of internal
revenue taxes and customs duties necessarily implies the power to authorize
and sanction the collection of the taxes and duties involved in such
assessments as may be upheld by the Court of Tax Appeals. At any rate, the
same now has the authority formerly vested in courts of first instance to hear
and decide cases involving disputed assessments of internal revenue taxes
and customs duties. Inasmuch as those cases filed with courts of first
instance constituted judicial actions, such is, likewise, the nature of the
proceedings before the Court of Tax Appeals, insofar as sections 316 and 332
of the Tax Code are concerned.
The question whether the sum of P16,860.31 is due from Gancayco as
deficiency income tax for 1949 hinges on the validity of his claim for
deduction of two (2) items, namely: (a) for farming expenses, P27,459.00;
and (b) for representation expenses, P8,933.45.
Section 30 of the Tax Code partly reads:
(a) Expenses:
(1) In General All the ordinary and necessary expenses paid or incurred
during the taxable year in carrying on any trade or business, including a
reasonable allowance for salaries or other compensation for personal
services actually rendered; traveling expenses while away from home in the
pursuit of a trade or business; and rentals or other payments required to be
made as a condition to the continued use or possession, for the purposes of
the trade or business, of property to which the taxpayer has not taken or is
not taking title or in which he has no equity. (Emphasis supplied.)
Referring to the item of P27,459, for farming expenses allegedly incurred by
Gancayco, the decision appealed from has the following to say:
No evidence has been presented as to the nature of the said "farming
expenses" other than the bare statement of petitioner that they were spent for
the "development and cultivation of (his) property". No specification has been
made as to the actual amount spent for purchase of tools, equipment or
materials, or the amount spent for improvement. Respondent claims that the
entire amount was spent exclusively for clearing and developing the farm
which were necessary to place it in a productive state. It is not, therefore, an
ordinary expense but a capitol expenditure. Accordingly, it is not deductible
but it may be amortized, in accordance with section 75 of Revenue
Regulations No. 2, cited above. See also, section 31 of the Revenue Code
which provides that in computing net income, no deduction shall in any case
be allowed in respect of any amount paid out for new buildings or for
permanent improvements, or betterments made to increase the value of any
property or estate. (Emphasis supplied.)
We concur in this view, which is a necessary consequence of section 31 of
the Tax Code, pursuant to which:
(a) General Rule In computing net income no deduction shall in any case
be allowed in respect of
(1) Personal, living, or family expenses;
(2) Any amount paid out for new buildings or for permanent improvements, or
betterments made to increase the value of any property or estate;
(3) Any amount expended in restoring property or in making good the
exhaustion thereof for which an allowance is or has been made; or
(4) Premiums paid on any life insurance policy covering the life of any officer
or employee, or any person financially interested in any trade or business
carried on by the taxpayer, individual or corporate, when the taxpayer is
directly or indirectly a beneficiary under such policy. (Emphasis supplied.)
Said view is, likewise, in accord with the consensus of the authorities on the
subject.
Expenses incident to the acquisition of property follow the same rule as
applied to payments made as direct consideration for the property. For
example, commission paid in acquiring property are considered as
representing part of the cost of the property acquired. The same treatment is
to be accorded to amounts expended for maps, abstracts, legal opinions on
titles, recording fees and surveys. Other non-deductible expenses include
amounts paid in connection with geological explorations, development and
subdividing of real estate; clearing and grading; restoration of soil, drilling
wells, architects's fees and similar types of expenditures. (4 Merten's Law of
Federal Income Taxation, Sec. 25.20, pp. 348-349; see also sec. 75 of the
income Regulation of the B.I.R.; Emphasis supplied.)
The cost of farm machinery, equipment and farm building represents a capital
investment and is not an allowable deduction as an item of expense. Amounts
expended in the development of farms, orchards, and ranches prior to the
time when the productive state is reached may be regarded as investments of
capital. (Merten's Law of Federal Income Taxation, supra, sec. 25.108, p.
525.)
Expenses for clearing off and grading lots acquired is a capital expenditure,
representing part of the cost of the land and was not deductible as an
expense. (Liberty Banking Co. v. Heiner 37 F [2d] 703 [8AFTR 100111] [CCA
3rd]; The B.L. Marble Chair Company v. U.S., 15 AFTR 746).
An item of expenditure, in order to be deductible under this section of the
statute providing for the deduction of ordinary and necessary business
expenses, must fall squarely within the language of the statutory provision.
This section is intended primarily, although not always necessarily, to cover
expenditures of a recurring nature where the benefit derived from the
payment is realized and exhausted within the taxable year. Accordingly, if the
result of the expenditure is the acquisition of an asset which has an
economically useful life beyond the taxable year, no deduction of such
payment may be obtained under the provisions of the statute. In such cases,
to the extent that a deduction is allowable, it must be obtained under the
provisions of the statute which permit deductions for amortization,
depreciation, depletion or loss. (W.B. Harbeson Co. 24 BTA, 542; Clark
Thread Co., 28 BTA 1128 aff'd 100 F [2d] 257 [CCA 3rd, 1938]; 4 Merten's
Law of Federal Income Taxation, Sec. 25.17, pp. 337-338.)
Gancayco's claim for representation expenses aggregated P31,753.97, of
which P22,820.52 was allowed, and P8,933.45 disallowed. Such
disallowance is justified by the record, for, apart from the absence of receipts,
invoices or vouchers of the expenditures in question, petitioner could not
specify the items constituting the same, or when or on whom or on what they
were incurred. The case of Cohan v. Commissioner, 39 F (2d) 540, cited by
petitioner is not in point, because in that case there was evidence on the
amounts spent and the persons entertained and the necessity of entertaining
them, although there were no receipts an vouchers of the expenditures
involved therein. Such is not the case of petitioner herein.
Being in accordance with the facts and law, the decision of the Court of Tax
Appeals is hereby affirmed therefore, with costs against petitioner Santiago
Cancayco. It is so ordered.

G.R. No. L-16661 January 31, 1962


CLARA DILUANGCO PALANCA, ET AL., petitioners,
vs.
COMMISSIONER OF INTERNAL REVENUE, ET AL., respondents.
Ramon A. Daz for petitioners.
Office of the Solicitor General for respondents.
BAUTISTA ANGELO, J.:
Gliceria Diluangco died on April 18, 1947 and testate proceedings were filed
with the Court of First Instance of Manila for her estate's settlement and
distribution. Upon discovering that the executor failed to file the return
required by law, the Commissioner of Internal Revenue required him to do so
and on March 27, 1951 he filed the requested estate and inheritance tax
return. The estate was tentatively assessed estate and inheritance taxes in
the total sum of P9,705.61, including 25% surcharge for failure to file the
return on time. Atty. Manuel V. San Jose, executor of the estate and counsel
for the heirs of the deceased, requested reconsideration of the imposition of
the 25%, which the Commissioner denied. Subsequently, Atty. San Jose
again requested reconsideration of the imposition of the same surcharge.
In a report submitted on August 25, 1961, internal revenue examiner
Francisco E. Testa stated that the estate subject to tax amounted to
P150,657.40 and not P67,187.86 as reported in the return and, accordingly,
an assessment notice was issued calling for he payment of P22,533.46 as
deficiency taxes. Atty. San Jose requested a reinvestigation of this
assessment which was referred for comment to examiner Testa who however
reiterated his recommendation because of the executor's failure to prove his
claim relative to the incorrectness of the valuation of the properties.
On March 5, 1952, the Commissioner of Internal Revenue issued a warrant of
distraint and levy for the satisfaction of the deficiency, estate and inheritance
taxes in the total amount of P24,790.21 informing thereof at the same time
the register of deeds pursuant to the provisions of Section 104 of the National
Internal Revenue Code. However, the warrant of distraint and levy was not
executed because the executor of the estate asked for a reinvestigation of the
case and for the placing of the real properties left by the deceased under
constructive levy in order to obviate the necessity of having to file surety bond
to guarantee the payment of the assessed taxes. This request was granted
and the case was again referred to examiner Testa for comment and
recommendation. Considering the market value of the properties as
appraised by C.M. Hoskins & Company, Inc. which was requested to do the
appraisal by the heirs themselves, examiner Testa submitted a report on July
26, 1952, and on the basis thereof a new assessment was made calling for
the payment of P10,437.76 on or before September 30, 1952. In view of this
new assessment the Commissioner ordered the warrant of distraint and levy
dated March 5, 1952 to be executed "except that the amount to be collected
should be P10,437.76 instead of P24,790.21 stated therein." In his
indorsement dated May 20, 1955, agent Manuel F. del Rosario reported that
Atty. San Jose refused to receive the said warrant of distraint and levy and
instead requested the suspension of the execution of said warrant in view of
certain discrepancies he allegedly found in the amount of the deficiency
transfer taxes. In view of this objection, another warrant was issued on June
23,1955, which was served on the secretary of Atty. San Jose, but instead of
paying the tax, Atty. San Jose sent a letter requesting that the heirs be
informed of the amounts that are respectively due from each with the
assurance that upon receipt of the information requested the heirs would
immediately make arrangement for the settlement of their tax liabilities. In
compliance with this request the Commissioner in a letter to Atty. San Jose
dated April 28, 1956 explained the breakdown of the amounts due from the
heirs the total of which amounted to P13,884.78, inclusive of surcharges,
interests and penalties. Atty. San Jose wrote another letter requesting
reconsideration of this assessment but the same was denied by the
Commissioner.
In a letter dated September 23, 1957, the heirs again requested a revaluation
of the properties of the deceased with the assurance that if the request is
granted they would be willing to file the requisite surety bond and a waiver of
limitations in accordance with the existing regulations, but before such
request could be acted upon which they assured was not intended for delay,
the heirs, thru counsel, made a turn-about by raising this time the defense of
prescription alleging that the right of the Government to collect by summary
method the estate and inheritance taxes in question had already prescribed.
The answer of the Commissioner was that the right of the Government to
collect has not as yet prescribed and that steps would be taken to sell the
properties left by the deceased. On March 3, 1958, the heirs filed a petition
for review with the Court of Tax Appeals disputing the right of the Government
to collect the taxes in question on the ground of prescription.
On November 24, 1959, the Court of Tax Appeals rendered decision holding
that the right of the Government to collect the sum of P10,437.76 has not
prescribed and ordered petitioners to pay respondent said amount, plus the
corresponding interest thereon to the date of payment. Petitioners have
appealed.
Section 332 (c) of the National Internal Revenue Code provides in part:
"Where the assessment of any internal revenue tax has been made within the
period of limitation above prescribed, such tax may be collected by distraint or
levy by the proceeding in court, but only if begun (1) within five years after the
assessment of the tax ..." It will be noted from this provision that all that is
required to start the running of the period of limitation therein prescribed is to
distraint or levy, or institute a proceeding in court, within 5 years after the
assessment on the tax. A judicial action for the collection of a tax is begun by
the filing of a complaint with the proper court of first instance, or where the
assessment is appealed to the Court of Tax Appeals, by filing an answer to
the taxpayer's petition for review wherein payment of the tax is prayed for
(Alhambra, Cigar and Cigarette Manufacturing Company v. The Collector of
Internal Revenue, G.R. Nos. L-12026 & L-12131, May 29, 1959). And the
summary remedy of distraint and levy is begun by the issuance of a warrant
of distraint and levy. This has been the practice long observed in the Bureau
of Internal Revenue, and this practice had been taken cognizance of by this
Court in a number of cases, wherein it held that the right of the Commissioner
of Internal Revenue to collect by summary method has the effect of stopping
the running of prescription once a warrant of distraint and levy is issued. (The
Collector of Internal Revenue v. Avelino, et al., L-9202, November 19, 1956;
The Collector of Internal Revenue v. Zulueta, et al., L-8840, February 8, 1957;
Collector of Internal Revenue v. Solano, et al., L-11475, July 31, 1958.) From
such pronouncement it can be inferred that the issuance of the warrant of
distraint and levy begins the summary remedy of distraint and levy and that it
is not necessary that it be actually executed to be made effective.1wph1.t

Here it is admitted that the estate and inheritance taxes in question were
finally assessed on August 18, 1952, and the warrant of distraint and levy was
issued within the 5-year period in Section 331 of the National Internal
Revenue Code from the time the return was filed. It is also admitted that the
warrant of distraint and levy was issued by respondent on June 23, 1955, but
that said warrant has not been fully executed in view of the request of counsel
for petitioners for an itemized statement of the amount due from each heir
and the assurance given by said counsel that upon receipt of respondent's
reply the heirs will immediately make arrangement for the settlement of their
shares. We, therefore, hold that the right of the Government to collect the
estate and inheritance taxes in question has not yet prescribed because the
warrant of distraint and levy for their collection was begun within the 5-year
period prescribed by law from the date of the assessment of said taxes.
Petitioners, however, contend that the issuance of the warrant in question
cannot be considered as having begun the summary remedy of distraint and
levy because (a) said warrant is defective and (b) it was not served upon the
taxpayer in accordance with law. With regard to the first contention,
petitioners allege that the warrant of distraint and levy is defective because
(1) it does not contain any description of the property sought to be levied
upon; (2) it was issued against the estate whose legal existence had long
terminated, and (3) it states the amount due in lump sum and does not
itemize the tax and penalty due from petitioners. And with regard to the
second contention, the defect is made to consist in that the warrant was
served not upon the taxpayer himself but upon one Arturo Cristi, secretary of
Atty. Manuel V. San Jose, contrary to what the law provides that a warrant
should be served upon the taxpayer himself except when he is absent from
the Philippines.
Petitioners apparently confuse the warrant of distraint and levy with the
certificate mentioned in Section 324 of the Tax Code. The warrant of distraint
and levy is not the certificate referred to in said section. Said warrant is the
order to distraint and levy upon the properties of the taxpayer. The certificate
mentioned in Section 324 is one prepared and issued by the agent
designated by the Commissioner of Internal Revenue to execute the order of
distraint and levy. It is issued after the seizure of the property distrained and
levied upon. In other words, the issuance of the warrant is merely the step
that starts the summary proceeding while the seizure of the properties is the
next step. It is for this reason that the warrant did not contain the details
regarding the properties to be distrained or levied upon because they are only
required in certificate referred to in Section 324.
As regards the claim that the warrant was issued against the estate which
has no longer legal existence because the testate proceedings were already
closed, suffice it to state that said warrant is a mere order to an agent of the
internal revenue office to collect the tax either from the estate or from the
heirs if said estate is closed. It is well-settled that an estate or inheritance tax,
whether assessed before or after the death of the deceased, can be collected
from the heirs even after the distribution of the properties of the decedent
(Pineda v. Court of First Instance of Tayabas, 52 Phil., 805).
The contention that the warrant is ineffective because it was not served upon
the taxpayers themselves is also untenable considering that the same was
served upon Atty. Manuel V. Jose, or his secretary, who has always acted
right along not only in behalf of the estate but also of the heirs of the
deceased. While the law provides that said warrant should be served upon
the taxpayer except when he is absent from the Philippines when it may be
served upon his agent or upon an occupant of the property, there is nothing
therein that would prevent the service to be made upon his authorized
representative. Here it is admitted that Atty. San Jose was the duly authorized
representative of the estate and of the heirs.
With regard to the contention that the issuance of the warrant is not sufficient
to begin the proceeding by summary method but that it is necessary that it be
actually executed, the Court of Tax Appeals said the following on the point:
Section 332 of the Revenue Code provides that the collection of an internal
revenue tax may be made by distraint and levy if the proceeding is begun
within five years after assessment. In this case, the distraint and levy and the
service thereof to Attorney Manuel V. San Jose. It is true that the warrant has
not been fully executed with the seizure and sale of any property subject to
the lien, but it was not due to the voluntary desistance of respondent; rather it
was because of the request of the then counsel for petitioners for a statement
of the amount due from each heir and for an opportunity to make
arrangement for the settlement of the obligation, which request was
considered reasonable by respondent. Under the law, it is not essential that
the warrant of distraint and levy be fully executed in order that it may have the
effect of suspending the running of the statute of limitation upon collection of
the tax. It is enough that the proceeding be validly begun or commenced and
that its execution has not been suspended by reason of the voluntary
desistance of the respondent. In our opinion, the warrant of distraint and levy
of June 23, 1955 was validly issued and was duly served upon counsel for
petitioners, and therefore, the five-year period for collection of the estate and
inheritance taxes in question was suspended. And it continued to be
suspended up to the date when the present appeal was filed by petitioners on
May 3, 1958. Accordingly, the right to collect said taxes has not prescribed.
We agree to the foregoing view. Indeed, the record shows that the warrant
was not actually executed or carried out with the seizure and sale of any
property of the deceased, not because of any voluntary desistance on the
part of respondent, but because of the many requests for postponement,
reinvestigation, revaluation, or other matters which had the effect of delaying
or postponing the execution of said warrant. Were it not for said requests for
postponement or revaluation, the warrant would have been fully executed
well within the period prescribed by law. Indeed, if by acceding to the request
for postponement of a taxpayer the period of prescription would be allowed to
run even if there is no voluntary desistance on the part of the tax collector, we
would not only countenance the commission of an injustice but would place
the collection of the tax at the mercy or caprice of the taxpayer to the
prejudice of the Government. Such a theory certainly cannot be entertained.
WHEREFORE, the decision appealed from is affirmed, with costs against
appellants.

G.R. Nos. 106949-50 December 1, 1995


PAPER INDUSTRIES CORPORATION OF THE PHILIPPINES (PICOP),
petitioner,
vs.
COURT OF APPEALS, COMMISSIONER OF INTERNAL REVENUE and
COURT OF TAX APPEALS, respondents.
G.R. Nos. 106984-85 December 1, 1995
COMMISSIONER INTERNAL REVENUE, petitioner,
vs.
PAPER INDUSTRIES CORPORATION OF THE PHILIPPINES, THE COURT
OF APPEALS and THE COURT OF TAX APPEALS, respondents.

FELICIANO, J.:
The Paper Industries Corporation of the Philippines ("Picop"), which is
petitioner in G.R. Nos. 106949-50 and private respondent in G.R. Nos.
106984-85, is a Philippine corporation registered with the Board of
Investments ("BOI") as a preferred pioneer enterprise with respect to its
integrated pulp and paper mill, and as a preferred non-pioneer enterprise with
respect to its integrated plywood and veneer mills.
On 21 April 1983, Picop received from the Commissioner of Internal Revenue
("CIR") two (2) letters of assessment and demand both dated 31 March 1983:
(a) one for deficiency transaction tax and for documentary and science stamp
tax; and (b) the other for deficiency income tax for 1977, for an aggregate
amount of P88,763,255.00. These assessments were computed as follows:
Transaction Tax
Interest payments on
money market
borrowings P 45,771,849.00

35% Transaction tax due


thereon 16,020,147.00
Add: 25% surcharge 4,005,036.75

T o t a l P 20,025,183.75
Add:
14% int. fr.
1-20-78 to
7-31-80 P 7,093,302.57
20% int, fr.
8-1-80 to
3-31-83 10,675,523.58

17,768,826.15

P 37,794,009.90
Documentary and Science Stamps Tax
Total face value of
debentures P100,000,000.00
Documentary Stamps
Tax Due
(P0.30 x P100,000.000 )
( P200 ) P 150,000.00
Science Stamps Tax Due
(P0.30 x P100,000,000 )
( P200 ) P 150,000.00

T o t a l P 300,000.00
Add: Compromise for
non-affixture 300.00

300,300.00

TOTAL AMOUNT DUE AND COLLECTIBLE P 38,094,309.90


===========
Deficiency Income Tax for 1977
Net income per return P 258,166.00
Add: Unallowable deductions
1) Disallowed deductions
availed of under
R.A. No. 5186 P 44,332,980.00
2) Capitalized interest
expenses on funds
used for acquisition
of machinery & other
equipment 42,840,131.00
3) Unexplained financial
guarantee expense 1,237,421.00
4) Understatement
of sales 2,391,644.00
5) Overstatement of
cost of sales 604,018.00

P91,406,194.00
Net income per investigation P91,664,360.00
Income tax due thereon 34,734,559.00
Less: Tax already assessed per return 80,358.00

Deficiency P34,654,201.00
Add:
14% int. fr.
4-15-78 to
7-31-81 P 11,128,503.56
20% int. fr.
8-1-80 to
4-15-81 4,886,242.34

P16,014,745.90

TOTAL AMOUNT DUE AND COLLECTIBLE P 50,668,946.90 1


===========
On 26 April 1983, Picop protested the assessment of deficiency transaction
tax and documentary and science stamp taxes. Picop also protested on 21
May 1983 the deficiency income tax assessment for 1977. These protests
were not formally acted upon by respondent CIR. On 26 September 1984, the
CIR issued a warrant of distraint on personal property and a warrant of levy
on real property against Picop, to enforce collection of the contested
assessments; in effect, the CIR denied Picop's protests.
Thereupon, Picop went before the Court of Tax Appeals ("CTA") appealing the
assessments. After trial, the CTA rendered a decision dated 15 August 1989,
modifying the findings of the CIR and holding Picop liable for the reduced
aggregate amount of P20,133,762.33, which was itemized in the dispositive
portion of the decision as follows:
35% Transaction Tax P 16,020,113.20
Documentary & Science
Stamp Tax 300,300.00
Deficiency Income Tax Due 3,813,349.33

TOTAL AMOUNT DUE AND PAYABLE P 20,133,762.53 2


===========
Picop and the CIR both went to the Supreme Court on separate Petitions for
Review of the above decision of the CTA. In two (2) Resolutions dated 7
February 1990 and 19 February 1990, respectively, the Court referred the two
(2) Petitions to the Court of Appeals. The Court of Appeals consolidated the
two (2) cases and rendered a decision, dated 31 August 1992, which further
reduced the liability of Picop to P6,338,354.70. The dispositive portion of the
Court of Appeals decision reads as follows:
WHEREFORE, the appeal of the Commissioner of Internal Revenue is
denied for lack of merit. The judgment against PICOP is modified, as follows:
1. PICOP is declared liable for the 35% transaction tax in the amount of
P3,578,543.51;
2. PICOP is absolved from the payment of documentary and science stamp
tax of P300,000.00 and the compromise penalty of P300.00;
3. PICOP shall pay 20% interest per annum on the deficiency income tax of
P1,481,579.15, for a period of three (3) years from 21 May 1983, or in the
total amount of P888,947.49, and a surcharge of 10% on the latter amount, or
P88,984.75.
No pronouncement as to costs.
SO ORDERED.
Picop and the CIR once more filed separate Petitions for Review before the
Supreme Court. These cases were consolidated and, on 23 August 1993, the
Court resolved to give due course to both Petitions in G.R. Nos. 106949-50
and 106984-85 and required the parties to file their Memoranda.
Picop now maintains that it is not liable at all to pay any of the assessments
or any part thereof. It assails the propriety of the thirty-five percent (35%)
deficiency transaction tax which the Court of Appeals held due from it in the
amount of P3,578,543.51. Picop also questions the imposition by the Court of
Appeals of the deficiency income tax of P1,481,579.15, resulting from
disallowance of certain claimed financial guarantee expenses and claimed
year-end adjustments of sales and cost of sales figures by Picop's external
auditors. 3
The CIR, upon the other hand, insists that the Court of Appeals erred in
finding Picop not liable for surcharge and interest on unpaid transaction tax
and for documentary and science stamp taxes and in allowing Picop to claim
as deductible expenses:
(a) the net operating losses of another corporation (i.e., Rustan Pulp and
Paper Mills, Inc.); and
(b) interest payments on loans for the purchase of machinery and equipment.
The CIR also claims that Picop should be held liable for interest at fourteen
percent (14%) per annum from 15 April 1978 for three (3) years, and interest
at twenty percent (20%) per annum for a maximum of three (3) years; and for
a surcharge of ten percent (10%), on Picop's deficiency income tax. Finally,
the CIR contends that Picop is liable for the corporate development tax
equivalent to five percent (5%) of its correct 1977 net income.
The issues which we must here address may be sorted out and grouped in
the following manner:
I. Whether Picop is liable for:
(1) the thirty-five percent (35%) transaction tax;
(2) interest and surcharge on unpaid transaction tax; and
(3) documentary and science stamp taxes;
II. Whether Picop is entitled to deductions against income of:
(1) interest payments on loans for the purchase of machinery and equipment;
(2) net operating losses incurred by the Rustan Pulp and Paper Mills, Inc.;
and
(3) certain claimed financial guarantee expenses; and
III. (1) Whether Picop had understated its sales and overstated its cost of
sales for 1977; and
(2) Whether Picop is liable for the corporate development tax of five percent
(5%) of its net income for 1977.
We will consider these issues in the foregoing sequence.
I.
(1) Whether Picop is liable
for the thirty-five percent
(35%) transaction tax.
With the authorization of the Securities and Exchange Commission, Picop
issued commercial paper consisting of serially numbered promissory notes
with the total face value of P229,864,000.00 and a maturity period of one (1)
year, i.e., from 24 December 1977 to 23 December 1978. These promissory
notes were purchased by various commercial banks and financial institutions.
On these promissory notes, Picop paid interest in the aggregate amount of
P45,771,849.00. In respect of these interest payments, the CIR required
Picop to pay the thirty-five percent (35%) transaction tax.
The CIR based this assessment on Presidential Decree No. 1154 dated 3
June 1977, which reads in part as follows:
Sec. 1. The National Internal Revenue Code, as amended, is hereby further
amended by adding a new section thereto to read as follows:
Sec. 195-C. Tax on certain interest. There shall be levied, assessed,
collected and paid on every commercial paper issued in the primary market
as principal instrument, a transaction tax equivalent to thirty-five percent
(35%) based on the gross amount of interest thereto as defined hereunder,
which shall be paid by the borrower/issuer: Provided, however, that in the
case of a long-term commercial paper whose maturity exceeds more than
one year, the borrower shall pay the tax based on the amount of interest
corresponding to one year, and thereafter shall pay the tax upon accrual or
actual payment (whichever is earlier) of the untaxed portion of the interest
which corresponds to a period not exceeding one year.
The transaction tax imposed in this section shall be a final tax to be paid by
the borrower and shall be allowed as a deductible item for purposes of
computing the borrower's taxable income.
For purposes of this tax
(a) "Commercial paper" shall be defined as an instrument evidencing
indebtedness of any person or entity, including banks and non-banks
performing quasi-banking functions, which is issued, endorsed, sold,
transferred or in any manner conveyed to another person or entity, either with
or without recourse and irrespective of maturity. Principally, commercial
papers are promissory notes and/or similar instruments issued in the primary
market and shall not include repurchase agreements, certificates of
assignments, certificates of participations, and such other debt instruments
issued in the secondary market.
(b) The term "interest" shall mean the difference between what the principal
borrower received and the amount it paid upon maturity of the commercial
paper which shall, in no case, be lower than the interest rate prevailing at the
time of the issuance or renewal of the commercial paper. Interest shall be
deemed synonymous with discount and shall include all fees, commissions,
premiums and other payments which form integral parts of the charges
imposed as a consequence of the use of money.
In all cases, where no interest rate is stated or if the rate stated is lower than
the prevailing interest rate at the time of the issuance or renewal of
commercial paper, the Commissioner of Internal Revenue, upon consultation
with the Monetary Board of the Central Bank of the Philippines, shall adjust
the interest rate in accordance herewith, and assess the tax on the basis
thereof.
The tax herein imposed shall be remitted by the borrower to the
Commissioner of Internal Revenue or his Collection Agent in the municipality
where such borrower has its principal place of business within five (5) working
days from the issuance of the commercial paper. In the case of long term
commercial paper, the tax upon the untaxed portion of the interest which
corresponds to a period not exceeding one year shall be paid upon accrual
payment, whichever is earlier. (Emphasis supplied)
Both the CTA and the Court of Appeals sustained the assessment of
transaction tax.
In the instant Petition, Picop reiterates its claim that it is exempt from the
payment of the transaction tax by virtue of its tax exemption under R.A. No.
5186, as amended, known as the Investment Incentives Act, which in the
form it existed in 1977-1978, read in relevant part as follows:
Sec. 8. Incentives to a Pioneer Enterprise. In addition to the incentives
provided in the preceding section, pioneer enterprises shall be granted the
following incentive benefits:
(a) Tax Exemption. Exemption from all taxes under the National Internal
Revenue Code, except income tax, from the date the area of investment is
included in the Investment Priorities Plan to the following extent:
(1) One hundred per cent (100%) for the first five years;
(2) Seventy-five per cent (75%) for the sixth through the eighth years;
(3) Fifty per cent (50%) for the ninth and tenth years;
(4) Twenty per cent (20%) for the eleventh and twelfth years; and
(5) Ten per cent (10%) for the thirteenth through the fifteenth year.
xxx xxx xxx 4
We agree with the CTA and the Court of Appeals that Picop's tax exemption
under R.A. No. 5186, as amended, does not include exemption from the
thirty-five percent (35%) transaction tax. In the first place, the thirty-five
percent (35%) transaction tax 5 is an income tax, that is, it is a tax on the interest
income of the lenders or creditors. In Western Minolco Corporation v.
Commissioner of Internal Revenue, 6 the petitioner corporation borrowed funds from
several financial institutions from June 1977 to October 1977 and paid the
corresponding thirty-five (35%) transaction tax thereon in the amount of
P1,317,801.03, pursuant to Section 210 (b) of the 1977 Tax Code. Western Minolco
applied for refund of that amount alleging it was exempt from the thirty-five (35%)
transaction tax by reason of Section 79-A of C.A. No. 137, as amended, which
granted new mines and old mines resuming operation "five (5) years complete tax
exemptions, except income tax, from the time of its actual bonafide orders for
equipment for commercial production." In denying the claim for refund, this Court
held:
The petitioner's contentions deserve scant consideration. The 35%
transaction tax is imposed on interest income from commercial papers issued
in the primary money market. Being a tax on interest, it is a tax on income.
As correctly ruled by the respondent Court of Tax Appeals:
Accordingly, we need not and do not think it necessary to discuss further the
nature of the transaction tax more than to say that the incipient scheme in the
issuance of Letter of Instructions No. 340 on November 24, 1975 (O.G. Dec.
15, 1975), i.e., to achieve operational simplicity and effective administration in
capturing the interest-income "windfall" from money market operations as a
new source of revenue, has lost none of its animating principle in parturition
of amendatory Presidential Decree No. 1154, now Section 210 (b) of the Tax
Code. The tax thus imposed is actually a tax on interest earnings of the
lenders or placers who are actually the taxpayers in whose income is
imposed. Thus "the borrower withholds the tax of 35% from the interest he
would have to pay the lender so that he (borrower) can pay the 35% of the
interest to the Government." (Citation omitted) . . . . Suffice it to state that the
broad consensus of fiscal and monetary authorities is that "even if nominally,
the borrower is made to pay the tax, actually, the tax is on the interest earning
of the immediate and all prior lenders/placers of the money. . . ." (Rollo, pp.
36-37)
The 35% transaction tax is an income tax on interest earnings to the lenders
or placers. The latter are actually the taxpayers. Therefore, the tax cannot be
a tax imposed upon the petitioner. In other words, the petitioner who
borrowed funds from several financial institutions by issuing commercial
papers merely withheld the 35% transaction tax before paying to the financial
institutions the interests earned by them and later remitted the same to the
respondent Commissioner of Internal Revenue. The tax could have been
collected by a different procedure but the statute chose this method.
Whatever collecting procedure is adopted does not change the nature of the
tax.
xxx xxx xxx 7
(Emphasis supplied)
Much the same issue was passed upon in Marinduque Mining Industrial
Corporation v. Commissioner of Internal Revenue 8 and resolved in the same
way:
It is very obvious that the transaction tax, which is a tax on interest derived
from commercial paper issued in the money market, is not a tax contemplated
in the above-quoted legal provisions. The petitioner admits that it is subject to
income tax. Its tax exemption should be strictly construed.
We hold that petitioner's claim for refund was justifiably denied. The
transaction tax, although nominally categorized as a business tax, is in reality
a withholding tax as positively stated in LOI No. 340. The petitioner could
have shifted the tax to the lenders or recipients of the interest. It did not
choose to do so. It cannot be heard now to complain about the tax. LOI No.
340 is an extraneous or extrinsic aid to the construction of section 210 (b).
xxx xxx xxx 9
(Emphasis supplied)
It is thus clear that the transaction tax is an income tax and as such, in any
event, falls outside the scope of the tax exemption granted to registered
pioneer enterprises by Section 8 of R.A. No. 5186, as amended. Picop was
the withholding agent, obliged to withhold thirty-five percent (35%) of the
interest payable to its lenders and to remit the amounts so withheld to the
Bureau of Internal Revenue ("BIR"). As a withholding agent, Picop is made
personally liable for the thirty-five percent (35%) transaction tax 10 and if it did
not actually withhold thirty-five percent (35%) of the interest monies it had paid to its
lenders, Picop had only itself to blame.
Picop claims that it had relied on a ruling, dated 6 October 1977, issued by
the CIR, which held that Picop was not liable for the thirty-five (35%)
transaction tax in respect of debenture bonds issued by Picop. Prior to the
issuance of the promissory notes involved in the instant case, Picop had also
issued debenture bonds P100,000,000.00 in aggregate face value. The
managing underwriter of this debenture bond issue, Bancom Development
Corporation, requested a formal ruling from the Bureau of Internal Revenue
on the liability of Picop for the thirty-five percent (35%) transaction tax in
respect of such bonds. The ruling rendered by the then Acting Commissioner
of Internal Revenue, Efren I. Plana, stated in relevant part:
It is represented that PICOP will be offering to the public primary bonds in the
aggregate principal sum of one hundred million pesos (P100,000,000.00);
that the bonds will be issued as debentures in denominations of one
thousand pesos (P1,000.00) or multiples, to mature in ten (10) years at 14%
interest per annum payable semi-annually; that the bonds are convertible into
common stock of the issuer at the option of the bond holder at an agreed
conversion price; that the issue will be covered by a "Trust Indenture" with a
duly authorized trust corporation as required by the Securities and Exchange
Commission, which trustee will act for and in behalf of the debenture bond
holders as beneficiaries; that once issued, the bonds cannot be
preterminated by the holder and cannot be redeemed by the issuer until after
eight (8) years from date of issue; that the debenture bonds will be
subordinated to present and future debts of PICOP; and that said bonds are
intended to be listed in the stock exchanges, which will place them alongside
listed equity issues.
In reply, I have the honor to inform you that although the bonds hereinabove
described are commercial papers which will be issued in the primary market,
however, it is clear from the abovestated facts that said bonds will not be
issued as money market instruments. Such being the case, and considering
that the purposes of Presidential Decree No. 1154, as can be gleaned from
Letter of Instruction No. 340, dated November 21, 1975, are (a) to regulate
money market transactions and (b) to ensure the collection of the tax on
interest derived from money market transactions by imposing a withholding
tax thereon, said bonds do not come within the purview of the "commercial
papers" intended to be subjected to the 35% transaction tax prescribed in
Presidential Decree No. 1154, as implemented by Revenue Regulations No.
7-77. (See Section 2 of said Regulation) Accordingly, PICOP is not subject to
35% transaction tax on its issues of the aforesaid bonds. However, those
investing in said bonds should be made aware of the fact that the transaction
tax is not being imposed on the issuer of said bonds by printing or stamping
thereon, in bold letters, the following statement: "ISSUER NOT SUBJECT TO
TRANSACTION TAX UNDER P.D. 1154. BONDHOLDER SHOULD
DECLARE INTEREST EARNING FOR INCOME TAX." 11 (Emphases supplied)
In the above quoted ruling, the CIR basically held that Picop's debenture
bonds did not constitute "commercial papers" within the meaning of P.D. No.
1154, and that, as such, those bonds were not subject to the thirty-five
percent (35%) transaction tax imposed by P.D. No. 1154.
The above ruling, however, is not applicable in respect of the promissory
notes which are the subject matter of the instant case. It must be noted that
the debenture bonds which were the subject matter of Commissioner Plana's
ruling were long-term bonds maturing in ten (10) years and which could not
be pre-terminated and could not be redeemed by Picop until after eight (8)
years from date of issue; the bonds were moreover subordinated to present
and future debts of Picop and convertible into common stock of Picop at the
option of the bondholder. In contrast, the promissory notes involved in the
instant case are short-term instruments bearing a one-year maturity period.
These promissory notes constitute the very archtype of money market
instruments. For money market instruments are precisely, by custom and
usage of the financial markets, short-term instruments with a tenor of one (1)
year or less. 12 Assuming, therefore, (without passing upon) the correctness of the
6 October 1977 BIR ruling, Picop's short-term promissory notes must be
distinguished, and treated differently, from Picop's long-term debenture bonds.
We conclude that Picop was properly held liable for the thirty-five percent
(35%) transaction tax due in respect of interest payments on its money
market borrowings.
At the same time, we agree with the Court of Appeals that the transaction tax
may be levied only in respect of the interest earnings of Picop's money
market lenders accruing after P.D. No. 1154 went into effect, and not in
respect of all the 1977 interest earnings of such lenders. The Court of
Appeals pointed out that:
PICOP, however contends that even if the tax has to be paid, it should be
imposed only for the interests earned after 20 September 1977 when PD
1154 creating the tax became effective. We find merit in this contention. It
appears that the tax was levied on interest earnings from January to October,
1977. However, as found by the lower court, PD 1154 was published in the
Official Gazette only on 5 September 1977, and became effective only fifteen
(15) days after the publication, or on 20 September 1977, no other effectivity
date having been provided by the PD. Based on the Worksheet prepared by
the Commissioner's office, the interests earned from 20 September to
October 1977 was P10,224,410.03. Thirty-five (35%) per cent of this is
P3,578,543.51 which is all PICOP should pay as transaction tax. 13 (Emphasis
supplied)
P.D. No. 1154 is not, in other words, to be given retroactive effect by imposing
the thirty-five percent (35%) transaction tax in respect of interest earnings
which accrued before the effectivity date of P.D. No. 1154, there being nothing
in the statute to suggest that the legislative authority intended to bring about
such retroactive imposition of the tax.
(2) Whether Picop is liable
for interest and surcharge
on unpaid transaction tax.
With respect to the transaction tax due, the CIR prays that Picop be held
liable for a twenty-five percent (25%) surcharge and for interest at the rate of
fourteen percent (14%) per annum from the date prescribed for its payment.
In so praying, the CIR relies upon Section 10 of Revenue Regulation 7-77
dated 3 June 1977, 14 issued by the Secretary of Finance. This Section reads:
Sec. 10. Penalties. Where the amount shown by the taxpayer to be due on
its return or part of such payment is not paid on or before the date prescribed
for its payment, the amount of the tax shall be increased by twenty-five (25%)
per centum, the increment to be a part of the tax and the entire amount shall
be subject to interest at the rate of fourteen (14%) per centum per annum
from the date prescribed for its payment.
In the case of willful neglect to file the return within the period prescribed
herein or in case a false or fraudulent return is willfully made, there shall be
added to the tax or to the deficiency tax in case any payment has been made
on the basis of such return before the discovery of the falsity or fraud, a
surcharge of fifty (50%) per centum of its amount. The amount so added to
any tax shall be collected at the same time and in the same manner and as
part of the tax unless the tax has been paid before the discovery of the falsity
or fraud, in which case the amount so added shall be collected in the same
manner as the tax.
In addition to the above administrative penalties, the criminal and civil
penalties as provided for under Section 337 of the Tax Code of 1977 shall be
imposed for violation of any provision of Presidential Decree No. 1154. 15
(Emphases supplied)
The 1977 Tax Code itself, in Section 326 in relation to Section 4 of the same
Code, invoked by the Secretary of Finance in issuing Revenue Regulation 7-
77, set out, in comprehensive terms, the rule-making authority of the
Secretary of Finance:
Sec. 326. Authority of Secretary of Finance to Promulgate Rules and
Regulations. The Secretary of Finance, upon recommendation of the
Commissioner of Internal Revenue, shall promulgate all needful rules and
regulations for the effective enforcement of the provisions of this Code.
(Emphasis supplied)
Section 4 of the same Code contains a list of subjects or areas to be dealt
with by the Secretary of Finance through the medium of an exercise of his
quasi-legislative or rule-making authority. This list, however, while it purports
to be open-ended, does not include the imposition of administrative or civil
penalties such as the payment of amounts additional to the tax due. Thus, in
order that it may be held to be legally effective in respect of Picop in the
present case, Section 10 of Revenue Regulation 7-77 must embody or rest
upon some provision in the Tax Code itself which imposes surcharge and
penalty interest for failure to make a transaction tax payment when due.
P.D. No. 1154 did not itself impose, nor did it expressly authorize the
imposition of, a surcharge and penalty interest in case of failure to pay the
thirty-five percent (35%) transaction tax when due. Neither did Section 210
(b) of the 1977 Tax Code which re-enacted Section 195-C inserted into the
Tax Code by P.D. No. 1154.
The CIR, both in its petition before the Court of Appeals and its Petition in the
instant case, points to Section 51 (e) of the 1977 Tax Code as its source of
authority for assessing a surcharge and penalty interest in respect of the
thirty-five percent (35%) transaction tax due from Picop. This Section needs
to be quoted in extenso:
Sec. 51. Payment and Assessment of Income Tax.
(c) Definition of deficiency. As used in this Chapter in respect of a tax
imposed by this Title, the term "deficiency" means:
(1) The amount by which the tax imposed by this Title exceeds the amount
shown as the tax by the taxpayer upon his return; but the amount so shown
on the return shall first be increased by the amounts previously assessed (or
collected without assessment) as a deficiency, and decreased by the amount
previously abated, credited, returned, or otherwise in respect of such tax; . . .
xxx xxx xxx
(e) Additions to the tax in case of non-payment.
(1) Tax shown on the return. Where the amount determined by the
taxpayer as the tax imposed by this Title or any installment thereof, or any
part of such amount or installment is not paid on or before the date prescribed
for its payment, there shall be collected as a part of the tax, interest upon
such unpaid amount at the rate of fourteen per centum per annum from the
date prescribed for its payment until it is paid: Provided, That the maximum
amount that may be collected as interest on deficiency shall in no case
exceed the amount corresponding to a period of three years, the present
provisions regarding prescription to the contrary notwithstanding.
(2) Deficiency. Where a deficiency, or any interest assessed in connection
therewith under paragraph (d) of this section, or any addition to the taxes
provided for in Section seventy-two of this Code is not paid in full within thirty
days from the date of notice and demand from the Commissioner of Internal
Revenue, there shall be collected upon the unpaid amount as part of the tax,
interest at the rate of fourteen per centum per annum from the date of such
notice and demand until it is paid: Provided, That the maximum amount that
may be collected as interest on deficiency shall in no case exceed the
amount corresponding to a period of three years, the present provisions
regarding prescription to the contrary notwithstanding.
(3) Surcharge. If any amount of tax included in the notice and demand
from the Commissioner of Internal Revenue is not paid in full within thirty
days after such notice and demand, there shall be collected in addition to the
interest prescribed herein and in paragraph (d) above and as part of the tax a
surcharge of five per centum of the amount of tax unpaid. (Emphases
supplied)
Section 72 of the 1977 Tax Code referred to in Section 51 (e) (2) above,
provides:
Sec. 72. Surcharges for failure to render returns and for rendering false and
fraudulent returns. In case of willful neglect to file the return or list required
by this Title within the time prescribed by law, or in case a false or fraudulent
return or list is wilfully made, the Commissioner of Internal Revenue shall add
to the tax or to the deficiency tax, in case any payment has been made on the
basis of such return before the discovery of the falsity or fraud, as surcharge
of fifty per centum of the amount of such tax or deficiency tax. In case of any
failure to make and file a return or list within the time prescribed by law or by
the Commissioner or other Internal Revenue Officer, not due to willful neglect,
the Commissioner of Internal Revenue shall add to the tax twenty-five per
centum of its amount, except that, when a return is voluntarily and without
notice from the Commissioner or other officer filed after such time, and it is
shown that the failure to file it was due to a reasonable cause, no such
addition shall be made to the tax. The amount so added to any tax shall be
collected at the same time, in the same manner and as part of the tax unless
the tax has been paid before the discovery of the neglect, falsity, or fraud, in
which case the amount so added shall be collected in the same manner as
the tax. (Emphases supplied)
It will be seen that Section 51 (c) (1) and (e) (1) and (3), of the 1977 Tax
Code, authorize the imposition of surcharge and interest only in respect of a
"tax imposed by this Title," that is to say, Title II on "Income Tax." It will also
be seen that Section 72 of the 1977 Tax Code imposes a surcharge only in
case of failure to file a return or list "required by this Title," that is, Title II on
"Income Tax." The thirty-five percent (35%) transaction tax is, however,
imposed in the 1977 Tax Code by Section 210 (b) thereof which Section is
embraced in Title V on "Taxes on Business" of that Code. Thus, while the
thirty-five percent (35%) transaction tax is in truth a tax imposed on interest
income earned by lenders or creditors purchasing commercial paper on the
money market, the relevant provisions, i.e., Section 210 (b), were not inserted
in Title II of the 1977 Tax Code. The end result is that the thirty-five percent
(35%) transaction tax is not one of the taxes in respect of which Section 51
(e) authorized the imposition of surcharge and interest and Section 72 the
imposition of a fraud surcharge.
It is not without reluctance that we reach the above conclusion on the basis of
what may well have been an inadvertent error in legislative draftsmanship, a
type of error common enough during the period of Martial Law in our country.
Nevertheless, we are compelled to adopt this conclusion. We consider that
the authority to impose what the present Tax Code calls (in Section 248) civil
penalties consisting of additions to the tax due, must be expressly given in
the enabling statute, in language too clear to be mistaken. The grant of that
authority is not lightly to be assumed to have been made to administrative
officials, even to one as highly placed as the Secretary of Finance.
The state of the present law tends to reinforce our conclusion that Section 51
(c) and (e) of the 1977 Tax Code did not authorize the imposition of a
surcharge and penalty interest for failure to pay the thirty-five percent (35%)
transaction tax imposed under Section 210 (b) of the same Code. The
corresponding provision in the current Tax Code very clearly embraces failure
to pay all taxes imposed in the Tax Code, without any regard to the Title of
the Code where provisions imposing particular taxes are textually located.
Section 247 (a) of the NIRC, as amended, reads:
Title X
Statutory Offenses and Penalties
Chapter I
Additions to the Tax
Sec. 247. General Provisions. (a) The additions to the tax or deficiency tax
prescribed in this Chapter shall apply to all taxes, fees and charges imposed
in this Code. The amount so added to the tax shall be collected at the same
time, in the same manner and as part of the tax. . . .
Sec. 248. Civil Penalties. (a) There shall be imposed, in addition to the tax
required to be paid, penalty equivalent to twenty-five percent (25%) of the
amount due, in the following cases:
xxx xxx xxx
(3) failure to pay the tax within the time prescribed for its payment; or
xxx xxx xxx
(c) the penalties imposed hereunder shall form part of the tax and the entire
amount shall be subject to the interest prescribed in Section 249.
Sec. 249. Interest. (a) In General. There shall be assessed and
collected on any unpaid amount of tax, interest at the rate of twenty percent
(20%) per annum or such higher rate as may be prescribed by regulations,
from the date prescribed for payment until the amount is fully paid. . . .
(Emphases supplied)
In other words, Section 247 (a) of the current NIRC supplies what did not
exist back in 1977 when Picop's liability for the thirty-five percent (35%)
transaction tax became fixed. We do not believe we can fill that legislative
lacuna by judicial fiat. There is nothing to suggest that Section 247 (a) of the
present Tax Code, which was inserted in 1985, was intended to be given
retroactive application by the legislative authority. 16
(3) Whether Picop is Liable
for Documentary and
Science Stamp Taxes.
As noted earlier, Picop issued sometime in 1977 long-term subordinated
convertible debenture bonds with an aggregate face value of
P100,000,000.00. Picop stated, and this was not disputed by the CIR, that the
proceeds of the debenture bonds were in fact utilized to finance the BOI-
registered operations of Picop. The CIR assessed documentary and science
stamp taxes, amounting to P300,000.00, on the issuance of Picop's
debenture bonds. It is claimed by Picop that its tax exemption "exemption
from all taxes under the National Internal Revenue Code, except income tax"
on a declining basis over a certain period of time includes exemption from
the documentary and science stamp taxes imposed under the NIRC.
The CIR, upon the other hand, stresses that the tax exemption under the
Investment Incentives Act may be granted or recognized only to the extent
that the claimant Picop was engaged in registered operations, i.e., operations
forming part of its integrated pulp and paper project. 17 The borrowing of funds
from the public, in the submission of the CIR, was not an activity included in Picop's
registered operations. The CTA adopted the view of the CIR and held that "the
issuance of convertible debenture bonds [was] not synonymous [with] the
manufactur[ing] operations of an integrated pulp and paper mill." 18
The Court of Appeals took a less rigid view of the ambit of the tax exemption
granted to registered pioneer enterprises. Said the Court of Appeals:
. . . PICOP's explanation that the debenture bonds were issued to finance its
registered operation is logical and is unrebutted. We are aware that tax
exemptions must be applied strictly against the beneficiary in order to deter
their abuse. It would indeed be altogether a different matter if there is a
showing that the issuance of the debenture bonds had no bearing
whatsoever on the registered operations PICOP and that they were issued in
connection with a totally different business undertaking of PICOP other than
its registered operation. There is, however, a dearth of evidence in this
regard. It cannot be denied that PICOP needed funds for its operations. One
of the means it used to raise said funds was to issue debenture bonds. Since
the money raised thereby was to be used in its registered operation, PICOP
should enjoy the incentives granted to it by R.A. 5186, one of which is the
exemption from payment of all taxes under the National Internal Revenue
Code, except income taxes, otherwise the purpose of the incentives would be
defeated. Documentary and science stamp taxes on debenture bonds are
certainly not income taxes. 19 (Emphasis supplied)
Tax exemptions are, to be sure, to be "strictly construed," that is, they are not
to be extended beyond the ordinary and reasonable intendment of the
language actually used by the legislative authority in granting the exemption.
The issuance of debenture bonds is certainly conceptually distinct from
pulping and paper manufacturing operations. But no one contends that
issuance of bonds was a principal or regular business activity of Picop; only
banks or other financial institutions are in the regular business of raising
money by issuing bonds or other instruments to the general public. We
consider that the actual dedication of the proceeds of the bonds to the
carrying out of Picop's registered operations constituted a sufficient nexus
with such registered operations so as to exempt Picop from stamp taxes
ordinarily imposed upon or in connection with issuance of such bonds. We
agree, therefore, with the Court of Appeals on this matter that the CTA and
the CIR had erred in rejecting Picop's claim for exemption from stamp taxes.
It remains only to note that after commencement of the present litigation
before the CTA, the BIR took the position that the tax exemption granted by
R.A. No. 5186, as amended, does include exemption from documentary
stamp taxes on transactions entered into by BOI-registered enterprises. BIR
Ruling No. 088, dated 28 April 1989, for instance, held that a registered
preferred pioneer enterprise engaged in the manufacture of integrated
circuits, magnetic heads, printed circuit boards, etc., is exempt from the
payment of documentary stamp taxes. The Commissioner said:
You now request a ruling that as a preferred pioneer enterprise, you are
exempt from the payment of Documentary Stamp Tax (DST).
In reply, please be informed that your request is hereby granted. Pursuant to
Section 46 (a) of Presidential Decree No. 1789, pioneer enterprises
registered with the BOI are exempt from all taxes under the National Internal
Revenue Code, except from all taxes under the National Internal Revenue
Code, except income tax, from the date the area of investment is included in
the Investment Priorities Plan to the following extent:
xxx xxx xxx
Accordingly, your company is exempt from the payment of documentary
stamp tax to the extent of the percentage aforestated on transactions
connected with the registered business activity. (BIR Ruling No. 111-81)
However, if said transactions conducted by you require the execution of a
taxable document with other parties, said parties who are not exempt shall be
the one directly liable for the tax. (Sec. 173, Tax Code, as amended; BIR
Ruling No. 236-87) In other words, said parties shall be liable to the same
percentage corresponding to your tax exemption. (Emphasis supplied)
Similarly, in BIR Ruling No. 013, dated 6 February 1989, the Commissioner
held that a registered pioneer enterprise producing polyester filament yarn
was entitled to exemption "from the documentary stamp tax on [its] sale of
real property in Makati up to December 31, 1989." It appears clear to the
Court that the CIR, administratively at least, no longer insists on the position it
originally took in the instant case before the CTA.
II
(1) Whether Picop is entitled
to deduct against current
income interest payments
on loans for the purchase
of machinery and equipment.
In 1969, 1972 and 1977, Picop obtained loans from foreign creditors in order
to finance the purchase of machinery and equipment needed for its
operations. In its 1977 Income Tax Return, Picop claimed interest payments
made in 1977, amounting to P42,840,131.00, on these loans as a deduction
from its 1977 gross income.
The CIR disallowed this deduction upon the ground that, because the loans
had been incurred for the purchase of machinery and equipment, the interest
payments on those loans should have been capitalized instead and claimed
as a depreciation deduction taking into account the adjusted basis of the
machinery and equipment (original acquisition cost plus interest charges)
over the useful life of such assets.
Both the CTA and the Court of Appeals sustained the position of Picop and
held that the interest deduction claimed by Picop was proper and allowable.
In the instant Petition, the CIR insists on its original position.
We begin by noting that interest payments on loans incurred by a taxpayer
(whether BOI-registered or not) are allowed by the NIRC as deductions
against the taxpayer's gross income. Section 30 of the 1977 Tax Code
provided as follows:
Sec. 30. Deduction from Gross Income. The following may be deducted
from gross income:
(a) Expenses:
xxx xxx xxx
(b) Interest:
(1) In general. The amount of interest paid within the taxable year on
indebtedness, except on indebtedness incurred or continued to purchase or
carry obligations the interest upon which is exempt from taxation as income
under this Title: . . . (Emphasis supplied)
Thus, the general rule is that interest expenses are deductible against gross
income and this certainly includes interest paid under loans incurred in
connection with the carrying on of the business of the taxpayer. 20 In the instant
case, the CIR does not dispute that the interest payments were made by Picop on
loans incurred in connection with the carrying on of the registered operations of
Picop, i.e., the financing of the purchase of machinery and equipment actually used
in the registered operations of Picop. Neither does the CIR deny that such interest
payments were legally due and demandable under the terms of such loans, and in
fact paid by Picop during the tax year 1977.
The CIR has been unable to point to any provision of the 1977 Tax Code or
any other Statute that requires the disallowance of the interest payments
made by Picop. The CIR invokes Section 79 of Revenue Regulations No. 2
as amended which reads as follows:
Sec. 79. Interest on Capital. Interest calculated for cost-keeping or other
purposes on account of capital or surplus invested in the business, which
does not represent a charge arising under an interest-bearing obligation, is
not allowable deduction from gross income. (Emphases supplied)
We read the above provision of Revenue Regulations No. 2 as referring to so
called "theoretical interest," that is to say, interest "calculated" or computed
(and not incurred or paid) for the purpose of determining the "opportunity
cost" of investing funds in a given business. Such "theoretical" or imputed
interest does not arise from a legally demandable interest-bearing obligation
incurred by the taxpayer who however wishes to find out, e.g., whether he
would have been better off by lending out his funds and earning interest
rather than investing such funds in his business. One thing that Section 79
quoted above makes clear is that interest which does constitute a charge
arising under an interest-bearing obligation is an allowable deduction from
gross income.
It is claimed by the CIR that Section 79 of Revenue Regulations No. 2 was
"patterned after" paragraph 1.266-1 (b), entitled "Taxes and Carrying Charges
Chargeable to Capital Account and Treated as Capital Items" of the U.S.
Income Tax Regulations, which paragraph reads as follows:
(B) Taxes and Carrying Charges. The items thus chargeable to capital
accounts are
(11) In the case of real property, whether improved or unimproved and
whether productive or nonproductive.
(a) Interest on a loan (but not theoretical interest of a taxpayer using his own
funds). 21
The truncated excerpt of the U.S. Income Tax Regulations quoted by the CIR
needs to be related to the relevant provisions of the U.S. Internal Revenue
Code, which provisions deal with the general topic of adjusted basis for
determining allowable gain or loss on sales or exchanges of property and
allowable depreciation and depletion of capital assets of the taxpayer:
Present Rule. The Internal Revenue Code, and the Regulations promulgated
thereunder provide that "No deduction shall be allowed for amounts paid or
accrued for such taxes and carrying charges as, under regulations prescribed
by the Secretary or his delegate, are chargeable to capital account with
respect to property, if the taxpayer elects, in accordance with such
regulations, to treat such taxes or charges as so chargeable."
At the same time, under the adjustment of basis provisions which have just
been discussed, it is provided that adjustment shall be made for all
"expenditures, receipts, losses, or other items" properly chargeable to a
capital account, thus including taxes and carrying charges; however, an
exception exists, in which event such adjustment to the capital account is not
made, with respect to taxes and carrying charges which the taxpayer has not
elected to capitalize but for which a deduction instead has been taken. 22
(Emphasis supplied)
The "carrying charges" which may be capitalized under the above quoted
provisions of the U.S. Internal Revenue Code include, as the CIR has pointed
out, interest on a loan "(but not theoretical interest of a taxpayer using his
own funds)." What the CIR failed to point out is that such "carrying charges"
may, at the election of the taxpayer, either be (a) capitalized in which case the
cost basis of the capital assets, e.g., machinery and equipment, will be
adjusted by adding the amount of such interest payments or alternatively, be
(b) deducted from gross income of the taxpayer. Should the taxpayer elect to
deduct the interest payments against its gross income, the taxpayer cannot at
the same time capitalize the interest payments. In other words, the taxpayer
is not entitled to both the deduction from gross income and the adjusted
(increased) basis for determining gain or loss and the allowable depreciation
charge. The U.S. Internal Revenue Code does not prohibit the deduction of
interest on a loan obtained for purchasing machinery and equipment against
gross income, unless the taxpayer has also or previously capitalized the
same interest payments and thereby adjusted the cost basis of such assets.
We have already noted that our 1977 NIRC does not prohibit the deduction of
interest on a loan incurred for acquiring machinery and equipment. Neither
does our 1977 NIRC compel the capitalization of interest payments on such a
loan. The 1977 Tax Code is simply silent on a taxpayer's right to elect one or
the other tax treatment of such interest payments. Accordingly, the general
rule that interest payments on a legally demandable loan are deductible from
gross income must be applied.
The CIR argues finally that to allow Picop to deduct its interest payments
against its gross income would be to encourage fraudulent claims to double
deductions from gross income:
[t]o allow a deduction of incidental expense/cost incurred in the purchase of
fixed asset in the year it was incurred would invite tax evasion through
fraudulent application of double deductions from gross income. 23 (Emphases
supplied)
The Court is not persuaded. So far as the records of the instant cases show,
Picop has not claimed to be entitled to double deduction of its 1977 interest
payments. The CIR has neither alleged nor proved that Picop had previously
adjusted its cost basis for the machinery and equipment purchased with the
loan proceeds by capitalizing the interest payments here involved. The Court
will not assume that the CIR would be unable or unwilling to disallow "a
double deduction" should Picop, having deducted its interest cost from its
gross income, also attempt subsequently to adjust upward the cost basis of
the machinery and equipment purchased and claim, e.g., increased
deductions for depreciation.
We conclude that the CTA and the Court of Appeals did not err in allowing the
deductions of Picop's 1977 interest payments on its loans for capital
equipment against its gross income for 1977.
(2) Whether Picop is entitled
to deduct against current
income net operating losses
incurred by Rustan Pulp
and Paper Mills, Inc.
On 18 January 1977, Picop entered into a merger agreement with the Rustan
Pulp and Paper Mills, Inc. ("RPPM") and Rustan Manufacturing Corporation
("RMC"). Under this agreement, the rights, properties, privileges, powers and
franchises of RPPM and RMC were to be transferred, assigned and
conveyed to Picop as the surviving corporation. The entire subscribed and
outstanding capital stock of RPPM and RMC would be exchanged for
2,891,476 fully paid up Class "A" common stock of Picop (with a par value of
P10.00) and 149,848 shares of preferred stock of Picop (with a par value of
P10.00), to be issued by Picop, the result being that Picop would wholly own
both RPPM and RMC while the stockholders of RPPM and RMC would join
the ranks of Picop's shareholders. In addition, Picop paid off the obligations of
RPPM to the Development Bank of the Philippines ("DBP") in the amount of
P68,240,340.00, by issuing 6,824,034 shares of preferred stock (with a par
value of P10.00) to the DBP. The merger agreement was approved in 1977
by the creditors and stockholders of Picop, RPPM and RMC and by the
Securities and Exchange Commission. Thereupon, on 30 November 1977,
apparently the effective date of merger, RPPM and RMC were dissolved. The
Board of Investments approved the merger agreement on 12 January 1978.
It appears that RPPM and RMC were, like Picop, BOI-registered companies.
Immediately before merger effective date, RPPM had over preceding years
accumulated losses in the total amount of P81,159,904.00. In its 1977
Income Tax Return, Picop claimed P44,196,106.00 of RPPM's accumulated
losses as a deduction against Picop's 1977 gross income. 24
Upon the other hand, even before the effective date of merger, on 30 August
1977, Picop sold all the outstanding shares of RMC stock to San Miguel
Corporation for the sum of P38,900,000.00, and reported a gain of
P9,294,849.00 from this transaction. 25
In claiming such deduction, Picop relies on section 7 (c) of R.A. No. 5186
which provides as follows:
Sec. 7. Incentives to Registered Enterprise. A registered enterprise, to the
extent engaged in a preferred area of investment, shall be granted the
following incentive benefits:
xxx xxx xxx
(c) Net Operating Loss Carry-over. A net operating loss incurred in any of
the first ten years of operations may be carried over as a deduction from
taxable income for the six years immediately following the year of such loss.
The entire amount of the loss shall be carried over to the first of the six
taxable years following the loss, and any portion of such loss which exceeds
the taxable income of such first year shall be deducted in like manner from
the taxable income of the next remaining five years. The net operating loss
shall be computed in accordance with the provisions of the National Internal
Revenue Code, any provision of this Act to the contrary notwithstanding,
except that income not taxable either in whole or in part under this or other
laws shall be included in gross income. (Emphasis supplied)
Picop had secured a letter-opinion from the BOI dated 21 February 1977
that is, after the date of the agreement of merger but before the merger
became effective relating to the deductibility of the previous losses of
RPPM under Section 7 (c) of R.A. No. 5186 as amended. The pertinent
portions of this BOI opinion, signed by BOI Governor Cesar Lanuza, read as
follows:
2) PICOP will not be allowed to carry over the losses of Rustan prior to the
legal dissolution of the latter because at that time the two (2) companies still
had separate legal personalities;
3) After BOI approval of the merger, PICOP can no longer apply for the
registration of the registered capacity of Rustan because with the approved
merger, such registered capacity of Rustan transferred to PICOP will have the
same registration date as that of Rustan. In this case, the previous losses of
Rustan may be carried over by PICOP, because with the merger, PICOP
assumes all the rights and obligations of Rustan subject, however, to the
period prescribed for carrying over of such
losses. 26 (Emphasis supplied)
Curiously enough, Picop did not also seek a ruling on this matter, clearly a
matter of tax law, from the Bureau of Internal Revenue. Picop chose to rely
solely on the BOI letter-opinion.
The CIR disallowed all the deductions claimed on the basis of RPPM's
losses, apparently on two (2) grounds. Firstly, the previous losses were
incurred by "another taxpayer," RPPM, and not by Picop in connection with
Picop's own registered operations. The CIR took the view that Picop, RPPM
and RMC were merged into one (1) corporate personality only on 12 January
1978, upon approval of the merger agreement by the BOI. Thus, during the
taxable year 1977, Picop on the one hand and RPPM and RMC on the other,
still had their separate juridical personalities. Secondly, the CIR alleged that
these losses had been incurred by RPPM "from the borrowing of funds" and
not from carrying out of RPPM's registered operations. We focus on the first
ground. 27
The CTA upheld the deduction claimed by Picop; its reasoning, however, is
less than crystal clear, especially in respect of its view of what the U.S. tax
law was on this matter. In any event, the CTA apparently fell back on the BOI
opinion of 21 February 1977 referred to above. The CTA said:
Respondent further averred that the incentives granted under Section 7 of
R.A. No. 5186 shall be available only to the extent in which they are engaged
in registered operations, citing Section 1 of Rule IX of the Basic Rules and
Regulations to Implement the Intent and Provisions of the Investment
Incentives Act, R.A. No. 5186.
We disagree with respondent. The purpose of the merger was to rationalize
the container board industry and not to take advantage of the net losses
incurred by RPPMI prior to the stock swap. Thus, when stock of a corporation
is purchased in order to take advantage of the corporation's net operating
loss incurred in years prior to the purchase, the corporation thereafter
entering into a trade or business different from that in which it was previously
engaged, the net operating loss carry-over may be entirely lost. [IRC (1954),
Sec. 382(a), Vol. 5, Mertens, Law of Federal Income Taxation, Chap. 29.11a,
p. 103]. 28 Furthermore, once the BOI approved the merger agreement, the registered capacity of Rustan
shall be transferred to PICOP, and the previous losses of Rustan may be carried over by PICOP by operation
of law. [BOI ruling dated February 21, 1977 (Exh. J-1)] It is clear therefrom, that the deduction availed of
under Section 7(c) of R.A. No. 5186 was only proper." (pp. 38-43, Rollo of SP No. 20070) 29 (Emphasis
supplied)
In respect of the above underscored portion of the CTA decision, we must
note that the CTA in fact overlooked the statement made by petitioner's
counsel before the CTA that:
Among the attractions of the merger to Picop was the accumulated net
operating loss carry-over of RMC that it might possibly use to relieve it
(Picop) from its income taxes, under Section 7 (c) of R.A. 5186. Said section
provides:
xxx xxx xxx
With this benefit in mind, Picop addressed three (3) questions to the BOI in a
letter dated November 25, 1976. The BOI replied on February 21, 1977
directly answering the three (3) queries. 30 (Emphasis supplied)
The size of RPPM's accumulated losses as of the date of the merger more
than P81,000,000.00 must have constituted a powerful attraction indeed
for Picop.
The Court of Appeals followed the result reached by the CTA. The Court of
Appeals, much like the CTA, concluded that since RPPM was dissolved on 30
November 1977, its accumulated losses were appropriately carried over by
Picop in the latter's 1977 Income Tax Return "because by that time RPPMI
and Picop were no longer separate and different taxpayers." 31
After prolonged consideration and analysis of this matter, the Court is unable
to agree with the CTA and Court of Appeals on the deductibility of RPPM's
accumulated losses against Picop's 1977 gross income.
It is important to note at the outset that in our jurisdiction, the ordinary rule
that is, the rule applicable in respect of corporations not registered with the
BOI as a preferred pioneer enterprise is that net operating losses cannot
be carried over. Under our Tax Code, both in 1977 and at present, losses
may be deducted from gross income only if such losses were actually
sustained in the same year that they are deducted or charged off. Section 30
of the 1977 Tax Code provides:
Sec. 30. Deductions from Gross Income. In computing net income, there
shall be allowed as deduction
xxx xxx xxx
(d) Losses:
(1) By Individuals. In the case of an individual, losses actually sustained
during the taxable year and not compensated for by an insurance or
otherwise
(A) If incurred in trade or business;
xxx xxx xxx
(2) By Corporations. In a case of a corporation, all losses actually
sustained and charged off within the taxable year and not compensated for by
insurance or otherwise.
(3) By Non-resident Aliens or Foreign Corporations. In the case of a non-
resident alien individual or a foreign corporation, the losses deductible are
those actually sustained during the year incurred in business or trade
conducted within the Philippines, . . . 32 (Emphasis supplied)
Section 76 of the Philippine Income Tax Regulations (Revenue Regulation
No. 2, as amended) is even more explicit and detailed:
Sec. 76. When charges are deductible. Each year's return, so far as
practicable, both as to gross income and deductions therefrom should be
complete in itself, and taxpayers are expected to make every reasonable
effort to ascertain the facts necessary to make a correct return. The
expenses, liabilities, or deficit of one year cannot be used to reduce the
income of a subsequent year. A taxpayer has the right to deduct all
authorized allowances and it follows that if he does not within any year
deduct certain of his expenses, losses, interests, taxes, or other charges,
he can not deduct them from the income of the next or any succeeding year. .
..
xxx xxx xxx
. . . . If subsequent to its occurrence, however, a taxpayer first ascertains the
amount of a loss sustained during a prior taxable year which has not been
deducted from gross income, he may render an amended return for such
preceding taxable year including such amount of loss in the deduction from
gross income and may in proper cases file a claim for refund of the excess
paid by reason of the failure to deduct such loss in the original return. A loss
from theft or embezzlement occurring in one year and discovered in another
is ordinarily deductible for the year in which sustained. (Emphases supplied)
It is thus clear that under our law, and outside the special realm of BOI-
registered enterprises, there is no such thing as a carry-over of net operating
loss. To the contrary, losses must be deducted against current income in the
taxable year when such losses were incurred. Moreover, such losses may be
charged off only against income earned in the same taxable year when the
losses were incurred.
Thus it is that R.A. No. 5186 introduced the carry-over of net operating losses
as a very special incentive to be granted only to registered pioneer
enterprises and only with respect to their registered operations. The statutory
purpose here may be seen to be the encouragement of the establishment
and continued operation of pioneer industries by allowing the registered
enterprise to accumulate its operating losses which may be expected during
the early years of the enterprise and to permit the enterprise to offset such
losses against income earned by it in later years after successful
establishment and regular operations. To promote its economic development
goals, the Republic foregoes or defers taxing the income of the pioneer
enterprise until after that enterprise has recovered or offset its earlier losses.
We consider that the statutory purpose can be served only if the accumulated
operating losses are carried over and charged off against income
subsequently earned and accumulated by the same enterprise engaged in
the same registered operations.
In the instant case, to allow the deduction claimed by Picop would be to
permit one corporation or enterprise, Picop, to benefit from the operating
losses accumulated by another corporation or enterprise, RPPM. RPPM far
from benefiting from the tax incentive granted by the BOI statute, in fact gave
up the struggle and went out of existence and its former stockholders joined
the much larger group of Picop's stockholders. To grant Picop's claimed
deduction would be to permit Picop to shelter its otherwise taxable income
(an objective which Picop had from the very beginning) which had not been
earned by the registered enterprise which had suffered the accumulated
losses. In effect, to grant Picop's claimed deduction would be to permit Picop
to purchase a tax deduction and RPPM to peddle its accumulated operating
losses. Under the CTA and Court of Appeals decisions, Picop would benefit
by immunizing P44,196,106.00 of its income from taxation thereof although
Picop had not run the risks and incurred the losses which had been
encountered and suffered by RPPM. Conversely, the income that would be
shielded from taxation is not income that was, after much effort, eventually
generated by the same registered operations which earlier had sustained
losses. We consider and so hold that there is nothing in Section 7 (c) of R.A.
No. 5186 which either requires or permits such a result. Indeed, that result
makes non-sense of the legislative purpose which may be seen clearly to be
projected by Section 7 (c), R.A. No. 5186.
The CTA and the Court of Appeals allowed the offsetting of RPPM's
accumulated operating losses against Picop's 1977 gross income, basically
because towards the end of the taxable year 1977, upon the arrival of the
effective date of merger, only one (1) corporation, Picop, remained. The
losses suffered by RPPM's registered operations and the gross income
generated by Picop's own registered operations now came under one and the
same corporate roof. We consider that this circumstance relates much more
to form than to substance. We do not believe that that single purely technical
factor is enough to authorize and justify the deduction claimed by Picop.
Picop's claim for deduction is not only bereft of statutory basis; it does
violence to the legislative intent which animates the tax incentive granted by
Section 7 (c) of R.A. No. 5186. In granting the extraordinary privilege and
incentive of a net operating loss carry-over to BOI-registered pioneer
enterprises, the legislature could not have intended to require the Republic to
forego tax revenues in order to benefit a corporation which had run no risks
and suffered no losses, but had merely purchased another's losses.
Both the CTA and the Court of Appeals appeared much impressed not only
with corporate technicalities but also with the U.S. tax law on this matter. It
should suffice, however, simply to note that in U.S. tax law, the availability to
companies generally of operating loss carry-overs and of operating loss
carry-backs is expressly provided and regulated in great detail by statute. 33 In
our jurisdiction, save for Section 7 (c) of R.A. No. 5186, no statute recognizes or
permits loss carry-overs and loss carry-backs. Indeed, as already noted, our tax law
expressly rejects the very notion of loss carry-overs and carry-backs.
We conclude that the deduction claimed by Picop in the amount of
P44,196,106.00 in its 1977 Income Tax Return must be disallowed.
(3) Whether Picop is entitled
to deduct against current
income certain claimed
financial guarantee expenses.
In its Income Tax Return for 1977, Picop also claimed a deduction in the
amount of P1,237,421.00 as financial guarantee expenses.
This deduction is said to relate to chattel and real estate mortgages required
from Picop by the Philippine National Bank ("PNB") and DBP as guarantors of
loans incurred by Picop from foreign creditors. According to Picop, the
claimed deduction represents registration fees and other expenses incidental
to registration of mortgages in favor of DBP and PNB.
In support of this claimed deduction, Picop allegedly showed its own
vouchers to BIR Examiners to prove disbursements to the Register of Deeds
of Tandag, Surigao del Sur, of particular amounts. In the proceedings before
the CTA, however, Picop did not submit in evidence such vouchers and
instead presented one of its employees to testify that the amount claimed had
been disbursed for the registration of chattel and real estate mortgages.
The CIR disallowed this claimed deduction upon the ground of insufficiency of
evidence. This disallowance was sustained by the CTA and the Court of
Appeals. The CTA said:
No records are available to support the abovementioned expenses. The
vouchers merely showed that the amounts were paid to the Register of
Deeds and simply cash account. Without the supporting papers such as the
invoices or official receipts of the Register of Deeds, these vouchers standing
alone cannot prove that the payments made were for the accrued expenses
in question. The best evidence of payment is the official receipts issued by
the Register of Deeds. The testimony of petitioner's witness that the official
receipts and cash vouchers were shown to the Bureau of Internal Revenue
will not suffice if no records could be presented in court for proper marking
and identification. 34 Emphasis supplied)
The Court of Appeals added:
The mere testimony of a witness for PICOP and the cash vouchers do not
suffice to establish its claim that registration fees were paid to the Register of
Deeds for the registration of real estate and chattel mortgages in favor of
Development Bank of the Philippines and the Philippine National Bank as
guarantors of PICOP's loans. The witness could very well have been merely
repeating what he was instructed to say regardless of the truth, while the
cash vouchers, which we do not find on file, are not said to provide the
necessary details regarding the nature and purpose of the expenses reflected
therein. PICOP should have presented, through the guarantors, its owner's
copy of the registered titles with the lien inscribed thereon as well as an
official receipt from the Register of Deeds evidencing payment of the
registration fee. 35 (Emphasis supplied)
We must support the CTA and the Court of Appeals in their foregoing rulings.
A taxpayer has the burden of proving entitlement to a claimed deduction. 36 In
the instant case, even Picop's own vouchers were not submitted in evidence and
the BIR Examiners denied that such vouchers and other documents had been
exhibited to them. Moreover, cash vouchers can only confirm the fact of
disbursement but not necessarily the purpose thereof. 37 The best evidence that
Picop should have presented to support its claimed deduction were the invoices
and official receipts issued by the Register of Deeds. Picop not only failed to
present such documents; it also failed to explain the loss thereof, assuming they
had existed before. 38 Under the best evidence rule, 39 therefore, the testimony of
Picop's employee was inadmissible and was in any case entitled to very little, if any,
credence.
We consider that entitlement to Picop's claimed deduction of P1,237,421.00
was not adequately shown and that such deduction must be disallowed.
III
(1) Whether Picop had understated
its sales and overstated its
cost of sales for 1977.
In its assessment for deficiency income tax for 1977, the CIR claimed that
Picop had understated its sales by P2,391,644.00 and, upon the other hand,
overstated its cost of sales by P604,018.00. Thereupon, the CIR added back
both sums to Picop's net income figure per its own return.
The 1977 Income Tax Return of Picop set forth the following figures:
Sales (per Picop's Income Tax Return):
Paper P 537,656,719.00
Timber P 263,158,132.00

Total Sales P 800,814,851.00


============
Upon the other hand, Picop's Books of Accounts reflected higher sales
figures:
Sales (per Picop's Books of Accounts):
Paper P 537,656,719.00
Timber P 265,549,776.00

Total Sales P 803,206,495.00


============
The above figures thus show a discrepancy between the sales figures
reflected in Picop's Books of Accounts and the sales figures reported in its
1977 Income Tax Return, amounting to: P2,391,644.00.
The CIR also contended that Picop's cost of sales set out in its 1977 Income
Tax Return, when compared with the cost figures in its Books of Accounts,
was overstated:
Cost of Sales
(per Income Tax Return) P607,246,084.00
Cost of Sales
(per Books of Accounts) P606,642,066.00

Discrepancy P 604,018.00
============
Picop did not deny the existence of the above noted discrepancies. In the
proceedings before the CTA, Picop presented one of its officials to explain the
foregoing discrepancies. That explanation is perhaps best presented in
Picop's own words as set forth in its Memorandum before this Court:
. . . that the adjustment discussed in the testimony of the witness, represent
the best and most objective method of determining in pesos the amount of
the correct and actual export sales during the year. It was this correct and
actual export sales and costs of sales that were reflected in the income tax
return and in the audited financial statements. These corrections did not
result in realization of income and should not give rise to any deficiency tax.
xxx xxx xxx
What are the facts of this case on this matter? Why were adjustments
necessary at the year-end?
Because of PICOP's procedure of recording its export sales (reckoned in U.S.
dollars) on the basis of a fixed rate, day to day and month to month,
regardless of the actual exchange rate and without waiting when the actual
proceeds are received. In other words, PICOP recorded its export sales at a
pre-determined fixed exchange rate. That pre-determined rate was decided
upon at the beginning of the year and continued to be used throughout the
year.
At the end of the year, the external auditors made an examination. In that
examination, the auditors determined with accuracy the actual dollar
proceeds of the export sales received. What exchange rate was used by the
auditors to convert these actual dollar proceeds into Philippine pesos? They
used the average of the differences between (a) the recorded fixed exchange
rate and (b) the exchange rate at the time the proceeds were actually
received. It was this rate at time of receipt of the proceeds that determined
the amount of pesos credited by the Central Bank (through the agent banks)
in favor of PICOP. These accumulated differences were averaged by the
external auditors and this was what was used at the year-end for income tax
and other government-report purposes. (T.s.n., Oct. 17/85, pp. 20-25) 40
The above explanation, unfortunately, at least to the mind of the Court, raises
more questions than it resolves. Firstly, the explanation assumes that all of
Picop's sales were export sales for which U.S. dollars (or other foreign
exchange) were received. It also assumes that the expenses summed up as
"cost of sales" were all dollar expenses and that no peso expenses had been
incurred. Picop's explanation further assumes that a substantial part of
Picop's dollar proceeds for its export sales were not actually surrendered to
the domestic banking system and seasonably converted into pesos; had all
such dollar proceeds been converted into pesos, then the peso figures could
have been simply added up to reflect the actual peso value of Picop's export
sales. Picop offered no evidence in respect of these assumptions, no
explanation why and how a "pre-determined fixed exchange rate" was chosen
at the beginning of the year and maintained throughout. Perhaps more
importantly, Picop was unable to explain why its Books of Accounts did not
pick up the same adjustments that Picop's External Auditors were alleged to
have made for purposes of Picop's Income Tax Return. Picop attempted to
explain away the failure of its Books of Accounts to reflect the same
adjustments (no correcting entries, apparently) simply by quoting a passage
from a case where this Court refused to ascribe much probative value to the
Books of Accounts of a corporate taxpayer in a tax case. 41 What appears to
have eluded Picop, however, is that its Books of Accounts, which are kept by its
own employees and are prepared under its control and supervision, reflect what
may be deemed to be admissions against interest in the instant case. For Picop's
Books of Accounts precisely show higher sales figures and lower cost of sales
figures than Picop's Income Tax Return.
It is insisted by Picop that its Auditors' adjustments simply present the "best
and most objective" method of reflecting in pesos the "correct and ACTUAL
export sales" 42 and that the adjustments or "corrections" "did not result in
realization of [additional] income and should not give rise to any deficiency tax."
The correctness of this contention is not self-evident. So far as the record of this
case shows, Picop did not submit in evidence the aggregate amount of its U.S.
dollar proceeds of its export sales; neither did it show the Philippine pesos it had
actually received or been credited for such U.S. dollar proceeds. It is clear to this
Court that the testimonial evidence submitted by Picop fell far short of
demonstrating the correctness of its explanation.
Upon the other hand, the CIR has made out at least a prima facie case that
Picop had understated its sales and overstated its cost of sales as set out in
its Income Tax Return. For the CIR has a right to assume that Picop's Books
of Accounts speak the truth in this case since, as already noted, they embody
what must appear to be admissions against Picop's own interest.
Accordingly, we must affirm the findings of the Court of Appeals and the CTA.
(2) Whether Picop is liable for
the corporate development
tax of five percent (5%)
of its income for 1977.
The five percent (5%) corporate development tax is an additional corporate
income tax imposed in Section 24 (e) of the 1977 Tax Code which reads in
relevant part as follows:
(e) Corporate development tax. In addition to the tax imposed in
subsection (a) of this section, an additional tax in an amount equivalent to 5
per cent of the same taxable net income shall be paid by a domestic or a
resident foreign corporation; Provided, That this additional tax shall be
imposed only if the net income exceeds 10 per cent of the net worth, in case
of a domestic corporation, or net assets in the Philippines in case of a
resident foreign corporation: . . . .
The additional corporate income tax imposed in this subsection shall be
collected and paid at the same time and in the same manner as the tax
imposed in subsection (a) of this section.
Since this five percent (5%) corporate development tax is an income tax,
Picop is not exempted from it under the provisions of Section 8 (a) of R.A. No.
5186.
For purposes of determining whether the net income of a corporation exceeds
ten percent (10%) of its net worth, the term "net worth" means the
stockholders' equity represented by the excess of the total assets over
liabilities as reflected in the corporation's balance sheet provided such
balance sheet has been prepared in accordance with generally accepted
accounting principles employed in keeping the books of the corporation. 43
The adjusted net income of Picop for 1977, as will be seen below, is
P48,687,355.00. Its net worth figure or total stockholders' equity as reflected
in its Audited Financial Statements for 1977 is P464,749,528.00. Since its
adjusted net income for 1977 thus exceeded ten percent (10%) of its net
worth, Picop must be held liable for the five percent (5%) corporate
development tax in the amount of P2,434,367.75.
Recapitulating, we hold:
(1) Picop is liable for the thirty-five percent (35%) transaction tax in the
amount of P3,578,543.51.
(2) Picop is not liable for interest and surcharge on unpaid transaction tax.
(3) Picop is exempt from payment of documentary and science stamp taxes
in the amount of P300,000.00 and the compromise penalty of P300.00.
(4) Picop is entitled to its claimed deduction of P42,840,131.00 for interest
payments on loans for, among other things, the purchase of machinery and
equipment.
(5) Picop's claimed deduction in the amount of P44,196,106.00 for the
operating losses previously incurred by RPPM, is disallowed for lack of merit.
(6) Picop's claimed deduction for certain financial guarantee expenses in the
amount P1,237,421.00 is disallowed for failure adequately to prove such
expenses.
(7) Picop has understated its sales by P2,391,644.00 and overstated its cost
of sales by P604,018.00, for 1977.
(8) Picop is liable for the corporate development tax of five percent (5%) of its
adjusted net income for 1977 in the amount of P2,434,367.75.
Considering conclusions nos. 4, 5, 6, 7 and 8, the Court is compelled to hold
Picop liable for deficiency income tax for the year 1977 computed as follows:
Deficiency Income Tax
Net Income Per Return P 258,166.00
Add:
Unallowable Deductions
(1) Deduction of net
operating losses
incurred by RPPM P 44,196,106.00
(2) Unexplained financial
guarantee expenses P 1,237,421.00
(3) Understatement of
Sales P 2,391,644.00
(4) Overstatement of
Cost of Sales P 604,018.00

Total P 48,429,189.00

Net Income as Adjusted P 48,687,355.00


===========
Income Tax Due Thereon 44 P 17,030,574.00
Less:
Tax Already Assessed per
Return 80,358.00

Deficiency Income Tax P 16,560,216.00


Add:
Five percent (5%) Corporate
Development Tax P 2,434,367.00
Total Deficiency Income Tax P 18,994,583.00
===========
Add:
Five percent (5%) surcharge 45 P 949,729.15

Total Deficiency Income Tax


with surcharge P 19,944,312.15
Add:
Fourteen percent (14%)
interest from 15 April
1978 to 14 April 1981 46 P 8,376,610.80
Fourteen percent (14%)
interest from 21 April
1983 to 20 April 1986 47 P 11,894,787.00

Total Deficiency Income Tax


Due and Payable P 40,215,709.00
===========
WHEREFORE, for all the foregoing, the Decision of the Court of Appeals is
hereby MODIFIED and Picop is hereby ORDERED to pay the CIR the
aggregate amount of P43,794,252.51 itemized as follows:
(1) Thirty-five percent (35%)
transaction tax P 3,578,543.51
(2) Total Deficiency Income
Tax Due 40,215,709.00

Aggregate Amount Due and Payable P 43,794,252.51


============
No pronouncement as to costs.
SO ORDERED.
Narvasa, C.J., Regalado, Davide, Jr., Romero, Bellosillo, Melo, Puno,
Kapunan, Mendoza, Francisco, Hermosisima, Jr. and Panganiban, JJ.,
concur.
Padilla, J., took no part.

Separate Opinions
VITUG, J., concurring and dissenting:
In usual erudite manner, Mr. Justice Florentino P. Feliciano has written for the
Court the ponencia that presents in clear and logical sequence the issues, the
facts and the law involved. While I share, in most part, the conclusions
expressed in the opinion, I regrettably find it difficult, nevertheless, not to
propose a re-examination of the Court's holding in Western Minolco
Corporation vs. Commissioner of Internal Revenue (124 SCRA 121),
reiterated in Marinduque Mining and Industrial Corporation vs. Commissioner
of Internal Revenue (137 SCRA 88), that has taken the 35% transaction tax
on commercial papers issued in the primary market under the 1977 Revenue
Code, in relation to Republic Act ("R.A.") 5186, to be an income tax.
R.A. No. 5186, also known as the Investment Incentives Act, has provided for
incentives by, among other things, granting to registered pioneer enterprises
an exemption from all taxes, except income tax, under the National Internal
Revenue Code. The income tax, referred to, in my view, is that imposed in
Title II, entitled "Income Tax," of the Revenue Code. Nowhere under that title
is there a 35% transaction tax.
There was, to be sure, a 35% transaction tax still in effect in 1977 but it was a
tax not on the investor-lender in whose favor the interest income on the
commercial paper accrues. The tax was, instead, levied on the borrower-
issuer of commercial papers transacted in the primary market. Being the
principal taxpayer, the borrower-issuer could not have been likewise
contemplated to be a mere tax withholding agent. The tax was conceived as
a tax on business transaction, and so it was rightly incorporated in Title V,
entitled "Privilege Taxes on Business and Occupation" of the Tax Code.
The fact that a taxpayer on whom the tax is imposed can shift, characteristic
of indirect taxes, the burden thereof to another does not make the latter the
taxpayer and the former the withholding agent. Indeed, the facility of shifting
the burden of the tax is opposed to the idea of a direct tax to which class the
income tax actually belongs.
Accordingly, I vote to so reduce the tax liability of petitioners as adjudged by
the amount corresponding to the 35% transaction tax. In all other respects, I
concur with the majority in the judgment.
Separate Opinions
VITUG, J., concurring and dissenting:
In usual erudite manner, Mr. Justice Florentino P. Feliciano has written for the
Court the ponencia that presents in clear and logical sequence the issues, the
facts and the law involved. While I share, in most part, the conclusions
expressed in the opinion, I regrettably find it difficult, nevertheless, not to
propose a re-examination of the Court's holding in Western Minolco
Corporation vs. Commissioner of Internal Revenue (124 SCRA 121),
reiterated in Marinduque Mining and Industrial Corporation vs. Commissioner
of Internal Revenue (137 SCRA 88), that has taken the 35% transaction tax
on commercial papers issued in the primary market under the 1977 Revenue
Code, in relation to Republic Act ("R.A.") 5186, to be an income tax.
R.A. No. 5186, also known as the Investment Incentives Act, has provided for
incentives by, among other things, granting to registered pioneer enterprises
an exemption from all taxes, except income tax, under the National Internal
Revenue Code. The income tax, referred to, in my view, is that imposed in
Title II, entitled "Income Tax," of the Revenue Code. Nowhere under that title
is there a 35% transaction tax.
There was, to be sure, a 35% transaction tax still in effect in 1977 but it was a
tax not on the investor-lender in whose favor the interest income on the
commercial paper accrues. The tax was, instead, levied on the borrower-
issuer of commercial papers transacted in the primary market. Being the
principal taxpayer, the borrower-issuer could not have been likewise
contemplated to be a mere tax withholding agent. The tax was conceived as
a tax on business transaction, and so it was rightly incorporated in Title V,
entitled "Privilege Taxes on Business and Occupation" of the Tax Code.
The fact that a taxpayer on whom the tax is imposed can shift, characteristic
of indirect taxes, the burden thereof to another does not make the latter the
taxpayer and the former the withholding agent. Indeed, the facility of shifting
the burden of the tax is opposed to the idea of a direct tax to which class the
income tax actually belongs.
Accordingly, I vote to so reduce the tax liability of petitioners as adjudged by
the amount corresponding to the 35% transaction tax. In all other respects, I
concur with the majority in the judgment.

G.R. No. L-11976 September 26, 1961


COLLECTOR OF INTERNAL REVENUE, petitioner,
vs.
ANTONIO PRIETO, ET AL., respondents.
Office of the Solicitor General for petitioner.
Ramirez, Ortigas and Formilleza and Latorre for respondents.
RESOLUTION
DIZON, J.:
Before us is petitioner's motion praying that our decision of August 29, 1961
be reconsidered and set aside "insofar as said decision affirms the ruling of
the Court of Tax Appeals charging interest against the Government on the tax
payment declared refundable to the respondents." He relies upon our
decision in Collector of Internal Revenue vs. St. Paul's Hospital of Iloilo,
promulgated on May 25, 1959, in which we said:
We agree, however, with the Solicitor General that the Court of Tax Appeals
erred in ordering the payment of interest on the amount to be refunded to
respondent herein. In the absence of a statutory provision clearly or expressly
directing or authorizing such payment, and none has been cited by
respondent, the National Government cannot be required to pay interest (H.E.
Heacock Co. v. Collector of Customs, 37 Phil 970; Marine Trading Co. v.
Gov't of the P.I., 39 Phil. 29; Sarasola v. Trinidad, 40 Phil. 252). So much of
the decision appealed from as requires the payment of interest should,
therefore, be eliminated.
Prior, however, to the decision relied upon we held in Carcar Electric & Ice
Plant Co., Inc. vs. Collector of Internal Revenue (G.R. No. L-9257, Oct. 17,
1956, 53 O.G. No. 4, 1068) that "under the present Internal Revenue Code
the Collector of Internal Revenue may be made to answer for interest at the
legal rate on taxes improperly collected. Such liability serves as additional
safeguard in favor of the taxpayer against arbitrariness in the exaction or
collection of taxes and imposts." (See Resolution on the Motion for
Reconsideration filed by the Collector of Internal Revenue, 53 O.G. No. 4,
1071-1075).
In reasoning our Resolution in the Carcar case we said that "Under the
Internal Revenue Act of 1914, the Collector of Internal Revenue was liable for
interest on taxes improperly collected as held in Hongkong-Shanghai Bank
vs. Rafferty, 39 Phil. 153; Heacock Co. vs. Collector of Customs, 37 Phil. 970;
Vda. e Hijos de P. Roxas vs. Rafferty, 37 Phil. 957"; that, subsequently,
Section 1579 of the Administrative Code of 1917 expressly authorized suits
against the Collector of Internal Revenue "for the recovery without interest of
the sum alleged to have been illegally collected"; that for this reason,
thereafter no judgments for interest were rendered against the Collector; that
in 1939, the National Internal Revenue Code, in its section 306, authorized
recovery of taxes erroneously or illegally collected, but omitting the
expression "without interest" employed in the aforesaid section of the
Administrative Code of 1917, which it superseded; that considering our
repeated rulings holding the Collector of Internal Revenue liable for interest
on taxes improperly collected, in the absence of express exemption, it was
clear that the Legislature's failure to reenact the words "without interest" of
the Administrative Code of 1917 showed a clear desire to return to the rule in
force before said year.
Our decision in the Carcar case, however, must be understood as holding the
Collector of Internal Revenue liable for interest on taxes improperly collected
only if the collection was attended with "arbitrariness". The facts involved in
the case relied upon by petitioner the St. Paul's Hospital of Iloilo case
do not seem to justify the conclusion that arbitrariness attended or
characterized the collection of the taxes in question therein. Said facts are as
follows:
Petitioner is a corporation "dedicated to charitable, educational and religious
purposes", operating a hospital giving medical assistance to destitute
persons. (See St. Paul's Hospital of Iloilo v. Collector of Internal Revenue,
C.T.A. Case No. 6, promulgated on December 4, 1954). It maintains a
pharmacy department within the premises of its hospital to supply drugs and
medicines only to charity and paying patients confined therein. However, only
the paying patients are required to pay the medicines supplied to them and
the charge consists of the cost of such medicines plus an additional 10%
thereof to partly offset the cost of medicines supplied free of charge to charity
patients. On May 6, 1954, respondent assessed and demanded from
petitioner the sum of P485.00 allegedly representing business tax on its
operation of a pharmacy department. From this assessment petitioner
appealed to this Court.1awphl.nt

The question of whether or not the sale of drugs and medicines made at the
pharmacy department of the St. Paul's Hospital of Iloilo were taxable was, in
our opinion, a fairly debatable issue. The Collector, therefore, can not be said
to have acted arbitrarily in assessing the corresponding tax on the hospital.
This being the case, we see no real conflict between our decision in the
Carcar case, on the one hand, and the one rendered in the St. Paul's Hospital
of Iloilo case.
The question we now have to decide is whether the first or the second ruling
is the one applicable to the present case. Upon consideration of the facts
appearing of record we believe that it is the first. The Collector of Internal
Revenue had no reason to insist in collecting the inheritance tax from
respondents on the basis of the value of the properties allotted to each of
them, in accordance with the project of partition submitted to and approved by
the court without deducting therefrom the cash payments which, in
accordance with their agreement with their co-heirs, they had to pay to the
latter for the purpose of making the share of each heir equal in value to that of
the others as ordained in the will of the deceased Doa Teresa Tuason y
de la Paz, and as agreed among the heirs. What each of the respondents
really received as his share in the estate of said deceased was the value of
the properties allotted to each of them minus the cash payments each had to
make in order to equalize their respective share with that of the other heirs.
The collection of the inheritance taxes herein involved being clearly
unjustified, we are constrained, as already stated above, to hold the ruling in
the Carcar case applicable to the present.
WHEREFORE, petitioner's motion for reconsideration is hereby denied.

G.R. Nos. L-18169, L-18262 & L-21434 July 31, 1964


COMMISSIONER OF INTERNAL REVENUES, petitioner,
vs.
V.E. LEDNICKY and MARIA VALERO LEDNICKY, respondents.
Office of the Solicitor General for petitioner.
Ozaeta, Gibbs and Ozaeta for respondents.
REYES, J.B.L., J.:
The above-captioned cases were elevated to this Court under separate
petitions by the Commissioner for review of the corresponding decisions of
the Court of Tax Appeals. Since these cases involve the same parties and
issues akin to each case presented, they are herein decided jointly.
The respondents, V. E. Lednicky and Maria Valero Lednicky, are husband and
wife, respectively, both American citizens residing in the Philippines, and have
derived all their income from Philippine sources for the taxable years in
question.
In compliance with local law, the aforesaid respondents, on 27 March 1957,
filed their income tax return for 1956, reporting therein a gross income of
P1,017,287. 65 and a net income of P733,809.44 on which the amount of
P317,395.4 was assessed after deducting P4,805.59 as withholding tax.
Pursuant to the petitioner's assessment notice, the respondents paid the total
amount of P326,247.41, inclusive of the withheld taxes, on 15 April 1957.
On 17 March 1959, the respondents Lednickys filed an amended income tax
return for 1956. The amendment consists in a claimed deduction of
P205,939.24 paid in 1956 to the United States government as federal income
tax for 1956. Simultaneously with the filing of the amended return, the
respondents requested the refund of P112,437.90.
When the petitioner Commissioner of Internal Revenue failed to answer the
claim for refund, the respondents filed their petition with the Tax Court on 11
April 1959 as CTA Case No. 646, which is now G. R. No. L-18286 in the
Supreme Court.
G. R. No. L-18169 (formerly CTA Case No. 570) is also a claim for refund in
the amount of P150,269.00, as alleged overpaid income tax for 1955, the
facts of which are as follows:
On 28 February 1956, the same respondents-spouses filed their domestic
income tax return for 1955, reporting a gross income of P1,771,124.63 and a
net income of P1,052,550.67. On 19 April 1956, they filed an amended
income tax return, the amendment upon the original being a lesser net
income of P1,012,554.51, and, on the basis of this amended return, they paid
P570,252.00, inclusive of withholding taxes. After audit, the petitioner
determined a deficiency of P16,116.00, which amount, the respondents paid
on 5 December 1956.
Back in 1955, however, the Lednickys filed with the U.S. Internal Revenue
Agent in Manila their federal income tax return for the years 1947, 1951,
1952, 1953, and 1954 on income from Philippine sources on a cash basis.
Payment of these federal income taxes, including penalties and delinquency
interest in the amount of P264,588.82, were made in 1955 to the U.S.
Director of Internal Revenue, Baltimore, Maryland, through the National City
Bank of New York, Manila Branch. Exchange and bank charges in remitting
payment totaled P4,143.91.
Wherefore, the parties respectfully pray that the foregoing stipulation of facts
be admitted and approved by this Honorable Court, without prejudice to the
parties adducing other evidence to prove their case not covered by this
stipulation of facts.
1wph1.t

On 11 August 1958, the said respondents amended their Philippine income


tax return for 1955 to include the following deductions:
U.S. Federal income taxes
P471,867.32
Interest accrued up to May 15, 1955
40,333.92
Exchange and bank charges
4,143.91
Total

P516,345.15
and therewith filed a claim for refund of the sum of P166,384.00, which was
later reduced to P150,269.00.
The respondents Lednicky brought suit in the Tax Court, which was docketed
therein as CTA Case No. 570.
In G. R. No. 21434 (CTA Case No. 783), the facts are similar, but refer to
respondents Lednickys' income tax return for 1957, filed on 28 February
1958, and for which respondents paid a total sum of P196,799.65. In 1959,
they filed an amended return for 1957, claiming deduction of P190,755.80,
representing taxes paid to the U.S. Government on income derived wholly
from Philippine sources. On the strength thereof, respondents seek refund of
P90 520.75 as overpayment. The Tax Court again decided for respondents.
The common issue in all three cases, and one that is of first impression in this
jurisdiction, is whether a citizen of the United States residing in the
Philippines, who derives income wholly from sources within the Republic of
the Philippines, may deduct from his gross income the income taxes he has
paid to the United States government for the taxable year on the strength of
section 30 (C-1) of the Philippine Internal Revenue Code, reading as follows:
SEC. 30. Deduction from gross income. In computing net income there
shall be allowed as deductions
(a) ...
(b) ...
(c) Taxes:
(1) In general. Taxes paid or accrued within the taxable year, except
(A) The income tax provided for under this Title;
(B) Income, war-profits, and excess profits taxes imposed by the authority of
any foreign country; but this deduction shall be allowed in the case of a
taxpayer who does not signify in his return his desire to have to any extent
the benefits of paragraph (3) of this subsection (relating to credit for foreign
countries);
(C) Estate, inheritance and gift taxes; and
(D) Taxes assessed against local benefits of a kind tending to increase the
value of the property assessed. (Emphasis supplied)
The Tax Court held that they may be deducted because of the undenied fact
that the respondent spouses did not "signify" in their income tax return a
desire to avail themselves of the benefits of paragraph 3 (B) of the
subsection, which reads:
Par. (c) (3) Credits against tax for taxes of foreign countries. If the taxpayer
signifies in his return his desire to have the benefits of this paragraph, the tax
imposed by this Title shall be credited with
(A) ...;
(B) Alien resident of the Philippines. In the case of an alien resident of the
Philippines, the amount of any such taxes paid or accrued during the taxable
year to any foreign country, if the foreign country of which such alien resident
is a citizen or subject, in imposing such taxes, allows a similar credit to
citizens of the Philippines residing in such country;
It is well to note that the tax credit so authorized is limited under paragraph 4
(A and B) of the same subsection, in the following terms:
Par. (c) (4) Limitation on credit. The amount of the credit taken under this
section shall be subject to each of the following limitations:
(A) The amount of the credit in respect to the tax paid or accrued to any
country shall not exceed the same proportion of the tax against which such
credit is taken, which the taxpayer's net income from sources within such
country taxable under this Title bears to his entire net income for the same
taxable year; and
(B) The total amount of the credit shall not exceed the same proportion of the
tax against which such credit is taken, which the taxpayer's net income from
sources without the Philippines taxable under this Title bears to his entire net
income for the same taxable year.
We agree with appellant Commissioner that the Construction and wording of
Section 30 (c) (1) (B) of the Internal Revenue Act shows the law's intent that
the right to deduct income taxes paid to foreign government from the
taxpayer's gross income is given only as an alternative or substitute to his
right to claim a tax credit for such foreign income taxes under section 30 (c)
(3) and (4); so that unless the alien resident has a right to claim such tax
credit if he so chooses, he is precluded from deducting the foreign income
taxes from his gross income. For it is obvious that in prescribing that such
deduction shall be allowed in the case of a taxpayer who does not signify in
his return his desire to have to any extent the benefits of paragraph (3)
(relating to credits for taxes paid to foreign countries), the statute assumes
that the taxpayer in question also may signify his desire to claim a tax credit
and waive the deduction; otherwise, the foreign taxes would always be
deductible, and their mention in the list of non-deductible items in Section
30(c) might as well have been omitted, or at least expressly limited to taxes
on income from sources outside the Philippine Islands.
Had the law intended that foreign income taxes could be deducted from gross
income in any event, regardless of the taxpayer's right to claim a tax credit, it
is the latter right that should be conditioned upon the taxpayer's waiving the
deduction; in which Case the right to reduction under subsection (c-1-B)
would have been made absolute or unconditional (by omitting foreign taxes
from the enumeration of non-deductions), while the right to a tax credit under
subsection (c-3) would have been expressly conditioned upon the taxpayer's
not claiming any deduction under subsection (c-1). In other words, if the law
had been intended to operate as contended by the respondent taxpayers and
by the Court of Tax Appeals section 30 (subsection (c-1) instead of providing
as at present:
SEC. 30. Deduction from gross income. In computing net income there
shall be allowed as deductions
(a) ...
(b) ...
(c) Taxes:
(1) In general. Taxes paid or accrued within the taxable year, except
(A) The income tax provided for under this Title;
(B) Income, war-profits, and excess profits taxes imposed by the authority of
any foreign country; but this deduction shall be allowed in the case of a
taxpayer who does not signify in his return his desire to have to any extent
the benefits of paragraph (3) of this subsection (relating to credit for taxes of
foreign countries);
(C) Estate, inheritance and gift taxes; and
(D) Taxes assessed against local benefits of a kind tending to increase the
value of the property assessed.
would have merely provided:
SEC. 30. Decision from grow income. In computing net income there shall
be allowed as deductions:
(a) ...
(b) ...
(c) Taxes paid or accrued within the taxable year, EXCEPT
(A) The income tax provided for in this Title;
(B) Omitted or else worded as follows:
Income, war profits and excess profits taxes imposed by authority of any
foreign country on income earned within the Philippines if the taxpayer does
not claim the benefits under paragraph 3 of this subsection;
(C) Estate, inheritance or gift taxes;
(D) Taxes assessed against local benefits of a kind tending to increase the
value of the property assessed.
while subsection (c-3) would have been made conditional in the following or
equivalent terms:
(3) Credits against tax for taxes of foreign countries. If the taxpayer has
not deducted such taxes from his gross income but signifies in his return his
desire to have the benefits of this paragraph, the tax imposed by Title shall be
credited with ... (etc.).
Petitioners admit in their brief that the purpose of the law is to prevent the
taxpayer from claiming twice the benefits of his payment of foreign taxes, by
deduction from gross income (subs. c-1) and by tax credit (subs. c-3). This
danger of double credit certainly can not exist if the taxpayer can not claim
benefit under either of these headings at his option, so that he must be
entitled to a tax credit (respondent taxpayers admittedly are not so entitled
because all their income is derived from Philippine sources), or the option to
deduct from gross income disappears altogether.
Much stress is laid on the thesis that if the respondent taxpayers are not
allowed to deduct the income taxes they are required to pay to the
government of the United States in their return for Philippine income tax, they
would be subjected to double taxation. What respondents fail to observe is
that double taxation becomes obnoxious only where the taxpayer is taxed
twice for the benefit of the same governmental entity (cf. Manila vs.
Interisland Gas Service, 52 Off. Gaz. 6579; Manuf. Life Ins. Co. vs. Meer, 89
Phil. 357). In the present case, while the taxpayers would have to pay two
taxes on the same income, the Philippine government only receives the
proceeds of one tax. As between the Philippines, where the income was
earned and where the taxpayer is domiciled, and the United States, where
that income was not earned and where the taxpayer did not reside, it is
indisputable that justice and equity demand that the tax on the income should
accrue to the benefit of the Philippines. Any relief from the alleged double
taxation should come from the United States, and not from the Philippines,
since the former's right to burden the taxpayer is solely predicated on his
citizenship, without contributing to the production of the wealth that is being
taxed.
Aside from not conforming to the fundamental doctrine of income taxation that
the right of a government to tax income emanates from its partnership in the
production of income, by providing the protection, resources, incentive, and
proper climate for such production, the interpretation given by the
respondents to the revenue law provision in question operates, in its
application, to place a resident alien with only domestic sources of income in
an equal, if not in a better, position than one who has both domestic and
foreign sources of income, a situation which is manifestly unfair and short of
logic.
Finally, to allow an alien resident to deduct from his gross income whatever
taxes he pays to his own government amounts to conferring on the latter the
power to reduce the tax income of the Philippine government simply by
increasing the tax rates on the alien resident. Everytime the rate of taxation
imposed upon an alien resident is increased by his own government, his
deduction from Philippine taxes would correspondingly increase, and the
proceeds for the Philippines diminished, thereby subordinating our own taxes
to those levied by a foreign government. Such a result is incompatible with
the status of the Philippines as an independent and sovereign state.
IN VIEW OF THE FOREGOING, the decisions of the Court of Tax Appeals
are reversed, and, the disallowance of the refunds claimed by the
respondents Lednicky is affirmed, with costs against said respondents-
appellees.

G.R. No. L-19537 May 20, 1965


The late LINO GUTIERREZ substituted by ANDREA C. VDA. DE
GUTIERREZ, ANTONIO D. GUTIERREZ, GUILLERMO D. GUTIERREZ,
SANTIAGO D. GUTIERREZ and TOMAS D. GUTIERREZ,petitioners,
vs.
COLLECTOR (now COMMISSIONER) OF INTERNAL REVENUE,
respondent.
Rosendo J. Tansinsin, Sr., Rosendo Tansinsin, Jr. and Juan C. Nabong, Jr.for
petitioners.
Office of the Solicitor General for respondent.
BENGZON, J.P., J.:
Lino Gutierrez was primarily engaged in the business of leasing real property
for which he paid estate broker's privilege tax. He filed his income tax returns
for the years 1951, 1952, 1953 and 1954 on the following dates:
Year
Date Filed
1951
March 1, 1952
1952
February 28, 1953
1953
February 22, 1954
1954
February 23, 1955
and paid the corresponding tax declared therein.
On July 10, 1956 the Commissioner (formerly Collector) of Internal Revenue
assessed against Gutierrez the following defiency income tax:
1951 . . . . . . . . . . . . . .
P 1,400.00
1952 . . . . . . . . . . . . . .
672.00
1953 . . . . . . . . . . . . . .
5,161.00
1954 . . . . . . . . . . . . . .
4,608.00
Total . . . . . . . . . . . . . .

P 11,841.00
==========
The above defiency tax came about by the disallowance of deductions from
gross income representing depreciation, expenses Gutierrez allegely incurred
in carrying on his business, and the addition to gross income of receipts
which he did not report in his income tax returns. The disallowed business
expenses which were considered by the Commissioner either as personal or
capital expenditures consisted of:
1951
Personal expenses:
Transportation expenses to attend funeral of various persons
P 96.50
Repair of car and salary of driver
59.80
Expenses in attending National Convention of Filipino Businessmen in Baguio
121.35
Alms to indigent family
15.00
Capital expenditures:
Electrical fixtures and supplies
100.00
Transportation and other expenses to watch laborers in construction work
516.00
Realty tax not paid by former owner of property acquired by Gutierrez
350.00
Litigation expenses to collect rental and eject lessee
702.65
Other disallowed deductions:
Fines and penalties for late payment of taxes
64.48
1952
Personal expenses:
Car expenses, salary of driver and car depreciation
P1,454.37
Contribution to Lydia Samson and G. Trinidad
52.00
Officers' jewels and aprons donated to Biak-na-Bato Lodge No. 7, Free
Masons
280.00
Luncheon of Homeowners' Association
5.50
Ticket to opera "Aida"
15.00
1953
Personal expenses:
Car expenses, salary of driver, car depreciation
P 1,409.24
Cruise to Corregidor with Homeowners' Association
43.00
Contribution to alms to various individuals
70.00
Tickets to operas
28.00
Capital expenditures:
Cost of one set of Comments on the Rules of Court by Moran
P 145.00
1954
Personal expenses:
Car expenses, salary of driver and car depreciation
P 1,413.67
Furniture given as commission in connection with business transaction
115.00
Cost of iron door of Gutierrez' residence
55.00
Capital expenditures:
Painting of rental apartments
P 908.00
Carpentry and lumber for rental apartments
335.83
Tinsmith and plumbing for rental apartments
605.25
Cement, tiles, gravel, sand and masonry for rental apartments
199.48
Iron bars, venetian blind, water pumps for rental apartments
1,340.00
Relocation and registration of property used in taxpayer's business
1,758.12
He also claimed the depreciation of his residence as follows:
1952 . . . . . .
P 992.22
1953 . . . . . .
942.61
1954 . . . . . .
895.45
The following are the items of income which Gutierrez did not declare in his
income tax returns:
1951
Income of wife (admitted by Gutierrez)
P 2,749.90.
1953
Overstatement of purchase price of real estate
P 8,476.92
Understatement of profits from sale of real estate
5,803.74
1954
Understatement of profits from sale of real estate
P 5,444.24
The overstatement of purchase price of real estate refers to the sale of two
pieces of property in 1953. In 1943 Gutierrez bought a parcel of land situated
along Padre Faura St. in Manila for P35,000.00. Sometime in 1953, he sold
the same for P30,400.00. Expenses of sale amounted to P631.80. In his
return he claimed a loss of P5,231.80. 1 However, the Commissioner,
including the said property was bought in Japanese military notes, converting
the buying price to its equivalent in PhilippineCommonwealth peso by the use
of the Ballantyne Scale of Values. At P1.30 Japanese military notes per
Commonwealth peso, the acquisition cost of P35,000.00 Japanese military
notes was valued at P26,923.00 PhilippineCommonwealth peso. Accordingly,
the Commissioner determined a profit of P3,476.92 after restoring to
Gutierrez' gross income the P5,231.80 deductionfor loss.
In another transaction, Gutierrez sold a piece of land for P1,200.00. Alleging
the said property was purchased for P1,200.00, he reported no profit
hereunder. However, after verifying the deed of acquisition, the Commissioner
discovered the purchase price to be only P800.00. Consequently, he
determined a profit of P400.00 which was added to the gross income for
1953.1wph1.t

The understatement of profit from the sale of real estate may be explained
thus: In 1953 and 1954 Gutierrez sold four other properties upon which he
made substantial profits.2Convinced that said properties were capital assets,
he declared only 50% of the profits from their sale. However, treating said
properties as ordinary assets (as property held and used byGutierrez in his
business), the Commissioner taxed 100% of the profits from their disposition
pursuant to Section 35 of the Tax Code.
Having unsuccessfully questioned the legality and correctness of the
aforesaid assessment, Gutierrez instituted on February 17, 1958, the
Commissioner issued a warrant of distraint and levy on one of Gutierrez' real
properties but desisted from enforcing the same when Gutierrez filed a bond
to assure payment of his tax liability.
In a decision dated January 28, 1962, the Court of Tax Appeals upheld in toto
the assessment of the Commissioner of Internal Revenue. Hence, this
appeal.
On October 18, 1962, Lino Guttierrez died and he was substituted by Andrea
C. Vda. de Gutierrez, Antonio D. Gutierrez, Santiago D. Gutierrez, Guillermo
D. Gutierrez and Tomas D. Gutierrez, his heirs,as party petitioners.
The issues are: (1) Are the taxpayer's aforementioned claims for deduction
proper and allowable? (2) May the Ballantyne Scale of Values be applied
indetermining the acquisition cost in 1943 of a real property sold in 1953, for
income tax purposes? (3) Are real properties used in the trade or business of
the taxpayer capital or ordinary assets? (4) Has the right of the Commissioner
of Internal Revenue to collect the deficiency income tax for the years 1951
and 1952 prescribed? (5) Has the right of the Commissioner of Internal
Revenue to collect by distraint and levy the deficiency income tax for 1953
prescribed? If not, may the taxpayer's rea lproperty be distrained and levied
upon without first exhausting his personal property?
We come first to question whether or not the deductions claimed by Gutierrez
are allowable. Section 30(a) of the Tax Code allows business expenses tobe
deducted from gross income. We quote:
SEC. 30. Deductions from gross income. In computing net income there
shall be allowed as deductions
(a) Expenses:
(1) In general. All the ordinary and necessary expenses paid or incurred
during the taxable year in carrying on any trade or business, including a
reasonable allowance for salaries or other compensation for personal
services actually rendered; travelling expenses while away from home in the
pursuit of a trade or business; and rentals or other payments required to be
made as a condition to be continued use of possession, for the purposes of
the trade or business, or property to which the taxpayer has not taken or is
not taking title or in which he has no equity.
To be deductible, therefore, an expense must be (1) ordinary and necessary;
(2) paid or incurred within the taxable year; and, (3) paid or incurred in
carrying on a trade or business. 3
The transportation expenses which petitioner incurred to attend the funeral of
his friends and the cost of admission tickets to operas were expenses relative
to his personal and social activities rather than to his business of leasing real
estate. Likewise, the procurement and installation of an iron door to is
residence is purely a personal expense. Personal, living, or family expenses
are not deductible. 4
On the other hand, the cost of furniture given by the taxpayer as commission
in furtherance of a business transaction, the expenses incurred in attending
the National Convention of Filipino Businessmen, luncheon meeting and
cruise to Corregidor of the Homeowners' Association were shown to have
been made in the pursuit of his business. Commissions given in consideration
for bringing about a profitable transaction are part of the cost of the business
transaction and are deductible.
The record shows that Gutierrez was an officer of the Junior Chamber of
Commerce which sponsored the National Convention of Filipino
Businessmen. He was also the president of the Homeowners' Association, an
organization established by those engaged in the real estate trade. Having
proved that his membership thereof and activities in connection therewith
were solely to enhance his business, the expenses incurred thereunder are
deductible as ordinary and necessary business expenses.
With respect to the taxpayer's claim for deduction for car expenses, salary of
his driver and car depreciation, one-third of the same was disallowed by the
Commissioner on the ground that the taxpayer used his car and driver both
for personal and business purposes. There is no clear showing, however, that
the car was devoted more for the taxpayer's business than for his personal
and business needs. 5 According to the evidence, the taxpayer's car was
utilized both for personal and business needs. We therefore find it reasonable
to allow as deduction one-half of the driver's salary, car expenses and
depreciation.
The electrical supplies, paint, lumber, plumbing, cement, tiles, gravel,
masonry and labor used to repair the taxpayer's rental apartments did not
increase the value of such apartments, or prolong their life. They merely kept
the apartments in an ordinary operating condition. Hence, the expenses
incurred therefor are deductible as necessary expenditures for the
maintenance of the taxpayer's business.
Similarly, the litigation expenses defrayed by Gutierrez to collect apartment
rentals and to eject delinquent tenants are ordinary and necessary expenses
in pursuing his business. It is routinary and necessary for one in the leasing
business to collect rentals and to eject tenants who refuse to pay their
accounts.
The following are not deductible business expenses but should be integrated
into the cost of the capital assets for which they were incurred and
depreciated yearly: (1) Expenses in watching over laborers in construction
work. Watching over laborers is an activity more akin to the construction work
than to running the taxpayer's business. Hence, the expenses incurred
therefor should form part of the construction cost. (2) Real estate tax which
remained unpaid by the former owner of Gutierrez' rental property but which
the latter paid, is an additional cost to acquire such property and ought
therefore to be treated as part of the property's purchase price. (3) The iron
bars, venetian blind and water pump augmented the value of the, apartments
where they were installed. Their cost is not a maintenance charge, 6 hence,
not deductible.. 7 (4) Expenses for the relocation, survey and registration of
property tend to strengthen title over the property, hence, they should be
considered as addition to the costs of such property. (5) The set of
"Comments on the Rules of Court" having a life span of more than one year
should be depreciated ratably during its whole life span instead of its total
cost being deducted in one year.
Coming to the claim for depreciation of Gutierrez' residence, we find the
same not deductible. A taxpayer may deduct from gross income a reasonable
allowance for deterioration of property arising out of its use or employment in
business or trade. 8 Gutierrez' residence was not used in his trade or business.
Gutierrez also claimed for deduction the fines and penalties which he paid for
late payment of taxes. While Section 30 allows taxes to be deducted from
gross income, it does not specifically allow fines and penalties to be so
deducted. Deductions from gross income are matters of legislative grace;
what is not expressly granted by Congress is withheld. Moreover, when acts
are condemned, by law and their commission is made punishable by fines or
forfeitures, to allow them to be deducted from the wrongdoer's gross income,
reduces, and so in part defeats, the prescribed punishment. .9
As regards the alms to an indigent family and various individuals,
contributions to Lydia Yamson and G. Trinidad and a donation consisting of
officers' jewels and aprons to Biak-na-Bato Lodge No. 7, the same are not
deductible from gross income inasmuch as their recipients have not been
shown to be among those specified by law. Contributions are deductible when
given to the Government of the Philippines, or any of its political subdivisions
for exclusively public purposes, to domestic corporations or associations
organized and operated exclusively for religious, charitable, scientific, athletic,
cultural or educational purposes, or for the rehabilitation of veterans, or to
societies for the prevention of cruelty to children or animals, no part of the net
income of which inures to the benefit of any private stockholder or individual.
10

We come to the question of whether or not the Ballantyne Scale of Values


can be applied to tax cases.
Sometime in 1943 Gutierrez bought a piece of real estate in Manila for a price
of P35,000.00. In 1953 he sold said property for P30,400.00, thereby
incurring a loss which he claimed as deduction in his income tax return for
1953. The Commissioner of Internal Revenue, convinced that the purchase
price of the property in 1943 was in Japanese military notes, converted said
purchase price into Philippine Commonwealth pesos by the use of the
Ballantyne Scale of Values. As a result, the Commissioner found Gutierrez to
have profited, instead of lost in the sale.
Firstly, Gutierrez maintains that the purchase price was paid for in
Commonwealth pesos. On the other hand the Commissioner insists that
inasmuch as the prevailing currency in the City of Manila in 1943 was the
Japanese military issue, the transaction could have been in said military
notes. The evidence offered by Gutierrez, consisting of the testimony of his
son to the effect that it was he who carried the bundle of Commonwealth
pesos and Japanese military notes when his father purchased the property,
did not convince the Tax Court. No cogent reason to alter the court a quo's
finding of fact in this regard has been given. There is no definite showing that
Gutierrez paid for the property in Commonwealth pesos. Considering that in
1943 the medium of exchange in Manila was the Japanese military notes, the
use of which the Japanese Military Government enforced with stringent
measures, we are inclined to concur with the finding that the purchase price
was in Japanese military notes. We are specifically mindful of the fact that
Gutierrez sold the property in 1953 for only P30,400.00 at a time when the
price of real estate in the City of Manila was much greater than in 1943.
It is further contended by Gutierrez that the money he used to pay for the
purchase of the property in question came from the proceeds of merchandise
acquired prior to World War II but which he sold after Manila was occupied by
the Japanese military forces, hence, the purchase price should be deemed to
have been made in Commonwealth pesos inasmuch as the aforesaid
merchandise was purchased in Commonwealth pesos. This contention, if
true, strengthens our conclusion that the real estate in question was bought in
Japanese military notes. For, at the time Gutierrez sold his merchandise, the
prevailing currency in the City of Manila was the Japanese military money.
Consequently, the proceeds therefrom, which were used to buy the real
estate in question, were Japanese military notes.
Gutierrez assails the use of the Ballantyne Scale of Values in converting the
purchase price of the real estate in question from Japanese military notes to
Philippine Commonwealth pesos on the ground that (1) the Ballantyne Scale
of Values was intended only for transactions entered into by parties
voluntarily during the Japanese occupation, wherein a portion of the contract
was left unperformed until liberation of the Philippines by the Americans; (2)
that such Scale of Values cannot be the basis of a tax, for it is not a law.
In determining the gain or loss from the sale of property the purchase price
and the selling price ought to be in the same currency. Since in this case the
purchase price was in Japanese military notes and the selling price was in
our present legal tender, the Japanese military notes should be converted to
the present currency. Since the only standard scale recognized by courts for
the purpose is the Ballantyne Scale of Values, we find it compelling to use
such table of values rather than adopt an arbitrary scale. It may not be amiss
to state in this connection that the Ballantyne Scale of Values is not being
used herein as the authority to impose the tax, but only as a medium of
computing the tax base upon which the tax is to be imposed.
It is furthermore proffered by the taxpayer that in determining gain or loss, the
real value of the Commonwealth peso at the time the property was purchased
and the value of the Republic peso at the time. the same property was sold
should be considered. The Commonwealth peso and the Republic peso are
the same currency, with the same intrinsic value, sanctioned by the same
authorities. Both are legal tender and accepted at face value regardless of
fluctuation in their buying power. The 1941 Commonwealth peso when used
to buy in 1963 or in 1965 is accorded the same value: one peso.
In his income tax returns for 1953 and 1954, Gutierrez reported only 50% of
profits he realized from the sale of real properties during the years 1953 and
1954 on the ground that said properties were capital assets. Profits from the
sale of capital assets are taxable to the extent of 50% thereof pursuant to
Section 34 of the Tax Code.
Section 34 provides:
SEC. 34. Capital gains and losses. (a) Definitions. As used in this title

(1) Capital assets. The term "capital assets" means property held by the
taxpayer (whether or not connected with his trade or business), but does not
include stock in trade of the taxpayer or other property of a kind which would
properly be included in the inventory of the taxpayer if on hand at the close of
the taxable year, or property held by the taxpayer primarily for sale to
customers in the ordinary course of his trade or business, or property used in
the trade or business, of a character which is subject to the allowance for
depreciation provided in subsection (f) of section thirty; or real property used
in the trade or business of the taxpayer.
xxx xxx xxx
(b) Percentage taken into account. In the case of a taxpayer, other than a
corporation, only the following percentages of the gain or loss recognized
upon the sale or exchange of capital asset hall be taken into account in
computing net capital gain, net capital loss, and net income:
(1) One hundred per centum if the capital asset has been held for not more
than twelve months;
(2) Fifty per centum if the capital asset has been held for not more than
twelve months.
Section 34, before it was amended by Republic Act 82 in 1947, considered as
capital assets real property used in the trade or business of a taxpayer.
However, with the passage of Republic Act 82, Congress classified "real
property used in the trade or business of the taxpayer" is ordinary asset. The
explanatory note to Republic Act 82 says "... the words "or real property
used in the trade or business of the taxpayer" have been included among the
non-capital assets. This has the effect of withdrawing the gain or loss from the
sale or exchange of real property used in the trade or business of the
taxpayer from the operation of the capital gains and losses provisions. As
such real property is used in the trade or business of the taxpayer, it is logical
that the gain or loss from the sale or exchange thereof should be treated as
ordinary income or loss. 11 Accordingly, the real estate, admittedly used by
Gutierrez in his business, which he sold in 1953 and 1954 should be treated
as ordinary assets and the gain from the sale thereof, as ordinary gain,
hence, fully taxable. 12
With regard to the issue of the prescription of the Commissioner's right to
collect deficiency tax for 1951 and 1952, Gutierrez claims that the counting of
the 5-year period to collect income tax should start from the time the income
tax returns were filed. He, therefore, urges us to declare the Commissioner's
right to collect the deficiency tax for 1951 and 1952 to have prescribed, the
income tax returns for 1951 and 1952 having been filed in March 1952 and on
February 28, 1953, respectively, and the action to collect the tax having been
instituted on March 5, 1958 when the Commissioner filed his answer to the
petition for review in C.T.A. Case No. 504. On the other hand, the
Commissioner argues that the running of the prescriptive period to collect
commences from the time of assessment. Inasmuch as the tax for 1951 and
1952 were assessed only on July 10, 1956, less than five years lapsed when
he filed his answer on March 5, 1958.
The period of limitation to collect income tax is counted from the assessment
of the tax as provided for in paragraph (c) of Section 332 quoted below:
SEC. 332(c). Where the assessment of any internal revenue tax has been
made within the period of limitation above prescribed such tax may be
collected by distraint or levy or by a proceeding in court, but only if begun (1)
within five years after the assessment of the tax, or (2) prior to the expiration
of any period for collection agreed upon in writing by the Collector of Internal
Revenue and the taxpayer before the expiration of such five-year period. The
period so agreed upon may be extended by subsequent agreements in
writing made before the expiration of the period previously agreed upon.
Inasmuch as the assessment for deficiency income tax was made on July 10,
1956 which is 7 months and 25 days prior to the action for collection, the right
of the Commissioner to collect such tax has not prescribed.
The next issue relates to the prescription of the right of the Commissioner of
Internal Revenue to collect the deficiency tax for 1954 by distraint and levy.
The pertinent provision of the Tax Code states:
SEC. 51(d). Refusal or neglect to make returns; fraudulent returns, etc. In
cases of refusal or neglect to make a return and in cases of erroneous, false,
or fraudulent returns, the Collector of Internal Revenue shall, upon the
discovery thereof, at any time within three years after said return is due or
has been made, make a return upon information obtained as provided for in
this code or by existing law, or require the necessary corrections to be made,
and the assessment made by the Collector of Internal Revenue thereon shall
be paid by such person or corporation immediately upon notification of the
amount of such assessment.
On February 23, 1955 Gutierrez filed his income tax return for 1954 and on
February 24, 1958 the Commissioner of Internal Revenue issued a warrant of
distraint and levy to collect the tax due thereunder. Gutierrez contends that
the Commissioner's right to issue said warrant is barred, for the same was
issued more than 3 years from the time he filed his income tax return. On the
other hand, the Commissioner of Internal Revenue maintains that his right did
not lapse inasmuch as from the last day prescribed by law for the filing of the
1954 return to the date when he issued the warrant of distraint and levy, less
than 3 years passed. The question now is: should the counting of the
prescriptive period commence from the actual filing of the return or from the
last day prescribed by law for the filing thereof?
We observe that Section 51(d) speaks of erroneous, false or fraudulent
returns, and refusal or neglect of the taxpayer to file a return. It also provides
for two dates from which to count the three-year prescriptive period, namely,
the date when the return is due and the date the return has been made. We
are inclined to conclude that the date when the return is due refers to cases
where the taxpayer refused or neglected to file a return, and the date when
the return has been made refers to instances where the taxpayer filed
erroneous, false or fraudulent returns. Since Gutierrez filed an income tax
return, the three-year prescriptive period should be counted from the time he
filed such return. From February 23, 1955 when the income tax return for
1954 was filed, to February 24, 1958, when the warrant of distraint and levy
was issued, 3 years and 2 days elapsed. The right of the Commissioner to
issue said warrant of distraint and levy having lapsed by two days, the
warrant issued is null and void.
The above finding has made academic the question of whether or not the
warrant of distraint and levy can be enforced against the taxpayer's real
property without first exhausting his personal properties.
In resume the tax liability of Lino Gutierrez for 1951, 1952, 1953 and 1954
may be computed as follows:

1951
Net income per investigation
P29,471.81
Add: Disallowed deductions for salary of
driver and car expenses
29.90

P29,501.81
Less: Allowable deductions:
Expenses in attending National
Convention of Filipino Businessmen
P 121.35
Repair of rental apartments
802.65
924.00
Net income

P30,425.71
Less: Personal exemption
3,600.00
Amount subject to tax

P26,825.71

Tax due thereon


P 5,668.00
Less tax already paid
3,981.00
Deficiency income tax due

P 1,687.00
==========
1952
Net income per investigation
P21,632.22
Add: Disallowed deductions:
Salary of driver
P 260.67
Car expenses
401.51
Car depreciation
65.00
727.18

P22,359.40
Less Allowable deduction:
Luncheon, Homeowners' Association
5.50
Net income

P22,364.90
Less: Personal exemption
3,600.00
Amount subject to tax

P18,764.90

Tax due thereon


P 3,324.00
Less tax already paid
2,476.00
Deficiency income tax due

848.00
==========
1953
Net income per investigation
P69,180.91
Add: Disallowed deductions:
Salary of driver
P 140.00
Car expenses
406.00
Car depreciation
58.50
604.50
P69,785.40
Less: Allowable deduction:
Cruise to Corregidor with Homeowners'
Association
42.00
Net Income

P69,828 40
Less: Personal exemption
3,600.00
Amount subject to tax

P66,228.40

Tax due thereon


P15,179.00
Less tax already paid
9,805.00
Deficiency income tax due

P 5,374.00
==========
1954
Net income per investigation
P43,881.92
Add: Disallowed deductions:
Salary of driver
P 140.00
Car expenses
414.18
Car depreciation
72.65
626.83

P44,508.75
Less: Allowable deductions:
Furniture given in connection with business transaction
P 115.00
Repairs of rental apartments
2,048.56
2,163.56
Net income
P42,345.19
Less: Personal exemption
3,000.00
Amount subject to tax

P39,345.19

Tax due thereon


P 9,984.00
Less tax already paid
5,964.00
Deficiency income tax due

P 4,020.00
==========
SUMMARY
1951 . . . . . . . . . . . . . . . .
P 1,687.00
1952 . . . . . . . . . . . . . . . .
848.00
1953 . . . . . . . . . . . . . . . .
5,374.00
1954 . . . . . . . . . . . . . . . .
4,020.00
TOTAL . . . . . . . . . .

P 11,929.00
=========
WHEREFORE, the decision appealed from is modified and Lino Gutierrez
and/or his heirs, namely, Andrea C. Vda. de Gutierrez, Antonio D. Gutierrez,
Santiago D. Gutierrez, Guillermo D. Gutierrez and Tomas D. Gutierrez, are
ordered to pay the sums of P1,687.00, P848.00, P5,374.00, and P4,020.00,
as deficiency income tax for the years 1951, 1952, 1953 and 1954,
respectively, or a total of P11,929.00, plus the statutory penalties in case of
delinquency. No costs. So ordered.

G.R. No. L-21520 December 11, 1967


PLARIDEL SURETY and INSURANCE COMPANY, petitioner,
vs.
COMMISSIONER OF INTERNAL REVENUE, respondent.
Gil R. Carlos and Associates for petitioner.
Office of the Solicitor General for respondent.
BENGZON, J.P., J.:
Petitioner Plaridel Surety & Insurance Co., is a domestic corporation engaged
in the bonding business. On November 9, 1950, petitioner, as surety, and
Constancio San Jose, as principal, solidarily executed a performance bond in
the penal sum of P30,600.00 in favor of the P. L. Galang Machinery Co., Inc.,
to secure the performance of San Jose's contractual obligation to produce
and supply logs to the latter.
To afford itself adequate protection against loss or damage on the
performance bond, petitioner required San Jose and one Ramon Cuervo to
execute an indemnity agreement obligating themselves, solidarily, to
indemnify petitioner for whatever liability it may incur by reason of said
performance bond. Accordingly, San Jose constituted a chattel mortgage on
logging machineries and other movables in petitioner's favor 1 while Ramon
Cuervo executed a real estate mortgage.2
San Jose later failed to deliver the logs to Galang Machinery 3 and the latter
sued on the performance bond. On October 1, 1952, the Court of First
Instance adjudged San Jose and petitioner liable; it also directed San Jose
and Cuervo to reimburse petitioner for whatever amount it would pay Galang
Machinery. The Court of Appeals, on June 17, 1955, affirmed the judgment of
the lower court. The same judgment was likewise affirmed by this Court 4 on
January 11, 1957 except for a slight modification apropos the award of
attorney's fees.
On February 19 and March 20, 1957, petitioner effected payment in favor of
Galang Machinery in the total sum of P44,490.00 pursuant to the final
decision.
In its income tax return for the year 1957, petitioner claimed the said amount
of P44,490.00 as deductible loss from its gross income and, accordingly, paid
the amount of P136.00 as its income tax for 1957.
The Commissioner of Internal Revenue disallowed the claimed deduction of
P44,490.00 and assessed against petitioner the sum of P8,898.00, plus
interest, as deficiency income tax for the year 1957. Petitioner filed its protest
which was denied. Whereupon, appeal was taken to the Tax Court, petitioner
insisting that the P44,490.00 which it paid to Galang Machinery was a
deductible loss.
The Tax Court dismissed the appeal, ruling that petitioner was duly
compensated for otherwise than by insurance thru the mortgages in its
favor executed by San Jose and Cuervo and it had not yet exhausted all
its available remedies, especially as against Cuervo, to minimize its loss.
When its motion to reconsider was denied, petitioner elevated the present
appeal.
Of the sum of P44,490.00, the amount of P30,600.00 which is the principal
sum stipulated in the performance bond is being claimed as loss deduction
under Sec. 30 (d) (2) of the Tax Code and P10,000.00 which is the interest
that had accrued on the principal sum is now being claimed as interest
deduction under Sec. 30 (b) (1).
Loss is deductible only in the taxable year it actually happens or is sustained.
However, if it is compensable by insurance or otherwise, deduction for the
loss suffered is postponed to a subsequent year, which, to be precise, is that
year in which it appears that no compensation at all can be had, or that there
is a remaining or net loss, i.e., no full compensation. 5
There is no question that the year in which the petitioner Insurance Co.
effected payment to Galang Machinery pursuant to a final decision occurred
in 1957. However, under the same court decision, San Jose and Cuervo were
obligated to reimburse petitioner for whatever payments it would make to
Galang Machinery. Clearly, petitioner's loss is compensable otherwise (than
by insurance). It should follow, then, that the loss deduction can not be
itc-alf

claimed in 1957.
Now, petitioner's submission is that its case is an exception. Citing Cu
Unjieng Sons, Inc. v. Board of Tax Appeals,6 and American cases also,
petitioner argues that even if there is a right to compensation by insurance or
otherwise, the deduction can be taken in the year of actual loss where the
possibility of recovery is remote. The pronouncement, however to this effect
in the Cu Unjieng case is not as authoritative as petitioner would have it since
it was there found that the taxpayer had no legal right to compensation either
by insurance or otherwise.7 And the American cases cited8 are not in point.
None of them involved a taxpayer who had, as in the present case, obtained
a final judgment against third persons for reimbursement of payments made.
In those cases, there was either no legally enforceable right at all or such
claimed right was still to be, or being, litigated.
On the other hand, the rule is that loss deduction will be denied if there is a
measurable right to compensation for the loss, with ultimate collection
reasonably clear. So where there is reasonable ground for reimbursement,
the taxpayer must seek his redress and may not secure a loss deduction until
he establishes that no recovery may be had.9 In other words, as the Tax Court
put it, the taxpayer (petitioner) must exhaust his remedies first to recover or
reduce his loss.
It is on record that petitioner had not exhausted its remedies, especially
against Ramon Cuervo who was solidarily liable with San Jose for
reimbursement to it. Upon being prodded by the Tax Court to go after Cuervo,
Hermogenes Dimaguiba, president of petitioner corporation, said that they
would10 but no evidence was submitted that anything was really done on the
matter. Moreover, petitioner's evidence on remote possibility of recovery is
fatally wanting. Its right to reimbursement is not only secured by the
mortgages executed by San Jose and Cuervo but also by a final and
executory judgment in the civil case itself. Thus, other properties of San Jose
and Cuervo were subject to levy and execution. But no writ of execution,
satisfied or unsatisfied, was ever submitted. Neither has it been established
that Cuervo was insolvent. The only evidence on record on the point is
Dimaguiba's testimony that he does not really know if Cuervo has other
properties.11 This is not substantial proof of insolvency. Thus, it was too
itc-alf

premature for petitioner to claim a loss deduction.


But assuming that there was no reasonable expectation of recovery, still no
loss deduction can be had. Sec. 30 (d) (2) of the Tax Code requires a charge-
off as one of the conditions for loss deduction:
In the case of a corporation, all losses actually sustained and charged-off
within the taxable year and not compensated for by insurance or otherwise.
(Emphasis supplied)
Mertens12 states only four (4) requisites because the United States Internal
Revenue Code of 193913 has no charge-off requirement. Sec. 23(f) thereof
itc-alf

provides merely:
In the case of a corporation, losses sustained during the taxable year and not
compensated for by insurance or otherwise.
Petitioner, who had the burden of proof 14 failed to adduce evidence that there
was a charge-off in connection with the P44,490.00or P30,600.00 which
it paid to Galang Machinery.
In connection with the claimed interest deduction of P10,000.00, the Solicitor
General correctly points out that this question was never raised before the
Tax Court. Petitioner, thru counsel, had admitted before said court 15 and in the
memorandum it filed16 that the only issue in the case was whether the entire
P44,490.00 paid by it was or was not a deductible loss under Sec. 30 (d) (2)
of the Tax Code. Even in petitioner's return, the P44,490.00 was claimed
wholly as losses on its bond.17 The alleged interest deduction not having been
properly litigated as an issue before the Tax Court, it is now too late to raise
and assert it before this Court.
WHEREFORE, the appealed decision is, as it is hereby, affirmed. Costs
against petitioner Plaridel Surety & Insurance Co. So ordered.

G.R. No. L-21551 September 30, 1969


FERNANDEZ HERMANOS, INC., petitioner,
vs.
COMMISSIONER OF INTERNAL REVENUE and COURT OF TAX
APPEALS, respondents.
-----------------------------
G.R. No. L-21557 September 30, 1969
COMMISSIONER OF INTERNAL REVENUE, petitioner,
vs.
FERNANDEZ HERMANOS, INC., and COURT OF TAX APPEALS,
respondents.
-----------------------------
G.R. No. L-24972 September 30, 1969
COMMISSIONER OF INTERNAL REVENUE, petitioner,
vs.
FERNANDEZ HERMANOS INC., and the COURT OF TAX APPEALS,
respondents.
-----------------------------
G.R. No. L-24978 September 30, 1969
FERNANDEZ HERMANOS, INC., petitioner,
vs.
THE COMMISSIONER OF INTERNAL REVENUE, and HON. ROMAN A.
UMALI, COURT OF TAX APPEALS, respondents.
L-21551:
Rafael Dinglasan for petitioner.
Office of the Solicitor General Arturo A. Alafriz, Solicitor Alejandro B. Afurong
and Special Attorney Virgilio G. Saldajeno for respondent.
L-21557:
Office of the Solicitor General for petitioner.
Rafael Dinglasan for respondent Fernandez Hermanos, Inc.
L-24972:
Office of the Solicitor General Antonio P. Barredo, Assistant Solicitor General
Felicisimo R. Rosete and Special Attorney Virgilio G. Saldajeno for petitioner.
Rafael Dinglasan for respondent Fernandez Hermanos, Inc.
L-24978:
Rafael Dinglasan for petitioner.
Office of the Solicitor General Antonio P. Barredo, Assistant Solicitor General
Antonio G. Ibarra and Special Attorney Virgilio G. Saldajeno for respondent.

TEEHANKEE, J.:
These four appears involve two decisions of the Court of Tax Appeals
determining the taxpayer's income tax liability for the years 1950 to 1954 and
for the year 1957. Both the taxpayer and the Commissioner of Internal
Revenue, as petitioner and respondent in the cases a quo respectively,
appealed from the Tax Court's decisions, insofar as their respective
contentions on particular tax items were therein resolved against them. Since
the issues raised are interrelated, the Court resolves the four appeals in this
joint decision.
Cases L-21551 and L-21557
The taxpayer, Fernandez Hermanos, Inc., is a domestic corporation
organized for the principal purpose of engaging in business as an "investment
company" with main office at Manila. Upon verification of the taxpayer's
income tax returns for the period in question, the Commissioner of Internal
Revenue assessed against the taxpayer the sums of P13,414.00,
P119,613.00, P11,698.00, P6,887.00 and P14,451.00 as alleged deficiency
income taxes for the years 1950, 1951, 1952, 1953 and 1954, respectively.
Said assessments were the result of alleged discrepancies found upon the
examination and verification of the taxpayer's income tax returns for the said
years, summarized by the Tax Court in its decision of June 10, 1963 in CTA
Case No. 787, as follows:
1. Losses
a. Losses in Mati Lumber Co. (1950) P 8,050.00
b. Losses in or bad debts of Palawan Manganese Mines, Inc. (1951)
353,134.25
c. Losses in Balamban Coal Mines
1950
8,989.76
1951
27,732.66
d. Losses in Hacienda Dalupiri
1950
17,418.95
1951
29,125.82
1952
26,744.81
1953
21,932.62
1954
42,938.56
e. Losses in Hacienda Samal
1951
8,380.25
1952
7,621.73
2. Excessive depreciation of Houses
1950
P 8,180.40
1951
8,768.11
1952
18,002.16
1953
13,655.25
1954
29,314.98
3. Taxable increase in net worth
1950
P 30,050.00
1951
1,382.85
4. Gain realized from sale of real property in 1950 P 11,147.2611
The Tax Court sustained the Commissioner's disallowances of Item 1, sub-
items (b) and (e) and Item 2 of the above summary, but overruled the
Commissioner's disallowances of all the remaining items. It therefore
modified the deficiency assessments accordingly, found the total deficiency
income taxes due from the taxpayer for the years under review to amount to
P123,436.00 instead of P166,063.00 as originally assessed by the
Commissioner, and rendered the following judgment:
RESUME
1950
P2,748.00
1951
108,724.00
1952
3,600.00
1953
2,501.00
1954
5,863.00
Total

P123,436.00
WHEREFORE, the decision appealed from is hereby modified, and petitioner
is ordered to pay the sum of P123,436.00 within 30 days from the date this
decision becomes final. If the said amount, or any part thereof, is not paid
within said period, there shall be added to the unpaid amount as surcharge of
5%, plus interest as provided in Section 51 of the National Internal Revenue
Code, as amended. With costs against petitioner. (Pp. 75, 76, Taxpayer's
Brief as appellant)
Both parties have appealed from the respective adverse rulings against them
in the Tax Court's decision. Two main issues are raised by the parties: first,
the correctness of the Tax Court's rulings with respect to the disputed items of
disallowances enumerated in the Tax Court's summary reproduced above,
and second, whether or not the government's right to collect the deficiency
income taxes in question has already prescribed.
On the first issue, we will discuss the disputed items of disallowances
seriatim.
1. Re allowances/disallowances of losses.
(a) Allowance of losses in Mati Lumber Co. (1950). The Commissioner of
Internal Revenue questions the Tax Court's allowance of the taxpayer's
writing off as worthless securities in its 1950 return the sum of P8,050.00
representing the cost of shares of stock of Mati Lumber Co. acquired by the
taxpayer on January 1, 1948, on the ground that the worthlessness of said
stock in the year 1950 had not been clearly established. The Commissioner
contends that although the said Company was no longer in operation in 1950,
it still had its sawmill and equipment which must be of considerable value.
The Court, however, found that "the company ceased operations in 1949
when its Manager and owner, a certain Mr. Rocamora, left for Spain ,where
he subsequently died. When the company eased to operate, it had no assets,
in other words, completely insolvent. This information as to the insolvency of
the Company reached (the taxpayer) in 1950," when it properly claimed
the loss as a deduction in its 1950 tax return, pursuant to Section 30(d) (4) (b)
or Section 30 (e) (3) of the National Internal Revenue Code. 2
We find no reason to disturb this finding of the Tax Court. There was
adequate basis for the writing off of the stock as worthless securities.
Assuming that the Company would later somehow realize some proceeds
from its sawmill and equipment, which were still existing as claimed by the
Commissioner, and that such proceeds would later be distributed to its
stockholders such as the taxpayer, the amount so received by the taxpayer
would then properly be reportable as income of the taxpayer in the year it is
received.
(b) Disallowance of losses in or bad debts of Palawan Manganese Mines,
Inc. (1951). The taxpayer appeals from the Tax Court's disallowance of its
writing off in 1951 as a loss or bad debt the sum of P353,134.25, which it had
advanced or loaned to Palawan Manganese Mines, Inc. The Tax Court's
findings on this item follow:
Sometime in 1945, Palawan Manganese Mines, Inc., the controlling
stockholders of which are also the controlling stockholders of petitioner
corporation, requested financial help from petitioner to enable it to resume it
mining operations in Coron, Palawan. The request for financial assistance
was readily and unanimously approved by the Board of Directors of petitioner,
and thereafter a memorandum agreement was executed on August 12, 1945,
embodying the terms and conditions under which the financial assistance was
to be extended, the pertinent provisions of which are as follows:
"WHEREAS, the FIRST PARTY, by virtue of its resolution adopted on August
10, 1945, has agreed to extend to the SECOND PARTY the requested
financial help by way of accommodation advances and for this purpose has
authorized its President, Mr. Ramon J. Fernandez to cause the release of
funds to the SECOND PARTY.
"WHEREAS, to compensate the FIRST PARTY for the advances that it has
agreed to extend to the SECOND PARTY, the latter has agreed to pay to the
former fifteen per centum (15%) of its net profits.
"NOW THEREFORE, for and in consideration of the above premises, the
parties hereto have agreed and covenanted that in consideration of the
financial help to be extended by the FIRST PARTY to the SECOND PARTY to
enable the latter to resume its mining operations in Coron, Palawan, the
SECOND PARTY has agreed and undertaken as it hereby agrees and
undertakes to pay to the FIRST PARTY fifteen per centum (15%) of its net
profits." (Exh. H-2)
Pursuant to the agreement mentioned above, petitioner gave to Palawan
Manganese Mines, Inc. yearly advances starting from 1945, which advances
amounted to P587,308.07 by the end of 1951. Despite these advances and
the resumption of operations by Palawan Manganese Mines, Inc., it continued
to suffer losses. By 1951, petitioner became convinced that those advances
could no longer be recovered. While it continued to give advances, it decided
to write off as worthless the sum of P353,134.25. This amount "was arrived at
on the basis of the total of advances made from 1945 to 1949 in the sum of
P438,981.39, from which amount the sum of P85,647.14 had to be deducted,
the latter sum representing its pre-war assets. (t.s.n., pp. 136-139, Id)." (Page
4, Memorandum for Petitioner.) Petitioner decided to maintain the advances
given in 1950 and 1951 in the hope that it might be able to recover the same,
as in fact it continued to give advances up to 1952. From these facts, and as
admitted by petitioner itself, Palawan Manganese Mines, Inc., was still in
operation when the advances corresponding to the years 1945 to 1949 were
written off the books of petitioner. Under the circumstances, was the sum of
P353,134.25 properly claimed by petitioner as deduction in its income tax
return for 1951, either as losses or bad debts?
It will be noted that in giving advances to Palawan Manganese Mine Inc.,
petitioner did not expect to be repaid. It is true that some testimonial evidence
was presented to show that there was some agreement that the advances
would be repaid, but no documentary evidence was presented to this effect.
The memorandum agreement signed by the parties appears to be very clear
that the consideration for the advances made by petitioner was 15% of the
net profits of Palawan Manganese Mines, Inc. In other words, if there were no
earnings or profits, there was no obligation to repay those advances. It has
been held that the voluntary advances made without expectation of
repayment do not result in deductible losses. 1955 PH Fed. Taxes, Par. 13,
329, citing W. F. Young, Inc. v. Comm., 120 F 2d. 159, 27 AFTR 395; George
B. Markle, 17 TC. 1593.
Is the said amount deductible as a bad debt? As already stated, petitioner
gave advances to Palawan Manganese Mines, Inc., without expectation of
repayment. Petitioner could not sue for recovery under the memorandum
agreement because the obligation of Palawan Manganese Mines, Inc. was to
pay petitioner 15% of its net profits, not the advances. No bad debt could
arise where there is no valid and subsisting debt.
Again, assuming that in this case there was a valid and subsisting debt and
that the debtor was incapable of paying the debt in 1951, when petitioner
wrote off the advances and deducted the amount in its return for said year,
yet the debt is not deductible in 1951 as a worthless debt. It appears that the
debtor was still in operation in 1951 and 1952, as petitioner continued to give
advances in those years. It has been held that if the debtor corporation,
although losing money or insolvent, was still operating at the end of the
taxable year, the debt is not considered worthless and therefore not
deductible. 3
The Tax Court's disallowance of the write-off was proper. The Solicitor
General has rightly pointed out that the taxpayer has taken an "ambiguous
position " and "has not definitely taken a stand on whether the amount
involved is claimed as losses or as bad debts but insists that it is either a loss
or a bad debt." 4 We sustain the government's position that the advances
made by the taxpayer to its 100% subsidiary, Palawan Manganese Mines,
Inc. amounting to P587,308,07 as of 1951 were investments and not loans. 5
The evidence on record shows that the board of directors of the two
companies since August, 1945, were identical and that the only capital of
Palawan Manganese Mines, Inc. is the amount of P100,000.00 entered in the
taxpayer's balance sheet as its investment in its subsidiary company. 6 This
fact explains the liberality with which the taxpayer made such large advances
to the subsidiary, despite the latter's admittedly poor financial condition.
The taxpayer's contention that its advances were loans to its subsidiary as
against the Tax Court's finding that under their memorandum agreement, the
taxpayer did not expect to be repaid, since if the subsidiary had no earnings,
there was no obligation to repay those advances, becomes immaterial, in the
light of our resolution of the question. The Tax Court correctly held that the
subsidiary company was still in operation in 1951 and 1952 and the taxpayer
continued to give it advances in those years, and, therefore, the alleged debt
or investment could not properly be considered worthless and deductible in
1951, as claimed by the taxpayer. Furthermore, neither under Section 30 (d)
(2) of our Tax Code providing for deduction by corporations of losses actually
sustained and charged off during the taxable year nor under Section 30 (e)
(1) thereof providing for deduction of bad debts actually ascertained to be
worthless and charged off within the taxable year, can there be a partial
writing off of a loss or bad debt, as was sought to be done here by the
taxpayer. For such losses or bad debts must be ascertained to be so and
written off during the taxable year, are therefore deductible in full or not at all,
in the absence of any express provision in the Tax Code authorizing partial
deductions.
The Tax Court held that the taxpayer's loss of its investment in its subsidiary
could not be deducted for the year 1951, as the subsidiary was still in
operation in 1951 and 1952. The taxpayer, on the other hand, claims that its
advances were irretrievably lost because of the staggering losses suffered by
its subsidiary in 1951 and that its advances after 1949 were "only limited to
the purpose of salvaging whatever ore was already available, and for the
purpose of paying the wages of the laborers who needed help." 7 The
correctness of the Tax Court's ruling in sustaining the disallowance of the
write-off in 1951 of the taxpayer's claimed losses is borne out by subsequent
events shown in Cases L-24972 and L-24978 involving the taxpayer's 1957
income tax liability. (Infra, paragraph 6.) It will there be seen that by 1956, the
obligation of the taxpayer's subsidiary to it had been reduced from
P587,398.97 in 1951 to P442,885.23 in 1956, and that it was only on January
1, 1956 that the subsidiary decided to cease operations. 8
(c) Disallowance of losses in Balamban Coal Mines (1950 and 1951). The
Court sustains the Tax Court's disallowance of the sums of P8,989.76 and
P27,732.66 spent by the taxpayer for the operation of its Balamban coal
mines in Cebu in 1950 and 1951, respectively, and claimed as losses in the
taxpayer's returns for said years. The Tax Court correctly held that the losses
"are deductible in 1952, when the mines were abandoned, and not in 1950
and 1951, when they were still in operation." 9 The taxpayer's claim that these
expeditions should be allowed as losses for the corresponding years that they
were incurred, because it made no sales of coal during said years, since the
promised road or outlet through which the coal could be transported from the
mines to the provincial road was not constructed, cannot be sustained. Some
definite event must fix the time when the loss is sustained, and here it was
the event of actual abandonment of the mines in 1952. The Tax Court held
that the losses, totalling P36,722.42 were properly deductible in 1952, but the
appealed judgment does not show that the taxpayer was credited therefor in
the determination of its tax liability for said year. This additional deduction of
P36,722.42 from the taxpayer's taxable income in 1952 would result in the
elimination of the deficiency tax liability for said year in the sum of P3,600.00
as determined by the Tax Court in the appealed judgment.
(d) and (e) Allowance of losses in Hacienda Dalupiri (1950 to 1954) and
Hacienda Samal (1951-1952). The Tax Court overruled the
Commissioner's disallowance of these items of losses thus:
Petitioner deducted losses in the operation of its Hacienda Dalupiri the sums
of P17,418.95 in 1950, P29,125.82 in 1951, P26,744.81 in 1952, P21,932.62
in 1953, and P42,938.56 in 1954. These deductions were disallowed by
respondent on the ground that the farm was operated solely for pleasure or
as a hobby and not for profit. This conclusion is based on the fact that the
farm was operated continuously at a loss. 1awphl.nt

From the evidence, we are convinced that the Hacienda Dalupiri was
operated by petitioner for business and not pleasure. It was mainly a cattle
farm, although a few race horses were also raised. It does not appear that the
farm was used by petitioner for entertainment, social activities, or other non-
business purposes. Therefore, it is entitled to deduct expenses and losses in
connection with the operation of said farm. (See 1955 PH Fed. Taxes, Par.
13, 63, citing G.C.M. 21103, CB 1939-1, p.164)
Section 100 of Revenue Regulations No. 2, otherwise known as the Income
Tax Regulations, authorizes farmers to determine their gross income on the
basis of inventories. Said regulations provide:
"If gross income is ascertained by inventories, no deduction can be made for
livestock or products lost during the year, whether purchased for resale,
produced on the farm, as such losses will be reflected in the inventory by
reducing the amount of livestock or products on hand at the close of the
year."
Evidently, petitioner determined its income or losses in the operation of said
farm on the basis of inventories. We quote from the memorandum of counsel
for petitioner:
"The Taxpayer deducted from its income tax returns for the years from 1950
to 1954 inclusive, the corresponding yearly losses sustained in the operation
of Hacienda Dalupiri, which losses represent the excess of its yearly
expenditures over the receipts; that is, the losses represent the difference
between the sales of livestock and the actual cash disbursements or
expenses." (Pages 21-22, Memorandum for Petitioner.)
As the Hacienda Dalupiri was operated by petitioner for business and since it
sustained losses in its operation, which losses were determined by means of
inventories authorized under Section 100 of Revenue Regulations No. 2, it
was error for respondent to have disallowed the deduction of said losses. The
same is true with respect to loss sustained in the operation of the Hacienda
Samal for the years 1951 and 1952. 10
The Commissioner questions that the losses sustained by the taxpayer were
properly based on the inventory method of accounting. He concedes,
however, "that the regulations referred to does not specify how the
inventories are to be made. The Tax Court, however, felt satisfied with the
evidence presented by the taxpayer ... which merely consisted of an alleged
physical count of the number of the livestock in Hacienda Dalupiri for the
years involved." 11 The Tax Court was satisfied with the method adopted by
the taxpayer as a farmer breeding livestock, reporting on the basis of receipts
and disbursements. We find no Compelling reason to disturb its findings.
2. Disallowance of excessive depreciation of buildings (1950-1954). During
the years 1950 to 1954, the taxpayer claimed a depreciation allowance for its
buildings at the annual rate of 10%. The Commissioner claimed that the
reasonable depreciation rate is only 3% per annum, and, hence, disallowed
as excessive the amount claimed as depreciation allowance in excess of 3%
annually. We sustain the Tax Court's finding that the taxpayer did not submit
adequate proof of the correctness of the taxpayer's claim that the depreciable
assets or buildings in question had a useful life only of 10 years so as to
justify its 10% depreciation per annum claim, such finding being supported by
the record. The taxpayer's contention that it has many zero or one-peso
assets, 12 representing very old and fully depreciated assets serves but to
support the Commissioner's position that a 10% annual depreciation rate was
excessive.
3. Taxable increase in net worth (1950-1951). The Tax Court set aside the
Commissioner's treatment as taxable income of certain increases in the
taxpayer's net worth. It found that:
For the year 1950, respondent determined that petitioner had an increase in
net worth in the sum of P30,050.00, and for the year 1951, the sum of
P1,382.85. These amounts were treated by respondent as taxable income of
petitioner for said years.
It appears that petitioner had an account with the Manila Insurance Company,
the records bearing on which were lost. When its records were reconstituted
the amount of P349,800.00 was set up as its liability to the Manila Insurance
Company. It was discovered later that the correct liability was only
319,750.00, or a difference of P30,050.00, so that the records were adjusted
so as to show the correct liability. The correction or adjustment was made in
1950. Respondent contends that the reduction of petitioner's liability to Manila
Insurance Company resulted in the increase of petitioner's net worth to the
extent of P30,050.00 which is taxable. This is erroneous. The principle
underlying the taxability of an increase in the net worth of a taxpayer rests on
the theory that such an increase in net worth, if unreported and not explained
by the taxpayer, comes from income derived from a taxable source. (See
Perez v. Araneta, G.R. No. L-9193, May 29, 1957; Coll. vs. Reyes, G.R. Nos.
L- 11534 & L-11558, Nov. 25, 1958.) In this case, the increase in the net
worth of petitioner for 1950 to the extent of P30,050.00 was not the result of
the receipt by it of taxable income. It was merely the outcome of the
correction of an error in the entry in its books relating to its indebtedness to
the Manila Insurance Company. The Income Tax Law imposes a tax on
income; it does not tax any or every increase in net worth whether or not
derived from income. Surely, the said sum of P30,050.00 was not income to
petitioner, and it was error for respondent to assess a deficiency income tax
on said amount.
The same holds true in the case of the alleged increase in net worth of
petitioner for the year 1951 in the sum of P1,382.85. It appears that certain
items (all amounting to P1,382.85) remained in petitioner's books as
outstanding liabilities of trade creditors. These accounts were discovered in
1951 as having been paid in prior years, so that the necessary adjustments
were made to correct the errors. If there was an increase in net worth of the
petitioner, the increase in net worth was not the result of receipt by petitioner
of taxable income." 13 The Commissioner advances no valid grounds in his
brief for contesting the Tax Court's findings. Certainly, these increases in the
taxpayer's net worth were not taxable increases in net worth, as they were
not the result of the receipt by it of unreported or unexplained taxable income,
but were shown to be merely the result of the correction of errors in its entries
in its books relating to its indebtednesses to certain creditors, which had been
erroneously overstated or listed as outstanding when they had in fact been
duly paid. The Tax Court's action must be affirmed.
4. Gain realized from sale of real property (1950). We likewise sustain as
being in accordance with the evidence the Tax Court's reversal of the
Commissioner's assessment on all alleged unreported gain in the sum of
P11,147.26 in the sale of a certain real property of the taxpayer in 1950. As
found by the Tax Court, the evidence shows that this property was acquired in
1926 for P11,852.74, and was sold in 1950 for P60,000.00, apparently,
resulting in a gain of P48,147.26. 14 The taxpayer reported in its return a gain
of P37,000.00, or a discrepancy of P11,147.26. 15 It was sufficiently proved
from the taxpayer's books that after acquiring the property, the taxpayer had
made improvements totalling P11,147.26, 16 accounting for the apparent
discrepancy in the reported gain. In other words, this figure added to the
original acquisition cost of P11,852.74 results in a total cost of P23,000.00,
and the gain derived from the sale of the property for P60,000.00 was
correctly reported by the taxpayer at P37,000.00.
On the second issue of prescription, the taxpayer's contention that the
Commissioner's action to recover its tax liability should be deemed to have
prescribed for failure on the part of the Commissioner to file a complaint for
collection against it in an appropriate civil action, as contradistinguished from
the answer filed by the Commissioner to its petition for review of the
questioned assessments in the case a quo has long been rejected by this
Court. This Court has consistently held that "a judicial action for the collection
of a tax is begun by the filing of a complaint with the proper court of first
instance, or where the assessment is appealed to the Court of Tax Appeals,
by filing an answer to the taxpayer's petition for review wherein payment of
the tax is prayed for." 17 This is but logical for where the taxpayer avails of the
right to appeal the tax assessment to the Court of Tax Appeals, the said Court
is vested with the authority to pronounce judgment as to the taxpayer's
liability to the exclusion of any other court. In the present case, regardless of
whether the assessments were made on February 24 and 27, 1956, as
claimed by the Commissioner, or on December 27, 1955 as claimed by the
taxpayer, the government's right to collect the taxes due has clearly not
prescribed, as the taxpayer's appeal or petition for review was filed with the
Tax Court on May 4, 1960, with the Commissioner filing on May 20, 1960 his
Answer with a prayer for payment of the taxes due, long before the expiration
of the five-year period to effect collection by judicial action counted from the
date of assessment.
Cases L-24972 and L-24978
These cases refer to the taxpayer's income tax liability for the year 1957.
Upon examination of its corresponding income tax return, the Commissioner
assessed it for deficiency income tax in the amount of P38,918.76, computed
as follows:
Net income per return
P29,178.70

Add: Unallowable deductions:


(1) Net loss claimed on Ha. Dalupiri
89,547.33
(2) Amortization of Contractual right claimed as an expense under Mines
Operations
48,481.62
Net income per investigation

P167,297.65
Tax due thereon
38,818.00
Less: Amount already assessed

5,836.00
Balance
P32,982.00
Add: 1/2% monthly interest from 6-20-59 to 6-20-62
5,936.76
TOTAL AMOUNT DUE AND COLLECTIBLE

P38,918.76
18

The Tax Court overruled the Commissioner's disallowance of the taxpayer's


losses in the operation of its Hacienda Dalupiri in the sum of P89,547.33 but
sustained the disallowance of the sum of P48,481.62, which allegedly
represented 1/5 of the cost of the "contractual right" over the mines of its
subsidiary, Palawan Manganese Mines, Inc. which the taxpayer had acquired.
It found the taxpayer liable for deficiency income tax for the year 1957 in the
amount of P9,696.00, instead of P32,982.00 as originally assessed, and
rendered the following judgment:
WHEREFORE, the assessment appealed from is hereby modified. Petitioner
is hereby ordered to pay to respondent the amount of P9,696.00 as
deficiency income tax for the year 1957, plus the corresponding interest
provided in Section 51 of the Revenue Code. If the deficiency tax is not paid
in full within thirty (30) days from the date this decision becomes final and
executory, petitioner shall pay a surcharge of five per cent (5%) of the unpaid
amount, plus interest at the rate of one per cent (1%) a month, computed
from the date this decision becomes final until paid, provided that the
maximum amount that may be collected as interest shall not exceed the
amount corresponding to a period of three (3) years. Without pronouncement
as to costs. 19
Both parties again appealed from the respective adverse rulings against them
in the Tax Court's decision.
5. Allowance of losses in Hacienda Dalupiri (1957). The Tax Court cited its
previous decision overruling the Commissioner's disallowance of losses
suffered by the taxpayer in the operation of its Hacienda Dalupiri, since it was
convinced that the hacienda was operated for business and not for pleasure.
And in this appeal, the Commissioner cites his arguments in his appellant's
brief in Case No. L-21557. The Tax Court, in setting aside the
Commissioner's principal objections, which were directed to the accounting
method used by the taxpayer found that:
It is true that petitioner followed the cash basis method of reporting income
and expenses in the operation of the Hacienda Dalupiri and used the accrual
method with respect to its mine operations. This method of accounting,
otherwise known as the hybrid method, followed by petitioner is not without
justification.
... A taxpayer may not, ordinarily, combine the cash and accrual bases. The
1954 Code provisions permit, however, the use of a hybrid method of
accounting, combining a cash and accrual method, under circumstances and
requirements to be set out in Regulations to be issued. Also, if a taxpayer is
engaged in more than one trade or business he may use a different method
of accounting for each trade or business. And a taxpayer may report income
from a business on accrual basis and his personal income on the cash basis.'
(See Mertens, Law of Federal Income Taxation, Zimet & Stanley Revision,
Vol. 2, Sec. 12.08, p. 26.) 20
The Tax Court, having satisfied itself with the adequacy of the taxpayer's
accounting method and procedure as properly reflecting the taxpayer's
income or losses, and the Commissioner having failed to show the contrary,
we reiterate our ruling [supra, paragraph 1 (d) and (e)] that we find no
compelling reason to disturb its findings.
6. Disallowance of amortization of alleged "contractual rights." The
reasons for sustaining this disallowance are thus given by the Tax Court:
It appears that the Palawan Manganese Mines, Inc., during a special meeting
of its Board of Directors on January 19, 1956, approved a resolution, the
pertinent portions of which read as follows:
"RESOLVED, as it is hereby resolved, that the corporation's current assets
composed of ores, fuel, and oil, materials and supplies, spare parts and
canteen supplies appearing in the inventory and balance sheet of the
Corporation as of December 31, 1955, with an aggregate value of
P97,636.98, contractual rights for the operation of various mining claims in
Palawan with a value of P100,000.00, its title on various mining claims in
Palawan with a value of P142,408.10 or a total value of P340,045.02 be, as
they are hereby ceded and transferred to Fernandez Hermanos, Inc., as
partial settlement of the indebtedness of the corporation to said Fernandez
Hermanos Inc. in the amount of P442,895.23." (Exh. E, p. 17, CTA rec.)
On March 29, 1956, petitioner's corporation accepted the above offer of
transfer, thus:
"WHEREAS, the Palawan Manganese Mines, Inc., due to its yearly
substantial losses has decided to cease operation on January 1, 1956 and in
order to satisfy at least a part of its indebtedness to the Corporation, it has
proposed to transfer its current assets in the amount of NINETY SEVEN
THOUSAND SIX HUNDRED THIRTY SIX PESOS & 98/100 (P97,636.98) as
per its balance sheet as of December 31, 1955, its contractual rights valued
at ONE HUNDRED THOUSAND PESOS (P100,000.00) and its title over
various mining claims valued at ONE HUNDRED FORTY TWO THOUSAND
FOUR HUNDRED EIGHT PESOS & 10/100 (P142,408.10) or a total
evaluation of THREE HUNDRED FORTY THOUSAND FORTY FIVE PESOS
& 08/100 (P340,045.08) which shall be applied in partial settlement of its
obligation to the Corporation in the amount of FOUR HUNDRED FORTY
TWO THOUSAND EIGHT HUNDRED EIGHTY FIVE PESOS & 23/100
(P442,885.23)," (Exh. E-1, p. 18, CTA rec.)
Petitioner determined the cost of the mines at P242,408.10 by adding the
value of the contractual rights (P100,000.00) and the value of its mining
claims (P142,408.10). Respondent disallowed the deduction on the following
grounds: (1) that the Palawan Manganese Mines, Inc. could not transfer
P242,408.10 worth of assets to petitioner because the balance sheet of the
said corporation for 1955 shows that it had only current as worth P97,636.96;
and (2) that the alleged amortization of "contractual rights" is not allowed by
the Revenue Code.
The law in point is Section 30(g) (1) (B) of the Revenue Code, before its
amendment by Republic Act No. 2698, which provided in part:
"(g) Depletion of oil and gas wells and mines.:
"(1) In general. ... (B) in the case of mines, a reasonable allowance for
depletion thereof not to exceed the market value in the mine of the product
thereof, which has been mined and sold during the year for which the return
and computation are made. The allowances shall be made under rules and
regulations to be prescribed by the Secretary of Finance: Provided, That
when the allowances shall equal the capital invested, ... no further allowance
shall be made."
Assuming, arguendo, that the Palawan Manganese Mines, Inc. had assets
worth P242,408.10 which it actually transferred to the petitioner in 1956, the
latter cannot just deduct one-fifth (1/5) of said amount from its gross income
for the year 1957 because such deduction in the form of depletion charge
was not sanctioned by Section 30(g) (1) (B) of the Revenue Code, as above-
quoted.
xxx xxx xxx
The sole basis of petitioner in claiming the amount of P48,481.62 as a
deduction was the memorandum of its mining engineer (Exh. 1, pp. 31-32,
CTA rec.), who stated that the ore reserves of the Busuange Mines (Mines
transferred by the Palawan Manganese Mines, Inc. to the petitioner) would be
exhausted in five (5) years, hence, the claim for P48,481.62 or one-fifth (1/5)
of the alleged cost of the mines corresponding to the year 1957 and every
year thereafter for a period of 5 years. The said memorandum merely showed
the estimated ore reserves of the mines and it probable selling price. No
evidence whatsoever was presented to show the produced mine and for how
much they were sold during the year for which the return and computation
were made. This is necessary in order to determine the amount of depletion
that can be legally deducted from petitioner's gross income. The method
employed by petitioner in making an outright deduction of 1/5 of the cost of
the mines is not authorized under Section 30(g) (1) (B) of the Revenue Code.
Respondent's disallowance of the alleged "contractual rights" amounting to
P48,481.62 must therefore be sustained. 21
The taxpayer insists in this appeal that it could use as a method for depletion
under the pertinent provision of the Tax Code its "capital investment,"
representing the alleged value of its contractual rights and titles to mining
claims in the sum of P242,408.10 and thus deduct outright one-fifth (1/5) of
this "capital investment" every year. regardless of whether it had actually
mined the product and sold the products. The very authorities cited in its brief
give the correct concept of depletion charges that they "allow for the
exhaustion of the capital value of the deposits by production"; thus, "as the
cost of the raw materials must be deducted from the gross income before the
net income can be determined, so the estimated cost of the reserve used up
is allowed." 22 The alleged "capital investment" method invoked by the
taxpayer is not a method of depletion, but the Tax Code provision, prior to its
amendment by Section 1, of Republic Act No. 2698, which took effect on
June 18, 1960, expressly provided that "when the allowances shall equal the
capital invested ... no further allowances shall be made;" in other words, the
"capital investment" was but the limitation of the amount of depletion that
could be claimed. The outright deduction by the taxpayer of 1/5 of the cost of
the mines, as if it were a "straight line" rate of depreciation, was correctly held
by the Tax Court not to be authorized by the Tax Code.
ACCORDINGLY, the judgment of the Court of Tax Appeals, subject of the
appeals in Cases Nos. L-21551 and L-21557, as modified by the crediting of
the losses of P36,722.42 disallowed in 1951 and 1952 to the taxpayer for the
year 1953 as directed in paragraph 1 (c) of this decision, is hereby affirmed.
The judgment of the Court of Tax Appeals appealed from in Cases Nos. L-
24972 and L-24978 is affirmed in toto. No costs. So ordered.

G.R. No. L-22492 September 5, 1967


BASILAN ESTATES, INC., petitioner,
vs.
THE COMMISSIONER OF INTERNAL REVENUE and THE COURT OF TAX
APPEALS, respondents.
Felix A. Gulfin and Antonio S. Alano for petitioner.
Office of the Solicitor General for respondents.

BENGZON, J.P., J.:


A Philippine corporation engaged in the coconut industry, Basilan Estates,
Inc., with principal offices in Basilan City, filed on March 24, 1954 its income
tax returns for 1953 and paid an income tax of P8,028. On February 26,
1959, the Commissioner of Internal Revenue, per examiners' report of
February 19, 1959, assessed Basilan Estates, Inc., a deficiency income tax of
P3,912 for 1953 and P86,876.85 as 25% surtax on unreasonably
accumulated profits as of 1953 pursuant to Section 25 of the Tax Code. On
non-payment of the assessed amount, a warrant of distraint and levy was
issued but the same was not executed because Basilan Estates, Inc.
succeeded in getting the Deputy Commissioner of Internal Revenue to order
the Director of the district in Zamboanga City to hold execution and maintain
constructive embargo instead. Because of its refusal to waive the period of
prescription, the corporation's request for reinvestigation was not given due
course, and on December 2, 1960, notice was served the corporation that the
warrant of distraint and levy would be executed.
On December 20, 1960, Basilan Estates, Inc. filed before the Court of Tax
Appeals a petition for review of the Commissioner's assessment, alleging
prescription of the period for assessment and collection; error in disallowing
claimed depreciations, travelling and miscellaneous expenses; and error in
finding the existence of unreasonably accumulated profits and the imposition
of 25% surtax thereon. On October 31, 1963, the Court of Tax Appeals found
that there was no prescription and affirmed the deficiency assessment in toto.
On February 21, 1964, the case was appealed to Us by the taxpayer, upon
the following issues:
1. Has the Commissioner's right to collect deficiency income tax prescribed?
2. Was the disallowance of items claimed as deductible proper?
3. Have there been unreasonably accumulated profits? If so, should the 25%
surtax be imposed on the balance of the entire surplus from 1947-1953, or
only for 1953?
4. Is the petitioner exempt from the penalty tax under Republic Act 1823
amending Section 25 of the Tax Code?
PRESCRIPTION
There is no dispute that the assessment of the deficiency tax was made on
February 26, 1959; but the petitioner claims that it never received notice of
such assessment or if it did, it received the notice beyond the five-year
prescriptive period. To show prescription, the annotation on the notice (Exhibit
10, No. 52, ACR, p. 54-A of the BIR records) "No accompanying letter 11/25/"
is advanced as indicative of the fact that receipt of the notice was after March
24, 1959, the last date of the five-year period within which to assess
deficiency tax, since the original returns were filed on March 24, 1954.
Although the evidence is not clear on this point, We cannot accept this
interpretation of the petitioner, considering the presence of circumstances that
lead Us to presume regularity in the performance of official functions. The
notice of assessment shows the assessment to have been made on February
26, 1959, well within the five-year period. On the right side of the notice is
also stamped "Feb. 26, 1959" denoting the date of release, according to
Bureau of Internal Revenue practice. The Commissioner himself in his letter
(Exh. H, p. 84 of BIR records) answering petitioner's request to lift, the
warrant of distraint and levy, asserts that notice had been sent to petitioner. In
the letter of the Regional Director forwarding the case to the Chief of the
Investigation Division which the latter received on March 10, 1959 (p. 71 of
the BIR records), notice of assessment was said to have been sent to
petitioner. Subsequently, the Chief of the Investigation Division indorsed on
March 18, 1959 (p. 24 of the BIR records) the case to the Chief of the Law
Division. There it was alleged that notice was already sent to petitioner on
February 26, 1959. These circumstances pointing to official performance of
duty must necessarily prevail over petitioner's contrary interpretation.
Besides, even granting that notice had been received by the petitioner late,
as alleged, under Section 331 of the Tax Code requiring five years within
which to assess deficiency taxes, the assessment is deemed made when
notice to this effect is released, mailed or sent by the Collector to the taxpayer
and it is not required that the notice be received by the taxpayer within the
aforementioned five-year period.1
ASSESSMENT
The questioned assessment is as follows:
Net Income per return
P40,142.90
Add:
Over-claimed depreciation
P10,500.49
Mis. expenses disallowed
6,759.17
Officer's travelling expenses disallowed

2,300.40

19,560.06

Net Income per Investigation


P59,702.96
20% tax on P59,702.96
11,940.00
Less: Tax already assessed
8,028.00

Deficiency income tax


P3,912.00
Add: Additional tax of 25% on P347,507.01
86,876.75
Tax Due & Collectible

P90,788.75
=========
The Commissioner disallowed:
Over-claimed depreciation
P10,500.49
Miscellaneous expenses
6,759.17
Officer's travelling expenses
2,300.40
DEDUCTIONS
A. Depreciation. Basilan Estates, Inc. claimed deductions for the
depreciation of its assets up to 1949 on the basis of their acquisition cost. As
of January 1, 1950 it changed the depreciable value of said assets by
increasing it to conform with the increase in cost for their replacement.
Accordingly, from 1950 to 1953 it deducted from gross income the value of
depreciation computed on the reappraised value.
In 1953, the year involved in this case, taxpayer claimed the following
depreciation deduction:
Reappraised assets
P47,342.53
New assets consisting of hospital building and equipment
3,910.45
Total depreciation

P51,252.98
Upon investigation and examination of taxpayer's books and papers, the
Commissioner of Internal Revenue found that the reappraised assets
depreciated in 1953 were the same ones upon which depreciation was
claimed in 1952. And for the year 1952, the Commissioner had already
determined, with taxpayer's concurrence, the depreciation allowable on said
assets to be P36,842.04, computed on their acquisition cost at rates fixed by
the taxpayer. Hence, the Commissioner pegged the deductible depreciation
for 1953 on the same old assets at P36,842.04 and disallowed the excess
thereof in the amount of P10,500.49.
The question for resolution therefore is whether depreciation shall be
determined on the acquisition cost or on the reappraised value of the assets.
Depreciation is the gradual diminution in the useful value of tangible property
resulting from wear and tear and normal obsolescense. The term is also
applied to amortization of the value of intangible assets, the use of which in
the trade or business is definitely limited in duration. 2 Depreciation
commences with the acquisition of the property and its owner is not bound to
see his property gradually waste, without making provision out of earnings for
its replacement. It is entitled to see that from earnings the value of the
property invested is kept unimpaired, so that at the end of any given term of
years, the original investment remains as it was in the beginning. It is not only
the right of a company to make such a provision, but it is its duty to its bond
and stockholders, and, in the case of a public service corporation, at least, its
plain duty to the public.3 Accordingly, the law permits the taxpayer to recover
gradually his capital investment in wasting assets free from income tax. 4
Precisely, Section 30 (f) (1) which states:
(1)In general. A reasonable allowance for deterioration of property arising
out of its use or employment in the business or trade, or out of its not being
used: Provided, That when the allowance authorized under this subsection
shall equal the capital invested by the taxpayer . . . no further allowance shall
be made. . . .
allows a deduction from gross income for depreciation but limits the recovery
to the capital invested in the asset being depreciated.
The income tax law does not authorize the depreciation of an asset beyond
its acquisition cost. Hence, a deduction over and above such cost cannot be
claimed and allowed. The reason is that deductions from gross income are
privileges,5 not matters of right.6 They are not created by implication but upon
clear expression in the law.7
Moreover, the recovery, free of income tax, of an amount more than the
invested capital in an asset will transgress the underlying purpose of a
depreciation allowance. For then what the taxpayer would recover will be, not
only the acquisition cost, but also some profit. Recovery in due time thru
depreciation of investment made is the philosophy behind depreciation
allowance; the idea of profit on the investment made has never been the
underlying reason for the allowance of a deduction for depreciation.
Accordingly, the claim for depreciation beyond P36,842.04 or in the amount of
P10,500.49 has no justification in the law. The determination, therefore, of the
Commissioner of Internal Revenue disallowing said amount, affirmed by the
Court of Tax Appeals, is sustained.
B. Expenses. The next item involves disallowed expenses incurred in
1953, broken as follows:
Miscellaneous expenses
P6,759.17
Officer's travelling expenses
2,300.40
Total

P9,059.57
These were disallowed on the ground that the nature of these expenses could
not be satisfactorily explained nor could the same be supported by
appropriate papers.
Felix Gulfin, petitioner's accountant, explained the P6,759.17 was actual
expenses credited to the account of the president of the corporation incurred
in the interest of the corporation during the president's trip to Manila (pp. 33-
34 of TSN of Dec. 5, 1962); he stated that the P2,300.40 was the president's
travelling expenses to and from Manila as to the vouchers and receipts of
these, he said the same were made but got burned during the Basilan fire on
March 30, 1962 (p. 40 of same TSN). Petitioner further argues that when it
sent its records to Manila in February, 1959, the papers in support of these
miscellaneous and travelling expenses were not included for the reason that
by February 9, 1959, when the Bureau of Internal Revenue decided to
investigate, petitioner had no more obligation to keep the same since five
years had lapsed from the time these expenses were incurred (p. 41 of same
TSN). On this ground, the petitioner may be sustained, for under Section 337
of the Tax Code, receipts and papers supporting such expenses need be kept
by the taxpayer for a period of five years from the last entry. At the time of the
investigation, said five years had lapsed. Taxpayer's stand on this issue is
therefore sustained.
UNREASONABLY ACCUMULATED PROFITS
Section 25 of the Tax Code which imposes a surtax on profits unreasonably
accumulated, provides:
Sec. 25. Additional tax on corporations improperly accumulating profits or
surplus (a) Imposition of tax. If any corporation, except banks,
insurance companies, or personal holding companies, whether domestic or
foreign, is formed or availed of for the purpose of preventing the imposition of
the tax upon its shareholders or members or the shareholders or members of
another corporation, through the medium of permitting its gains and profits to
accumulate instead of being divided or distributed, there is levied and
assessed against such corporation, for each taxable year, a tax equal to
twenty-five per centum of the undistributed portion of its accumulated profits
or surplus which shall be in addition to the tax imposed by section twenty-
four, and shall be computed, collected and paid in the same manner and
subject to the same provisions of law, including penalties, as that tax.
1awphl.nt

The Commissioner found that in violation of the abovequoted section,


petitioner had unreasonably accumulated profits as of 1953 in the amount of
P347,507.01, based on the following circumstances (Examiner's Report pp.
62-68 of BIR records):
1. Strong financial position of the petitioner as of December 31, 1953. Assets
were P388,617.00 while the liabilities amounted to only P61,117.31 or a ratio
of 6:1.
2. As of 1953, the corporation had considerable capital adequate to meet the
reasonable needs of the business amounting to P327,499.69 (assets less
liabilities).
3. The P200,000 reserved for electrification of drier and mechanization and
the P50,000 reserved for malaria control were reverted to its surplus in 1953.
4. Withdrawal by shareholders, of large sums of money as personal loans.
5. Investment of undistributed earnings in assets having no proximate
connection with the business as hospital building and equipment worth
P59,794.72.
6. In 1953, with an increase of surplus amounting to P677,232.01, the capital
stock was increased to P500,000 although there was no need for such
increase.
Petitioner tried to show that in considering the surplus, the examiner did not
take into account the possible expenses for cultivation, labor, fertilitation,
drainage, irrigation, repair, etc. (pp. 235-237 of TSN of Dec. 7, 1962). As aptly
answered by the examiner himself, however, they were already included as
part of the working capital (pp. 237-238 of TSN of Dec. 7, 1962).
In the unreasonable accumulation of P347,507.01 are included P200,000 for
electrification of driers and mechanization and P50,000 for malaria control
which were reserved way back in 1948 (p. 67 of the BIR records) but reverted
to the general fund only in 1953. If there were any plans for these amounts to
be used in further expansion through projects, it did not appear in the records
as was properly indicated in 1948 when such amounts were reserved. Thus,
while in 1948 it was already clear that the money was intended to go to future
projects, in 1953 upon reversion to the general fund, no such intention was
shown. Such reversion therefore gave occasion for the Government to
consider the same for tax purposes. The P250,000 reverted to the general
fund was sought to be explained as later used elsewhere: "part of it in the
Hilano Industries, Inc. in building the factory site and buildings to house
technical men . . . part of it was spent in the facilities for the waterworks
system and for industrialization of the coconut industry" (p. 117 of TSN of
Dec. 6, 1962). This is not sufficient explanation. Persuasive jurisprudence on
the matter such as those in the United States from where our tax law was
derived,8 has it that: "In order to determine whether profits were accumulated
for the reasonable needs of the business or to avoid the surtax upon
shareholders, the controlling intention of the taxpayer is that which is
manifested at the time of the accumulation, not subsequently declared
intentions which are merely the products of after-thought." 9 The reversion
here was made because the reserved amount was not enough for the
projects intended, without any intent to channel the same to some particular
future projects in mind.
Petitioner argues that since it has P560,717.44 as its expenses for the year
1953, a surplus of P347,507.01 is not unreasonably accumulated. As rightly
contended by the Government, there is no need to have such a large amount
at the beginning of the following year because during the year, current assets
are converted into cash and with the income realized from the business as
the year goes, these expenses may well be taken care of (pp. 238 of TSN of
Dec. 7, 1962). Thus, it is erroneous to say that the taxpayer is entitled to
retain enough liquid net assets in amounts approximately equal to current
operating needs for the year to cover "cost of goods sold and operating
expenses" for "it excludes proper consideration of funds generated by the
collection of notes receivable as trade accounts during the course of the
year."10 In fact, just because the fatal accumulations are less than 70% of the
annual operating expenses of the year, it does not mean that the
accumulations are reasonable as a matter of law." 11
Petitioner tried to show that investments were made with Basilan Coconut
Producers Cooperative Association and Basilan Hospital (pp. 103-105 of TSN
of Dec. 6, 1962) totalling P59,794.72 as of December 31, 1953. This shows
all the more the unreasonable accumulation. As of December 31, 1953
already P59,794.72 was spent yet as of that date there was still a surplus
of P347,507.01.
Petitioner questions why the examiner covered the period from 1948-1953
when the taxable year on review was 1953. The surplus of P347,507.01 was
taken by the examiner from the balance sheet of petitioner for 1953. To check
the figure arrived at, the examiner traced the accumulation process from 1947
until 1953, and petitioner's figure stood out to be correct. There was no error
in the process applied, for previous accumulations should be considered in
determining unreasonable accumulations for the year concerned. "In
determining whether accumulations of earnings or profits in a particular year
are within the reasonable needs of a corporation, it is neccessary to take into
account prior accumulations, since accumulations prior to the year involved
may have been sufficient to cover the business needs and additional
accumulations during the year involved would not reasonably be necessary." 12
Another factor that stands out to show unreasonable accumulation is the fact
that large amounts were withdrawn by or advanced to the stockholders. For
the year 1953 alone these totalled P197,229.26. Yet the surplus of
P347,507.01 was left as of December 31, 1953. We find unacceptable
petitioner's explanation that these were advances made in furtherance of the
business purposes of the petitioner. As correctly held by the Court of Tax
Appeals, while certain expenses of the corporation were credited against
these amounts, the unspent balance was retained by the stockholders without
refunding them to petitioner at the end of each year. These advances were in
fact indirect loans to the stockholders indicating the unreasonable
accumulation of surplus beyond the needs of the business.
ALLEGED EXEMPTION
Petitioner wishes to avail of the exempting proviso in Sec. 25 of the Internal
Revenue Code as amended by R.A. 1823, approved June 22, 1957, whereby
accumulated profits or surplus if invested in any dollar-producing or dollar-
earning industry or in the purchase of bonds issued by the Central Bank, may
not be subject to the 25% surtax. We have but to point out that the
unreasonable accumulation was in 1953. The exemption was by virtue of
Republic Act 1823 which amended Sec. 25 only on June 22, 1957 more
than three years after the period covered by the assessment.
In resume, Basilan Estates, Inc. is liable for the payment of deficiency income
tax and surtax for the year 1953 in the amount of P88,977.42, computed as
follows:
Net Income per return
P40,142.90
Add: Over-claimed depreciation
10,500.49
Net income per finding

P50,643.39
20% tax on P50,643.39
P10,128.67
Less: Tax already assessed
8,028.00
Deficiency income tax

P2,100.67
Add: 25% surtax on P347,507.01
86,876.75
Total tax due and collectible

P88,977.42
===========
WHEREFORE, the judgment appealed from is modified to the extent that
petitioner is allowed its deductions for travelling and miscellaneous expenses,
but affirmed insofar as the petitioner is liable for P2,100.67 as deficiency
income tax for 1953 and P86,876.75 as 25% surtax on the unreasonably
accumulated profit of P347,507.01. No costs. So ordered.

G.R. No. L-25043 April 26, 1968


ANTONIO ROXAS, EDUARDO ROXAS and ROXAS Y CIA., in their own
respective behalf and as judicial co-guardians of JOSE ROXAS,
petitioners,
vs.
COURT OF TAX APPEALS and COMMISSIONER OF INTERNAL
REVENUE, respondents.
Leido, Andrada, Perez and Associates for petitioners.
Office of the Solicitor General for respondents.
BENGZON, J.P., J.:
Don Pedro Roxas and Dona Carmen Ayala, Spanish subjects, transmitted to
their grandchildren by hereditary succession the following properties:
(1) Agricultural lands with a total area of 19,000 hectares, situated in the
municipality of Nasugbu, Batangas province;
(2) A residential house and lot located at Wright St., Malate, Manila; and
(3) Shares of stocks in different corporations.
To manage the above-mentioned properties, said children, namely, Antonio
Roxas, Eduardo Roxas and Jose Roxas, formed a partnership called Roxas y
Compania.
AGRICULTURAL LANDS
At the conclusion of the Second World War, the tenants who have all been
tilling the lands in Nasugbu for generations expressed their desire to
purchase from Roxas y Cia. the parcels which they actually occupied. For its
part, the Government, in consonance with the constitutional mandate to
acquire big landed estates and apportion them among landless tenants-
farmers, persuaded the Roxas brothers to part with their landholdings.
Conferences were held with the farmers in the early part of 1948 and finally
the Roxas brothers agreed to sell 13,500 hectares to the Government for
distribution to actual occupants for a price of P2,079,048.47 plus P300,000.00
for survey and subdivision expenses.
It turned out however that the Government did not have funds to cover the
purchase price, and so a special arrangement was made for the
Rehabilitation Finance Corporation to advance to Roxas y Cia. the amount of
P1,500,000.00 as loan. Collateral for such loan were the lands proposed to
be sold to the farmers. Under the arrangement, Roxas y Cia. allowed the
farmers to buy the lands for the same price but by installment, and contracted
with the Rehabilitation Finance Corporation to pay its loan from the proceeds
of the yearly amortizations paid by the farmers.
In 1953 and 1955 Roxas y Cia. derived from said installment payments a net
gain of P42,480.83 and P29,500.71. Fifty percent of said net gain was
reported for income tax purposes as gain on the sale of capital asset held for
more than one year pursuant to Section 34 of the Tax Code.
RESIDENTIAL HOUSE
During their bachelor days the Roxas brothers lived in the residential house at
Wright St., Malate, Manila, which they inherited from their grandparents. After
Antonio and Eduardo got married, they resided somewhere else leaving only
Jose in the old house. In fairness to his brothers, Jose paid to Roxas y Cia.
rentals for the house in the sum of P8,000.00 a year.
ASSESSMENTS
On June 17, 1958, the Commissioner of Internal Revenue demanded from
Roxas y Cia the payment of real estate dealer's tax for 1952 in the amount of
P150.00 plus P10.00 compromise penalty for late payment, and P150.00 tax
for dealers of securities for 1952 plus P10.00 compromise penalty for late
payment. The assessment for real estate dealer's tax was based on the fact
that Roxas y Cia. received house rentals from Jose Roxas in the amount of
P8,000.00. Pursuant to Sec. 194 of the Tax Code, an owner of a real estate
who derives a yearly rental income therefrom in the amount of P3,000.00 or
more is considered a real estate dealer and is liable to pay the corresponding
fixed tax.
The Commissioner of Internal Revenue justified his demand for the fixed tax
on dealers of securities against Roxas y Cia., on the fact that said partnership
made profits from the purchase and sale of securities.
In the same assessment, the Commissioner assessed deficiency income
taxes against the Roxas Brothers for the years 1953 and 1955, as follows:
1953
1955
Antonio Roxas
P7,010.00
P5,813.00
Eduardo Roxas
7,281.00
5,828.00
Jose Roxas
6,323.00
5,588.00
The deficiency income taxes resulted from the inclusion as income of Roxas y
Cia. of the unreported 50% of the net profits for 1953 and 1955 derived from
the sale of the Nasugbu farm lands to the tenants, and the disallowance of
deductions from gross income of various business expenses and
contributions claimed by Roxas y Cia. and the Roxas brothers. For the reason
that Roxas y Cia. subdivided its Nasugbu farm lands and sold them to the
farmers on installment, the Commissioner considered the partnership as
engaged in the business of real estate, hence, 100% of the profits derived
therefrom was taxed.
The following deductions were disallowed:

ROXAS Y CIA.:
1953

Tickets for Banquet in honor of


S. Osmea
P 40.00
Gifts of San Miguel beer
28.00
Contributions to
Philippine Air Force Chapel
100.00
Manila Police Trust Fund
150.00
Philippines Herald's fund for Manila's neediest families
100.00
1955

Contributions to Contribution to
Our Lady of Fatima Chapel, FEU
50.00
ANTONIO ROXAS:
1953

Contributions to
Pasay City Firemen Christmas Fund
25.00
Pasay City Police Dept. X'mas fund
50.00
1955

Contributions to
Baguio City Police Christmas fund
25.00
Pasay City Firemen Christmas fund
25.00
Pasay City Police Christmas fund
50.00

EDUARDO ROXAS:
1953

Contributions to
Hijas de Jesus' Retiro de Manresa
450.00
Philippines Herald's fund for Manila's neediest families
100.00
1955

Contributions to Philippines
Herald's fund for Manila's
neediest families
120.00

JOSE ROXAS:
1955

Contributions to Philippines
Herald's fund for Manila's
neediest families
120.00
The Roxas brothers protested the assessment but inasmuch as said protest
was denied, they instituted an appeal in the Court of Tax Appeals on January
9, 1961. The Tax Court heard the appeal and rendered judgment on July 31,
1965 sustaining the assessment except the demand for the payment of the
fixed tax on dealer of securities and the disallowance of the deductions for
contributions to the Philippine Air Force Chapel and Hijas de Jesus' Retiro de
Manresa. The Tax Court's judgment reads:
WHEREFORE, the decision appealed from is hereby affirmed with respect to
petitioners Antonio Roxas, Eduardo Roxas, and Jose Roxas who are hereby
ordered to pay the respondent Commissioner of Internal Revenue the
amounts of P12,808.00, P12,887.00 and P11,857.00, respectively, as
deficiency income taxes for the years 1953 and 1955, plus 5% surcharge and
1% monthly interest as provided for in Sec. 51(a) of the Revenue Code; and
modified with respect to the partnership Roxas y Cia. in the sense that it
should pay only P150.00, as real estate dealer's tax. With costs against
petitioners.
Not satisfied, Roxas y Cia. and the Roxas brothers appealed to this Court.
The Commissioner of Internal Revenue did not appeal.
The issues:
(1) Is the gain derived from the sale of the Nasugbu farm lands an ordinary
gain, hence 100% taxable?
(2) Are the deductions for business expenses and contributions deductible?
(3) Is Roxas y Cia. liable for the payment of the fixed tax on real estate
dealers?
The Commissioner of Internal Revenue contends that Roxas y Cia. could be
considered a real estate dealer because it engaged in the business of selling
real estate. The business activity alluded to was the act of subdividing the
Nasugbu farm lands and selling them to the farmers-occupants on
installment. To bolster his stand on the point, he cites one of the purposes of
Roxas y Cia. as contained in its articles of partnership, quoted below:
4. (a) La explotacion de fincas urbanes pertenecientes a la misma o que
pueden pertenecer a ella en el futuro, alquilandoles por los plazos y demas
condiciones, estime convenientes y vendiendo aquellas que a juicio de sus
gerentes no deben conservarse;
The above-quoted purpose notwithstanding, the proposition of the
Commissioner of Internal Revenue cannot be favorably accepted by Us in
this isolated transaction with its peculiar circumstances in spite of the fact that
there were hundreds of vendees. Although they paid for their respective
holdings in installment for a period of ten years, it would nevertheless not
make the vendor Roxas y Cia. a real estate dealer during the ten-year
amortization period.
It should be borne in mind that the sale of the Nasugbu farm lands to the very
farmers who tilled them for generations was not only in consonance with, but
more in obedience to the request and pursuant to the policy of our
Government to allocate lands to the landless. It was the bounden duty of the
Government to pay the agreed compensation after it had persuaded Roxas y
Cia. to sell its haciendas, and to subsequently subdivide them among the
farmers at very reasonable terms and prices. However, the Government could
not comply with its duty for lack of funds. Obligingly, Roxas y Cia. shouldered
the Government's burden, went out of its way and sold lands directly to the
farmers in the same way and under the same terms as would have been the
case had the Government done it itself. For this magnanimous act, the
municipal council of Nasugbu passed a resolution expressing the people's
gratitude.
The power of taxation is sometimes called also the power to destroy.
Therefore it should be exercised with caution to minimize injury to the
proprietary rights of a taxpayer. It must be exercised fairly, equally and
uniformly, lest the tax collector kill the "hen that lays the golden egg". And, in
order to maintain the general public's trust and confidence in the Government
this power must be used justly and not treacherously. It does not conform with
Our sense of justice in the instant case for the Government to persuade the
taxpayer to lend it a helping hand and later on to penalize him for duly
answering the urgent call.
In fine, Roxas y Cia. cannot be considered a real estate dealer for the sale in
question. Hence, pursuant to Section 34 of the Tax Code the lands sold to the
farmers are capital assets, and the gain derived from the sale thereof is
capital gain, taxable only to the extent of 50%.
DISALLOWED DEDUCTIONS
Roxas y Cia. deducted from its gross income the amount of P40.00 for tickets
to a banquet given in honor of Sergio Osmena and P28.00 for San Miguel
beer given as gifts to various persons. The deduction were claimed as
representation expenses. Representation expenses are deductible from gross
income as expenditures incurred in carrying on a trade or business under
Section 30(a) of the Tax Code provided the taxpayer proves that they are
reasonable in amount, ordinary and necessary, and incurred in connection
with his business. In the case at bar, the evidence does not show such link
between the expenses and the business of Roxas y Cia. The findings of the
Court of Tax Appeals must therefore be sustained.
The petitioners also claim deductions for contributions to the Pasay City
Police, Pasay City Firemen, and Baguio City Police Christmas funds, Manila
Police Trust Fund, Philippines Herald's fund for Manila's neediest families and
Our Lady of Fatima chapel at Far Eastern University.
The contributions to the Christmas funds of the Pasay City Police, Pasay City
Firemen and Baguio City Police are not deductible for the reason that the
Christmas funds were not spent for public purposes but as Christmas gifts to
the families of the members of said entities. Under Section 39(h), a
contribution to a government entity is deductible when used exclusively for
public purposes. For this reason, the disallowance must be sustained. On the
other hand, the contribution to the Manila Police trust fund is an allowable
deduction for said trust fund belongs to the Manila Police, a government
entity, intended to be used exclusively for its public functions.
The contributions to the Philippines Herald's fund for Manila's neediest
families were disallowed on the ground that the Philippines Herald is not a
corporation or an association contemplated in Section 30 (h) of the Tax Code.
It should be noted however that the contributions were not made to the
Philippines Herald but to a group of civic spirited citizens organized by the
Philippines Herald solely for charitable purposes. There is no question that
the members of this group of citizens do not receive profits, for all the funds
they raised were for Manila's neediest families. Such a group of citizens may
be classified as an association organized exclusively for charitable purposes
mentioned in Section 30(h) of the Tax Code.
Rightly, the Commissioner of Internal Revenue disallowed the contribution to
Our Lady of Fatima chapel at the Far Eastern University on the ground that
the said university gives dividends to its stockholders. Located within the
premises of the university, the chapel in question has not been shown to
belong to the Catholic Church or any religious organization. On the other
hand, the lower court found that it belongs to the Far Eastern University,
contributions to which are not deductible under Section 30(h) of the Tax Code
for the reason that the net income of said university injures to the benefit of its
stockholders. The disallowance should be sustained.
Lastly, Roxas y Cia. questions the imposition of the real estate dealer's fixed
tax upon it, because although it earned a rental income of P8,000.00 per
annum in 1952, said rental income came from Jose Roxas, one of the
partners. Section 194 of the Tax Code, in considering as real estate dealers
owners of real estate receiving rentals of at least P3,000.00 a year, does not
provide any qualification as to the persons paying the rentals. The law, which
states:1wph1.t

. . . "Real estate dealer" includes any person engaged in the business of


buying, selling, exchanging, leasing or renting property on his own account as
principal and holding himself out as a full or part-time dealer in real estate or
as an owner of rental property or properties rented or offered to rent for an
aggregate amount of three thousand pesos or more a year: . . . (Emphasis
supplied) .
is too clear and explicit to admit construction. The findings of the Court of Tax
Appeals or, this point is sustained.1wph1.t

To Summarize, no deficiency income tax is due for 1953 from Antonio Roxas,
Eduardo Roxas and Jose Roxas. For 1955 they are liable to pay deficiency
income tax in the sum of P109.00, P91.00 and P49.00, respectively,
computed as follows: *
ANTONIO ROXAS
Net income per return
P315,476.59
Add: 1/3 share, profits in Roxas y Cia.
P 153,249.15

Less amount declared


146,135.46
Amount understated

P 7,113.69
Contributions disallowed
115.00

P 7,228.69
Less 1/3 share of contributions amounting to P21,126.06 disallowed from
partnership but allowed to partners
7,042.02
186.67
Net income per review

P315,663.26
Less: Exemptions

4,200.00
Net taxable income

P311,463.26
Tax due
154,169.00

Tax paid
154,060.00
Deficiency

P 109.00
==========
EDUARDO ROXAS
Net income per return

P 304,166.92
Add: 1/3 share, profits in Roxas y Cia
P 153,249.15
Less profits declared
146,052.58
Amount understated

P 7,196.57
Less 1/3 share in contributions amounting to P21,126.06 disallowed from
partnership but allowed to partners
7,042.02
155.55
Net income per review

P304,322.47
Less: Exemptions

4,800.00
Net taxable income

P299,592.47
Tax Due
P147,250.00

Tax paid
147,159.00
Deficiency

P91.00
===========
JOSE ROXAS
Net income per return

P222,681.76
Add: 1/3 share, profits in Roxas y Cia.
P153,429.15

Less amount reported


146,135.46
Amount understated

7,113.69
Less 1/3 share of contributions disallowed from partnership but allowed as
deductions to partners
7,042.02
71.67
Net income per review

P222,753.43
Less: Exemption

1,800.00
Net income subject to tax

P220,953.43
Tax due
P102,763.00

Tax paid
102,714.00
Deficiency

P 49.00
===========
WHEREFORE, the decision appealed from is modified. Roxas y Cia. is
hereby ordered to pay the sum of P150.00 as real estate dealer's fixed tax for
1952, and Antonio Roxas, Eduardo Roxas and Jose Roxas are ordered to
pay the respective sums of P109.00, P91.00 and P49.00 as their individual
deficiency income tax all corresponding for the year 1955. No costs. So
ordered.

G.R. No. L-12287 August 7, 1918


VICENTE MADRIGAL and his wife, SUSANA PATERNO, plaintiffs-
appellants,
vs.
JAMES J. RAFFERTY, Collector of Internal Revenue, and VENANCIO
CONCEPCION, Deputy Collector of Internal Revenue, defendants-
appellees.
Gregorio Araneta for appellants.
Assistant Attorney Round for appellees.
MALCOLM, J.:
This appeal calls for consideration of the Income Tax Law, a law of American
origin, with reference to the Civil Code, a law of Spanish origin.
STATEMENT OF THE CASE.
Vicente Madrigal and Susana Paterno were legally married prior to January 1,
1914. The marriage was contracted under the provisions of law concerning
conjugal partnerships (sociedad de gananciales). On February 25, 1915,
Vicente Madrigal filed sworn declaration on the prescribed form with the
Collector of Internal Revenue, showing, as his total net income for the year
1914, the sum of P296,302.73. Subsequently Madrigal submitted the claim
that the said P296,302.73 did not represent his income for the year 1914, but
was in fact the income of the conjugal partnership existing between himself
and his wife Susana Paterno, and that in computing and assessing the
additional income tax provided by the Act of Congress of October 3, 1913, the
income declared by Vicente Madrigal should be divided into two equal parts,
one-half to be considered the income of Vicente Madrigal and the other half
of Susana Paterno. The general question had in the meantime been
submitted to the Attorney-General of the Philippine Islands who in an opinion
dated March 17, 1915, held with the petitioner Madrigal. The revenue officers
being still unsatisfied, the correspondence together with this opinion was
forwarded to Washington for a decision by the United States Treasury
Department. The United States Commissioner of Internal Revenue reversed
the opinion of the Attorney-General, and thus decided against the claim of
Madrigal.
After payment under protest, and after the protest of Madrigal had been
decided adversely by the Collector of Internal Revenue, action was begun by
Vicente Madrigal and his wife Susana Paterno in the Court of First Instance of
the city of Manila against Collector of Internal Revenue and the Deputy
Collector of Internal Revenue for the recovery of the sum of P3,786.08,
alleged to have been wrongfully and illegally collected by the defendants from
the plaintiff, Vicente Madrigal, under the provisions of the Act of Congress
known as the Income Tax Law. The burden of the complaint was that if the
income tax for the year 1914 had been correctly and lawfully computed there
would have been due payable by each of the plaintiffs the sum of P2,921.09,
which taken together amounts of a total of P5,842.18 instead of P9,668.21,
erroneously and unlawfully collected from the plaintiff Vicente Madrigal, with
the result that plaintiff Madrigal has paid as income tax for the year 1914,
P3,786.08, in excess of the sum lawfully due and payable.
The answer of the defendants, together with an analysis of the tax
declaration, the pleadings, and the stipulation, sets forth the basis of
defendants' stand in the following way: The income of Vicente Madrigal and
his wife Susana Paterno of the year 1914 was made up of three items: (1)
P362,407.67, the profits made by Vicente Madrigal in his coal and shipping
business; (2) P4,086.50, the profits made by Susana Paterno in her
embroidery business; (3) P16,687.80, the profits made by Vicente Madrigal in
a pawnshop company. The sum of these three items is P383,181.97, the
gross income of Vicente Madrigal and Susana Paterno for the year 1914.
General deductions were claimed and allowed in the sum of P86,879.24. The
resulting net income was P296,302.73. For the purpose of assessing the
normal tax of one per cent on the net income there were allowed as specific
deductions the following: (1) P16,687.80, the tax upon which was to be paid
at source, and (2) P8,000, the specific exemption granted to Vicente Madrigal
and Susana Paterno, husband and wife. The remainder, P271,614.93 was
the sum upon which the normal tax of one per cent was assessed. The
normal tax thus arrived at was P2,716.15.
The dispute between the plaintiffs and the defendants concerned the
additional tax provided for in the Income Tax Law. The trial court in an
exhausted decision found in favor of defendants, without costs.
ISSUES.
The contentions of plaintiffs and appellants having to do solely with the
additional income tax, is that is should be divided into two equal parts,
because of the conjugal partnership existing between them. The learned
argument of counsel is mostly based upon the provisions of the Civil Code
establishing the sociedad de gananciales. The counter contentions of
appellees are that the taxes imposed by the Income Tax Law are as the name
implies taxes upon income tax and not upon capital and property; that the fact
that Madrigal was a married man, and his marriage contracted under the
provisions governing the conjugal partnership, has no bearing on income
considered as income, and that the distinction must be drawn between the
ordinary form of commercial partnership and the conjugal partnership of
spouses resulting from the relation of marriage.
DECISION.
From the point of view of test of faculty in taxation, no less than five answers
have been given the course of history. The final stage has been the selection
of income as the norm of taxation. (See Seligman, "The Income Tax,"
Introduction.) The Income Tax Law of the United States, extended to the
Philippine Islands, is the result of an effect on the part of the legislators to put
into statutory form this canon of taxation and of social reform. The aim has
been to mitigate the evils arising from inequalities of wealth by a progressive
scheme of taxation, which places the burden on those best able to pay. To
carry out this idea, public considerations have demanded an exemption
roughly equivalent to the minimum of subsistence. With these exceptions, the
income tax is supposed to reach the earnings of the entire non-governmental
property of the country. Such is the background of the Income Tax Law.
Income as contrasted with capital or property is to be the test. The essential
difference between capital and income is that capital is a fund; income is a
flow. A fund of property existing at an instant of time is called capital. A flow of
services rendered by that capital by the payment of money from it or any
other benefit rendered by a fund of capital in relation to such fund through a
period of time is called an income. Capital is wealth, while income is the
service of wealth. (See Fisher, "The Nature of Capital and Income.") The
Supreme Court of Georgia expresses the thought in the following figurative
language: "The fact is that property is a tree, income is the fruit; labor is a
tree, income the fruit; capital is a tree, income the fruit." (Waring vs. City of
Savannah [1878], 60 Ga., 93.) A tax on income is not a tax on property.
"Income," as here used, can be defined as "profits or gains." (London County
Council vs. Attorney-General [1901], A. C., 26; 70 L. J. K. B. N. S., 77; 83 L.
T. N. S., 605; 49 Week. Rep., 686; 4 Tax Cas., 265. See further Foster's
Income Tax, second edition [1915], Chapter IV; Black on Income Taxes,
second edition [1915], Chapter VIII; Gibbons vs. Mahon [1890], 136 U.S.,
549; and Towne vs. Eisner, decided by the United States Supreme Court,
January 7, 1918.)
A regulation of the United States Treasury Department relative to returns by
the husband and wife not living apart, contains the following:
The husband, as the head and legal representative of the household and
general custodian of its income, should make and render the return of the
aggregate income of himself and wife, and for the purpose of levying the
income tax it is assumed that he can ascertain the total amount of said
income. If a wife has a separate estate managed by herself as her own
separate property, and receives an income of more than $3,000, she may
make return of her own income, and if the husband has other net income,
making the aggregate of both incomes more than $4,000, the wife's return
should be attached to the return of her husband, or his income should be
included in her return, in order that a deduction of $4,000 may be made from
the aggregate of both incomes. The tax in such case, however, will be
imposed only upon so much of the aggregate income of both shall exceed
$4,000. If either husband or wife separately has an income equal to or in
excess of $3,000, a return of annual net income is required under the law,
and such return must include the income of both, and in such case the return
must be made even though the combined income of both be less than
$4,000. If the aggregate net income of both exceeds $4,000, an annual return
of their combined incomes must be made in the manner stated, although
neither one separately has an income of $3,000 per annum. They are jointly
and separately liable for such return and for the payment of the tax. The
single or married status of the person claiming the specific exemption shall be
determined as one of the time of claiming such exemption which return is
made, otherwise the status at the close of the year."
With these general observations relative to the Income Tax Law in force in the
Philippine Islands, we turn for a moment to consider the provisions of the Civil
Code dealing with the conjugal partnership. Recently in two elaborate
decisions in which a long line of Spanish authorities were cited, this court in
speaking of the conjugal partnership, decided that "prior to the liquidation the
interest of the wife and in case of her death, of her heirs, is an interest
inchoate, a mere expectancy, which constitutes neither a legal nor an
equitable estate, and does not ripen into title until there appears that there are
assets in the community as a result of the liquidation and settlement." (Nable
Jose vs. Nable Jose [1916], 15 Off. Gaz., 871; Manuel and Laxamana vs.
Losano [1918], 16 Off. Gaz., 1265.)
Susana Paterno, wife of Vicente Madrigal, has an inchoate right in the
property of her husband Vicente Madrigal during the life of the conjugal
partnership. She has an interest in the ultimate property rights and in the
ultimate ownership of property acquired as income after such income has
become capital. Susana Paterno has no absolute right to one-half the income
of the conjugal partnership. Not being seized of a separate estate, Susana
Paterno cannot make a separate return in order to receive the benefit of the
exemption which would arise by reason of the additional tax. As she has no
estate and income, actually and legally vested in her and entirely distinct from
her husband's property, the income cannot properly be considered the
separate income of the wife for the purposes of the additional tax. Moreover,
the Income Tax Law does not look on the spouses as individual partners in an
ordinary partnership. The husband and wife are only entitled to the exemption
of P8,000 specifically granted by the law. The higher schedules of the
additional tax directed at the incomes of the wealthy may not be partially
defeated by reliance on provisions in our Civil Code dealing with the conjugal
partnership and having no application to the Income Tax Law. The aims and
purposes of the Income Tax Law must be given effect.
The point we are discussing has heretofore been considered by the Attorney-
General of the Philippine Islands and the United States Treasury Department.
The decision of the latter overruling the opinion of the Attorney-General is as
follows:
TREASURY DEPARTMENT, Washington.
Income Tax.
FRANK MCINTYRE,
Chief, Bureau of Insular Affairs, War Department,
Washington, D. C.
SIR: This office is in receipt of your letter of June 22, 1915, transmitting copy
of correspondence "from the Philippine authorities relative to the method of
submission of income tax returns by marred person."
You advise that "The Governor-General, in forwarding the papers to the
Bureau, advises that the Insular Auditor has been authorized to suspend
action on the warrants in question until an authoritative decision on the points
raised can be secured from the Treasury Department."
From the correspondence it appears that Gregorio Araneta, married and living
with his wife, had an income of an amount sufficient to require the imposition
of the net income was properly computed and then both income and
deductions and the specific exemption were divided in half and two returns
made, one return for each half in the names respectively of the husband and
wife, so that under the returns as filed there would be an escape from the
additional tax; that Araneta claims the returns are correct on the ground under
the Philippine law his wife is entitled to half of his earnings; that Araneta has
dominion over the income and under the Philippine law, the right to determine
its use and disposition; that in this case the wife has no "separate estate"
within the contemplation of the Act of October 3, 1913, levying an income tax.
It appears further from the correspondence that upon the foregoing
explanation, tax was assessed against the entire net income against Gregorio
Araneta; that the tax was paid and an application for refund made, and that
the application for refund was rejected, whereupon the matter was submitted
to the Attorney-General of the Islands who holds that the returns were
correctly rendered, and that the refund should be allowed; and thereupon the
question at issue is submitted through the Governor-General of the Islands
and Bureau of Insular Affairs for the advisory opinion of this office.
By paragraph M of the statute, its provisions are extended to the Philippine
Islands, to be administered as in the United States but by the appropriate
internal-revenue officers of the Philippine Government. You are therefore
advised that upon the facts as stated, this office holds that for the Federal
Income Tax (Act of October 3, 1913), the entire net income in this case was
taxable to Gregorio Araneta, both for the normal and additional tax, and that
the application for refund was properly rejected.
The separate estate of a married woman within the contemplation of the
Income Tax Law is that which belongs to her solely and separate and apart
from her husband, and over which her husband has no right in equity. It may
consist of lands or chattels.
The statute and the regulations promulgated in accordance therewith provide
that each person of lawful age (not excused from so doing) having a net
income of $3,000 or over for the taxable year shall make a return showing the
facts; that from the net income so shown there shall be deducted $3,000
where the person making the return is a single person, or married and not
living with consort, and $1,000 additional where the person making the return
is married and living with consort; but that where the husband and wife both
make returns (they living together), the amount of deduction from the
aggregate of their several incomes shall not exceed $4,000.
The only occasion for a wife making a return is where she has income from a
sole and separate estate in excess of $3,000, but together they have an
income in excess of $4,000, in which the latter event either the husband or
wife may make the return but not both. In all instances the income of husband
and wife whether from separate estates or not, is taken as a whole for the
purpose of the normal tax. Where the wife has income from a separate estate
makes return made by her husband, while the incomes are added together
for the purpose of the normal tax they are taken separately for the purpose of
the additional tax. In this case, however, the wife has no separate income
within the contemplation of the Income Tax Law.
Respectfully,
DAVID A. GATES.
Acting Commissioner.
In connection with the decision above quoted, it is well to recall a few basic
ideas. The Income Tax Law was drafted by the Congress of the United States
and has been by the Congress extended to the Philippine Islands. Being thus
a law of American origin and being peculiarly intricate in its provisions, the
authoritative decision of the official who is charged with enforcing it has
peculiar force for the Philippines. It has come to be a well-settled rule that
great weight should be given to the construction placed upon a revenue law,
whose meaning is doubtful, by the department charged with its execution.
(U.S. vs. Cerecedo Hermanos y Cia. [1907], 209 U.S., 338; In re Allen [1903],
2 Phil., 630; Government of the Philippine Islands vs. Municipality of
Binalonan, and Roman Catholic Bishop of Nueva Segovia [1915], 32 Phil.,
634.) We conclude that the judgment should be as it is hereby affirmed with
costs against appellants. So ordered.

TAX BASE AND TAX RATES

G.R. No. L-46029 June 23, 1988


N.V. REEDERIJ "AMSTERDAM" and ROYAL INTEROCEAN LINES,
petitioners,
vs.
COMMISSIONER OF INTERNAL REVENUE, respondent.
GANCAYCO, J.:
The issue posed in this petition is the income tax liability of a foreign shipping
corporation which called on Philippine ports to load cargoes for foreign
destination on two occasions in 1963 and 1964, respectively, and which
collected freight fees on these transactions.
From March 27 to April 30, 1963, M.V. Amstelmeer and from September 24 to
October 28, 1964, MV "Amstelkroon, " both of which are vessels of petitioner
N.B. Reederij "AMSTERDAM," called on Philippine ports to load cargoes for
foreign destination. The freight fees for these transactions were paid abroad
in the amount of US $98,175.00 in 1963 and US $137,193.00 in 1964. In
these two instances, petitioner Royal Interocean Lines acted as husbanding
agent for a fee or commission on said vessels. No income tax appears to
have been paid by petitioner N.V. Reederij "AMSTERDAM" on the freight
receipts.
Respondent Commissioner of Internal Revenue, through his examiners, filed
the corresponding income tax returns for and in behalf of the former under
Section 15 of the National Internal Revenue Code. Applying the then
prevailing market conversion rate of P3.90 to the US $1.00, the gross receipts
of petitioner N.V. Reederij "Amsterdam" for 1963 and 1964 amounted to
P382,882.50 and P535,052.00, respectively. On June 30, 1967, respondent
Commissioner assessed said petitioner in the amounts of P193,973.20 and
P262,904.94 as deficiency income tax for 1963 and 1964, respectively, as "a
non-resident foreign corporation not engaged in trade or business in the
Philippines under Section 24 (b) (1) of the Tax Code.
On the assumption that the said petitioner is a foreign corporation engaged in
trade or business in the Philippines, on August 28, 1967, petitioner Royal
Interocean Lines filed an income tax return of the aforementioned vessels
computed at the exchange rate of P2.00 to USs1.00 1 and paid the tax thereon
in the amount of P1,835.52 and P9,448.94, respectively, pursuant to Section 24 (b)
(2) in relation to Section 37 (B) (e) of the National Internal Revenue Code and
Section 163 of Revenue Regulations No. 2. On the same two dates, petitioner
Royal Interocean Lines as the husbanding agent of petitioner N.V. Reederij
"AMSTERDAM" filed a written protest against the abovementioned assessment
made by the respondent Commissioner which protest was denied by said
respondent in a letter dated March 3, 1969: On March 31, 1969, petitioners filed a
petition for review with the respondent Court of Tax Appeals praying for the
cancellation of the subject assessment. After due hearing, the respondent court, on
December 1, 1976, rendered a decision modifying said assessments by eliminating
the 50% fraud compromise penalties imposed upon petitioners. Petitioners filed a
motion for reconsideration of said decision but this was denied by the respondent
court.
Hence, this petition for review where petitioners raised the following issues:
A. WHETHER N.V. REEDERIJ "AMSTERDAM" NOT HAVING ANY OFFICE
OR PLACE OF BUSINESS IN THE PHILIPPINES, WHOSE VESSELS
CALLED ON THE PHILIPPINE PORTS FOR THE PURPOSE OF LOADING
CARGOES ONLY TWICE-ONE IN 1963 AND ANOTHER IN 1964
SHOULD BE TAXED AS A FOREIGN CORPORATION NOT ENGAGED IN
TRADE OR BUSINESS IN THE PHILIPPINES UNDER SECTION 24(b) (1)
OF THE TAX CODE OR SHOULD BE TAXED AS A FOREIGN
CORPORATION ENGAGED IN TRADE OR BUSINESS IN THE
PHILIPPINES UNDER SECTION 24(b) (2) IN RELATION TO SECTION 37
(e) OF THE SAME CODE; AND
B. WHETHER THE FOREIGN EXCHANGE RECEIPTS OF N.V. REEDERIJ
"AMSTERDAM" SHOULD BE CONVERTED INTO PHILI PINE PESOS AT
THE OFFICIAL RATE OF P2.00 TO US $1.00, OR AT P3.90 TO US $1.00.
Petitioners contend that respondent court erred in holding that petitioner N.V.
Reederij "AMSTERDAM" is a non-resident foreign corporation because it
allegedly disregarded Section 163 of Revenue Regulations No. 2 (providing
for the determination of the net income of foreign corporations doing business
in the Philippines) and in holding that the foreign exchange ang e receipts of
said petitioner for purposes of computing its income tax should be converted
into Philippine pesos at the rate of P3.90 to US $1.00 instead of P2.00 to US
$1.00.
The petition is devoid of merit.
Petitioner N.V. Reederij "AMSTERDAM" is a foreign corporation not
authorized or licensed to do business in the Philippines. It does not have a
branch office in the Philippines and it made only two calls in Philippine ports,
one in 1963 and the other in 1964. In order that a foreign corporation may be
considered engaged in trade or business, its business transactions must be
continuous. A casual business activity in the Philippines by a foreign
corporation, as in the present case, does not amount to engaging in trade or
business in the Philippines for income tax purposes.
The Court reproduces with approval the following disquisition of the
respondent court
A corporation is itself a taxpaying entity and speaking generally, for purposes
of income tax, corporations are classified into (a) domestic corporations and
(b) foreign corporations. (Sec. 24(a) and (b), Tax Code.) Foreign corporations
are further classified into (1) resident foreign corporations and (2) non-
resident foreign corporations. (Sec. 24(b) (1) and (2). Tax Code.) A resident
foreign corporation is a foreign corporation engaged in trade or business
within the Philippines or having an office or place of business therein (Sec.
84(g), Tax Code) while a non- resident foreign corporation is a foreign
corporation not engaged in trade or business within the Philippines and not
having any office or place of business therein. (Sec. 84(h), Tax Code.)
A domestic corporation is taxed on its income from sources within and without
the Philippines, but a foreign corporation is taxed only on its income from
sources within the Philippines. (Sec. 24(a), Tax Code; Sec. 16, Rev. Regs.
No. 2.) However, while a foreign corporation doing business in the Philippines
is taxable on income solely from sources within the Philippines, it is permitted
to deductions from gross income but only to the extent connected with
income earned in the Philippines. (Secs. 24(b) (2) and 37, Tax Code.) On the
other hand, foreign corporations not doing business in the Philippines are
taxable on income from all sources within the Philippines, as interest,
dividends, rents, salaries, wages, premiums, annuities Compensations,
remunerations, emoluments, or other fixed or determinable annual or
periodical or casual gains, profits and income and capital gains" The tax is
30% (now 35%) of such gross income. (Sec. 24 (b) (1), Tax Code.)
At the time material to this case, certain corporations were given special
treatment, namely, building and loan associations operating as such in
accordance with Section 171 of the Corporation Law, educational institutions,
domestic life insurance companies and for" foreign life insurance companies
doing business in the Philippines. (Sec. 24(a) & (c), Tax Code.) It bears
emphasis, however, that foreign life insurance companies which were not
doing business in the Philippines were taxable as other foreign corporations
not authorized to do business in the Philippines. (Sec. 24(c) Tax Code.)
Now to the case at bar. Here, petitioner N.V. Reederij "Amsterdam" is a non-
resident foreign corporation, organized and existing under the laws of The
Netherlands with principal office in Amsterdam and not licensed to do
business in the Philippines. (pp. 8-81, CTA records.) As a non-resident foreign
corporation, it is thus a foreign corporation, not engaged in trade or business
within the Philippines and not having any office or place of business therein.
(Sec. 84(h), Tax Code.) As stated above, it is therefore taxable on income
from all sources within the Philippines, as interest, dividends, rents, salaries,
wages, premiums, annuities, compensations, remunerations, emoluments, or
other fixed or determinable annual or periodical or casual gains, profits and
income and capital gains, and the tax is equal to thirty per centum of such
amount, under Section 24(b) (1) of the Tax Code. The accent is on the words
of--`such amount." Accordingly, petitioner N. V. Reederij "Amsterdam" being a
non-resident foreign corporation, its taxable income for purposes of our
income tax law consists of its gross income from all sources within the
Philippines.
The law seems clear and specific. It thus calls for its application as worded as
it leaves no leeway for interpretation. The applicable provision imposes a tax
on foreign corporations falling under the classification of non-resident
corporations without any exceptions or conditions, unlike in the case of
foreign corporations engaged in trade or business within the Philippines
which contained (at the time material to this case) an exception with respect
to foreign life insurance companies. Adherence to the provision of the law,
which specifies and determines the taxable income of, and the rate of income
tax applicable to, non-resident foreign corporations, without mentioning any
exceptions, would therefore lead to the conclusion that petitioner N.V.
Reederij "Amsterdam" is subject to income tax on gross income from all
sources within the Philippines.
A foreign corporation engaged in trade or business within the Philippines, or
which has an office or place of business therein, is taxed on its total net
income received from all sources within the Philippines at the rate of 25%
upon the amount but which taxable net income does not exceed
P100,000.00, and 35% upon the amount but which taxable net income
exceeds P100,000.00. 2 On the other hand, a foreign corporation not engaged in
trade or business within the Philippmes and which does not have any office or place
of business therein is taxed on income received from all sources within the
Philippines at the rate of 35% of the gross income. 3
Petitioner relies on Section 24 (b) (2) and Section 37 (B) (e) of the Tax Code
and implementing Section 163 of the Income Tax Regulations but these
provisions refer to a foreign corporation engaged in trade or business in the
Philippines and not to a foreign corporation not engaged in trade or business
in the Philippines like petitioner-ship-owner herein. Thus, the respondent
court aptly ruled:
It must be stressed, however, that Section 37 (e) of the Code, as
implemented by Section 163 of the Regulations, provides the rule of the
determination of the net income taxable in the Philippines of a foreign
steamship company doing business in the Philippines. To assure that non-
resident foreign steamship companies not engaged in business in the
Philippines and not having any office or place of business herein are not
covered therein, the regulations explicitly and clearly provide that "the net
income of a foreign steamship co company doing business in or from this
country is ascertained," under the formula contained therein, "for the purpose
of the income tax.! The reason is easily discernible. As stated above, the
taxable income of non-resident foreign corporations consists of its gross
income from all sources within the Philippines. Accordingly, a foreign
steamgship corporation derives income partly from sources within and partly
from sources without the Philippines if it is carrying on a business of
transportation service between points in the Philippines and points outside
the Philippines. (Vol. 3, 1965, Federal Taxes, Par. 16389.) Only then does
Section 37 (e) of the Tax Code, are implemented by Section 163 of the
Regulations, apply in computing net income subject to tax. There is no basis
therefore for an assertion "that Section 37 (e) does not distinguish between a
foreign corporation engaged in business in the Philippines and a foreign
corporation not engaged in business in the Philippines."" (p. 84, CTA
records.) (Decision, pp. 11-12.)
The conversion rate of P2.00 to US $1.00 which petitioners claim should be
applicable to the income of petitioners for income tax purposes instead of
P3.90 to s1.00 is likewise untenable. The transactions involved in this case
are for the taxable years 1963 and 1964. Under Rep. Act No. 2609, the
monetary board was authorized to fix the legal conversion rate for foreign
exchange. The free market conversion rate during those years was P3.90 to
US $1.00.
This conversion rate issue was definitely settled by this Court in the case of
Commissioner of Internal Revenue vs. Royal Interocean Lines and the Court
of Tax Appeals 4 to wit:
It should be noted that on July 1 6, 1959, the policy incorporated in Circular
No. 20 and implemented in subsequent circulars was relaxed with the
enactment of Republic Act No. 2609 which directed the monetary authorities
to take steps for the adoption of a four-year program of gradual decontrol,
during which the Monetary Board, with the approval of the President, could
and did fix the conversion rate of the Philippine peso to the US dollar at a
ratio other than that prescribed in Section 48 of Republic Act 265. During the
period involved in the case at bar, the free market conversion rate ranged
from P3.47 to P3.65 to a US dollar at which rate the freight fees in question
were computed in the contested assessment. Inasmuch said frees were
revenues derived from foreign exchange transactions, it follows necessarily
that the petitioner was fully justified in computing the taxpayer's receipts at Id
free market rates.
xxx xxx xxx
The case of the United States Lines, on which the appealed decision of the
Court of Tax Appeals is anchored, refers to transactions that took place
before the approval of Republic Act 2609 on July 16, 1959 when the only legal rate of exchange obtaining
in the Philippines was P2 to US $1, and all foreign exchange had to be surrendered to the Central Bank
subject to its disposition pursuant to its own rules and regulations. Upon the other hand, the present case
refers to transactions that took place during the effectivity of Republic Act 2609 when there was, apart from
the parity rate, a legal free market conversion rate for foreign exchange transactions, which rate had been
fixed in open trading, such as those involved in the case at bar.
Indeed, in the course of the investigation conducted by the Commissioner on
the accounting records of petitioner Royal Interocean Lines, it was verified
that when said petitioner paid its agency fees for services rendered as
husbanding agent of the said vessels, it used the conversion rate of P3.90 to
US $1.00. 5 It is now estopped from claiming otherwise in this case.
WHEREFORE, the petition is DENIED with costs against petitioners. This decision
is immediately executory and no extension of time to file motion for reconsideration
shall be entertained.

G.R. No. 76573 September 14, 1989


MARUBENI CORPORATION (formerly Marubeni Iida, Co., Ltd.),
petitioner,
vs.
COMMISSIONER OF INTERNAL REVENUE AND COURT OF TAX
APPEALS, respondents.
Melquiades C. Gutierrez for petitioner.
The Solicitor General for respondents.

FERNAN, C.J.:
Petitioner, Marubeni Corporation, representing itself as a foreign corporation
duly organized and existing under the laws of Japan and duly licensed to
engage in business under Philippine laws with branch office at the 4th Floor,
FEEMI Building, Aduana Street, Intramuros, Manila seeks the reversal of the
decision of the Court of Tax Appeals 1 dated February 12, 1986 denying its claim
for refund or tax credit in the amount of P229,424.40 representing alleged
overpayment of branch profit remittance tax withheld from dividends by Atlantic Gulf
and Pacific Co. of Manila (AG&P).
The following facts are undisputed: Marubeni Corporation of Japan has equity
investments in AG&P of Manila. For the first quarter of 1981 ending March 31,
AG&P declared and paid cash dividends to petitioner in the amount of
P849,720 and withheld the corresponding 10% final dividend tax thereon.
Similarly, for the third quarter of 1981 ending September 30, AG&P declared
and paid P849,720 as cash dividends to petitioner and withheld the
corresponding 10% final dividend tax thereon. 2
AG&P directly remitted the cash dividends to petitioner's head office in Tokyo,
Japan, net not only of the 10% final dividend tax in the amounts of P764,748
for the first and third quarters of 1981, but also of the withheld 15% profit
remittance tax based on the remittable amount after deducting the final
withholding tax of 10%. A schedule of dividends declared and paid by AG&P
to its stockholder Marubeni Corporation of Japan, the 10% final intercorporate
dividend tax and the 15% branch profit remittance tax paid thereon, is shown
below:

1981
FIRST QUARTER (three months ended 3.31.81) (In Pesos)
THIRD QUARTER (three months ended 9.30.81)
TOTAL OF FIRST and THIRD quarters
Cash Dividends Paid
849,720.44
849,720.00
1,699,440.00
10% Dividend Tax Withheld
84,972.00
84,972.00
169,944.00
Cash Dividend net of 10% Dividend Tax Withheld
764,748.00
764,748.00
1,529,496.00
15% Branch Profit Remittance Tax Withheld
114,712.20
114,712.20
229,424.40 3
Net Amount Remitted to Petitioner
650,035.80
650,035.80
1,300,071.60

The 10% final dividend tax of P84,972 and the 15% branch profit remittance
tax of P114,712.20 for the first quarter of 1981 were paid to the Bureau of
Internal Revenue by AG&P on April 20, 1981 under Central Bank Receipt No.
6757880. Likewise, the 10% final dividend tax of P84,972 and the 15%
branch profit remittance tax of P114,712 for the third quarter of 1981 were
paid to the Bureau of Internal Revenue by AG&P on August 4, 1981 under
Central Bank Confirmation Receipt No. 7905930. 4
Thus, for the first and third quarters of 1981, AG&P as withholding agent paid
15% branch profit remittance on cash dividends declared and remitted to
petitioner at its head office in Tokyo in the total amount of P229,424.40 on
April 20 and August 4, 1981. 5
In a letter dated January 29, 1981, petitioner, through the accounting firm
Sycip, Gorres, Velayo and Company, sought a ruling from the Bureau of
Internal Revenue on whether or not the dividends petitioner received from
AG&P are effectively connected with its conduct or business in the
Philippines as to be considered branch profits subject to the 15% profit
remittance tax imposed under Section 24 (b) (2) of the National Internal
Revenue Code as amended by Presidential Decrees Nos. 1705 and 1773.
In reply to petitioner's query, Acting Commissioner Ruben Ancheta ruled:
Pursuant to Section 24 (b) (2) of the Tax Code, as amended, only profits
remitted abroad by a branch office to its head office which are effectively
connected with its trade or business in the Philippines are subject to the 15%
profit remittance tax. To be effectively connected it is not necessary that the
income be derived from the actual operation of taxpayer-corporation's trade
or business; it is sufficient that the income arises from the business activity in
which the corporation is engaged. For example, if a resident foreign
corporation is engaged in the buying and selling of machineries in the
Philippines and invests in some shares of stock on which dividends are
subsequently received, the dividends thus earned are not considered
'effectively connected' with its trade or business in this country. (Revenue
Memorandum Circular No. 55-80).
In the instant case, the dividends received by Marubeni from AG&P are not
income arising from the business activity in which Marubeni is engaged.
Accordingly, said dividends if remitted abroad are not considered branch
profits for purposes of the 15% profit remittance tax imposed by Section 24
(b) (2) of the Tax Code, as amended . . . 6
Consequently, in a letter dated September 21, 1981 and filed with the
Commissioner of Internal Revenue on September 24, 1981, petitioner
claimed for the refund or issuance of a tax credit of P229,424.40
"representing profit tax remittance erroneously paid on the dividends remitted
by Atlantic Gulf and Pacific Co. of Manila (AG&P) on April 20 and August 4,
1981 to ... head office in Tokyo. 7
On June 14, 1982, respondent Commissioner of Internal Revenue denied
petitioner's claim for refund/credit of P229,424.40 on the following grounds:
While it is true that said dividends remitted were not subject to the 15% profit
remittance tax as the same were not income earned by a Philippine Branch of
Marubeni Corporation of Japan; and neither is it subject to the 10%
intercorporate dividend tax, the recipient of the dividends, being a non-
resident stockholder, nevertheless, said dividend income is subject to the 25
% tax pursuant to Article 10 (2) (b) of the Tax Treaty dated February 13, 1980
between the Philippines and Japan.
Inasmuch as the cash dividends remitted by AG&P to Marubeni Corporation,
Japan is subject to 25 % tax, and that the taxes withheld of 10 % as
intercorporate dividend tax and 15 % as profit remittance tax totals (sic) 25 %,
the amount refundable offsets the liability, hence, nothing is left to be
refunded. 8
Petitioner appealed to the Court of Tax Appeals which affirmed the denial of
the refund by the Commissioner of Internal Revenue in its assailed judgment
of February 12, 1986. 9
In support of its rejection of petitioner's claimed refund, respondent Tax Court
explained:
Whatever the dialectics employed, no amount of sophistry can ignore the fact
that the dividends in question are income taxable to the Marubeni Corporation
of Tokyo, Japan. The said dividends were distributions made by the Atlantic,
Gulf and Pacific Company of Manila to its shareholder out of its profits on the
investments of the Marubeni Corporation of Japan, a non-resident foreign
corporation. The investments in the Atlantic Gulf & Pacific Company of the
Marubeni Corporation of Japan were directly made by it and the dividends on
the investments were likewise directly remitted to and received by the
Marubeni Corporation of Japan. Petitioner Marubeni Corporation Philippine
Branch has no participation or intervention, directly or indirectly, in the
investments and in the receipt of the dividends. And it appears that the funds
invested in the Atlantic Gulf & Pacific Company did not come out of the funds
infused by the Marubeni Corporation of Japan to the Marubeni Corporation
Philippine Branch. As a matter of fact, the Central Bank of the Philippines, in
authorizing the remittance of the foreign exchange equivalent of (sic) the
dividends in question, treated the Marubeni Corporation of Japan as a non-
resident stockholder of the Atlantic Gulf & Pacific Company based on the
supporting documents submitted to it.
Subject to certain exceptions not pertinent hereto, income is taxable to the
person who earned it. Admittedly, the dividends under consideration were
earned by the Marubeni Corporation of Japan, and hence, taxable to the said
corporation. While it is true that the Marubeni Corporation Philippine Branch
is duly licensed to engage in business under Philippine laws, such dividends
are not the income of the Philippine Branch and are not taxable to the said
Philippine branch. We see no significance thereto in the identity concept or
principal-agent relationship theory of petitioner because such dividends are
the income of and taxable to the Japanese corporation in Japan and not to
the Philippine branch. 10
Hence, the instant petition for review.
It is the argument of petitioner corporation that following the principal-agent
relationship theory, Marubeni Japan is likewise a resident foreign corporation
subject only to the 10 % intercorporate final tax on dividends received from a
domestic corporation in accordance with Section 24(c) (1) of the Tax Code of
1977 which states:
Dividends received by a domestic or resident foreign corporation liable to tax
under this Code (1) Shall be subject to a final tax of 10% on the total
amount thereof, which shall be collected and paid as provided in Sections 53
and 54 of this Code ....
Public respondents, however, are of the contrary view that Marubeni, Japan,
being a non-resident foreign corporation and not engaged in trade or
business in the Philippines, is subject to tax on income earned from Philippine
sources at the rate of 35 % of its gross income under Section 24 (b) (1) of the
same Code which reads:
(b) Tax on foreign corporations (1) Non-resident corporations. A foreign
corporation not engaged in trade or business in the Philippines shall pay a tax
equal to thirty-five per cent of the gross income received during each taxable
year from all sources within the Philippines as ... dividends ....
but expressly made subject to the special rate of 25% under Article 10(2) (b)
of the Tax Treaty of 1980 concluded between the Philippines and Japan. 11
Thus:
Article 10 (1) Dividends paid by a company which is a resident of a
Contracting State to a resident of the other Contracting State may be taxed in
that other Contracting State.
(2) However, such dividends may also be taxed in the Contracting State of
which the company paying the dividends is a resident, and according to the
laws of that Contracting State, but if the recipient is the beneficial owner of
the dividends the tax so charged shall not exceed;
(a) . . .
(b) 25 per cent of the gross amount of the dividends in all other cases.
Central to the issue of Marubeni Japan's tax liability on its dividend income
from Philippine sources is therefore the determination of whether it is a
resident or a non-resident foreign corporation under Philippine laws.
Under the Tax Code, a resident foreign corporation is one that is "engaged in
trade or business" within the Philippines. Petitioner contends that precisely
because it is engaged in business in the Philippines through its Philippine
branch that it must be considered as a resident foreign corporation. Petitioner
reasons that since the Philippine branch and the Tokyo head office are one
and the same entity, whoever made the investment in AG&P, Manila does not
matter at all. A single corporate entity cannot be both a resident and a non-
resident corporation depending on the nature of the particular transaction
involved. Accordingly, whether the dividends are paid directly to the head
office or coursed through its local branch is of no moment for after all, the
head office and the office branch constitute but one corporate entity, the
Marubeni Corporation, which, under both Philippine tax and corporate laws, is
a resident foreign corporation because it is transacting business in the
Philippines.
The Solicitor General has adequately refuted petitioner's arguments in this
wise:
The general rule that a foreign corporation is the same juridical entity as its
branch office in the Philippines cannot apply here. This rule is based on the
premise that the business of the foreign corporation is conducted through its
branch office, following the principal agent relationship theory. It is understood
that the branch becomes its agent here. So that when the foreign corporation
transacts business in the Philippines independently of its branch, the
principal-agent relationship is set aside. The transaction becomes one of the
foreign corporation, not of the branch. Consequently, the taxpayer is the
foreign corporation, not the branch or the resident foreign corporation.
Corollarily, if the business transaction is conducted through the branch office,
the latter becomes the taxpayer, and not the foreign corporation. 12
In other words, the alleged overpaid taxes were incurred for the remittance of
dividend income to the head office in Japan which is a separate and distinct
income taxpayer from the branch in the Philippines. There can be no other
logical conclusion considering the undisputed fact that the investment
(totalling 283.260 shares including that of nominee) was made for purposes
peculiarly germane to the conduct of the corporate affairs of Marubeni Japan,
but certainly not of the branch in the Philippines. It is thus clear that petitioner,
having made this independent investment attributable only to the head office,
cannot now claim the increments as ordinary consequences of its trade or
business in the Philippines and avail itself of the lower tax rate of 10 %.
But while public respondents correctly concluded that the dividends in dispute
were neither subject to the 15 % profit remittance tax nor to the 10 %
intercorporate dividend tax, the recipient being a non-resident stockholder,
they grossly erred in holding that no refund was forthcoming to the petitioner
because the taxes thus withheld totalled the 25 % rate imposed by the
Philippine-Japan Tax Convention pursuant to Article 10 (2) (b).
To simply add the two taxes to arrive at the 25 % tax rate is to disregard a
basic rule in taxation that each tax has a different tax basis. While the tax on
dividends is directly levied on the dividends received, "the tax base upon
which the 15 % branch profit remittance tax is imposed is the profit actually
remitted abroad." 13
Public respondents likewise erred in automatically imposing the 25 % rate
under Article 10 (2) (b) of the Tax Treaty as if this were a flat rate. A closer
look at the Treaty reveals that the tax rates fixed by Article 10 are the
maximum rates as reflected in the phrase "shall not exceed." This means that
any tax imposable by the contracting state concerned should not exceed the
25 % limitation and that said rate would apply only if the tax imposed by our
laws exceeds the same. In other words, by reason of our bilateral
negotiations with Japan, we have agreed to have our right to tax limited to a
certain extent to attain the goals set forth in the Treaty.
Petitioner, being a non-resident foreign corporation with respect to the
transaction in question, the applicable provision of the Tax Code is Section 24
(b) (1) (iii) in conjunction with the Philippine-Japan Treaty of 1980. Said
section provides:
(b) Tax on foreign corporations. (1) Non-resident corporations ... (iii) On
dividends received from a domestic corporation liable to tax under this
Chapter, the tax shall be 15% of the dividends received, which shall be
collected and paid as provided in Section 53 (d) of this Code, subject to the
condition that the country in which the non-resident foreign corporation is
domiciled shall allow a credit against the tax due from the non-resident
foreign corporation, taxes deemed to have been paid in the Philippines
equivalent to 20 % which represents the difference between the regular tax
(35 %) on corporations and the tax (15 %) on dividends as provided in this
Section; ....
Proceeding to apply the above section to the case at bar, petitioner, being a
non-resident foreign corporation, as a general rule, is taxed 35 % of its gross
income from all sources within the Philippines. [Section 24 (b) (1)].
However, a discounted rate of 15% is given to petitioner on dividends
received from a domestic corporation (AG&P) on the condition that its
domicile state (Japan) extends in favor of petitioner, a tax credit of not less
than 20 % of the dividends received. This 20 % represents the difference
between the regular tax of 35 % on non-resident foreign corporations which
petitioner would have ordinarily paid, and the 15 % special rate on dividends
received from a domestic corporation.
Consequently, petitioner is entitled to a refund on the transaction in question
to be computed as follows:
Total cash dividend paid ................P1,699,440.00
less 15% under Sec. 24
(b) (1) (iii ) .........................................254,916.00
------------------
Cash dividend net of 15 % tax
due petitioner ...............................P1,444.524.00
less net amount
actually remitted .............................1,300,071.60
-------------------
Amount to be refunded to petitioner
representing overpayment of
taxes on dividends remitted ..............P 144 452.40
===========
It is readily apparent that the 15 % tax rate imposed on the dividends
received by a foreign non-resident stockholder from a domestic corporation
under Section 24 (b) (1) (iii) is easily within the maximum ceiling of 25 % of
the gross amount of the dividends as decreed in Article 10 (2) (b) of the Tax
Treaty.
There is one final point that must be settled. Respondent Commissioner of
Internal Revenue is laboring under the impression that the Court of Tax
Appeals is covered by Batas Pambansa Blg. 129, otherwise known as the
Judiciary Reorganization Act of 1980. He alleges that the instant petition for
review was not perfected in accordance with Batas Pambansa Blg. 129 which
provides that "the period of appeal from final orders, resolutions, awards,
judgments, or decisions of any court in all cases shall be fifteen (15) days
counted from the notice of the final order, resolution, award, judgment or
decision appealed from ....
This is completely untenable. The cited BP Blg. 129 does not include the
Court of Tax Appeals which has been created by virtue of a special law,
Republic Act No. 1125. Respondent court is not among those courts
specifically mentioned in Section 2 of BP Blg. 129 as falling within its scope.
Thus, under Section 18 of Republic Act No. 1125, a party adversely affected
by an order, ruling or decision of the Court of Tax Appeals is given thirty (30)
days from notice to appeal therefrom. Otherwise, said order, ruling, or
decision shall become final.
Records show that petitioner received notice of the Court of Tax Appeals's
decision denying its claim for refund on April 15, 1986. On the 30th day, or on
May 15, 1986 (the last day for appeal), petitioner filed a motion for
reconsideration which respondent court subsequently denied on November
17, 1986, and notice of which was received by petitioner on November 26,
1986. Two days later, or on November 28, 1986, petitioner simultaneously
filed a notice of appeal with the Court of Tax Appeals and a petition for review
with the Supreme Court. 14 From the foregoing, it is evident that the instant appeal
was perfected well within the 30-day period provided under R.A. No. 1125, the
whole 30-day period to appeal having begun to run again from notice of the denial
of petitioner's motion for reconsideration.
WHEREFORE, the questioned decision of respondent Court of Tax Appeals
dated February 12, 1986 which affirmed the denial by respondent
Commissioner of Internal Revenue of petitioner Marubeni Corporation's claim
for refund is hereby REVERSED. The Commissioner of Internal Revenue is
ordered to refund or grant as tax credit in favor of petitioner the amount of
P144,452.40 representing overpayment of taxes on dividends received. No
costs.

G.R. No. 103092 July 21, 1994


BANK OF AMERICA NT & SA, petitioner,
vs.
HONORABLE COURT OF APPEALS, AND THE COMMISSIONER OF
INTERNAL REVENUE, respondents.
G.R. No. 103106 July 21, 1994
BANK OF AMERICA NT & SA, petitioner,
vs.
THE HONORABLE COURT OF APPEALS AND THE COMMISSIONER OF
INTERNAL REVENUE, respondents.
Sycip, Salazar, Hernandez & Gatmaitan and Agcaoili & Associates for
petitioner.

VITUG, J.:
Section 24(b) (2) (ii) of the National Internal Revenue Code, in the language it
was worded in 1982 (the taxable period relevant to the case at bench),
provided, in part, thusly:
Sec. 24. Rates of tax on corporations. . . .
(b) Tax on foreign corporations. . . .
(2) (ii) Tax on branch profit and remittances.
Any profit remitted abroad by a branch to its head office shall be subject to a
tax of fifteen per cent (15%) . . . ."
Petitioner Bank of America NT & SA argues that the 15% branch profit
remittance tax on the basis of the above provision should be assessed on the
amount actually remitted abroad, which is to say that the 15% profit
remittance tax itself should not form part of the tax base. Respondent
Commissioner of Internal Revenue, contending otherwise, holds the position
that, in computing the 15% remittance tax, the tax should be inclusive of the
sum deemed remitted.
The statement of facts made by the Court of Tax Appeals, later adopted by
the Court of Appeals, and not in any serious dispute by the parties, can be
quoted thusly:
Petitioner is a foreign corporation duly licensed to engage in business in the
Philippines with Philippine branch office at BA Lepanto Bldg., Paseo de
Roxas, Makati, Metro Manila. On July 20, 1982 it paid 15% branch profit
remittance tax in the amount of P7,538,460.72 on profit from its regular
banking unit operations and P445,790.25 on profit from its foreign currency
deposit unit operations or a total of P7,984,250.97. The tax was based on net
profits after income tax without deducting the amount corresponding to the
15% tax.
Petitioner filed a claim for refund with the Bureau of Internal Revenue of that
portion of the payment which corresponds to the 15% branch profit remittance
tax, on the ground that the tax should have been computed on the basis of
profits actually remitted, which is P45,244,088.85, and not on the amount
before profit remittance tax, which is P53,228,339.82. Subsequently, without
awaiting respondent's decision, petitioner filed a petition for review on June
14, 1984 with this Honorable Court for the recovery of the amount of
P1,041,424.03 computed as follows:
Net Profits After Profit Tax Due Alleged
Income Tax But Remittance Alleged by Overpayment
Before Profit Tax Paid Petitioner Item 1-2
Remittance Tax _________ _________ ___________
A. Regular Banking
Unit Operations
(P50,256,404.82)
1. Computation of BIR
15% x P50,256,404.82 - P7,538,460.72
2. Computation of
Petitioner
- P50,256,404.82 x 15% P6,555,183.24 P983,277.48
1.15
B. Foreign Currency
Deposit Unit
Operations
(P2,971,935)
1. Computation of BIR
15% x - P2,971,935.00 P445,790.25
2. Computation of
Petitioner
- P2,971,935.00 x 15% P387,643.70 P58,146.55
T O T A L. . P7,984,250.97 P6,942,286.94 P1,041,424.02" 1
The Court of Tax Appeals upheld petitioner bank in its claim for refund. The
Commissioner of Internal Revenue filed a timely appeal to the Supreme Court
(docketed G.R. No. 76512) which referred it to the Court of Appeals following
this Court's pronouncement in Development Bank of the Philippines vs. Court
of Appeals, et al. (180 SCRA 609). On 19 September 1990, the Court of
Appeals set aside the decision of the Court of Tax Appeals. Explaining its
reversal of the tax court's decision, the appellate court said:
The Court of Tax Appeals sought to deduce legislative intent vis-a-vis the
aforesaid law through an analysis of the wordings thereof, which to their
minds reveal an intent to mitigate at least the harshness of successive
taxation. The use of the word remitted may well be understood as referring to
that part of the said total branch profits which would be sent to the head office
as distinguished from the total profits of the branch (not all of which need be
sent or would be ordered remitted abroad). If the legislature indeed had
wanted to mitigate the harshness of successive taxation, it would have been
simpler to just lower the rates without in effect requiring the relatively novel
and complicated way of computing the tax, as envisioned by the herein
private respondent. The same result would have been achieved. 2
Hence, these petitions for review in G.R. No. 103092 and G.R.
No. 103106 (filed separately due to inadvertence) by the law firms of "Agcaoili
and Associates" and of "Sycip, Salazar, Hernandez and Gatmaitan" in
representation of petitioner bank.
We agree with the Court of Appeals that not much reliance can be made on
our decision in Burroughs Limited vs. Commission of Internal Revenue (142
SCRA 324), for there we ruled against the Commissioner mainly on the basis
of what the Court so then perceived as his position in a 21 January 1980
ruling the reversal of which, by his subsequent ruling of 17 March 1982, could
not apply retroactively against Burroughs in conformity with Section 327 (now
Section 246, re: non-retroactivity of rulings) of the National Internal Revenue
Code. Hence, we held:
Petitioner's aforesaid contention is without merit. What is applicable in the
case at bar is still the Revenue Ruling of January 21, 1980 because private
respondent Burroughs Limited paid the branch profit remittance tax in
question on March 14, 1979. Memorandum Circular
No. 8-82 dated March 17, 1982 cannot be given retroactive effect in the light
of Section 327 of the National Internal Revenue Code which
provides
Sec. 327. Non-retroactivity of rulings. Any revocation, modification, or reversal
of any of the rules and regulations promulgated in accordance with the
preceding section or any of the rulings or circulars promulgated by the
Commissioner shall not be given retroactive application if the revocation,
modification, or reversal will be prejudicial to the taxpayer except in the
following cases (a) where the taxpayer deliberately misstates or omits
material facts from his return or in any document required of him by the
Bureau of Internal Revenue; (b) where the facts subsequently gathered by
the Bureau of Internal Revenue are materially different from the facts on
which the ruling is based, or (c) where the taxpayer acted in bad faith. (ABS-
CBN Broadcasting Corp. v. CTA, 108 SCRA 151-152)
The prejudice that would result to private respondent Burroughs Limited by a
retroactive application of Memorandum Circular No. 8-82 is beyond question
for it would be deprived of the substantial amount of P172,058.90. And,
insofar as the enumerated exceptions are concerned, admittedly, Burroughs
Limited does not fall under any of them.
The Court of Tax Appeals itself commented similarly when it observed thusly
in its decision:
In finding the Commissioner's contention without merit, this Court however
ruled against the applicability of Revenue Memorandum Circular No. 8-82
dated March 17, 1982 to the Burroughs Limited case because the taxpayer
paid the branch profit remittance tax involved therein on March 14, 1979 in
accordance with the ruling of the Commissioner of Internal Revenue dated
January 21, 1980. In view of Section 327 of the then in force National Internal
Revenue Code, Revenue Memorandum Circular No. 8-82 dated March 17,
1982 cannot be given retroactive effect because any revocation or
modification of any ruling or circular of the Bureau of Internal Revenue should
not be given retroactive application if such revocation or modification will,
subject to certain exceptions not pertinent thereto, prejudice taxpayers. 3
The Solicitor General correctly points out that almost invariably in an ad
valorem tax, the tax paid or withheld is not deducted from the tax base. Such
impositions as the ordinary income tax, estate and gift taxes, and the value
added tax are generally computed in like manner. In these cases, however, it
is so because the law, in defining the tax base and in providing for tax
withholding, clearly spells it out to be such. As so well expounded by the Tax
Court
. . . In all the situations . . . where the mechanism of withholding of taxes at
source operates to ensure collection of the tax, and which respondent claims
the base on which the tax is computed is the amount to be paid or remitted,
the law applicable expressly, specifically and unequivocally mandates that the
tax is on the total amount thereof which shall be collected and paid as
provided in Sections 53 and 54 of the Tax Code. Thus:
Dividends received by an individual who is a citizen or resident of the
Philippines from a domestic corporation, shall be subject to a final tax at the
rate of fifteen (15%) per cent on the total amount thereof, which shall be
collected and paid as provided in Sections 53 and 54 of this Code. (Emphasis
supplied; Sec. 21, Tax Code)
Interest from Philippine Currency bank deposits and yield from deposit
substitutes whether received by citizens of the Philippines or by resident alien
individuals, shall be subject to a final tax as follows: (a) 15% of the interest or
savings deposits, and (b) 20% of the interest on time deposits and yield from
deposits substitutes, which shall be collected and paid as provided in
Sections 53 and 54 of this Code: . . . (Emphasis supplied; Sec. 21, Tax Code
applicable.)
And on rental payments payable by the lessee to the lessor (at 5%), also
cited by respondent, Section 1, paragraph (C), of Revenue Regulations No.
13-78, November 1, 1978, provides that:
Section 1. Income payments subject to withholding tax and rates prescribed
therein. Except as therein otherwise provided, there shall be withheld a
creditable income tax at the rates herein specified for each class of payee
from the following items of income payments to persons residing in the
Philippines.
xxx xxx xxx
(C) Rentals When the gross rental or the payment required to be made as
a condition to the continued use or possession of property, whether real or
personal, to which the payor or obligor has not taken or is not taking title or in
which he has no equity, exceeds five hundred pesos (P500.00) per contract
or payment whichever is greater five per centum (5%).
Note that the basis of the 5% withholding tax, as expressly and
unambiguously provided therein, is on the gross rental. Revenue Regulations
No. 13-78 was promulgated pursuant to Section 53(f) of the then in force
National Internal Revenue Code which authorized the Minister of Finance,
upon recommendation of the Commissioner of Internal Revenue, to require
the withholding of income tax on the same items of income payable to
persons (natural or judicial) residing in the Philippines by the persons making
such payments at the rate of not less than 2 1/2% but not more than 35%
which are to be credited against the income tax liability of the taxpayer for the
taxable year.
On the other hand, there is absolutely nothing in Section 24(b) (2) (ii), supra,
which indicates that the 15% tax on branch profit remittance is on the total
amount of profit to be remitted abroad which shall be collected and paid in
accordance with the tax withholding device provided in Sections 53 and 54 of
the Tax Code. The statute employs "Any profit remitted abroad by a branch to
its head office shall be subject to a tax of fifteen per cent (15%)" without
more. Nowhere is there said of "base on the total amount actually applied for
by the branch with the Central Bank of the Philippines as profit to be remitted
abroad, which shall be collected and paid as provided in Sections 53 and 54
of this Code." Where the law does not qualify that the tax is imposed and
collected at source based on profit to be remitted abroad, that qualification
should not be read into the law. It is a basic rule of statutory construction that
there is no safer nor better canon of interpretation than that when the
language of the law is clear and unambiguous, it should be applied as written.
And to our mind, the term "any profit remitted abroad" can only mean such
profit as is "forwarded, sent, or transmitted abroad" as the word "remitted" is
commonly and popularly accepted and understood. To say therefore that the
tax on branch profit remittance is imposed and collected at source and
necessarily the tax base should be the amount actually applied for the branch
with the Central Bank as profit to be remitted abroad is to ignore the
unmistakable meaning of plain words. 4
In the 15% remittance tax, the law specifies its own tax base to be on the
"profit remitted abroad." There is absolutely nothing equivocal or uncertain
about the language of the provision. The tax is imposed on the amount sent
abroad, and the law (then in force) calls for nothing further. The taxpayer is a
single entity, and it should be understandable if, such as in this case, it is the
local branch of the corporation, using its own local funds, which remits the tax
to the Philippine Government.
The remittance tax was conceived in an attempt to equalize the income tax
burden on foreign corporations maintaining, on the one hand, local branch
offices and organizing, on the other hand, subsidiary domestic corporations
where at least a majority of all the latter's shares of stock are owned by such
foreign corporations. Prior to the amendatory provisions of the Revenue
Code, local branches were made to pay only the usual corporate income tax
of 25%-35% on net income (now a uniform 35%) applicable to resident
foreign corporations (foreign corporations doing business in the Philippines).
While Philippine subsidiaries of foreign corporations were subject to the same
rate of 25%-35% (now also a uniform 35%) on their net income, dividend
payments, however, were additionally subjected to a 15% (withholding) tax
(reduced conditionally from 35%). In order to avert what would otherwise
appear to be an unequal tax treatment on such subsidiaries vis-a-vis local
branch offices, a 20%, later reduced to 15%, profit remittance tax was
imposed on local branches on their remittances of profits abroad. But this is
where the tax pari-passu ends between domestic branches and subsidiaries
of foreign corporations.
The Solicitor General suggests that the analogy should extend to the ordinary
application of the withholding tax system and so with the rule on constructive
remittance concept as well. It is difficult to accept the proposition. In the
operation of the withholding tax system, the payee is the taxpayer, the person
on whom the tax is imposed, while the payor, a separate entity, acts no more
than an agent of the government for the collection of the tax in order to
ensure its payment. Obviously, the amount thereby used to settle the tax
liability is deemed sourced from the proceeds constitutive of the tax base.
Since the payee, not the payor, is the real taxpayer, the rule on constructive
remittance (or receipt) can be easily rationalized, if not indeed, made clearly
manifest. It is hardly the case, however, in the imposition of the 15%
remittance tax where there is but one taxpayer using its own domestic funds
in the payment of the tax. To say that there is constructive remittance even of
such funds would be stretching far too much that imaginary rule. Sound logic
does not defy but must concede to facts.
We hold, accordingly, that the written claim for refund of the excess tax
payment filed, within the two-year prescriptive period, with the Court of Tax
Appeals has been lawfully made.
WHEREFORE, the decision of the Court of Appeals appealed from is
REVERSED and SET ASIDE, and that of the Court of Tax Appeals is
REINSTATED.

G.R. No. L-66838 December 2, 1991


COMMISSIONER OF INTERNAL REVENUE, petitioner,
vs.
PROCTER & GAMBLE PHILIPPINE MANUFACTURING CORPORATION
and THE COURT OF TAX APPEALS, respondents.
T.A. Tejada & C.N. Lim for private respondent.

RESOLUTION

FELICIANO, J.:p
For the taxable year 1974 ending on 30 June 1974, and the taxable year
1975 ending 30 June 1975, private respondent Procter and Gamble
Philippine Manufacturing Corporation ("P&G-Phil.") declared dividends
payable to its parent company and sole stockholder, Procter and Gamble Co.,
Inc. (USA) ("P&G-USA"), amounting to P24,164,946.30, from which dividends
the amount of P8,457,731.21 representing the thirty-five percent (35%)
withholding tax at source was deducted.
On 5 January 1977, private respondent P&G-Phil. filed with petitioner
Commissioner of Internal Revenue a claim for refund or tax credit in the
amount of P4,832,989.26 claiming, among other things, that pursuant to
Section 24 (b) (1) of the National Internal Revenue Code ("NITC"), 1 as amended
by Presidential Decree No. 369, the applicable rate of withholding tax on the dividends remitted was only
fifteen percent (15%) (and not thirty-five percent [35%]) of the dividends.
There being no responsive action on the part of the Commissioner, P&G-
Phil., on 13 July 1977, filed a petition for review with public respondent Court
of Tax Appeals ("CTA") docketed as CTA Case No. 2883. On 31 January
1984, the CTA rendered a decision ordering petitioner Commissioner to
refund or grant the tax credit in the amount of P4,832,989.00.
On appeal by the Commissioner, the Court through its Second Division
reversed the decision of the CTA and held that:
(a) P&G-USA, and not private respondent P&G-Phil., was the proper party to
claim the refund or tax credit here involved;
(b) there is nothing in Section 902 or other provisions of the US Tax Code that
allows a credit against the US tax due from P&G-USA of taxes deemed to
have been paid in the Philippines equivalent to twenty percent (20%) which
represents the difference between the regular tax of thirty-five percent (35%)
on corporations and the tax of fifteen percent (15%) on dividends; and
(c) private respondent P&G-Phil. failed to meet certain conditions necessary
in order that "the dividends received by its non-resident parent company in
the US (P&G-USA) may be subject to the preferential tax rate of 15% instead
of 35%."
These holdings were questioned in P&G-Phil.'s Motion for Re-consideration
and we will deal with them seriatim in this Resolution resolving that Motion.
I
1. There are certain preliminary aspects of the question of the capacity of
P&G-Phil. to bring the present claim for refund or tax credit, which need to be
examined. This question was raised for the first time on appeal, i.e., in the
proceedings before this Court on the Petition for Review filed by the
Commissioner of Internal Revenue. The question was not raised by the
Commissioner on the administrative level, and neither was it raised by him
before the CTA.
We believe that the Bureau of Internal Revenue ("BIR") should not be allowed
to defeat an otherwise valid claim for refund by raising this question of alleged
incapacity for the first time on appeal before this Court. This is clearly a
matter of procedure. Petitioner does not pretend that P&G-Phil., should it
succeed in the claim for refund, is likely to run away, as it were, with the
refund instead of transmitting such refund or tax credit to its parent and sole
stockholder. It is commonplace that in the absence of explicit statutory
provisions to the contrary, the government must follow the same rules of
procedure which bind private parties. It is, for instance, clear that the
government is held to compliance with the provisions of Circular No. 1-88 of
this Court in exactly the same way that private litigants are held to such
compliance, save only in respect of the matter of filing fees from which the
Republic of the Philippines is exempt by the Rules of Court.
More importantly, there arises here a question of fairness should the BIR,
unlike any other litigant, be allowed to raise for the first time on appeal
questions which had not been litigated either in the lower court or on the
administrative level. For, if petitioner had at the earliest possible opportunity,
i.e., at the administrative level, demanded that P&G-Phil. produce an express
authorization from its parent corporation to bring the claim for refund, then
P&G-Phil. would have been able forthwith to secure and produce such
authorization before filing the action in the instant case. The action here was
commenced just before expiration of the two (2)-year prescriptive period.
2. The question of the capacity of P&G-Phil. to bring the claim for refund has
substantive dimensions as well which, as will be seen below, also ultimately
relate to fairness.
Under Section 306 of the NIRC, a claim for refund or tax credit filed with the
Commissioner of Internal Revenue is essential for maintenance of a suit for
recovery of taxes allegedly erroneously or illegally assessed or collected:
Sec. 306. Recovery of tax erroneously or illegally collected. No suit or
proceeding shall be maintained in any court for the recovery of any national
internal revenue tax hereafter alleged to have been erroneously or illegally
assessed or collected, or of any penalty claimed to have been collected
without authority, or of any sum alleged to have been excessive or in any
manner wrongfully collected, until a claim for refund or credit has been duly
filed with the Commissioner of Internal Revenue; but such suit or proceeding
may be maintained, whether or not such tax, penalty, or sum has been paid
under protest or duress. In any case, no such suit or proceeding shall be
begun after the expiration of two years from the date of payment of the tax or
penalty regardless of any supervening cause that may arise after payment: . .
. (Emphasis supplied)
Section 309 (3) of the NIRC, in turn, provides:
Sec. 309. Authority of Commissioner to Take Compromises and to Refund
Taxes.The Commissioner may:
xxx xxx xxx
(3) credit or refund taxes erroneously or illegally received, . . . No credit or
refund of taxes or penalties shall be allowed unless the taxpayer files in
writing with the Commissioner a claim for credit or refund within two (2) years
after the payment of the tax or penalty. (As amended by P.D. No. 69)
(Emphasis supplied)
Since the claim for refund was filed by P&G-Phil., the question which arises
is: is P&G-Phil. a "taxpayer" under Section 309 (3) of the NIRC? The term
"taxpayer" is defined in our NIRC as referring to "any person subject to tax
imposed by the Title [on Tax on Income]." 2 It thus becomes important to note that under
Section 53 (c) of the NIRC, the withholding agent who is "required to deduct and withhold any tax" is made "
personally liable for such tax" and indeed is indemnified against any claims and demands which the
stockholder might wish to make in questioning the amount of payments effected by the withholding agent in
accordance with the provisions of the NIRC. The withholding agent, P&G-Phil., is directly and independently
liable 3 for the correct amount of the tax that should be withheld from the dividend remittances. The
withholding agent is, moreover, subject to and liable for deficiency assessments, surcharges and penalties
should the amount of the tax withheld be finally found to be less than the amount that should have been
withheld under law.
A "person liable for tax" has been held to be a "person subject to tax" and
properly considered a "taxpayer." 4 The terms liable for tax" and "subject to tax" both connote
legal obligation or duty to pay a tax. It is very difficult, indeed conceptually impossible, to consider a person
who is statutorily made "liable for tax" as not "subject to tax." By any reasonable standard, such a person
should be regarded as a party in interest, or as a person having sufficient legal interest, to bring a suit for
refund of taxes he believes were illegally collected from him.
In Philippine Guaranty Company, Inc. v. Commissioner of Internal Revenue, 5
this Court pointed out that a withholding agent is in fact the agent both of the government and of the
taxpayer, and that the withholding agent is not an ordinary government agent:
The law sets no condition for the personal liability of the withholding agent to
attach. The reason is to compel the withholding agent to withhold the tax
under all circumstances. In effect, the responsibility for the collection of the
tax as well as the payment thereof is concentrated upon the person over
whom the Government has jurisdiction. Thus, the withholding agent is
constituted the agent of both the Government and the taxpayer. With respect
to the collection and/or withholding of the tax, he is the Government's agent.
In regard to the filing of the necessary income tax return and the payment of
the tax to the Government, he is the agent of the taxpayer. The withholding
agent, therefore, is no ordinary government agent especially because under
Section 53 (c) he is held personally liable for the tax he is duty bound to
withhold; whereas the Commissioner and his deputies are not made liable by
law. 6 (Emphasis supplied)
If, as pointed out in Philippine Guaranty, the withholding agent is also an
agent of the beneficial owner of the dividends with respect to the filing of the
necessary income tax return and with respect to actual payment of the tax to
the government, such authority may reasonably be held to include the
authority to file a claim for refund and to bring an action for recovery of such
claim. This implied authority is especially warranted where, is in the instant
case, the withholding agent is the wholly owned subsidiary of the parent-
stockholder and therefore, at all times, under the effective control of such
parent-stockholder. In the circumstances of this case, it seems particularly
unreal to deny the implied authority of P&G-Phil. to claim a refund and to
commence an action for such refund.
We believe that, even now, there is nothing to preclude the BIR from requiring
P&G-Phil. to show some written or telexed confirmation by P&G-USA of the
subsidiary's authority to claim the refund or tax credit and to remit the
proceeds of the refund., or to apply the tax credit to some Philippine tax
obligation of, P&G-USA, before actual payment of the refund or issuance of a
tax credit certificate. What appears to be vitiated by basic unfairness is
petitioner's position that, although P&G-Phil. is directly and personally liable
to the Government for the taxes and any deficiency assessments to be
collected, the Government is not legally liable for a refund simply because it
did not demand a written confirmation of P&G-Phil.'s implied authority from
the very beginning. A sovereign government should act honorably and fairly at
all times, even vis-a-vis taxpayers.
We believe and so hold that, under the circumstances of this case, P&G-Phil.
is properly regarded as a "taxpayer" within the meaning of Section 309,
NIRC, and as impliedly authorized to file the claim for refund and the suit to
recover such claim.
II
1. We turn to the principal substantive question before us: the applicability to
the dividend remittances by P&G-Phil. to P&G-USA of the fifteen percent
(15%) tax rate provided for in the following portion of Section 24 (b) (1) of the
NIRC:
(b) Tax on foreign corporations.
(1) Non-resident corporation. A foreign corporation not engaged in trade
and business in the Philippines, . . ., shall pay a tax equal to 35% of the gross
income receipt during its taxable year from all sources within the Philippines,
as . . . dividends . . . Provided, still further, that on dividends received from a
domestic corporation liable to tax under this Chapter, the tax shall be 15% of
the dividends, which shall be collected and paid as provided in Section 53 (d)
of this Code, subject to the condition that the country in which the non-
resident foreign corporation, is domiciled shall allow a credit against the tax
due from the non-resident foreign corporation, taxes deemed to have been
paid in the Philippines equivalent to 20% which represents the difference
between the regular tax (35%) on corporations and the tax (15%) on
dividends as provided in this Section . . .
The ordinary thirty-five percent (35%) tax rate applicable to dividend
remittances to non-resident corporate stockholders of a Philippine
corporation, goes down to fifteen percent (15%) if the country of domicile of
the foreign stockholder corporation "shall allow" such foreign corporation a
tax credit for "taxes deemed paid in the Philippines," applicable against the
tax payable to the domiciliary country by the foreign stockholder corporation.
In other words, in the instant case, the reduced fifteen percent (15%) dividend
tax rate is applicable if the USA "shall allow" to P&G-USA a tax credit for
"taxes deemed paid in the Philippines" applicable against the US taxes of
P&G-USA. The NIRC specifies that such tax credit for "taxes deemed paid in
the Philippines" must, as a minimum, reach an amount equivalent to twenty
(20) percentage points which represents the difference between the regular
thirty-five percent (35%) dividend tax rate and the preferred fifteen percent
(15%) dividend tax rate.
It is important to note that Section 24 (b) (1), NIRC, does not require that the
US must give a "deemed paid" tax credit for the dividend tax (20 percentage
points) waived by the Philippines in making applicable the preferred divided
tax rate of fifteen percent (15%). In other words, our NIRC does not require
that the US tax law deem the parent-corporation to have paid the twenty (20)
percentage points of dividend tax waived by the Philippines. The NIRC only
requires that the US "shall allow" P&G-USA a "deemed paid" tax credit in an
amount equivalent to the twenty (20) percentage points waived by the
Philippines.
2. The question arises: Did the US law comply with the above requirement?
The relevant provisions of the US Intemal Revenue Code ("Tax Code") are
the following:
Sec. 901 Taxes of foreign countries and possessions of United States.
(a) Allowance of credit. If the taxpayer chooses to have the benefits of this
subpart, the tax imposed by this chapter shall, subject to the applicable
limitation of section 904, be credited with the amounts provided in the
applicable paragraph of subsection (b) plus, in the case of a corporation, the
taxes deemed to have been paid under sections 902 and 960. Such choice
for any taxable year may be made or changed at any time before the
expiration of the period prescribed for making a claim for credit or refund of
the tax imposed by this chapter for such taxable year. The credit shall not be
allowed against the tax imposed by section 531 (relating to the tax on
accumulated earnings), against the additional tax imposed for the taxable
year under section 1333 (relating to war loss recoveries) or under section
1351 (relating to recoveries of foreign expropriation losses), or against the
personal holding company tax imposed by section 541.
(b) Amount allowed. Subject to the applicable limitation of section 904, the
following amounts shall be allowed as the credit under subsection (a):
(a) Citizens and domestic corporations. In the case of a citizen of the
United States and of a domestic corporation, the amount of any income, war
profits, and excess profits taxes paid or accrued during the taxable year to
any foreign country or to any possession of the United States; and
xxx xxx xxx
Sec. 902. Credit for corporate stockholders in foreign corporation.
(A) Treatment of Taxes Paid by Foreign Corporation. For purposes of this
subject, a domestic corporation which owns at least 10 percent of the voting
stock of a foreign corporation from which it receives dividends in any taxable
year shall
xxx xxx xxx
(2) to the extent such dividends are paid by such foreign corporation out of
accumulated profits [as defined in subsection (c) (1) (b)] of a year for which
such foreign corporation is a less developed country corporation, be deemed
to have paid the same proportion of any income, war profits, or excess profits
taxes paid or deemed to be paid by such foreign corporation to any foreign
country or to any possession of the United States on or with respect to such
accumulated profits, which the amount of such dividends bears to the amount
of such accumulated profits.
xxx xxx xxx
(c) Applicable Rules
(1) Accumulated profits defined. For purposes of this section, the term
"accumulated profits" means with respect to any foreign corporation,
(A) for purposes of subsections (a) (1) and (b) (1), the amount of its gains,
profits, or income computed without reduction by the amount of the income,
war profits, and excess profits taxes imposed on or with respect to such
profits or income by any foreign country. . . .; and
(B) for purposes of subsections (a) (2) and (b) (2), the amount of its gains,
profits, or income in excess of the income, war profits, and excess profits
taxes imposed on or with respect to such profits or income.
The Secretary or his delegate shall have full power to determine from the
accumulated profits of what year or years such dividends were paid, treating
dividends paid in the first 20 days of any year as having been paid from the
accumulated profits of the preceding year or years (unless to his satisfaction
shows otherwise), and in other respects treating dividends as having been
paid from the most recently accumulated gains, profits, or earning. . . .
(Emphasis supplied)
Close examination of the above quoted provisions of the US Tax Code 7 shows
the following:
a. US law (Section 901, Tax Code) grants P&G-USA a tax credit for the
amount of the dividend tax actually paid (i.e., withheld) from the dividend
remittances to P&G-USA;
b. US law (Section 902, US Tax Code) grants to P&G-USA a "deemed paid'
tax credit 8 for a proportionate part of the corporate income tax actually paid to the Philippines by P&G-
Phil.
The parent-corporation P&G-USA is "deemed to have paid" a portion of the
Philippine corporate income tax although that tax was actually paid by its
Philippine subsidiary, P&G-Phil., not by P&G-USA. This "deemed paid"
concept merely reflects economic reality, since the Philippine corporate
income tax was in fact paid and deducted from revenues earned in the
Philippines, thus reducing the amount remittable as dividends to P&G-USA.
In other words, US tax law treats the Philippine corporate income tax as if it
came out of the pocket, as it were, of P&G-USA as a part of the economic
cost of carrying on business operations in the Philippines through the medium
of P&G-Phil. and here earning profits. What is, under US law, deemed paid
by P&G- USA are not "phantom taxes" but instead Philippine corporate
income taxes actually paid here by P&G-Phil., which are very real indeed.
It is also useful to note that both (i) the tax credit for the Philippine dividend
tax actually withheld, and (ii) the tax credit for the Philippine corporate income
tax actually paid by P&G Phil. but "deemed paid" by P&G-USA, are tax
credits available or applicable against the US corporate income tax of P&G-
USA. These tax credits are allowed because of the US congressional desire
to avoid or reduce double taxation of the same income stream. 9
In order to determine whether US tax law complies with the requirements for
applicability of the reduced or preferential fifteen percent (15%) dividend tax
rate under Section 24 (b) (1), NIRC, it is necessary:
a. to determine the amount of the 20 percentage points dividend tax waived
by the Philippine government under Section 24 (b) (1), NIRC, and which
hence goes to P&G-USA;
b. to determine the amount of the "deemed paid" tax credit which US tax law
must allow to P&G-USA; and
c. to ascertain that the amount of the "deemed paid" tax credit allowed by US
law is at least equal to the amount of the dividend tax waived by the
Philippine Government.
Amount (a), i.e., the amount of the dividend tax waived by the Philippine
government is arithmetically determined in the following manner:
P100.00 Pretax net corporate income earned by P&G-Phil.
x 35% Regular Philippine corporate income tax rate

P35.00 Paid to the BIR by P&G-Phil. as Philippine


corporate income tax.
P100.00
-35.00

P65.00 Available for remittance as dividends to P&G-USA


P65.00 Dividends remittable to P&G-USA
x 35% Regular Philippine dividend tax rate under Section 24
(b) (1), NIRC
P22.75 Regular dividend tax
P65.00 Dividends remittable to P&G-USA
x 15% Reduced dividend tax rate under Section 24 (b) (1), NIRC

P9.75 Reduced dividend tax


P22.75 Regular dividend tax under Section 24 (b) (1), NIRC
-9.75 Reduced dividend tax under Section 24 (b) (1), NIRC

P13.00 Amount of dividend tax waived by Philippine


===== government under Section 24 (b) (1), NIRC.
Thus, amount (a) above is P13.00 for every P100.00 of pre-tax net income
earned by P&G-Phil. Amount (a) is also the minimum amount of the "deemed
paid" tax credit that US tax law shall allow if P&G-USA is to qualify for the
reduced or preferential dividend tax rate under Section 24 (b) (1), NIRC.
Amount (b) above, i.e., the amount of the "deemed paid" tax credit which US
tax law allows under Section 902, Tax Code, may be computed arithmetically
as follows:
P65.00 Dividends remittable to P&G-USA
- 9.75 Dividend tax withheld at the reduced (15%) rate

P55.25 Dividends actually remitted to P&G-USA


P35.00 Philippine corporate income tax paid by P&G-Phil.
to the BIR
Dividends actually
remitted by P&G-Phil.
to P&G-USA P55.25
= x P35.00 = P29.75 10
Amount of accumulated P65.00 ======
profits earned by
P&G-Phil. in excess
of income tax
Thus, for every P55.25 of dividends actually remitted (after withholding at the
rate of 15%) by P&G-Phil. to its US parent P&G-USA, a tax credit of P29.75 is
allowed by Section 902 US Tax Code for Philippine corporate income tax
"deemed paid" by the parent but actually paid by the wholly-owned
subsidiary.
Since P29.75 is much higher than P13.00 (the amount of dividend tax waived
by the Philippine government), Section 902, US Tax Code, specifically and
clearly complies with the requirements of Section 24 (b) (1), NIRC.
3. It is important to note also that the foregoing reading of Sections 901 and
902 of the US Tax Code is identical with the reading of the BIR of Sections
901 and 902 of the US Tax Code is identical with the reading of the BIR of
Sections 901 and 902 as shown by administrative rulings issued by the BIR.
The first Ruling was issued in 1976, i.e., BIR Ruling No. 76004, rendered by
then Acting Commissioner of Intemal Revenue Efren I. Plana, later Associate
Justice of this Court, the relevant portion of which stated:
However, after a restudy of the decision in the American Chicle Company
case and the provisions of Section 901 and 902 of the U.S. Internal Revenue
Code, we find merit in your contention that our computation of the credit
which the U.S. tax law allows in such cases is erroneous as the amount of tax
"deemed paid" to the Philippine government for purposes of credit against the
U.S. tax by the recipient of dividends includes a portion of the amount of
income tax paid by the corporation declaring the dividend in addition to the
tax withheld from the dividend remitted. In other words, the U.S. government
will allow a credit to the U.S. corporation or recipient of the dividend, in
addition to the amount of tax actually withheld, a portion of the income tax
paid by the corporation declaring the dividend. Thus, if a Philippine
corporation wholly owned by a U.S. corporation has a net income of
P100,000, it will pay P25,000 Philippine income tax thereon in accordance
with Section 24(a) of the Tax Code. The net income, after income tax, which
is P75,000, will then be declared as dividend to the U.S. corporation at 15%
tax, or P11,250, will be withheld therefrom. Under the aforementioned
sections of the U.S. Internal Revenue Code, U.S. corporation receiving the
dividend can utilize as credit against its U.S. tax payable on said dividends
the amount of P30,000 composed of:
(1) The tax "deemed paid" or indirectly paid on the dividend arrived at as
follows:
P75,000 x P25,000 = P18,750

100,000 **
(2) The amount of 15% of
P75,000 withheld = 11,250

P30,000
The amount of P18,750 deemed paid and to be credited against the U.S. tax
on the dividends received by the U.S. corporation from a Philippine subsidiary
is clearly more than 20% requirement of Presidential Decree No. 369 as 20%
of P75,000.00 the dividends to be remitted under the above example,
amounts to P15,000.00 only.
In the light of the foregoing, BIR Ruling No. 75-005 dated September 10,
1975 is hereby amended in the sense that the dividends to be remitted by
your client to its parent company shall be subject to the withholding tax at the
rate of 15% only.
This ruling shall have force and effect only for as long as the present pertinent
provisions of the U.S. Federal Tax Code, which are the bases of the ruling,
are not revoked, amended and modified, the effect of which will reduce the
percentage of tax deemed paid and creditable against the U.S. tax on
dividends remitted by a foreign corporation to a U.S. corporation. (Emphasis
supplied)
The 1976 Ruling was reiterated in, e.g., BIR Ruling dated 22 July 1981
addressed to Basic Foods Corporation and BIR Ruling dated 20 October
1987 addressed to Castillo, Laman, Tan and Associates. In other words, the
1976 Ruling of Hon. Efren I. Plana was reiterated by the BIR even as the
case at bar was pending before the CTA and this Court.
4. We should not overlook the fact that the concept of "deemed paid" tax
credit, which is embodied in Section 902, US Tax Code, is exactly the same
"deemed paid" tax credit found in our NIRC and which Philippine tax law
allows to Philippine corporations which have operations abroad (say, in the
United States) and which, therefore, pay income taxes to the US government.
Section 30 (c) (3) and (8), NIRC, provides:
(d) Sec. 30. Deductions from Gross Income.In computing net income, there
shall be allowed as deductions . . .
(c) Taxes. . . .
xxx xxx xxx
(3) Credits against tax for taxes of foreign countries. If the taxpayer
signifies in his return his desire to have the benefits of this paragraphs, the
tax imposed by this Title shall be credited with . . .
(a) Citizen and Domestic Corporation. In the case of a citizen of the
Philippines and of domestic corporation, the amount of net income, war
profits or excess profits, taxes paid or accrued during the taxable year to any
foreign country. (Emphasis supplied)
Under Section 30 (c) (3) (a), NIRC, above, the BIR must give a tax credit to a
Philippine corporation for taxes actually paid by it to the US government
e.g., for taxes collected by the US government on dividend remittances to the
Philippine corporation. This Section of the NIRC is the equivalent of Section
901 of the US Tax Code.
Section 30 (c) (8), NIRC, is practically identical with Section 902 of the US
Tax Code, and provides as follows:
(8) Taxes of foreign subsidiary. For the purposes of this subsection a
domestic corporation which owns a majority of the voting stock of a foreign
corporation from which it receives dividends in any taxable year shall be
deemed to have paid the same proportion of any income, war-profits, or
excess-profits taxes paid by such foreign corporation to any foreign country,
upon or with respect to the accumulated profits of such foreign corporation
from which such dividends were paid, which the amount of such dividends
bears to the amount of such accumulated profits: Provided, That the amount
of tax deemed to have been paid under this subsection shall in no case
exceed the same proportion of the tax against which credit is taken which the
amount of such dividends bears to the amount of the entire net income of the
domestic corporation in which such dividends are included. The term
"accumulated profits" when used in this subsection reference to a foreign
corporation, means the amount of its gains, profits, or income in excess of
the income, war-profits, and excess-profits taxes imposed upon or with
respect to such profits or income; and the Commissioner of Internal Revenue
shall have full power to determine from the accumulated profits of what year
or years such dividends were paid; treating dividends paid in the first sixty
days of any year as having been paid from the accumulated profits of the
preceding year or years (unless to his satisfaction shown otherwise), and in
other respects treating dividends as having been paid from the most recently
accumulated gains, profits, or earnings. In the case of a foreign corporation,
the income, war-profits, and excess-profits taxes of which are determined on
the basis of an accounting period of less than one year, the word "year" as
used in this subsection shall be construed to mean such accounting period.
(Emphasis supplied)
Under the above quoted Section 30 (c) (8), NIRC, the BIR must give a tax
credit to a Philippine parent corporation for taxes "deemed paid" by it, that is,
e.g., for taxes paid to the US by the US subsidiary of a Philippine-parent
corporation. The Philippine parent or corporate stockholder is "deemed"
under our NIRC to have paid a proportionate part of the US corporate income
tax paid by its US subsidiary, although such US tax was actually paid by the
subsidiary and not by the Philippine parent.
Clearly, the "deemed paid" tax credit which, under Section 24 (b) (1), NIRC,
must be allowed by US law to P&G-USA, is the same "deemed paid" tax
credit that Philippine law allows to a Philippine corporation with a wholly- or
majority-owned subsidiary in (for instance) the US. The "deemed paid" tax
credit allowed in Section 902, US Tax Code, is no more a credit for "phantom
taxes" than is the "deemed paid" tax credit granted in Section 30 (c) (8),
NIRC.
III
1. The Second Division of the Court, in holding that the applicable dividend
tax rate in the instant case was the regular thirty-five percent (35%) rate
rather than the reduced rate of fifteen percent (15%), held that P&G-Phil. had
failed to prove that its parent, P&G-USA, had in fact been given by the US tax
authorities a "deemed paid" tax credit in the amount required by Section 24
(b) (1), NIRC.
We believe, in the first place, that we must distinguish between the legal
question before this Court from questions of administrative implementation
arising after the legal question has been answered. The basic legal issue is of
course, this: which is the applicable dividend tax rate in the instant case: the
regular thirty-five percent (35%) rate or the reduced fifteen percent (15%)
rate? The question of whether or not P&G-USA is in fact given by the US tax
authorities a "deemed paid" tax credit in the required amount, relates to the
administrative implementation of the applicable reduced tax rate.
In the second place, Section 24 (b) (1), NIRC, does not in fact require that the
"deemed paid" tax credit shall have actually been granted before the
applicable dividend tax rate goes down from thirty-five percent (35%) to
fifteen percent (15%). As noted several times earlier, Section 24 (b) (1),
NIRC, merely requires, in the case at bar, that the USA "shall allow a credit
against the
tax due from [P&G-USA for] taxes deemed to have been paid in the
Philippines . . ." There is neither statutory provision nor revenue regulation
issued by the Secretary of Finance requiring the actual grant of the "deemed
paid" tax credit by the US Internal Revenue Service to P&G-USA before the
preferential fifteen percent (15%) dividend rate becomes applicable. Section
24 (b) (1), NIRC, does not create a tax exemption nor does it provide a tax
credit; it is a provision which specifies when a particular (reduced) tax rate is
legally applicable.
In the third place, the position originally taken by the Second Division results
in a severe practical problem of administrative circularity. The Second
Division in effect held that the reduced dividend tax rate is not applicable until
the US tax credit for "deemed paid" taxes is actually given in the required
minimum amount by the US Internal Revenue Service to P&G-USA. But, the
US "deemed paid" tax credit cannot be given by the US tax authorities unless
dividends have actually been remitted to the US, which means that the
Philippine dividend tax, at the rate here applicable, was actually imposed and
collected. 11 It is this practical or operating circularity that is in fact avoided by our BIR when it issues
rulings that the tax laws of particular foreign jurisdictions (e.g., Republic of Vanuatu 12 Hongkong, 13
Denmark, 14 etc.) comply with the requirements set out in Section 24 (b) (1), NIRC, for applicability of the
fifteen percent (15%) tax rate. Once such a ruling is rendered, the Philippine subsidiary begins to withhold at
the reduced dividend tax rate.
A requirement relating to administrative implementation is not properly
imposed as a condition for the applicability, as a matter of law, of a particular
tax rate. Upon the other hand, upon the determination or recognition of the
applicability of the reduced tax rate, there is nothing to prevent the BIR from
issuing implementing regulations that would require P&G Phil., or any
Philippine corporation similarly situated, to certify to the BIR the amount of the
"deemed paid" tax credit actually subsequently granted by the US tax
authorities to P&G-USA or a US parent corporation for the taxable year
involved. Since the US tax laws can and do change, such implementing
regulations could also provide that failure of P&G-Phil. to submit such
certification within a certain period of time, would result in the imposition of a
deficiency assessment for the twenty (20) percentage points differential. The
task of this Court is to settle which tax rate is applicable, considering the state
of US law at a given time. We should leave details relating to administrative
implementation where they properly belong with the BIR.
2. An interpretation of a tax statute that produces a revenue flow for the
government is not, for that reason alone, necessarily the correct reading of
the statute. There are many tax statutes or provisions which are designed,
not to trigger off an instant surge of revenues, but rather to achieve longer-
term and broader-gauge fiscal and economic objectives. The task of our
Court is to give effect to the legislative design and objectives as they are
written into the statute even if, as in the case at bar, some revenues have to
be foregone in that process.
The economic objectives sought to be achieved by the Philippine
Government by reducing the thirty-five percent (35%) dividend rate to fifteen
percent (15%) are set out in the preambular clauses of P.D. No. 369 which
amended Section 24 (b) (1), NIRC, into its present form:
WHEREAS, it is imperative to adopt measures responsive to the
requirements of a developing economy foremost of which is the financing of
economic development programs;
WHEREAS, nonresident foreign corporations with investments in the
Philippines are taxed on their earnings from dividends at the rate of 35%;
WHEREAS, in order to encourage more capital investment for large projects
an appropriate tax need be imposed on dividends received by non-resident
foreign corporations in the same manner as the tax imposed on interest on
foreign loans;
xxx xxx xxx
(Emphasis supplied)
More simply put, Section 24 (b) (1), NIRC, seeks to promote the in-flow of
foreign equity investment in the Philippines by reducing the tax cost of
earning profits here and thereby increasing the net dividends remittable to the
investor. The foreign investor, however, would not benefit from the reduction
of the Philippine dividend tax rate unless its home country gives it some relief
from double taxation (i.e., second-tier taxation) (the home country would
simply have more "post-R.P. tax" income to subject to its own taxing power)
by allowing the investor additional tax credits which would be applicable
against the tax payable to such home country. Accordingly, Section 24 (b) (1),
NIRC, requires the home or domiciliary country to give the investor
corporation a "deemed paid" tax credit at least equal in amount to the twenty
(20) percentage points of dividend tax foregone by the Philippines, in the
assumption that a positive incentive effect would thereby be felt by the
investor.
The net effect upon the foreign investor may be shown arithmetically in the
following manner:
P65.00 Dividends remittable to P&G-USA (please
see page 392 above
- 9.75 Reduced R.P. dividend tax withheld by P&G-Phil.

P55.25 Dividends actually remitted to P&G-USA


P55.25
x 46% Maximum US corporate income tax rate

P25.415US corporate tax payable by P&G-USA


without tax credits
P25.415
- 9.75 US tax credit for RP dividend tax withheld by P&G-Phil.
at 15% (Section 901, US Tax Code)

P15.66 US corporate income tax payable after Section 901


tax credit.
P55.25
- 15.66

P39.59 Amount received by P&G-USA net of R.P. and U.S.


===== taxes without "deemed paid" tax credit.
P25.415
- 29.75 "Deemed paid" tax credit under Section 902 US
Tax Code (please see page 18 above)
- 0 - US corporate income tax payable on dividends
====== remitted by P&G-Phil. to P&G-USA after
Section 902 tax credit.
P55.25 Amount received by P&G-USA net of RP and US
====== taxes after Section 902 tax credit.
It will be seen that the "deemed paid" tax credit allowed by Section 902, US
Tax Code, could offset the US corporate income tax payable on the dividends
remitted by P&G-Phil. The result, in fine, could be that P&G-USA would after
US tax credits, still wind up with P55.25, the full amount of the dividends
remitted to P&G-USA net of Philippine taxes. In the calculation of the
Philippine Government, this should encourage additional investment or re-
investment in the Philippines by P&G-USA.
3. It remains only to note that under the Philippines-United States Convention
"With Respect to Taxes on Income," 15 the Philippines, by a treaty commitment, reduced the
regular rate of dividend tax to a maximum of twenty percent (20%) of the gross amount of dividends paid to
US parent corporations:
Art 11. Dividends
xxx xxx xxx
(2) The rate of tax imposed by one of the Contracting States on dividends
derived from sources within that Contracting State by a resident of the other
Contracting State shall not exceed
(a) 25 percent of the gross amount of the dividend; or
(b) When the recipient is a corporation, 20 percent of the gross amount of the
dividend if during the part of the paying corporation's taxable year which
precedes the date of payment of the dividend and during the whole of its prior
taxable year (if any), at least 10 percent of the outstanding shares of the
voting stock of the paying corporation was owned by the recipient
corporation.
xxx xxx xxx
(Emphasis supplied)
The Tax Convention, at the same time, established a treaty obligation on the
part of the United States that it "shall allow" to a US parent corporation
receiving dividends from its Philippine subsidiary "a [tax] credit for the
appropriate amount of taxes paid or accrued to the Philippines by the
Philippine [subsidiary] .16 This is, of course, precisely the "deemed paid" tax credit provided for
in Section 902, US Tax Code, discussed above. Clearly, there is here on the part of the Philippines a
deliberate undertaking to reduce the regular dividend tax rate of twenty percent (20%) is a maximum rate,
there is still a differential or additional reduction of five (5) percentage points which compliance of US law
(Section 902) with the requirements of Section 24 (b) (1), NIRC, makes available in respect of dividends from
a Philippine subsidiary.
We conclude that private respondent P&G-Phil, is entitled to the tax refund or
tax credit which it seeks.
WHEREFORE, for all the foregoing, the Court Resolved to GRANT private
respondent's Motion for Reconsideration dated 11 May 1988, to SET ASIDE
the Decision of the and Division of the Court promulgated on 15 April 1988,
and in lieu thereof, to REINSTATE and AFFIRM the Decision of the Court of
Tax Appeals in CTA Case No. 2883 dated 31 January 1984 and to DENY the
Petition for Review for lack of merit. No pronouncement as to costs.

G.R. No. L-68375 April 15, 1988


COMMISSIONER OF INTERNAL REVENUE, petitioner,
vs.
WANDER PHILIPPINES, INC. AND THE COURT OF TAX APPEALS,
respondents.
The Solicitor General for petitioner.
Felicisimo R. Quiogue and Cirilo P. Noel for respondents.

BIDIN, J.:
This is a petition for review on certiorari of the January 19, 1984 Decision of
the Court of Tax Appeals * in C.T.A. Case No.2884, entitled Wander Philippines, Inc. vs.
Commissioner of Internal Revenue, holding that Wander Philippines, Inc. is entitled to the preferential rate of
15% withholding tax on the dividends remitted to its foreign parent company, the Glaro S.A. Ltd. of
Switzerland, a non-resident foreign corporation.
Herein private respondent, Wander Philippines, Inc. (Wander, for short), is a
domestic corporation organized under Philippine laws. It is wholly-owned
subsidiary of the Glaro S.A. Ltd. (Glaro for short), a Swiss corporation not
engaged in trade or business in the Philippines.
On July 18, 1975, Wander filed its withholding tax return for the second
quarter ending June 30, 1975 and remitted to its parent company, Glaro
dividends in the amount of P222,000.00, on which 35% withholding tax
thereof in the amount of P77,700.00 was withheld and paid to the Bureau of
Internal Revenue.
Again, on July 14, 1976, Wander filed a withholding tax return for the second
quarter ending June 30, 1976 on the dividends it remitted to Glaro amounting
to P355,200.00, on wich 35% tax in the amount of P124,320.00 was withheld
and paid to the Bureau of Internal Revenue.
On July 5, 1977, Wander filed with the Appellate Division of the Internal
Revenue a claim for refund and/or tax credit in the amount of P115,400.00,
contending that it is liable only to 15% withholding tax in accordance with
Section 24 (b) (1) of the Tax Code, as amended by Presidential Decree Nos.
369 and 778, and not on the basis of 35% which was withheld and paid to
and collected by the government.
Petitioner herein, having failed to act on the above-said claim for refund, on
July 15, 1977, Wander filed a petition with respondent Court of Tax Appeals.
On October 6, 1977, petitioner file his Answer.
On January 19, 1984, respondent Court of Tax Appeals rendered a Decision,
the decretal portion of which reads:
WHEREFORE, respondent is hereby ordered to grant a refund and/or tax
credit to petitioner in the amount of P115,440.00 representing overpaid
withholding tax on dividends remitted by it to the Glaro S.A. Ltd. of
Switzerland during the second quarter of the years 1975 and 1976.
On March 7, 1984, petitioner filed a Motion for Reconsideration but the same
was denied in a Resolution dated August 13, 1984. Hence, the instant
petition.
Petitioner raised two (2) assignment of errors, to wit:
I
ASSUMING THAT THE TAX REFUND IN THE CASE AT BAR IS
ALLOWABLE AT ALL, THE COURT OF TAX APPEALS ERRED INHOLDING
THAT THE HEREIN RESPONDENT WANDER PHILIPPINES, INC. IS
ENTITLED TO THE SAID REFUND.
II
THE COURT OF TAX APPEALS ERRED IN HOLDING THAT
SWITZERLAND, THE HOME COUNTRY OF GLARO S.A. LTD. (THE
PARENT COMPANY OF THE HEREIN RESPONDENT WANDER
PHILIPPINES, INC.), GRANTS TO SAID GLARO S.A. LTD. AGAINST ITS
SWISS INCOME TAX LIABILITY A TAX CREDIT EQUIVALENT TO THE 20
PERCENTAGE-POINT PORTION (OF THE 35 PERCENT PHILIPPINE
DIVIDEND TAX) SPARED OR WAIVED OR OTHERWISE DEEMED AS IF
PAID IN THE PHILIPPINES UNDER SECTION 24 (b) (1) OF THE
PHILIPPINE TAX CODE.
The sole issue in this case is whether or not private respondent Wander is
entitled to the preferential rate of 15% withholding tax on dividends declared
and remitted to its parent corporation, Glaro.
From this issue, two questions were posed by petitioner: (1) Whether or not
Wander is the proper party to claim the refund; and (2) Whether or not
Switzerland allows as tax credit the "deemed paid" 20% Philippine Tax on
such dividends.
Petitioner maintains and argues that it is Glaro the tax payer, and not Wander,
the remitter or payor of the dividend income and a mere withholding agent for
and in behalf of the Philippine Government, which should be legally entitled to
receive the refund if any.
It will be noted, however, that Petitioner's above-entitled argument is being
raised for the first time in this Court. It was never raised at the administrative
level, or at the Court of Tax Appeals. To allow a litigant to assume a different
posture when he comes before the court and challenge the position he had
accepted at the administrative level, would be to sanction a procedure
whereby the Courtwhich is supposed to review administrative
determinationswould not review, but determine and decide for the first time,
a question not raised at the administrative forum. Thus, it is well settled that
under the same underlying principle of prior exhaustion of administrative
remedies, on the judicial level, issues not raised in the lower court cannot be
raised for the first time on appeal (Aguinaldo Industries Corporation vs.
Commissioner of Internal Revenue, 112 SCRA 136; Pampanga Sugar Dev.
Co., Inc. vs. CIR, 114 SCRA 725; Garcia vs. Court of Appeals, 102 SCRA
597; Matialonzo vs. Servidad, 107 SCRA 726,
In any event, the submission of petitioner that Wander is but a withholding
agent of the government and therefore cannot claim reimbursement of the
alleged overpaid taxes, is untenable. It will be recalled, that said corporation
is first and foremost a wholly owned subsidiary of Glaro. The fact that it
became a withholding agent of the government which was not by choice but
by compulsion under Section 53 (b) of the Tax Code, cannot by any stretch of
the imagination be considered as an abdication of its responsibility to its
mother company. Thus, this Court construing Section 53 (b) of the Internal
Revenue Code held that "the obligation imposed thereunder upon the
withholding agent is compulsory." It is a device to insure the collection by the
Philippine Government of taxes on incomes, derived from sources in the
Philippines, by aliens who are outside the taxing jurisdiction of this Court
(Commissioner of Internal Revenue vs. Malayan Insurance Co., Inc., 21
SCRA 944). In fact, Wander may be assessed for deficiency withholding tax
at source, plus penalties consisting of surcharge and interest (Section 54,
NLRC). Therefore, as the Philippine counterpart, Wander is the proper entity
who should for the refund or credit of overpaid withholding tax on dividends
paid or remitted by Glaro.
Closely intertwined with the first assignment of error is the issue of whether or
not Switzerland, the foreign country where Glaro is domiciled, grants to Glaro
a tax credit against the tax due it, equivalent to 20%, or the difference
between the regular 35% rate of the preferential 15% rate. The dispute in this
issue lies on the fact that Switzerland does not impose any income tax on
dividends received by Swiss corporation from corporations domiciled in
foreign countries.
Section 24 (b) (1) of the Tax Code, as amended by P.D. 369 and 778, the law
involved in this case, reads:
Sec. 1. The first paragraph of subsection (b) of Section 24 of the National
Internal Revenue Code, as amended, is hereby further amended to read as
follows:
(b) Tax on foreign corporations. 1) Non-resident corporation. A foreign
corporation not engaged in trade or business in the Philippines, including a
foreign life insurance company not engaged in the life insurance business in
the Philippines, shall pay a tax equal to 35% of the gross income received
during its taxable year from all sources within the Philippines, as interest
(except interest on foreign loans which shall be subject to 15% tax),
dividends, premiums, annuities, compensations, remuneration for technical
services or otherwise, emoluments or other fixed or determinable, annual,
periodical or casual gains, profits, and income, and capital gains: ... Provided,
still further That on dividends received from a domestic corporation liable to
tax under this Chapter, the tax shall be 15% of the dividends received, which
shall be collected and paid as provided in Section 53 (d) of this Code, subject
to the condition that the country in which the non-resident foreign corporation
is domiciled shall allow a credit against the tax due from the non-resident
foreign corporation taxes deemed to have been paid in the Philippines
equivalent to 20% which represents the difference between the regular tax
(35%) on corporations and the tax (15%) dividends as provided in this
section: ...
From the above-quoted provision, the dividends received from a domestic
corporation liable to tax, the tax shall be 15% of the dividends received,
subject to the condition that the country in which the non-resident foreign
corporation is domiciled shall allow a credit against the tax due from the non-
resident foreign corporation taxes deemed to have been paid in the
Philippines equivalent to 20% which represents the difference between the
regular tax (35%) on corporations and the tax (15%) dividends.
In the instant case, Switzerland did not impose any tax on the dividends
received by Glaro. Accordingly, Wander claims that full credit is granted and
not merely credit equivalent to 20%. Petitioner, on the other hand, avers the
tax sparing credit is applicable only if the country of the parent corporation
allows a foreign tax credit not only for the 15 percentage-point portion actually
paid but also for the equivalent twenty percentage point portion spared,
waived or otherwise deemed as if paid in the Philippines; that private
respondent does not cite anywhere a Swiss law to the effect that in case
where a foreign tax, such as the Philippine 35% dividend tax, is spared
waived or otherwise considered as if paid in whole or in part by the foreign
country, a Swiss foreign-tax credit would be allowed for the whole or for the
part, as the case may be, of the foreign tax so spared or waived or
considered as if paid by the foreign country.
While it may be true that claims for refund are construed strictly against the
claimant, nevertheless, the fact that Switzerland did not impose any tax or the
dividends received by Glaro from the Philippines should be considered as a
full satisfaction of the given condition. For, as aptly stated by respondent
Court, to deny private respondent the privilege to withhold only 15% tax
provided for under Presidential Decree No. 369, amending Section 24 (b) (1)
of the Tax Code, would run counter to the very spirit and intent of said law
and definitely will adversely affect foreign corporations" interest here and
discourage them from investing capital in our country.
Besides, it is significant to note that the conclusion reached by respondent
Court is but a confirmation of the May 19, 1977 ruling of petitioner that "since
the Swiss Government does not impose any tax on the dividends to be
received by the said parent corporation in the Philippines, the condition
imposed under the above-mentioned section is satisfied. Accordingly, the
withholding tax rate of 15% is hereby affirmed."
Moreover, as a matter of principle, this Court will not set aside the conclusion
reached by an agency such as the Court of Tax Appeals which is, by the very
nature of its function, dedicated exclusively to the study and consideration of
tax problems and has necessarily developed an expertise on the subject
unless there has been an abuse or improvident exercise of authority (Reyes
vs. Commissioner of Internal Revenue, 24 SCRA 198, which is not present in
the instant case.
WHEREFORE, the petition filed is DISMISSED for lack of merit.
[G.R. No. L-26145. February 20, 1984.]

THE MANILA WINE MERCHANTS, INC., Petitioner, v. THE COMMISSIONER OF


INTERNAL REVENUE, Respondent.

Rafael D. Salcedo for Petitioner.

The Solicitor General for Respondent.

SYLLABUS

1. TAXATION; NATIONAL INTERNAL REVENUE CODE; CORPORATE INCOME TAX;


ADDITIONAL TAX ON ACCUMULATED EARNINGS; EXEMPTION THEREFROM. A
prerequisite to the imposition of the tax has been that the corporation be formed or
availed of for the purpose of avoiding the income tax (or surtax) on its shareholders,
or on the shareholders of any other corporation by permitting the earnings and profits
of the corporation to accumulate instead of dividing them among or distributing them
to the shareholders. If the earnings and profits were distributed, the shareholders
would be required to pay an income tax thereon whereas, if the distribution were not
made to them, they would incur no tax in respect to the undistributed earnings and
profits of the corporation (Mertens, Law on Federal Income Taxation, Vol. 7, Chapter
39, p. 44). The touchstone of liability is the purpose behind the accumulation of the
income and not the consequences of the accumulation (Ibid., p. 47). Thus, if the
failure to pay dividends is due to some other cause, such as the use of undistributed
earnings and profits for the reasonable needs of the business, such purpose does not
fall within the interdiction of the statute (Ibid., p. 45).

2. ID.; ID.; ID.; ID.; ID.; WHEN ACCUMULATION CONSIDERED UNREASONABLE.


An accumulation of earnings or profits (including undistributed earnings or profits of
prior years) is unreasonable if it is not required for the purpose of the business,
considering all the circumstances of the case (Sec. 21, Revenue Regulations No. 2).

3. ID.; ID.; ID.; ID.; ID.; "REASONABLE NEEDS OF THE BUSINESS," CONSTRUED.
To determine the "reasonable needs" of the business in order to justify an
accumulation of earnings, the Courts of the United States have invented the so-called
"Immediacy Test" which construed the words "reasonable needs of the business" to
mean the immediate needs of the business, and it was generally held that if the
corporation did not prove an immediate need for the accumulation of the earnings and
profits, the accumulation was not for the reasonable needs of the business, and the
penalty tax would apply. American cases likewise hold that investment of the earnings
and profits of the corporation in stock or securities of an unrelated business usually
indicates an accumulation beyond the reasonable needs of the business. (Helvering v.
Chicago Stockyards Co., 318 US 693; Helvering v. National Grocery Co., 304 US 282).

4. REMEDIAL LAW; APPEALS; FACTUAL FINDINGS OF THE COURT OF TAX APPEALS,


BINDING. The finding of the Court of Tax Appeals that the purchase of the U.S.A.
Treasury bonds were in no way related to petitioners business of importing and selling
wines whisky, liquors and distilled spirits, and thus construed as an investment
beyond the reasonable needs of the business is binding on Us, the same being factual
(Renato Raymundo v. Hon. De Jova, 101 SCRA 495). Furthermore, the wisdom behind
thus finding cannot be doubted, The case of J.M. Perry & Co. v. Commissioner of
Internal Revenue supports the same.

5. TAXATION; NATIONAL INTERNAL REVENUE CODE; INCOME TAX OF


CORPORATIONS; ADDITIONAL TAX ON ACCUMULATED EARNINGS; EXCEPTION
THEREFROM; ACCUMULATION OF EARNINGS, MUST BE USED FOR REASONABLE
NEEDS OF BUSINESS WITHIN A REASONABLE TIME. The records further reveal that
from May 1951 when petitioner purchased the U.S.A. Treasury shares, until 1962
when it finally liquidated the same, it (petitioner) never had the occasion to use the
said shares in aiding or financing its importation. This militates against the purpose
enunciated earlier by petitioner that the shares were purchased to finance its
importation business. To justify an accumulation of earnings and profits for the
reasonably anticipated future needs, such accumulation must be used within a
reasonable time after the close of the taxable year (Mertens, Ibid., p. 104).

6. ID.; ID.; ID.; ID.; ID.; ID.; INTENTION AT THE TIME OF ACCUMULATION, BASIS OF
THE TAX; ACCUMULATION OF PROFITS IN CASE AT BAR, UNREASONABLE. In order
to determine whether profits are accumulated for the reasonable needs of the
business as to avoid the surtax upon shareholders, the controlling intention of the
taxpayer is that which is manifested at the time of accumulation not subsequently
declared intentions which are merely the product of afterthought (Basilan Estates, Inc.
v. Comm. of Internal Revenue, 21 SCRA 17 citing Jacob Mertens, Jr., The law of
Federal Income Taxation, Vol. 7, Cumulative Supplement, p. 213; Smoot and San &
Gravel Corp. v. Comm., 241 F 2d 197). A speculative and indefinite purpose will not
suffice. The mere recognition of a future problem and the discussion of possible and
alternative solutions is not sufficient. Definiteness of plan coupled with action taken
towards its consummation are essential (Fuel Carriers, Inc. v. US 202 F supp. 497;
Smoot Sand & Gravel Corp. v. Comm., supra). Viewed on the foregoing analysis and
tested under the "immediacy doctrine," We are convinced that the Court of Tax
Appeals is correct in finding that the investment made by petitioner in the U.S.A.
Treasury shares in 1951 was an accumulation of profits in excess of the reasonable
needs of petitioners business.
chanroblesvirtuallawlibrary

7. ID.; ID.; ID.; ID.; ACCUMULATIONS OF PRIOR YEARS TAKEN INTO ACCOUNT IN
DETERMINATION OF LIABILITY THEREFOR. The rule is now settled in Our
jurisprudence that undistributed earnings or profits of prior years are taken into
consideration in determining unreasonable accumulation for purposes of the 25%
surtax. The case of Basilan Estates, Inc. v. Commissioner of Internal Revenue further
strengthen this rule in determining unreasonable accumulation for the year
concerned.In determining whether accumulations of earnings or profits in a particular
year are within the reasonable needs of a corporation, it is necessary to take into
account prior accumulations, since accumulations prior to the year involved may have
been sufficient to cover the business needs and additional accumulations during the
year involved would not reasonably be necessary.

DECISION
GUERRERO, J.:

In this Petition for Review on Certiorari, Petitioner, the Manila Wine Merchants, Inc.,
disputes the decision of the Court of Tax Appeals ordering it (petitioner) to pay
respondent, the Commissioner of Internal Revenue, the amount of P86,804.38 as
25% surtax plus interest which represents the additional tax due petitioner for
improperly accumulating profits or surplus in the taxable year 1957 under Sec. 25 of
the National Internal Revenue Code. chanrobles virtualawlibrary chanrobles.com:chanrobles.com.ph

The Court of Tax Appeals made the following finding of facts, to wit: jgc:chanrobles.com.ph

"Petitioner, a domestic corporation organized in 1937, is principally engaged in the


importation and sale of whisky, wines, liquors and distilled spirits. Its original
subscribed and paid capital was P500,000.00. Its capital of P500,000.00 was reduced
to P250,000.00 in 1950 with the approval of the Securities and Exchange Commission
but the reduction of the capital was never implemented. On June 21, 1958,
petitioners capital was increased to P1,000,000.00 with the approval of the said
Commission.

On December 31, 1957, herein respondent caused the examination of herein


petitioners book of account and found the latter of having unreasonably accumulated
surplus of P428,934.32 for the calendar year 1947 to 1957, in excess of the
reasonable needs of the business subject to the 25% surtax imposed by Section 25 of
the Tax Code.

On February 26, 1963, the Commissioner of Internal Revenue demanded upon the
Manila Wine Merchants, Inc. payment of P126,536.12 as 25% surtax and interest on
the latters unreasonable accumulation of profits and surplus for the year 1957,
computed as follows: chanrob1es virtual 1aw library

Unreasonable accumulation of surtax P428,934.42

25% surtax due thereon P107,234.00

Add: 1/2% monthly interest from June 20,

1959 to June 20, 1962 19,302.12

TOTAL AMOUNT DUE AND COLLECTIBLE P126,536.12

=========

Respondent contends that petitioner has accumulated earnings beyond the reasonable
needs of its business because the average ratio of the cash dividends declared and
paid by petitioner from 1947 to 1957 was 40.33% of the total surplus available for
distribution at the end of each calendar year. On the other hand, petitioner contends
that in 1957, it distributed 100% of its net earnings after income tax and part of the
surplus for prior years. Respondent further submits that the accumulated earnings tax
should be based on 25% of the total surplus available at the end of each calendar
year while petitioner maintains that the 25% surtax is imposed on the total surplus or
net income for the year after deducting therefrom the income tax due.

The records show the following analysis of petitioners net income, cash dividends and
earned surplus for the years 1946 to 1957: 1

Percentage of

Dividends to

Net Income Total Cash Net Income Balance

After Income Dividends After of Earned

Year Tax Paid Income Tax Surplus

1946 P 613,790.00 P 200,000. 32.58% P 234,104.81

1947 425,719.87 360,000. 84.56% 195,167.10

1948 415,591.83 375,000. 90.23% 272,991.38

1949 335,058.06 200,000. 59.69% 893,113.42

1950 399,698.09 600,000. 150.11% 234,987.07

1951 346,257.26 300,000. 86.64% 281,244.33

1952 196,161.97 200,000. 101.96% 277,406.30

1953 169,714.04 200,000. 117.85% 301,138.84

1954 238,124.85 250,000. 104.99% 289,262.69

1955 312,284.74 200,000. 64.04% 401,548.43

1956 374,240.28 300,000. 80.16% 475,788.71

1957 353,145.71 400,000. 113.27% 428,934.42

P4,179,787.36 P3,585.000. 85.77% P3,785.688.50

========== ========= ======= ==========

Another basis of respondent in assessing petitioner for accumulated earnings tax is its
substantial investment of surplus or profits in unrelated business. These investments
are itemized as follows:
chanrob1es virtual 1aw library
1. Acme Commercial Co., Inc. P 27,501.00

2. Union Insurance Society

of Canton 1,145.76

3. U.S.A. Treasury Bond 347,217.50

4. Wack Wack Golf &

Country Club 1.00

375,865.26

=========

As to the investment of P27,501.00 made by petitioner in the Acme Commercial Co.,


Inc., Mr. N.R.E. Hawkins, president of the petitioner corporation 2 explained as
follows:
chanrob1es virtual 1aw library

The first item consists of shares of Acme Commercial Co., Inc. which the Company
acquired in 1947 and 1949. In the said years, we thought it prudent to invest in a
business which patronizes us. As a supermarket, Acme Commercial Co., Inc. is one of
our best customers. The investment has proven to be beneficial to the stockholders of
this Company. As an example, the Company received cash dividends in 1961 totalling
P16,875.00 which was included in its income tax return for the said year.

As to the investments of petitioner in Union Insurance Society of Canton and Wack


Wack Golf Club in the sums of P1,145.76 and P1.00, respectively, the same official of
the petitioner-corporation stated that: 3

The second and fourth items are small amounts which we believe would not affect
this case substantially. As regards the Union Insurance Society of Canton shares, this
was a pre-war investment, when Wise & Co., Inc., Manila Wine Merchants and the said
insurance firm were common stockholders of the Wise Bldg. Co.,, Inc. and the three
companies were all housed in the same building. Union Insurance invested in Wise
Bldg. Co., Inc. but invited Manila Wine Merchants, Inc. to buy a few of its shares.

As to the U.S.A. Treasury Bonds amounting to P347,217.50, Mr. Hawkins explained as


follows: 4

With regards to the U.S.A. Treasury Bills in the amount of P347,217.50, in 1950, our
balance sheet for the said year shows the Company had deposited in current account
in various banks P629,403.64 which was not earning any interest. We decided to
utilize part of this money as reserve to finance our importations and to take care of
future expansion including acquisition of a lot and the construction of our own office
building and bottling plant.

At that time, we believed that a dollar reserve abroad would be useful to the Company
in meeting immediate urgent orders of its local customers. In order that the money
may earn interest, the Company, on May 31, 1951 purchased US Treasury bills with
90-day maturity and earning approximately 1% interest with the face value of
US$175,000.00. US Treasury Bills are easily convertible into cash and for the said
reason they may be better classified as cash rather than investments.

The Treasury Bills in question were held as such for many years in view of our
expectation that the Central Bank inspite of the controls would allow no-dollar licenses
importations. However, since the Central Bank did not relax its policy with respect
thereto, we decided sometime in 1957 to hold the bills for a few more years in view of
our plan to buy a lot and construct a building of our own. According to the lease
agreement over the building formerly occupied by us in Dasmarias St., the lease was
to expire sometime in 1957. At that time, the Company was not yet qualified to own
real property in the Philippines. We therefore waited until 60% of the stocks of the
Company would be owned by Filipino citizens before making definite plans. Then in
1959 when the Company was already more than 60% Filipino owned, we commenced
looking for a suitable location and then finally in 1961, we bought the man lot with an
old building on Otis St., Paco, our present site, for P665,000.00. Adjoining smaller lots
were bought later. After the purchase of the main property, we proceeded with the
remodelling of the old building and the construction of additions, which were
completed at a cost of P143,896.00 in April, 1962.

In view of the needs of the business of this Company and the purchase of the Otis lots
and the construction of the improvements thereon, most of its available funds
including the Treasury Bills had been utilized, but inspite of the said expenses the
Company consistently declared dividends to its stockholders. The Treasury Bills were
liquidated on February 15, 1962.

Respondent found that the accumulated surplus in question were invested to


unrelated business which were not considered in the immediate needs of the
Company such that the 25% surtax be imposed therefrom." cralaw virtua1aw library

Petitioner appealed to the Court of Tax Appeals.

On the basis of the tabulated figures, supra, the Court of Tax Appeals found that the
average percentage of cash dividends distributed was 85.77% for a period of 11 years
from 1946 to 1957 and not only 40.33% of the total surplus available for distribution
at the end of each calendar year actually distributed by the petitioner to its
stockholders, which is indicative of the view that the Manila Wine Merchants, Inc. was
not formed for the purpose of preventing the imposition of income tax upon its
shareholders. 5

With regards to the alleged substantial investment of surplus or profits in unrelated


business, the Court of Tax Appeals held that the investment of petitioner with Acme
Commercial Co., Inc., Union Insurance Society of Canton and with the Wack Wack Golf
and Country Club are harmless accumulation of surplus and, therefore, not subject to
the 25% surtax provided in Section 25 of the Tax Code. 6

As to the U.S.A. Treasury Bonds amounting to P347,217.50, the Court of Tax Appeals
ruled that its purchase was in no way related to petitioners business of importing and
selling wines, whisky, liquors and distilled spirits. Respondent Court was convinced
that the surplus of P347,217.50 which was invested in the U.S.A. Treasury Bonds was
availed of by petitioner for the purpose of preventing the imposition of the surtax
upon petitioners shareholders by permitting its earnings and profits to accumulate
beyond the reasonable needs of business. Hence, the Court of Tax Appeals modified
respondents decision by imposing upon petitioner the 25% surtax for 1957 only in
the amount of P86,804.38 computed as follows: chanrob1es virtual 1aw library

Unreasonable accumulation

of surplus P347,217.50

25% surtax due thereon P 86,804.38 7

On May 30, 1966, the Court of Tax Appeals denied the motion for reconsideration filed
by petitioner on March 30, 1966. Hence, this petition.

Petition assigns the following errors: chanrob1es virtual 1aw library

The Court of Tax Appeals erred in holding that petitioner was availed of for the
purpose of preventing the imposition of a surtax on its shareholders.

II

The Court of Tax Appeals erred in holding that petitioners purchase of U.S.A. Treasury
Bills in 1951 was an investment in unrelated business subject to the 25% surtax in
1957 as surplus profits improperly accumulated in the latter years.

III

The Court of Tax Appeals erred in not finding that petitioner did not accumulate its
surplus profits improperly in 1957, and in not holding that such surplus profits,
including the so-called unrelated investments, were necessary for its reasonable
business needs.

IV

The Court of Tax Appeals erred in not holding that petitioner had overcome the prima
facie presumption provided for in Section 25(c) of the Revenue Code.

The Court of Tax Appeals erred in finding petition liable for the payment of the surtax
of P86,804.38 and in denying petitioners Motion for Reconsideration and/or New Trial.
The issues in this case can be summarized as follows: (1) whether the purchase of the
U.S.A. Treasury bonds by petitioner in 1951 can be construed as an investment to an
unrelated business and hence, such was availed of by petitioner for the purpose of
preventing the imposition of the surtax upon petitioners shareholders by permitting
its earnings and profits to accumulate beyond the reasonable needs of the business,
and if so, (2) whether the penalty tax of twenty-five percent (25%) can be imposed
on such improper accumulation in 1957 despite the fact that the accumulation
occurred in 1951.chanrobles virtualawlibrary chanrobles.com:chanrobles.com.ph

The pertinent provision of the National Internal Revenue Code reads as follows: jgc:chanrobles.com.ph

"Sec. 25. Additional tax on corporations improperly accumulating profits or surplus.


(a) Imposition of Tax. If any corporation, except banks, insurance companies, or
personal holding companies whether domestic or foreign, is formed or availed of for
the purpose of preventing the imposition of the tax upon its shareholders or members
or the shareholders or members of another corporation, through the medium of
permitting its gains and profits to accumulate instead of being divided or distributed,
there is levied and assessed against such corporation, for each taxable year, a tax
equal to twenty-five per centum of the undistributed portion of its accumulated profits
or surplus which shall be in addition to the tax imposed by section twenty-four and
shall be computed, collected and paid in the same manner and subject to the same
provisions of law, including penalties, as that tax: Provided, that no such tax shall be
levied upon any accumulated profits or surplus, if they are invested in any dollar-
producing or dollar-saving industry or in the purchase of bonds issued by the Central
Bank of the Philippines.

x x x

(c) Evidence determinative of purpose. The fact that the earnings of profits of a
corporation are permitted to accumulate beyond the reasonable needs of the business
shall be determinative of the purpose to avoid the tax upon its shareholders or
members unless the corporation, by clear preponderance of evidence, shall prove the
contrary." (As amended by Republic Act No. 1823).

As correctly pointed out by the Court of Tax Appeals, inasmuch as the provisions of
Section 25 of the National Internal Revenue Code were bodily lifted from Section 102
of the U.S. Internal Revenue Code of 1939, including the regulations issued in
connection therewith, it would be proper to resort to applicable cases decided by the
American Federal Courts for guidance and enlightenment. chanrobles virtual lawlibrary

A prerequisite to the imposition of the tax has been that the corporation be formed or
availed of for the purpose of avoiding the income tax (or surtax) on its shareholders,
or on the shareholders of any other corporation by permitting the earnings and profits
of the corporation to accumulate instead of dividing them among or distributing them
to the shareholders. If the earnings and profits were distributed, the shareholders
would be required to pay an income tax thereon whereas, if the distribution were not
made to them, they would incur no tax in respect to the undistributed earnings and
profits of the corporation. 8 The touchstone of liability is the purpose behind the
accumulation of the income and not the consequences of the accumulation. 9 Thus, if
the failure to pay dividends is due to some other cause, such as the use of
undistributed earnings and profits for the reasonable needs of the business, such
purpose does not fall within the interdiction of the statute. 10

An accumulation of earnings or profits (including undistributed earnings or profits of


prior years) is unreasonable if it is not required for the purpose of the business,
considering all the circumstances of the case. 11

In purchasing the U.S.A. Treasury Bonds, in 1951, petitioner argues that these bonds
were so purchased (1) in order to finance their importation; and that a dollar reserve
abroad would be useful to the Company in meeting urgent orders of its local
customers and (2) to take care of future expansion including the acquisition of a lot
and the construction of their office building and bottling plant.

We find no merit in the petition.

To avoid the twenty-five percent (25%) surtax, petitioner has to prove that the
purchase of the U.S.A. Treasury Bonds in 1951 with a face value of $175,000.00 was
an investment within the reasonable needs of the Corporation.

To determine the "reasonable needs" of the business in order to justify an


accumulation of earnings, the Courts of the United States have invented the so-called
"Immediacy Test" which construed the words "reasonable needs of the business" to
mean the immediate needs of the business, and it was generally held that if the
corporation did not prove an immediate need for the accumulation of the earnings and
profits, the accumulation was not for the reasonable needs of the business, and the
penalty tax would apply. 12 American cases likewise hold that investment of the
earnings and profits of the corporation in stock or securities of an unrelated business
usually indicates an accumulation beyond the reasonable needs of the business. 13

The finding of the Court of Tax Appeals that the purchase of the U.S.A. Treasury bonds
were in no way related to petitioners business of importing and selling wines whisky,
liquors and distilled spirits, and thus construed as an investment beyond the
reasonable needs of the business 14 is binding on Us, the same being factual. 15
Furthermore, the wisdom behind thus finding cannot be doubted, The case of J.M.
Perry & Co. v. Commissioner of Internal Revenue 16 supports the same. In that case,
the U.S. Court said the following: jgc:chanrobles.com.ph

"It appears that the taxpayer corporation was engaged in the business of cold storage
and wareshousing in Yahima, Washington. It maintained a cold storage plant, divided
into four units, having a total capacity of 490,000 boxes of fruits. It presented
evidence to the effect that various alterations and repairs to its plant were
contemplated in the tax years, . . .

It also appeared that in spite of the fact that the taxpayer contended that it needed to
maintain this large cash reserve on hand, it proceeded to make various investments
which had no relation to its storage business. In 1934, it purchased mining stock
which it sold in 1935 at a profit of US $47,995.29. . . .

All these things may reasonably have appealed to the Board as incompatible with a
purpose to strengthen the financial position of the taxpayer and to provide for needed
alteration."
cralaw virtua1aw library
The records further reveal that from May 1951 when petitioner purchased the U.S.A.
Treasury shares, until 1962 when it finally liquidated the same, it (petitioner) never
had the occasion to use the said shares in aiding or financing its importation. This
militates against the purpose enunciated earlier by petitioner that the shares were
purchased to finance its importation business. To justify an accumulation of earnings
and profits for the reasonably anticipated future needs, such accumulation must be
used within a reasonable time after the close of the taxable year. 17

Petitioner advanced the argument that the U.S.A. Treasury shares were held for a few
more years from 1957, in view of a plan to buy a lot and construct a building of their
own; that at that time (1957), the Company was not yet qualified to own real
property in the Philippines, hence it (petitioner) had to wait until sixty percent (60%)
of the stocks of the Company would be owned by Filipino citizens before making
definite plans. 18

These arguments of petitioner indicate that it considers the U.S.A. Treasury shares not
only for the purpose of aiding or financing its importation but likewise for the purpose
of buying a lot and constructing a building thereon in the near future, but conditioned
upon the completion of the 60% citizenship requirement of stock ownership of the
Company in order to qualify it to purchase and own a lot. The time when the company
would be able to establish itself to meet the said requirement and the decision to
pursue the same are dependent upon various future contingencies. Whether these
contingencies would unfold favorably to the Company and if so, whether the Company
would decide later to utilize the U.S.A. Treasury shares according to its plan, remains
to be seen. From these assertions of petitioner, We cannot gather anything definite or
certain. This, We cannot approve. chanrobles law library

In order to determine whether profits are accumulated for the reasonable needs of the
business as to avoid the surtax upon shareholders, the controlling intention of the
taxpayer is that which is manifested at the time of accumulation not subsequently
declared intentions which are merely the product of afterthought. 19 A speculative
and indefinite purpose will not suffice. The mere recognition of a future problem and
the discussion of possible and alternative solutions is not sufficient. Definiteness of
plan coupled with action taken towards its consummation are essential. 20 The Court
of Tax Appeals correctly made the following ruling: 21

"As to the statement of Mr. Hawkins in Exh. "B" regarding the expansion program of
the petitioner by purchasing a lot and building of its own, we find no justifiable reason
for the retention in 1957 or thereafter of the US Treasury Bonds which were
purchased in 1951.

x x x

"Moreover, if there was any thought for the purchase of a lot and building for the
needs of petitioners business, the corporation may not with impunity permit its
earnings to pile up merely because at some future time certain outlays would have to
be made. Profits may only be accumulated for the reasonable needs of the business,
and implicit in this is further requirement of a reasonable time." cralaw virtua1aw library

Viewed on the foregoing analysis and tested under the "immediacy doctrine," We are
convinced that the Court of Tax Appeals is correct in finding that the investment made
by petitioner in the U.S.A. Treasury shares in 1951 was an accumulation of profits in
excess of the reasonable needs of petitioners business.

Finally, petitioner asserts that the surplus profits allegedly accumulated in the form of
U.S.A. Treasury shares in 1951 by it (petitioner) should not be subject to the surtax in
1957. In other words, petitioner claims that the surtax of 25% should be based on the
surplus accumulated in 1951 and not in 1957.

This is devoid of merit.

The rule is now settled in Our jurisprudence that undistributed earnings or profits of
prior years are taken into consideration in determining unreasonable accumulation for
purposes of the 25% surtax. 22 The case of Basilan Estates, Inc. v. Commissioner of
Internal Revenue 23 further strengthen this rule, and We quote: jgc:chanrobles.com.ph

"Petitioner questions why the examiner covered the period from 1948-1953 when the
taxable year on review was 1953. The surplus of P347,507.01 was taken by the
examiner from the balance sheet of the petitioner for 1953. To check the figure
arrived at, the examiner traced the accumulation process from 1947 until 1953, and
petitioners figure stood out to be correct. There was no error in the process applied,
for previous accumulations should be considered in determining unreasonable
accumulation for the year concerned.In determining whether accumulations of
earnings or profits in a particular year are within the reasonable needs of a
corporation, it is necessary to take into account prior accumulations, since
accumulations prior to the year involved may have been sufficient to cover the
business needs and additional accumulations during the year involved would not
reasonably be necessary." chanroblesvirtuallawlibrary

WHEREFORE, IN VIEW OF THE FOREGOING, the decision of the Court of Tax Appeals
is AFFIRMED in toto, with costs against petitioner.

G.R. No. 85749 May 15, 1989


COMMISSIONER OF INTERNAL REVENUE, petitioner,
vs.
ANTONIO TUASON, INC. and THE COURT OF TAX APPEALS,
respondents.
The Office of the Solicitor General for petitioner.
Mendoza & Papa and Roman M. Umali for private respondent.

GRIO-AQUINO, J.:
Elevated to this Court for review is the decision dated October 14, 1988 of the
Court of Tax Appeals in CTA Case No. 3865, entitled "Antonio Tuason, Inc.
vs. Commissioner of Internal Revenue," which set aside the petitioner
Revenue Commissioner's assessment of P1,151,146.98 as the 25% surtax
on the private respondent's unreasonable accumulation of surplus for the
years 1975-1978.
Under date of February 27, 1981, the petitioner, Commissioner of Internal
Revenue, assessed Antonio Tuason, Inc.
a
b Deficiency income tax for the years 1975,1976 and 1978 . . . . . . .
.. P37,491.83.
c
(b) Deficiency corporate quarterly income tax for the first quarter of 1975 . . .
. . . . . . . . . . . . . . . . . . 161.49.
(c) 25% surtax on unreasonable accumulation of surplus for the years 1975-
1978 . . . . . . . . . . . . 1,151,146.98.
The private respondent did not object to the first and second items and,
therefore, paid the amounts demanded. However, it protested the
assessment on a 25% surtax on the third item on the ground that the
accumulation of surplus profits during the years in question was solely for the
purpose of expanding its business operations as real estate broker. The
request for reinvestigation was granted on condition that a waiver of the
statute of limitations should be filed by the private respondent. The latter
replied that there was no need of a waiver of the statute of limitaitons
because the right of the Government to assess said tax does not prescribe.
No investigation was conducted nor a decision rendered on Antonio Tuazon
Inc.'s protest. meantime, the Revenue Commissioner issued warrants of
distraint and levy to enforce collection of the total amount originally assessed
including the amounts already paid.
The private respondent filed a petition for review in the Court of Tax Appeals
with a request that pending determination of the case on the merits, an order
be issued restraining the Commissioner and/or his representatives from
enforcing the warrants of distraint and levy. Since the right asserted by the
Commissioner to collect the taxes involved herein by the summary methods
of distraint and levy was not clear, and it was shown that portions of the tax
liabilities involved in the assessment had already been paid, a writ of
injunction was issued by the Tax Court on November 26, 1984, ordering the
Commissioner to refrain fron enforcing said warrants of distraint and levy. It
did not require the petitioner to file a bond (Annex A, pp. 28-30, Rollo).
In view of the reversal of the Commissioner's decision by the Court of Tax
Appeals, the petitioner appealed to this Court, raising the following issues:
1. Whether or not private respondent Antonio Tuason, Inc. is a holding
company and/or investment company;
2. Whether or not privaaate respondent Antonio Tuason, Inc. accumulated
surplus for the years 1975 to 1978; and
3. Whether or not Antonio Tuason, Inc. is liable for the 25% surtax on undue
accumulation of surplus for the years 1975 to 1978.
Section 25 of the Tax Code at the time the surtax was assessed, provided:
Sec. 25. Additional tax on corporation improperly accumulating profits or
surplus.
(a) Imposition of tax. If any corporation, except banks, insurance
companies, or personal holding companies, whether domestic or foreign, is
formed or availed of for the purpose of preventing the imposition of the tax
upon its shareholders or members or the shareholders or members of
another corporation, through the medium of permitting its gains and profits to
accumulate instead of being divided or distributed, there is levied and
assessed against such corporation, for each taxable year, a tax equal to
twenty-five per centum of the undistributed portion of its accumulated profits
or surplus which shall be in addition to the tax imposed by section twenty-
four, and shall be computed, collected and paid in the same manner and
subject to the same provisions of law, including penalties, as that tax.
(b) Prima facie evidence. The fact that any corporation is a mere holding
company shall be prima facie evidence of a purpose to avoid the tax upon its
shareholders or members. Similar presumption will lie in the case of an
investment company where at any time during the taxable year more than fifty
per centum in value of its outstanding stock is owned, directly or indirectly, by
one person.
(c) Evidence determinative of purpose. The fact that the earnings or profits
of a corporation are permitted to accumulate beyond the reasonable needs of
the business shall be determinative of the purpose to avoid the tax upon its
shareholders or members unless the corporation, by clear preponderance of
evidence, shall prove the contrary.
The petition for review is meritorious.
The Court of Tax Appeals conceded that the Revenue Commissioner's
determination that Antonio Tuason, Inc. was a mere holding or investment
company, was "presumptively correct" (p. 7, Annex A), for the corporation did
not involve itself in the development of subdivisions but merely subdivided its
own lots and sold them for bigger profits. It derived its income mostly from
interest, dividends and rental realized from the sale of realty.
Another circumstance supporting that presumption is that 99.99% in value of
the outstanding stock of Antonio Tuason, Inc., is owned by Antonio Tuason
himself. The Commissioner "conclusively presumed" that when the
corporation accumulated (instead of distributing to the shareholders) a
surplus of over P3 million fron its earnings in 1975 up to 1978, the purpose
was to avoid the imposition of the progressive income tax on its shareholders.
That Antonio Tuason, Inc. accumulated surplus profits amounting to
P3,263,305.88 for 1975 up to 1978 is not disputed. However, the private
respondent vehemently denies that its purpose was to evade payment of the
progressive income tax on such dividends by its stockholders. According to
the private respondent, surplus profits were set aside by the company to build
up sufficient capital for its expansion program which included the construction
in 1979-1981 of an apartment building, and the purchase in 1980 of a
condominium unit which was intended for resale or lease.
However, while these investments were actually made, the Commissioner
points out that the corporation did not use up its surplus profits. It allegation
that P1,525,672.74 was spent for the construction of an apartment building in
1979 and P1,752,332.87 for the purchase of a condominium unit in Urdaneta
Village in 1980 was refuted by the Declaration of Real Property on the
apartment building (Exh. C) which shows that its market value is only
P429,890.00, and the Tax Declaration on the condominium unit which reflects
a market value of P293,830.00 only (Exh. D-1). The enormous discrepancy
between the alleged investment cost and the declared market value of these
pieces of real estate was not denied nor explained by the private respondent.
Since the company as of the time of the assessment in 1981, had invested in
its business operations only P 773,720 out of its accumulated surplus profits
of P3,263,305.88 for 1975-1978, its remaining accumulated surplus profits of
P2,489,858.88 are subject to the 25% surtax.
All presumptions are in favor of the correctness of petitioner's assessment
against the private respondent. It is incumbent upon the taxpayer to prove the
contrary (Mindanao Bus Company vs. Commissioner of Internal Revenue, 1
SCRA 538). Unfortunately, the private respondent failed to overcome the
presumption of correctness of the Commissioner's assessment.
The touchstone of liability is the purpose behind the accumulation of the
income and not the consequences of the accumulation. Thus, if the failure to
pay dividends were for the purpose of using the undistributed earnings and
profits for the reasonable needs of the business, that purpose would not fall
within the interdiction of the statute" (Mertens Law of Federal Income
Taxation, Vol. 7, Chapter 39, p. 45 cited in Manila Wine Merchants, Inc. vs.
Commissioner of Internal Revenue, 127 SCRA 483, 493).
It is plain to see that the company's failure to distribute dividends to its
stockholders in 1975-1978 was for reasons other than the reasonable needs
of the business, thereby falling within the interdiction of Section 25 of the Tax
Code of 1977.
WHEREFORE, the appealed decision of the Court of Tax Appeals is hereby
set aside. The petitioner's assessment of a 25% surtax against the Antonio
Tuason, Inc. is reinstated but only on the latter's unspent accumulated
surplus profits of P2,489,585.88. No costs.

[G.R. No. 108067. January 20, 2000]


CYANAMID PHILIPPINES, INC., petitioner, vs. THE
COURT OF APPEALS, THE COURT OF TAX APPEALS
and COMMISSIONER OF INTERNAL REVENUE,
respondents.
DECISION
QUISUMBING, J.:
Petitioner disputes the decision of the Court of Appeals which
[if !supportFootnotes][1][endif]

affirmed the decision of the Court of Tax Appeals, ordering


[if !supportFootnotes][2][endif]

petitioner to pay respondent Commissioner of Internal Revenue the


amount of three million, seven hundred seventy-four thousand, eight
hundred sixty seven pesos and fifty centavos (P3,774,867.50) as 25%
surtax on improper accumulation of profits for 1981, plus 10% surcharge
and 20% annual interest from January 30, 1985 to January 30, 1987, under
Sec. 25 of the National Internal Revenue Code.
The Court of Tax Appeals made the following factual findings:
Petitioner, Cyanamid Philippines, Inc., a corporation organized under
Philippine laws, is a wholly owned subsidiary of American Cyanamid Co.
based in Maine, USA. It is engaged in the manufacture of pharmaceutical
products and chemicals, a wholesaler of imported finished goods, and an
importer/indentor.
On February 7, 1985, the CIR sent an assessment letter to petitioner and
demanded the payment of deficiency income tax of one hundred nineteen
thousand eight hundred seventeen (P119,817.00) pesos for taxable year
1981, as follows:
"Net income disclosed by the return as audited

14,575,210.00
Add: Discrepancies:

Professional fees/yr.
per investigation
17018
262,877.00
110,399.37

Total Adjustment
152,477.00
Net income per Investigation
14,727,687.00
Less: Personal and additional exemptions
___________
Amount subject to tax
14,727,687.00
Income tax due thereon .25% Surtax 2,385,231.50
3,237,495.00
Less: Amount already assessed .
5,161,788.00
BALANCE .
75,709.00
_______ monthly interest from ..1,389,636.00
44,108.00
_________
____________
Compromise penalties ...
___________
TOTAL AMOUNT DUE ..3,774,867.50
119,817.00"[if !supportFootnotes][3][endif]

Sc-lex
[if !supportEmptyParas] [endif]
[if!supportMisalignedColumns]

[endif]
On March 4, 1985, petitioner protested the assessments particularly, (1) the
25% Surtax Assessment of P3,774,867.50; (2) 1981 Deficiency Income
Assessment of P119,817.00; and 1981 Deficiency Percentage Assessment
of P8,846.72. Petitioner, through its external accountant,
[if !supportFootnotes][4][endif]

Sycip, Gorres, Velayo & Co., claimed, among others, that the surtax for the
undue accumulation of earnings was not proper because the said profits
were retained to increase petitioners working capital and it would be used
for reasonable business needs of the company. Petitioner contended that it
availed of the tax amnesty under Executive Order No. 41, hence enjoyed
amnesty from civil and criminal prosecution granted by the law.
On October 20, 1987, the CIR in a letter addressed to SGV & Co., refused
to allow the cancellation of the assessment notices and rendered its
resolution, as follows:
"It appears that your client availed of Executive Order No. 41 under File
No. 32A-F-000455-41B as certified and confirmed by our Tax Amnesty
Implementation Office on October 6, 1987.
In reply thereto, I have the honor to inform you that the availment of the
tax amnesty under Executive Order No. 41, as amended is sufficient basis,
in appropriate cases, for the cancellation of the assessment issued after
August 21, 1986. (Revenue Memorandum Order No. 4-87) Said availment
does not, therefore, result in cancellation of assessments issued before
August 21, 1986, as in the instant case. In other words, the assessments in
this case issued on January 30, 1985 despite your clients availment of the
tax amnesty under Executive Order No. 41, as amended still subsist.
Such being the case, you are therefore, requested to urge your client to pay
this Office the aforementioned deficiency income tax and surtax on undue
accumulation of surplus in the respective amounts of P119,817.00 and
P3,774,867.50 inclusive of interest thereon for the year 1981, within thirty
(30) days from receipt hereof, otherwise this office will be constrained to
enforce collection thereof thru summary remedies prescribed by law.
This constitutes the final decision of this Office on this matter."
[if !supportFootnotes][5]

[endif]

Petitioner appealed to the Court of Tax Appeals. During the pendency of


the case, however, both parties agreed to compromise the 1981 deficiency
income tax assessment of P119,817.00. Petitioner paid a reduced amount
--twenty-six thousand, five hundred seventy-seven pesos (P26,577.00) --
as compromise settlement. However, the surtax on improperly
accumulated profits remained unresolved.
Petitioner claimed that CIRs assessment representing the 25% surtax on its
accumulated earnings for the year 1981 had no legal basis for the
following reasons: (a) petitioner accumulated its earnings and profits for
reasonable business requirements to meet working capital needs and
retirement of indebtedness; (b) petitioner is a wholly owned subsidiary of
American Cyanamid Company, a corporation organized under the laws of
the State of Maine, in the United States of America, whose shares of stock
are listed and traded in New York Stock Exchange. This being the case, no
individual shareholder of petitioner could have evaded or prevented the
imposition of individual income taxes by petitioners accumulation of
earnings and profits, instead of distribution of the same. Scl-aw
In denying the petition, the Court of Tax Appeals made the following
pronouncements:
"Petitioner contends that it did not declare dividends for the year 1981 in
order to use the accumulated earnings as working capital reserve to meet
its "reasonable business needs". The law permits a stock corporation to set
aside a portion of its retained earnings for specified purposes (citing
Section 43, paragraph 2 of the Corporation Code of the Philippines). In the
case at bar, however, petitioners purpose for accumulating its earnings
does not fall within the ambit of any of these specified purposes.
More compelling is the finding that there was no need for petitioner to set
aside a portion of its retained earnings as working capital reserve as it
claims since it had considerable liquid funds. A thorough review of
petitioners financial statement (particularly the Balance Sheet, p. 127, BIR
Records) reveals that the corporation had considerable liquid funds
consisting of cash accounts receivable, inventory and even its sales for the
period is adequate to meet the normal needs of the business. This can be
determined by computing the current asset to liability ratio of the
company:
current ratio
= current assets / current liabilities

= P 47,052,535.00 / P21,275,544.00

= 2.21: 1

The significance of this ratio is to serve as a primary test of a companys


solvency to meet current obligations from current assets as a going
concern or a measure of adequacy of working capital.
xxx
We further reject petitioners argument that "the accumulated earnings tax
does not apply to a publicly-held corporation" citing American
jurisprudence to support its position. The reference finds no application in
the case at bar because under Section 25 of the NIRC as amended by
Section 5 of P.D. No. 1379 [1739] (dated September 17, 1980), the
exceptions to the accumulated earnings tax are expressly enumerated, to
wit: Bank, non-bank financial intermediaries, corporations organized
primarily, and authorized by the Central Bank of the Philippines to hold
shares of stock of banks, insurance companies, or personal holding
companies, whether domestic or foreign. The law on the matter is clear
and specific. Hence, there is no need to resort to applicable cases decided
by the American Federal Courts for guidance and enlightenment as to
whether the provision of Section 25 of the NIRC should apply to
petitioner. Rtc-spped
Equally clear and specific are the provisions of E.O. 41 particularly with
respect to its effectivity and coverage...
... Said availment does not result in cancellation of assessments issued
before August 21, 1986 as petitioner seeks to do in the case at bar.
Therefore, the assessments in this case, issued on January 30, 1985 despite
petitioners availment of the tax amnesty under E.O. 41 as amended, still
subsist."
xxx
WHEREFORE, petitioner Cyanamid Philippines, Inc., is ordered to pay
respondent Commissioner of Internal Revenue the sum of P3,774,867.50
representing 25% surtax on improper accumulation of profits for 1981,
plus 10% surcharge and 20% annual interest from January 30, 1985 to
January 30, 1987." [if !supportFootnotes][6][endif]

Petitioner appealed the Court of Tax Appeals decision to the Court of


Appeals. Affirming the CTA decision, the appellate court said:
"In reviewing the instant petition and the arguments raised herein, We find
no compelling reason to reverse the findings of the respondent Court. The
respondent Courts decision is supported by evidence, such as petitioner
corporations financial statement and balance sheets (p. 127, BIR Records).
On the other hand the petitioner corporation could only come up with an
alternative formula lifted from a decision rendered by a foreign court
(Bardahl Mfg. Corp. vs. Commissioner, 24 T.C.M. [CCH] 1030). Applying
said formula to its particular financial position, the petitioner corporation
attempts to justify its accumulated surplus earnings. To Our mind, the
petitioner corporations alternative formula cannot overturn the persuasive
findings and conclusion of the respondent Court based, as it is, on the
applicable laws and jurisprudence, as well as standards in the computation
of taxes and penalties practiced in this jurisdiction.
WHEREFORE, in view of the foregoing, the instant petition is hereby
DISMISSED and the decision of the Court of Tax Appeals dated August 6,
1992 in C.T.A. Case No. 4250 is AFFIRMED in toto." [if !supportFootnotes][7][endif]

Hence, petitioner now comes before us and assigns as sole issue:


WHETHER THE RESPONDENT COURT ERRED IN HOLDING THAT
THE PETITIONER IS LIABLE FOR THE ACCUMULATED
EARNINGS TAX FOR THE YEAR 1981. [if !supportFootnotes][8][endif]
Section 25 of the old National Internal Revenue Code of 1977
[if !supportFootnotes][9][endif]

states: Sd-aad-sc
"Sec. 25. Additional tax on corporation improperly accumulating profits
or surplus -
"(a) Imposition of tax. -- If any corporation is formed or availed of for the
purpose of preventing the imposition of the tax upon its
shareholders or members or the shareholders or members of
another corporation, through the medium of permitting its
gains and profits to accumulate instead of being divided or
distributed, there is levied and assessed against such
corporation, for each taxable year, a tax equal to twenty-
five per-centum of the undistributed portion of its
accumulated profits or surplus which shall be in addition to
the tax imposed by section twenty-four, and shall be
computed, collected and paid in the same manner and
subject to the same provisions of law, including penalties,
as that tax.
"(b) Prima facie evidence. -- The fact that any corporation is mere holding
company shall be prima facie evidence of a purpose to
avoid the tax upon its shareholders or members. Similar
presumption will lie in the case of an investment company
where at any time during the taxable year more than fifty
per centum in value of its outstanding stock is owned,
directly or indirectly, by one person.
"(c) Evidence determinative of purpose. -- The fact that the earnings or
profits of a corporation are permitted to accumulate beyond
the reasonable needs of the business shall be determinative
of the purpose to avoid the tax upon its shareholders or
members unless the corporation, by clear preponderance of
evidence, shall prove the contrary. M-issdaa
"(d) Exception -- The provisions of this sections shall not apply to banks,
non-bank financial intermediaries, corporation organized
primarily, and authorized by the Central Bank of the
Philippines to hold shares of stock of banks, insurance
companies, whether domestic or foreign.
The provision discouraged tax avoidance through corporate surplus
accumulation. When corporations do not declare dividends,
income taxes are not paid on the undeclared dividends
received by the shareholders. The tax on improper
accumulation of surplus is essentially a penalty tax
designed to compel corporations to distribute earnings so
that the said earnings by shareholders could, in turn, be
taxed.
Relying on decisions of the American Federal Courts, petitioner stresses
that the accumulated earnings tax does not apply to
Cyanamid, a wholly owned subsidiary of a publicly owned
company. Specifically, petitioner cites
[if !supportFootnotes][10][endif]

Golconda Mining Corp. vs. Commissioner, 507 F.2d 594,


whereby the U.S. Ninth Circuit Court of Appeals had taken
the position that the accumulated earnings tax could only
apply to a closely held corporation.
A review of American taxation history on accumulated earnings tax will
show that the application of the accumulated earnings tax to
publicly held corporations has been problematic. Initially,
the Tax Court and the Court of Claims held that the
accumulated earnings tax applies to publicly held
corporations. Then, the Ninth Circuit Court of Appeals
ruled in Golconda that the accumulated earnings tax could
only apply to closely held corporations. Despite Golconda,
the Internal Revenue Service asserted that the tax could be
imposed on widely held corporations including those not
controlled by a few shareholders or groups of shareholders.
The Service indicated it would not follow the Ninth Circuit
regarding publicly held corporations. In 1984,
[if !supportFootnotes][11][endif]

American legislation nullified the Ninth Circuits Golconda


ruling and made it clear that the accumulated earnings tax is
not limited to closely held corporations. [if !supportFootnotes][12][endif]

Clearly, Golconda is no longer a reliable precedent. Sl-xm-is


The amendatory provision of Section 25 of the 1977 NIRC, which was PD
1739, enumerated the corporations exempt from the
imposition of improperly accumulated tax: (a) banks; (b)
non-bank financial intermediaries; (c) insurance companies;
and (d) corporations organized primarily and authorized by
the Central Bank of the Philippines to hold shares of stocks
of banks. Petitioner does not fall among those exempt
classes. Besides, the rule on enumeration is that the express
mention of one person, thing, act, or consequence is
construed to exclude all others. Laws granting [if !supportFootnotes][13][endif]

exemption from tax are construed strictissimi juris against


the taxpayer and liberally in favor of the taxing power. [if !

Taxation is the rule and exemption is the


supportFootnotes][14][endif]

exception. The burden of proof rests upon the


[if !supportFootnotes][15][endif]

party claiming exemption to prove that it is, in fact, covered


by the exemption so claimed, a burden which [if !supportFootnotes][16][endif]

petitioner here has failed to discharge.


Another point raised by the petitioner in objecting to the assessment, is
that increase of working capital by a corporation justifies
accumulating income. Petitioner asserts that respondent
court erred in concluding that Cyanamid need not infuse
additional working capital reserve because it had
considerable liquid funds based on the 2.21:1 ratio of
current assets to current liabilities. Petitioner relies on the
so-called "Bardahl" formula, which allowed retention, as
working capital reserve, sufficient amounts of liquid assets
to carry the company through one operating cycle. The
"Bardahl" formula was developed to measure
[if !supportFootnotes][17][endif]

corporate liquidity. The formula requires an examination of


whether the taxpayer has sufficient liquid assets to pay all
of its current liabilities and any extraordinary expenses
reasonably anticipated, plus enough to operate the
business during one operating cycle. Operating cycle is the
period of time it takes to convert cash into raw materials,
raw materials into inventory, and inventory into sales,
including the time it takes to collect payment for the sales. [if !

supportFootnotes][18][endif]

Using this formula, petitioner contends, Cyanamid needed at least


P33,763,624.00 pesos as working capital. As of 1981, its
liquid asset was only P25,776,991.00. Thus, petitioner
asserts that Cyanamid had a working capital deficit of
P7,986,633.00. Therefore, the P9,540,926.00
[if !supportFootnotes][19][endif]

accumulated income as of 1981 may be validly


accumulated to increase the petitioners working capital for
the succeeding year.
We note, however, that the companies where the "Bardahl" formula was
applied, had operating cycles much shorter than that of
petitioner. In Atlas Tool Co., Inc. vs. CIR, the [if !supportFootnotes][20][endif]

companys operating cycle was only 3.33 months or 27.75%


of the year. In Cataphote Corp. of Mississippi vs. United
States, the corporations operating cycle was
[if !supportFootnotes][21][endif]

only 56.87 days, or 15.58% of the year. In the case of


Cyanamid, the operating cycle was 288.35 days, or 78.55%
of a year, reflecting that petitioner will need sufficient
liquid funds, of at least three quarters of the year, to cover
the operating costs of the business. There are variations in
the application of the "Bardahl" formula, such as average
operating cycle or peak operating cycle. In times when
there is no recurrence of a business cycle, the working
capital needs cannot be predicted with accuracy. As stressed
by American authorities, although the "Bardahl" formula is
well-established and routinely applied by the courts, it is
not a precise rule. It is used only for administrative
convenience. Petitioners application of the
[if !supportFootnotes][22][endif]

"Bardahl" formula merely creates a false illusion of


exactitude. Sl-xsc
Other formulas are also used, e.g. the ratio of current assets to current
liabilities and the adoption of the industry standard. [if !supportFootnotes]

The ratio of current assets to current liabilities is used


[23][endif]

to determine the sufficiency of working capital. Ideally, the


working capital should equal the current liabilities and there
must be 2 units of current assets for every unit of current
liability, hence the so-called "2 to 1" rule. [if !supportFootnotes][24][endif]

As of 1981 the working capital of Cyanamid was P25,776,991.00, or more


than twice its current liabilities. That current ratio of
Cyanamid, therefore, projects adequacy in working capital.
Said working capital was expected to increase further when
more funds were generated from the succeeding years sales.
Available income covered expenses or indebtedness for that
year, and there appeared no reason to expect an impending
working capital deficit which could have necessitated an
increase in working capital, as rationalized by petitioner.
In Basilan Estates, Inc. vs. Commissioner of Internal Revenue, [if !supportFootnotes][25][endif]

we held that:
"...[T]here is no need to have such a large amount at the beginning of the
following year because during the year, current assets are
converted into cash and with the income realized from the
business as the year goes, these expenses may well be taken
care of. [citation omitted]. Thus, it is erroneous to say that
the taxpayer is entitled to retain enough liquid net assets in
amounts approximately equal to current operating needs for
the year to cover cost of goods sold and operating expenses:
for it excludes proper consideration of funds generated by
the collection of notes receivable as trade accounts during
the course of the year." [if !supportFootnotes][26][endif]

If the CIR determined that the corporation avoided the tax on shareholders
by permitting earnings or profits to accumulate, and the
taxpayer contested such a determination, the burden of
proving the determination wrong, together with the
corresponding burden of first going forward with evidence,
is on the taxpayer. This applies even if the corporation is
not a mere holding or investment company and does not
have an unreasonable accumulation of earnings or profits. [if !

supportFootnotes][27][endif]

In order to determine whether profits are accumulated for the reasonable


needs of the business to avoid the surtax upon shareholders,
it must be shown that the controlling intention of the
taxpayer is manifested at the time of accumulation, not
intentions declared subsequently, which are mere
afterthoughts. Furthermore, the accumulated
[if !supportFootnotes][28][endif]

profits must be used within a reasonable time after the close


of the taxable year. In the instant case, petitioner did not
establish, by clear and convincing evidence, that such
accumulation of profit was for the immediate needs of the
business.
In Manila Wine Merchants, Inc. vs. Commissioner of Internal Revenue, [if !

we ruled: xl-aw
supportFootnotes][29][endif]

"To determine the reasonable needs of the business in order to justify an


accumulation of earnings, the Courts of the United States
have invented the so-called Immediacy Test which
construed the words reasonable needs of the business to
mean the immediate needs of the business, and it was
generally held that if the corporation did not prove an
immediate need for the accumulation of the earnings and
profits, the accumulation was not for the reasonable needs
of the business, and the penalty tax would apply. (Mertens,
Law of Federal Income Taxation, Vol. 7, Chapter 39, p.
103).
[if !supportFootnotes][30][endif]

In the present case, the Tax Court opted to determine the working capital
sufficiency by using the ratio between current assets to
current liabilities. The working capital needs of a business
depend upon the nature of the business, its credit policies,
the amount of inventories, the rate of turnover, the amount
of accounts receivable, the collection rate, the availability
of credit to the business, and similar factors. Petitioner, by
adhering to the "Bardahl" formula, failed to impress the tax
court with the required definiteness envisioned by the
statute. We agree with the tax court that the burden of proof
to establish that the profits accumulated were not beyond
the reasonable needs of the company, remained on the
taxpayer. This Court will not set aside lightly the conclusion
reached by the Court of Tax Appeals which, by the very
nature of its function, is dedicated exclusively to the
consideration of tax problems and has necessarily
developed an expertise on the subject, unless there has been
an abuse or improvident exercise of authority. [if !supportFootnotes][31][endif]

Unless rebutted, all presumptions generally are indulged in


favor of the correctness of the CIRs assessment against the
taxpayer. With petitioners failure to prove the CIR
incorrect, clearly and conclusively, this Court is constrained
to uphold the correctness of tax courts ruling as affirmed by
the Court of Appeals.
WHEREFORE, the instant petition is DENIED, and the decision of the
Court of Appeals, sustaining that of the Court of Tax Appeals, is hereby
AFFIRMED. Costs against petitioner.

Potrebbero piacerti anche